You are on page 1of 397

h na 's

K ris Text Book Solutions

B.Sc. MATHEMATICS-IV
(For B.A. and B.Sc. IVth Semester students of All Colleges affiliated to
Universities in Kumaun)
As per Kumaun University Syllabus

By

A. R. Vasishtha Hemlata Vasishtha


Retired Head, Dep’t. of Mathematics M.Sc. (Gold Medalist), Ph.D.
Meerut College, Meerut (U.P.) C.C.S. University, Meerut (U.P.)

T.B. Vector Spaces & Matrices


T.B. Real Analysis
T.B. Mathematical Methods

KRISHNA Prakashan Media (P) Ltd.


KRISHNA HOUSE, 11, Shivaji Road, Meerut-250 001 (U.P.), India
Jai Shri Radhey Shyam

Dedicated
to

Lord

Krishna
Authors & Publishers
This book (Solution Set B.Sc. IVth Semester) contains solutions of Text Books on
Vector Spaces & Matrices, Real Analysis and Mathematical Methods for
Kumaun University.
The subject matter has been discussed in such a simple way that the students will
find no difficulty to understand it. The solution of problems have been given with minute
details. Each chapter of this book contains fairly large number of solved problems which
have been selected from various university examination papers.
We have tried our best to keep the book free from misprints. The authors shall be
grateful to the readers who point out errors and omissions which, inspite of all care,
might have been there.
The authors, in general, hope that the present book will be warmly received by the
students and teachers. We shall indeed be very thankful to our colleagues for their
recommending this book to their students.
The authors wish to express their thanks to Mr. S.K. Rastogi, (M.D.),
Mr. Sugam Rastogi, (E.D.), Mrs. Kanupriya Rastogi, (Director) and entire team of
KRISHNA Prakashan Media (P) Ltd., Meerut for bringing out this book in the
present nice form.
The authors will feel amply rewarded if the book serves the purpose for which it is
meant. Suggestions for the improvement of the book are always welcome.
— Authors

(iv)
Syllabus
Vector Spaces & Matrices
Kumaun University

B.A./B.Sc. IV th Semester– Paper-I st M.M. : 60

Vector spaces: Vector space, sub spaces, Linear combinations, linear spans, Sums
and direct sums.
Bases and Dimensions: Linear dependence and independence, Bases and
dimensions, Dimensions and subspaces, Coordinates and change of bases.
Matrices: Idempotent, nilpotent, involutary, orthogonal and unitary matrices, singular
and nonsingular matrices, negative integral powers of a nonsingular matrix; Trace of a
matrix.
Rank of a matrix: Rank of a matrix, linear dependence of rows and columns of a
matrix, row rank, column rank, equivalence of row rank and column rank, elementary
transformations of a matrix and invariance of rank through elementary transformations,
normal form of a matrix, elementary matrices, rank of the sum and product of two
matrices, inverse of a non-singular matrix through elementary row transformations;
equivalence of matrices.
Applications of Matrices: Solutions of a system of linear homogeneous equations,
condition of consistency and nature of the general solution of a system of linear non-
homogeneous equations, matrices of rotation and reflection.

Real Analysis
Kumaun University

B.A./B.Sc. IV th Semester– Paper- II nd M.M. : 60

Continuity and Differentiability of functions: Continuity of functions, Uniform


continuity, Differentiability, Taylor's theorem with various forms of remainders.
Integration: Riemann integral-definition and properties, integrability of continuous
and monotonic functions, Fundamental theorem of integral calculus, Mean value
theorems of integral calculus.

(v)
Improper Integrals: Improper integrals and their convergence, Comparison test,
Dritchlet’s test, Absolute and uniform convergence, Weierstrass M-Test, Infinite integral
depending on a parameter.
Sequence and Series: Sequences, theorems on limit of sequences, Cauchy’s
convergence criterion, infinite series, series of non-negative terms, Absolute
convergence, tests for convergence, comparison test, Cauchy’s root Test, ratio Test,
Rabbe’s, Logarithmic test, De Morgan’s Test, Alternating series, Leibnitz’s theorem.
Uniform Convergence: Point wise convergence, Uniform convergence, Test of
uniform convergence, Weierstrass M-Test, Abel’s and Dritchlet’s test, Convergence and
uniform convergence of sequences and series of functions.

Mathematical Methods
Kumaun University

B.A./B.Sc. IV th Semester– Paper- II I rd M.M. : 60

Integral Transforms: Definition, Kernel.


Laplace Transforms: Definition, Existence theorem, Linearity property, Laplace
transforms of elementary functions, Heaviside Step and Dirac Delta Functions, First
Shifting Theorem, Second Shifting Theorem, Initial-Value Theorem, Final-Value
Theorem, The Laplace Transform of derivatives, integrals and Periodic functions.
Inverse Laplace Transforms: Inverse Laplace transforms of simple functions,
Inverse Laplace transforms using partial fractions, Convolution, Solutions of differential
and integro-differential equations using Laplace transforms. Dirichlet’s condition.
Fourier Transforms: Fourier Complex Transforms, Fourier sine and cosine
transforms, Properties of FourierTransforms, Inverse Fourier transforms.

(vi)
B rief C ontents
Dedication...............................................................(iII)
Preface.................................................................(iv)
Syllabus...............................................................(V-VI)
Brief Contents...................................................(VII-VIII)

Book 1: Vector Spaces & Matrices........................V-01-V-104


Chapter 1: Vector Spaces......................................................V-01—V-25

Chapter 2: Linear Dependence of Vectors..............................V-26—V-35

Chapter 3: Basis and Dimensions...........................................V-36—V-44

Chapter 4: Homomorphism of Vector Spaces.........................V-45—V-50

Chapter 5: Matrices..............................................................V-51—V-56

Chapter 6: Rank of a Matrix...................................................V-57—V-76

Chapter 7: Applications of Matrices.....................................V-77—V-104

Book 2: Real Analysis...........................................R-01-R-192

Chapter 1: Continuity...........................................................R-01—R-35

Chapter 2: Differentiability....................................................R-36—R-65

Chapter 3: Sequences...........................................................R-66—R-94

Chapter 4: Uniform Convergence of Sequences and


Series of Functions.............................................R-95—R-106

Chapter 5: Infinite Series...................................................R-107—R-162

Chapter 6: The Riemann Integral......................................R-163—R-175

Chapter 7: Convergence of Improper Integrals....................R-176—R-192

(vii)
Book 3: Mathematical Methods..............................M-01-M-96
Chapter 1: The Laplace Transform......................................M-01—M-25

Chapter 2: The Inverse Laplace Transform...........................M-26—M-51

Chapter 3: Applications of Laplace Transform......................M-52—M-75

Chapter 4: Fourier Transforms.............................................M-76—M-88

Chapter 5: Finite Fourier Transforms....................................M-89—M-96

(viii)
Krishna’s

VECTOR SPACES AND MATRICES

C hapters

1. Vector Spaces

2. Linear Dependence of Vectors

3. Basis and Dimensions


4. Homomorphism of Vector Spaces

5. Matrices

6. Rank of a Matrix

7. Applications of Matrices
L-3

Chapter-1
Vector Spaces

Comprehensive Problems 1
Problem 1: Show that a field F may be considered as a vector space over F if scalar
multiplication is identified with field multiplication.
Solution: Let ( F, + , • ) be a field. Take F as the set of vectors and also as the set of
scalars. Take the addition operation on the field F as the addition of vectors and the
multiplication operation on the field F as the operation of scalar multiplication i. e.,
multiplication of a vector by a scalar. Then F is a vector space over F as shown below :
Since ( F, + , • ) is a field, therefore ( F, + ) is an abelian group.
Further if a, b are any scalars i. e., a, b ∈ F and α, β are any vectors i. e., α, β ∈ F then
a (α + β ) = aα + aβ
and ( a + b ) α = aα + bα.
These results follow from the distributive laws in the field F.
Also ( ab ) α = a ( bα ) because the multiplication on the field F is associative.
Also if 1 is the unity element of the field F and α is any vector i. e.,α ∈ F, then1α = α.
Hence F ( F ) is a vector space.
Problem 2: Show that the complex field C is a vector space over the real field R.
Solution: Regard C as the set of vectors and R as the set of scalars.
Take the addition of complex numbers as the addition of vectors. Then (C , + ) is an
abelian group.
Let a be any scalar i. e., a ∈ R and α be any vector i. e., α ∈C . Now a ∈ R ⇒ a ∈ C . Thus
both a and α are in C. Regard the composition of scalar multiplication as the
multiplication of a and α in the field C. Since C is a field, therefore aα ∈C and thus C is
closed for scalar multiplication.
The real number 1 which is the unity element of the field R is also the unity element of
the field C.
Now let α, β be any vectors i. e.,α, β ∈C and a, b be any scalars i. e., a, b ∈ R . Since R ⊂ C ,
therefore
a, b ∈ R ⇒ a, b ∈ C.
We have a (α + β ) = aα + aβ and ( a + b ) α = aα + bα. These results follow from the
distributive laws in the field C.
Also ( ab ) α = a ( bα ) because the multiplication on the field C is associative.
Also 1α = α because 1 is also the multiplicative identity in the field C.
Hence C is a vector space over R.
V-4

Problem 3: Let V be the set of ordered pairs ( z1, z2 ) of complex numbers. Show that V is a vector
space over the real field R with addition in V and scalar multiplication on V defined by
( z1, z2 ) + ( w1, w2 ) = ( z1 + w1, z2 + w2 ) and a ( z1, z2 ) = ( az1, az2 ) where z1, z2 , w1, w2 ∈C
and a ∈ R.
Solution: Here V = {( z1, z2 ) : z1, z2 ∈ C}.
Addition composition in V, We have
α + β = ( z1 + w1, z2 + w2 ) V α = ( z1, z2 ), β = ( w1, w2 ) ∈ V .
Since z1 + w1, z2 + w2 are complex numbers, therefore α + β ∈V and thus V is closed
with respect to given addition of ordered pairs.
Scalar Multiplication composition in V over R. We have
aα = ( az1, az2 ) V a ∈ R, α = ( z1, z2 ) ∈ V .
Since az1, az2 are complex numbers, therefore aα ∈ V and thus V is closed with respect
to given scalar multiplication.
Associativity of addition in V.
Let α = ( z1, z2 ), β = ( w1, w2 ), γ = ( v1, v2 ) ∈ V .
We have {α + β} + γ = {( z1, z2 ) + ( w1, w2 )} + ( v1, v2 )
= ( z1 + w1, z2 + w2 ) + ( v1, v2 )
= ( {z1 + w1} + v1, {z2 + w2 } + v2 )
= ( z1 + {w1 + v1}, z2 + {w2 + v2 })
= ( z1, z2 ) + ( w1 + v1, w2 + v2 )
= ( z1, z2 ) + {( w1, w2 ) + ( v1, v2 )} = α + { β + γ}.
Commutativity of addition in V. We have
α + β = ( z1, z2 ) + ( w1, w2 ) = ( z1 + w1, z2 + w2 )
= ( w1 + z1, w2 + z2 ) = ( w1, w2 ) + ( z1, z2 ) = β + α.
Existence of additive identity in V. We have
(0, 0 ) ∈V . Also if ( z1, z2 ) ∈ V then
( z1, z2 ) + (0, 0 ) = ( z1 + 0, z2 + 0 ) = ( z1, z2 ).
∴ (0, 0 ) is the additive identity in V.
Existence of additive inverse of each element of V.
If ( z1, z2 ) ∈ V then ( − z1, − z2 ) ∈ V .
Also we have ( − z1, − z2 ) + ( z1, z2 ) = ( − z1 + z1, − z2 + z2 ) = (0, 0 ).
∴ ( − z1, − z2 ) is the additive inverse of ( z1, z2 ).
Thus V is an abelian group with respect to given addition. Further we observe that
1. If a ∈ R and α = ( z1, z2 ), β = ( w1, w2 ) ∈ V , then
a (α + β ) = a ( z1 + w1, z2 + w2 ) = ( a {z1 + w1}, a {z2 + w2 })
= ( az1 + aw1, az2 + aw2 ) = ( az1, az2 ) + ( aw1, aw2 )
= a ( z1, z2 ) + a ( w1, w2 ) = aα + aβ.
V-5

2. If a, b ∈ R and α = ( z1, z2 ) ∈ V , then


{a + b} α = ( {a + b} z1, {a + b} z2 ) = ( az1 + bz1, az2 + bz2 )
= ( az1, az2 ) + ( bz1, bz2 ) = a ( z1, z2 ) + b ( z1, z2 ) = aα + bα.
3. If a, b ∈ R and α = ( z1, z2 ) ∈ V , then
{ab} α = ( {ab} z1, {ab} z2 ) = ( a {bz1}, a {bz2 })
= a ( bz1, bz2 ) = a {b ( z1, z2 )} = a {bα}.
4. If 1 is the unity element of R and α = ( z1, z2 ) ∈ V , then
1α = (1z1, 1z2 ) = ( z1, z2 ) = α.
Hence V is a vector space over R.
Problem 4: Let F be the field I ( p), p being a prime number. Show that Vn( F ) is a vector space.
How many vectors are there in this vector space ?
Solution: The field I ( p) is the field
( {0, 1, 2 , … , p − 1}, + p , × p ).
The number of distinct elements in the field I ( p) is p.
We have V = {( a1, a2 , … , an ) : ai ∈ F}.
Let α = ( a1, a2 , … , an ), β = ( b1, b2 , … , bn ) ∈ V and a ∈ F.
We define vector addition and scalar multiplication in V over F as follows :
α + β = ( a1 + p b1, … , an + p bn ), aα = ( a × p a1, … , a × p an ).
We can easily show that Vn ( F )is a vector space for the operations defined as above.
Number of vectors in this space is pn.
Problem 5: Let S be any non-empty set and let F be any field. Let V be the set of all functions
from S to F i. e., let
V = { f : f : S → F}
Let us define sum of two elements f and g in V as follows :
( f + g ) ( x ) = f ( x ) + g ( x ) V x ∈ S.
Also let us define scalar multiplication of an element f in V by an element c in F as follows :
( c f ) ( x ) = c f ( x ) V x ∈ S.
Then V ( F ) is a vector space.
Solution: 1. We have V x ∈ S, ( f + g) ( x ) = f ( x ) + g ( x ). Since f ( x ) and g ( x ) are in F
and F is a field, therefore f ( x ) + g ( x ) is also in F. Thus f + g is also a function from S to
F. Therefore f + g ∈ V for all f , g ∈ V .
Associativity of addition. We have
[( f + g) + h] ( x ) = ( f + g) ( x ) + h ( x ) [by def.]
= [ f ( x ) + g ( x )] + h ( x ) [by def.]
= f ( x ) + [ g ( x ) + h ( x )]
[∵ f ( x ), g ( x ), h ( x ) are elements of F
and addition in F is associative]
= f ( x ) + ( g + h) ( x ) = [ f + ( g + h)] ( x ).
∴ ( f + g) + h = f + ( g + h).
V-6

Commutativity of addition. We have


( f + g) ( x) = f ( x) + g ( x)
= g ( x ) + f ( x ) [ ∵ addition in F is commutative]
= ( g + f ) ( x ).
∴ f + g = g + f.
^
Existence of additive identity. Let us define a function 0 : S → F such that
^
0 ( x ) = 0 V x ∈ S.
^
Then 0 ∈V and it is called zero function.
^ ^
We have ( f + 0 ) ( x ) = f ( x ) + 0 ( x ) = f ( x ) + 0 = f ( x ).
^
∴ f + 0 = f.
^
∴ the function 0 is the additive identity.
Existence of additive inverse. Let f ∈ V . Let us define a function − f : S → F by the
formula
( − f ) ( x ) = − [ f ( x )] V x ∈ S.
Then − f ∈ V and we have
[ f + ( − f )] ( x ) = f ( x ) + [( − f ) ( x )] = f ( x ) + [ − f ( x )]
^
= f ( x ) − f ( x ) = 0 = 0 ( x ).
^
∴ f + ( − f ) = 0.
∴ the function − f is the additive inverse of f.
Thus V is an abelian group with respect to addition composition.
2. If c ∈ F and f ∈ V , then V x ∈ S, we have
( c f ) ( x ) = c f ( x ).
Now f ( x ) ∈ F and c ∈ F. Therefore c f ( x ) is in F. Then c f is a function from S to F.
Therefore
c f ∈ V for all c ∈ F and for all f ∈ V .
Thus V is closed with respect to scalar multiplication.
3. We observe that
(i) If c ∈ F and f , g ∈ V , then
[ c ( f + g)] ( x ) = c [( f + g) ( x )] = c [ f ( x ) + g ( x )] = c f ( x ) + c g ( x )
= ( c f ) ( x ) + ( c g) ( x ) = ( c f + c g) ( x ).
∴ c ( f + g) = c f + c g.
(ii) If c1, c2 ∈ F and f ∈ V , then
[( c1 + c2 ) f ] ( x ) = ( c1 + c2 ) f ( x ) = c1 f ( x ) + c2 f ( x )
= ( c1 f ) ( x ) + ( c2 f ) ( x ) = ( c1 f + c2 f ) ( x ).
∴ ( c1 + c2 ) f = c1 f + c2 f .
(iii) If c1, c2 ∈ F and f ∈ V , then
[( c1 c2 ) f ] ( x ) = ( c1 c2 ) f ( x ) = c1 [ c2 f ( x )] = c1 [( c2 f ) ( x )]
= [ c1 ( c2 f )] ( x ).
∴ ( c1 c2 ) f = c1 ( c2 f ).
V-7

(iv) If 1 is the unity element of F and f ∈ V , then


(1 f ) ( x ) = 1 f ( x ) = f ( x ).
∴ 1 f = f.
Hence V is a vector space over F.
Problem 6: The vector space of all real valued continuous functions defined in some interval
[0, 1].
Solution: If f is a real valued function in the closed interval [0, 1], then we mean that
f ( x ) is a real number V x ∈[0, 1]. Let V denote the set of all real valued continuous
functions of x defined in the interval [0, 1]. Then V is a vector space over the field R of real
numbers with vector addition and scalar multiplication defined as below :
( f + g) ( x) = f ( x) + g ( x) V f , g ∈ V
and ( a f ) ( x ) = a f ( x ) V a ∈ R, V f ∈ V .
We know that the sum of two real numbers is also a real number. Also the sum of two
continuous functions is also a continuous function. Therefore V is closed with respect
to the addition of two functions defined above.
As in problem 5, we should first prove that V is an abelian group with respect to addition
composition.
V is closed with respect to scalar multiplication since a f is also a real valued continuous
function in [0, 1]. Further we observe that
1. If a ∈ R and f , g ∈ V , then
[ a ( f + g)] ( x ) = a [( f + g) ( x )] = a [ f ( x ) + g ( x )] = af ( x ) + ag ( x )
= ( af ) ( x ) + ( ag ) ( x ) = ( af + ag) ( x ).
∴ a ( f + g) = af + ag.
2. If a, b ∈ R and f ∈ V , then

[( a + b ) f ] ( x ) = ( a + b ) f ( x ) = af ( x ) + bf ( x ) = ( af ) ( x ) + ( bf ) ( x )
= ( af + bf ) ( x ).
∴ ( a + b ) f = af + bf .
3. If a, b ∈ R and f ∈ V , then
[( ab ) f ] ( x ) = ( ab ) f ( x ) = a [ bf ( x )] = a [( bf ) ( x )] = [ a ( bf )] ( x ).
∴ ( ab ) f = a ( bf ).
4. If 1 is the unity element of R and f ∈ V , then (1 f ) ( x ) = 1 f ( x ) = f ( x ).
∴ 1 f = f.
Hence V is a vector space over R.
Problem 7: Prove that the set of all vectors in a plane over the field of real numbers is a vector
space.
Solution: Let V be the set of all vectors in a plane defined as directed line segments. Let
R be the field of real numbers whose elements will be scalars.
V-8

→ → → → → →
Let α, β ∈V . If α = AB and β = BC, then we define α + β = AB + BC = AC. Since AC is
also a vector, therefore α, β ∈ V ⇒ α + β ∈ V and thus V is closed for addition of
vectors. Also from our knowledge of Vector Algebra we know that addition of vectors on

the set V is commutative as well as associative. The zero vector 0 = AA is identity for
→ →
addition of vectors. If α = AB, then the vector − α = BA is the additive inverse of α
→ → →
because − α + α = BA + AB = BB = the zero vector i. e., the identity for addition of
vectors.
Hence (V , + ) is an abelian group.
If α ∈V and m ∈ R i. e., m is any scalar, then the scalar multiplication mα is defined as a
vector whose direction is that of α or opposite to that of α according as m is +ive or −ive
and | mα | = | m |.|α |.
Since m ∈ R, α ∈ V ⇒ mα ∈ V , therefore V is closed for scalar multiplication.
Now if a, b ∈ R and α, β ∈V , then from our knowledge of Vector Algebra we know that
a (α + β ) = aα + aβ, ( a + b ) α = aα + bα and ( ab ) α = a ( bα ).
Also if α is any vector and 1 is the multiplicative identity of the field R , then by our
definition of scalar multiplication the vector1α is in the direction of the vector α and
|1 α | = |1|.|α | = 1 .|α | = |α |.
∴ by our definition of equality of two vectors, we have 1α = α.
Hence V is a vector space over the field R .
Problem 8: Let V be the set of all pairs ( x, y ) of real numbers, and let F be the field of real
numbers. Examine in each of the following cases whether V is a vector space over the field of real
numbers or not :
(i) ( x, y ) + ( x1, y1 ) = ( x + x1, y + y1 )
c ( x, y ) = ( | c | x,| c | y )
(ii) ( x, y ) + ( x1, y1 ) = ( x + x1, y + y1 )
c ( x, y ) = (0, c y )
(iii) ( x, y ) + ( x1, y1 ) = ( x + x1, y + y1 )
c ( x, y ) = ( c2 x, c2 y )
(iv) ( x, y ) + ( x1, y1 ) = ( x + y1, y + x1 )
c ( x, y ) = ( cx, cy )
Solution: (i) We shall show that in this case the postulate
( a + b ) α = aα + bα V a, b ∈ F and α ∈V fails.
Let α = ( x, y ) and a, b ∈ R. We have
( a + b ) α = ( a + b ) ( x, y ) = (| a + b | x,| a + b | y ), by def. …(1)
Also aα + bα = a ( x, y ) + b ( x, y )
= (| a | x,| a | y ) + (| b | x,| b | y ), by def. of scalar multiplication
= (| a | x + | b | x,| a | y + | b | y ), by def. of addition of vectors
= ({| a | + | b |} x,{| a | + | b |} y ). …(2)
V-9

Since | a + b |≤| a | + | b | , therefore from (1) and (2), we conclude that in general
( a + b ) α ≠ aα + bα. Hence V (R) is not a vector space.
(ii) We shall show that in this case the postulate 1α = α V α ∈ V fails. Let α = ( x, y )
where x, y ∈ R. By definition of scalar multiplication we have
1 α = 1 ( x, y ) = (0, 1 y ) = 0, y.
By (0, y ) ≠ ( x, y ) if x ≠ 0. Thus there exists α ∈V such that 1α ≠ α. Hence V ( R ) is not a
vector space.
(iii) Show that in this case the postulate ( a + b ) α = aα + bα V a, b ∈ F and α ∈V fails.
Note that in general ( a + b2) ≠ a2 + b2 .
(iv) Show that in this case the postulate α + β = β + α V α, β ∈ V
fails. Hence V ( R ) is not a vector space.
Problem 9: Show that the axiom 1α = α V α ∈ V and 1 ∈ F, defining the vector space is not a
consequence of the remaining axioms.
Solution: We shall prove this assertion by means of an example. Consider the set V of
all pairs ( x, y ) of real numbers and let F be the field of real numbers. We define vector
addition and scalar multiplication as follows :
( x, y ) + ( x1, y1 ) = ( x + x1, y + y1 ) and c ( x, y ) = (0, cy ).
Then it can be easily shown that all the axioms of vector space are satisfied except the
axiom
1α = α V α ∈ V , 1 ∈ F
as shown in problem 8 (ii). It follows that the axiom 1α = α V α ∈ V is not a conse-
quence of the remaining axioms.
Problem 10: Let V be the set of all pairs ( x, y ) of real numbers, and let F be the field of real
numbers. Define
( x, y ) + ( x1, y1 ) = ( x + x1, y + y1 )
c ( x, y ) = ( cx, y ).
Show that with these operations V is not a vector space over the field of real numbers.
Solution: Take a = 1, b = 2, α = (3, 4).
Then ( a + b ) α = 3 (3, 4) = (9, 4)
and aα + bα = 1 (3, 4) + 2 (3, 4) = (3, 4) + (6, 4) = (9, 8).
Thus we conclude that in general ( a + b ) α ≠ aα + bα.
Hence V ( R ) is not a vector space.
Problem 11: Let V be the set of ordered pairs ( x, y ) of real numbers. Show that V is not a vector
space over R with addition in V and scalar multiplication on V defined by :
(i) ( x, y ) + ( x1, y1 ) = ( x + x1, y + y1 ) and c ( x, y ) = ( x, y ) ;
(ii) ( x, y ) + ( x1, y1 ) = ( x, y ) and c ( x, y ) = ( cx, cy ) ;
(iii) ( x, y ) + ( x1, y1 ) = (0, 0 ) and c ( x, y ) = ( cx, cy ) ;
(iv) ( x, y ) + ( x1, y1 ) = ( xx1, yy1 ) and c ( x, y ) = ( cx, cy ).
V-10

Solution: (i) Take a = 1, b = 2, α = (3, 4).


Then ( a + b ) α = 3 (3, 4) = (3, 4)
and aα + bα = 1 (3, 4) + 2 (3, 4) = (3, 4) + (3, 4) = (6, 8).
Thus we conclude that in general ( a + b ) α ≠ aα + bα.
Hence V ( R ) is not a vector space.
(ii) Take α = (1, 2), β = (3, 4).
Then α + β = (1, 2) + (3, 4) = (1, 2) and β + α = (3, 4) + (1, 2) = (3, 4).
Thus we conclude that in general α + β ≠ β + α .
Hence V ( R ) is not a vector space.
(iii) For the operation of addition of vectors as defined in this case the identity element
does not exist.
Hence V ( R ) is not a vector space.
(iv) Take a = 1, b = 2, α = (2, 3).
Then ( a + b ) α = 3 (2, 3) = (6, 9) and aα + bα = 1 (2, 3) + 2 (2, 3)
= (2, 3) + (4, 6) = (8, 18).
Thus, in general ( a + b ) α ≠ aα + bα. Hence V ( R ) is not a vector space.
Problem 12: Let V be the set of all pairs ( x, y ) of real numbers and let F be the field of real
numbers. Define
( x, y ) + ( x1, y1 ) = ( x + x1, y + y1 )
c ( x, y ) = ( c x, 0 ).
Is V a vector space over the field of real numbers with these operations ?
Solution: We have 1 ∈ R and α = (2, 3) ∈ V .
Now 1α = (2, 0 ) ≠ (2, 3) = α.
Thus the axiom 1α = α V α ∈ V , 1 ∈ F fails.
Hence V is not a vector space over F for the given operations.
Problem 13: Let R be the field of real numbers and let Pn be the set of all polynomials (of degree
at most n) over the field R. Prove that Pn is a vector space over the field R.
Solution: Here Pn is the set of all polynomials of degree at most n over the field R. The
set Pn also includes the zero polynomial.
Thus Pn = { f ( x ) : f ( x ) = a0 + a1 x + a2 x2 + … + an x n, where a0 , a1, a2 , … , an ∈ R}.
If f ( x ) = a0 + a1 x + a2 x2 + … + an x n
and g ( x ) = b0 + b1 x + b2 x2 + … + bn x n

be any two members of Pn , then


n
f ( x ) + g ( x ) = ( a0 + b0 ) + ( a1 + b1 ) x + … + ( an + bn ) x
is also a member of Pn because it is also a polynomial of degree at most n over the field R .
Thus Pn is closed for addition of polynomials.
Also we know that addition of polynomials is commutative as well as associative. The
zero polynomial 0 is a member of Pn and is identity for addition of polynomials.
Also if f ( x ) = a0 + a1 x + … + an x n ∈ Pn , then
n
− f ( x ) = − a0 − a1 x − … − an x ∈ Pn
because it is also a polynomial of degree at most n over the field R .
We have − f ( x ) + f ( x ) = the zero polynomial.
∴ the polynomial − f ( x ) is the inverse of f ( x ) for addition of polynomials.
V-11

Hence Pn is an abelian group for addition of polynomials.


Now if f ( x ) = a0 + a1 x + a2 x2 + … + an x n
is any member of Pn and c ∈ R, we define
scalar multiplication c f ( x ) by the relation
c f ( x ) = ca0 + ( ca1 ) x + ( ca2 ) x2 + … + ( can ) x n.
Obviously, cf ( x ) ∈ Pn because it is also a polynomial of degree at most n over the field R .
Thus Pn is closed for scalar multiplication.
Now if a, b ∈ R and f ( x ), g ( x ) ∈ Pn , we have a [ f ( x ) + g ( x )] = af ( x ) + ag ( x ),
( a + b ) f ( x ) = a f ( x ) + bf ( x )
and ( ab ) f ( x ) = a [ b f ( x )] as can be easily shown.
Also 1 f ( x ) = f ( x ) V f ( x ) ∈ Pn .
Hence Pn is a vector space over the field R .
Problem 14: Let U and W be vector spaces over a field F. Let V be the set of ordered pairs i. e.,
V = {( u, w ) : u ∈ U , w ∈ W}.
Show that V is a vector space over F with addition in V and scalar multiplication on V defined by
( u, w ) + ( u1, w1 ) = ( u + u1, w + w1 )
and k ( u, w ) = ( ku, kw ) where u, u1 ∈ U and w, w1 ∈ W and k ∈ F.

Solution: Here V = {( u, w ): u ∈ U , w ∈ W}.


Addition composition in V. We have
α + β = ( u1, w1) + ( u2 , w2 ) = ( u1 + u2 , w1 + w2 ) V α = ( u1, w1) , β = ( u2 , w2 ) ∈ V .
Since u1 + u2 ∈ U and w1, w2 ∈ W therefore α + β ∈V and thus V is closed with respect
to the given addition of ordered pairs.
Scalar multiplication composition in V over F.
We have kα = ( ku1, kw1 ) V k ∈ F and α = ( u1, w1 ) ∈ V .
Since k u1 ∈ U and k w1 ∈ W therefore kα ∈ V and thus V is closed with respect to the
given scalar multiplication.
Associativity of addition in V.
Let α = ( u1, w1 ), β = ( u2 , w2 ), γ = ( u3 , w3 ) ∈ V .
We have {α + β} + γ = {( u1, w1 ) + ( u2 , w2 )} + ( u3 , w3 )
= ( u1 + u2 , w1 + w2 ) + ( u3 , w3 )
= ({u1 + u2 } + u3 ,{w1 + w2 } + w3 )
= ( u1 + {u2 + u3 }, w1 + {w2 + w3 })
= ( u1, w1) + ( u2 + u3 , w2 + w3 )
= ( u1, w1) + {( u2 , w2 ) + ( u3 , w3 )}
= α + {β + γ}.
V-12

Commutativity of addition in V. We have


α + β = ( u1, w1 ) + ( u2 , w2 ) = ( u1 + u2 , w1 + w2 )
= ( u2 + u1, w2 + w1 ) = ( u2 , w2 ) + ( u1, w1 ) = β + α.
Existence of addition identity in V. We have
(0, 0 ) ∈V where first coordinate is the additive identity of U and second coordinate
is the additive identity of W. Also if ( u1, w1 ) ∈ V then
( u1, w1 ) + (0, 0 ) = ( u1 + 0, w1 + 0 )
= ( u1, w1 ).
∴ (0, 0 ) is the additive identity in V .
Existence of additive inverse of each element of V : If ( u1, w1 ) ∈ V then
( − u1, − w1 ) ∈ V . Also we have
( − u1, − w1 ) + ( u1, w1 ) = ( − u1 + u1, − w1 + w1 ) = (0, 0 ).
∴ ( − u1, − w1 ) is the additive inverse of ( u1, w1 ).
This V is an abelian group with respect to given addition. Further we observe that
1. If α ∈ F and α = ( u1, w1 ), β = ( u2 , w2 ) ∈ V , then
a(α + β ) = a( u1 + u2 , w1 + w2 ) = ( a {u1 + u2 }, a {w1 + w2 })
= ( au1 + au2 , aw1 + aw2 )
= ( au1, aw1 ) + ( au2 , aw2 )
= a( u1, w1 ) + a ( u2, w2) = aα + aβ.
2. If a, b ∈ F and α = ( u1, w1 ) ∈ V , then
( a + b )α = ({a + b} u1,{a + b} w1 ) = ( au1 + bu1, aw1 + bw1 )
= ( au1, aw1) + ( bu1, bw1 )
= a( u1, w1 ) + b ( u1, w1 ) = aα + bα.
3. If a, b ∈ F and α = ( u1, w1 ) ∈ V , then {ab}α = ({ab} u1,{ab} w1 ) = ( a {bu1}, a {bw1})

= a( bu1, bw1 ) = a {b ( u1, w1 )} = a {bα}.


4. If 1 is the unity element of F and α = ( u1, w1 ) ∈ V , then
1α = (1u1, 1w1 ) = ( u1, w1 ) = α.
Hence V is a vector space over F.
This space V is called the external direct sum of U and W.

Comprehensive Problems 2
Problem 1: Show that the subset {( a, b, c ) : a + b + c = 0} of R3 is a subspace of R3 .
(Gorakhpur 2014)
Solution: Let W = {( a, b, c ) : a, b, c ∈ R and a + b + c = 0}.
We have 0 = (0, 0, 0 ) ∈ W since 0 + 0 + 0 = 0. Thus W is non-empty.
V-13

Let α = ( a1, b1, c1 ), β = ( a2 , b2 , c2 ) be any two elements of W. Then a1, b1, c1, a2 , b2 , c2
are real numbers such that
a1 + b1 + c1 = 0 …(1)
and a2 + b2 + c2 = 0. …(2)
If a, b ∈ R, we have
aα + bβ = a ( a1, b1, c1 ) + b ( a2 , b2 , c2 )
= ( aa1, ab1, ac1 ) + ( ba2 , bb2 , bc2 )
= ( aa1 + ba2 , ab1 + bb2 , ac1 + bc2 ).
Now ( aa1 + ba2 ) + ( ab1 + bb2 ) + ( ac1 + bc2 )
= a ( a1 + b1 + c1 ) + b ( a2 + b2 + c2 )
= a0 + b0 by (1) and (2)
= 0 + 0 = 0.
∴ aα + bβ ∈ W.
Hence W is a subspace of R3 .
Problem 2: Prove that the set of all solutions ( a, b, c ) of the equation a + b + 2c = 0 is a
subspace of the vector space V3 (R).
Solution: We have 0 = (0, 0, 0 ) ∈ W i. e., W is non-empty.
Let W = {( a, b, c ) : a, b, c ∈ R and a + b + 2c = 0}.
To prove that W is a subspace of V3 ( R ) or R3 .
Let α = ( a1, b1, c1 ) and β = ( a2 , b2 , c2 ) be any two elements of W. Then
a1 + b1 + 2c1 = 0 …(1)
and a2 + b2 + 2c2 = 0. …(2)
If a, b be any two elements of R, we have
aα + bβ = a ( a1, b1, c1 ) + b ( a2 , b2 , c2 )
= ( aa1, ab1, ac1 ) + ( ba2 , bb2 , bc2 )
= ( aa1 + ba2 , ab1 + bb2 , ac1 + bc2 ).
Now ( aa1 + ba2 ) + ( ab1 + bb2 ) + 2 ( ac1 + bc2 )
= a ( a1 + b1 + 2c1 ) + b ( a2 + b2 + 2c2 )
= a0 + b0 [from (1) and (2)]
= 0.
∴ aα + bβ = ( aa1 + ba2 , ab1 + bb2 , ac1 + bc2 ) ∈ W.
Thus α , β ∈W and a , b ∈ R ⇒ aα + bβ ∈ W.
Hence W is a subspace of V3 ( R ).
Problem 3: Show that the set W of the elements of the vector space V3 (R) of the form
( x + 2 y, y, − x + 3 y ) where x, y ∈ R is a subspace of V3 (R). (Kumaun 2013)
Solution: W is non-empty as (0, 0, 0 ) ∈W.
Let W = {( x + 2 y, y, − x + 3 y ) : x, y ∈ R}.
To prove that W is a subspace of V3 ( R ).
V-14

Let α = ( x1 + 2 y1, y1, − x1 + 3 y1 ) and β = ( x2 + 2 y2 , y2 , − x2 + 3 y2 ) be any two


elements of W.
If a, b be any two elements of R, we have
aα + bβ = a ( x1 + 2 y1, y1, − x1 + 3 y1 ) + b ( x2 + 2 y2 , y2 , − x2 + 3 y2 )
= ( ax1 + 2ay1, ay1, − ax1 + 3ay1 ) + ( bx2 + 2by2 , by2 , − bx2 + 3by2 )
= ( ax1 + 2ay1 + bx2 + 2by2 , ay1 + by2 , − ax1 + 3ay1 − bx2 + 3by2 )
= ([ ax1 + bx2 ] + 2 [ ay1 + by2 ], ay1 + by2 , − [ ax1 + bx2 ] + 3 [ ay1 + by2 ])
which is in W because it is of the form ( x + 2 y, y, − x + 3 y ).
Here in place of y we have ay1 + by2 and in place of x we have ax1 + bx2 .
Thus α, β ∈W and a, b ∈ R ⇒ aα + bβ ∈ W.
Hence W is a subspace of V3 ( R ).
Problem 4: Let V be the vector space of all polynomials over the field R. Determine whether or
not W is a subspace of V where (i) W contains all polynomials with integral coefficients (ii) W
consists of all polynomials b0 + b1 x2 + b2 x4 + … + bn x2 n i. e., polynomials with only even
powers of x.
Solution: (i) Any α ∈W can be expressed as
α = a0 + a1 x + a2 x2 + … + an x n,
where a0 , a1, … , an are integers.
1 a a a a
We have α = 0 + 1 x + 2 x2 + … + n x n.
3 3 3 3 3
1 a0 a1 an
Evidently α ∉W as , ,… are not necessarily integers.
3 3 3 3
1 1
Thus ∈ R, α ∈ W but α ∉W.
3 3
Hence W is not closed w.r.t. scalar multiplication. So W is not a subspace of V.
(ii) Proceed as in part (i) Ans. Yes.
Problem 5: Which of the following sets of vectors α = ( a1, … , an ) in R n are subspaces of
R n ? ( n ≥ 3).
(i) all α such that a1 ≥ 0; (ii) all α such that a1 + 3a2 = a3 ;
(iii) all α such that a2 = a12 ; (Kumaun 2011) (iv) all α such that a1 a2 = 0;
(v) all α such that a2 is rational.
Solution: (i) Let W = {α : α ∈ R n and a1 ≥ 0}.
Let α = ( a1, … , an ) and β = ( b1, … , bn ) be any two members of W. Then a1 ≥ 0 and b1 ≥ 0.
If a, b ∈ R, then aα + bβ = ( aa1 + bb1, … , aan + bbn ). If a and b are any two real numbers,
then aa1 + bb1 will not necessarily be ≥ 0. For example, if we take a1 = 3, b1 = 3, a = − 2
and b = − 2 , then aa1 + bb1 = − 6 − 6 = − 12 which is < 0. Thus α, β ∈W and
a, b ∈ R ⇒ aα + bβ ∉W. Hence W is not a subspace of R n .
(ii) Let W = {α : α ∈ R n and a1 + 3a2 = a3 }. W is non-empty since 0 = (0, 0, … , 0 ) ∈ W.
Let α = ( a1, … , an ) and β = ( b1, … , bn ) be any two members of W. Then a1 + 3a2 = a3 and
b1 + 3b2 = b3 .
If a, b ∈ R, then aα + bβ = ( aa1 + bb1, … , aan + bbn ).
V-15

We have ( aa1 + bb1 ) + 3 ( aa2 + bb2 ) = a ( a1 + 3a2 ) + b ( b1 + 3b2 ) = aa3 + bb3 . Thus
according to the definition of W, aα + bβ ∈ W. In this way α , β ∈W and a, b ∈ R
⇒ aα + bβ ∈ W. Hence W is a subspace of R n .
(iii) Let W = {α : α ∈ R n and a2 = a12 }. Let α = ( a1, … , an ) and β = ( b1, … , bn ) be any two
members of W. Then a2 = a12 and b2 = b12 .
If a, b ∈ R, then aα + bβ = ( aa1 + bb1, … , aan + bbn ).
Now aa2 + bb2 = aa12 + bb12 which is not necessarily equal to ( aa1 + bb1 )2 . For example,
take a1 = 2, a2 = 4, b1 = 3, b2 = 9, a = 2 , b = 3, then a2 = a12 and b2 = b12 .
Also aa2 + bb2 = 8 + 27 = 35 and ( aa1 + bb1 )2 = (13)2 = 169.
Thus aa2 + bb2 ≠ ( aa1 + bb1 )2 . In this way aα + bβ ∉W. Hence W is not a subspace of R n .
(iv) Let W = {α : α ∈ R n and a1 a2 = 0}.
Let α = ( a1, … , an ) and β = ( b1, … , bn ) be any two members of W. Then a1a2 = 0, b1b2 = 0.
If a, b ∈ R, then aα + bβ = ( aa1 + bb1, … , aan + bbn ).
We have
( aa1 + bb1 ) ( aa2 + bb2 ) = a2 a1a2 + ab ( a1b2 + a2 b1 ) + b2 b1b2 = ab ( a1b2 + a2 b1 )
which is not necessarily equal to zero. In this way aα + bβ is not necessarily a member of
W. Hence W is not a subspace of R n .
(v) Let W = {α : α ∈ R n and a2 is rational}. Let α = ( a1, … , an ) and β = ( b1, … , bn ) be any
two members of W. Then a2 is rational and b2 is rational. If a, b ∈ R , then
aα + bβ = ( aa1 + bb1, … , aan + bbn ). Now aa2 + bb2 is not necessarily rational. For
example if we take a = √ 3, b = √ 7, a2 = 3, b2 = 4, then aa2 + bb2 is not rational. Thus in
this case aα + bβ ∉W. Hence W is not a subspace of R n .
Problem 6: Let F be the field of integers modulo 2, V be the set of all 2 × 2 matrices over F. Show
that V ( F ) is a finite vector space. Give two non-trivial subspaces of this vector space.
Solution: Here F = ( {0, 1}, + 2 , ×2 ) is a field.
a c  
V =  : a, b, c, d ∈ F  ⋅
b d  
a c1  a c2  a c3 
Let A= 1  ,B=  2  ,C=  3 ,
 b1 d1   b2 d2   b3 d3 
where ai , bi , c i , di ∈ F for i = 1, 2 , 3.
Then A, B, C ∈ V .
We can easily prove that V is an abelian group with respect to addition of matrices.
0 0 
The matrix O =  ∈ V is the additive identity of the group. A ∈ V has its
0 0 
additive inverse X ∈ V s.t. A + X = X + A = O. For example if
 0 0 0 0 
A=  then X =  1 as 1 + 2 1 = 0.
 1 1   1 
V-16

If a ∈ F and α ∈V (i. e., α is a matrix of the type 2 × 2 with elements ∈ F), then aα ∈ V
because aα is also a matrix of the type 2 × 2 with elements ∈ F. Therefore V is closed with
respect to scalar multiplication. Also we observe that
(i) a (α + β ) = aα + aβ V a ∈ F and α, β ∈V .
(ii) ( a + b ) α = aα + bα V a, b ∈ F and V α ∈V.
(iii) ( ab ) α = a ( bα ) V a, b ∈ F and V α ∈V.
(iv) 1α = α V α ∈ V where 1 is the unity element of the field F.
Hence V ( F ) is a vector space.
Number of vectors (matrices) in
V ( F ) = 24 = 16.
Thus V ( F ) is a finite vector space.
Two non-trivial subspaces of V ( F ) are
 a 0    0 c  
W1 =   : a, b ∈ F  , W2 =  0 : c, d ∈ F .
 b 0    d  
Problem 7: Let V be the vector space of all n × n square matrices over a field F. Show that W is a
subspace of V if W consists of all matrices which are
(i) symmetric i. e., all matrices A = [ aij ] for which a ji = aij .
(ii) skew-symmetric.
(iii) upper triangular.
(iv) diagonal.
(v) scalar.
Solution: (i) W is non-empty since On × n ∈ W . Let A = [ aij ] and B = [ bij ] be any two
members of W i. e., a ji = aij and b ji = bij . If a, b ∈ F, aA + bB is the matrix whose ijth
element is aaij + bbij and ji th element is aa ji + bb ji . But aa ji + bb ji = a aij + b bij .
Thus aA + bB is also symmetric. So aA + bB ∈ W.
Hence W is a subspace of V.
(ii) W is non-empty since On × n ∈ W. Let A = [ aij ] and B = [ bij ] be any two members of
W i. e., a ji = − aij and b ji = − bij . If a, b ∈ F, aA + bB is the matrix whose ijth element is
aaij + bbij . But aa ji + bb ji = − ( aaij + bbij ). Thus aA + bB is also skew-symmetric.
So aA + bB ∈ W. Hence W is a subspace of V.
(iii) An upper triangular matrix is a square matrix whose entries below the main diagonal
are all zero. W is non-empty since On × n ∈ W. Let A = [ aij ] and B = [ bij ] be any two
members of W i. e., A and B are matrices whose entries below the main diagonal are all
zero. If a, b ∈ F. aA + bB is the matrix whose entries below the main diagonal are all
zero. Thus aA + bB is an upper triangular matrix.
So aA + bB ∈ W.
Hence W is a subspace of V.
(iv) A diagonal matrix is a square matrix whose non-diagonal entries are all zero. W is
non-empty since On × n ∈ W. Let A, B ∈ W. Then A, B are square matrices whose
non-diagonal entries are all zero. If a, b ∈ F, aA + bB is a square matrix whose
non-diagonal entries are all zero. Thus aA + bB is a diagonal matrix.
So aA + bB ∈ W. Hence W is a subspace of V .
V-17

(v) The matrix kI, for a scalar k ∈ F, is called a scalar matrix. W is non-empty since
On × n ∈ W. O = oI .
Let A, B ∈ W. Then A = k1 I , B = k2 I for some scalars k1, k2 ∈ F.
For a, b ∈ F, aA + bB = ( ak1 + bk2 )I .
Thus aA + bB is a scalar matrix, for a scalar ak1 + bk2 ∈ F.
So aA + bB ∈ W. Hence W is a subspace of V .
Problem 8: Let V be a vector space of all real n × n matrices. Prove that the set W consisting of
all n × n real matrices which commute with a given matrix T of V forms a subspace of V .
Solution: We have On × n ∈ W since OT = O = TO.
Now let A, B ∈ W i. e., AT = TA and BT = TB.
If a, b ∈ R , then ( aA + bB) T = ( aA ) T + ( bB) T
= a ( AT ) + b ( BT ) = a (TA ) + b (TB ) = T ( aA ) + T ( bB)
= T ( aA + bB).
Thus aA + bB commutes with T i. e., aA + bB ∈ W.
Hence W is a subspace of V.
Problem 9: Let V be the vector space of all 2 × 2 matrices over the real field R. Show that the
subset of V consisting of all matrices A for which A2 = A is not a subspace of V .
1 0  2 1 0  1 0  1 0 
Solution: We have I = , I =  0 1   0 1  =  0 1  = I .
0 1      
2 0  2 2 0 2 0  4 0 
Now 2I =  , (2I ) =  = ≠ 2I.
0 2  0 2   0 2   0 4 
Thus I ∈ W, but 2I ∉W.
Hence W is not a subspace of V.
Problem 10: Determine whether or not W is a subspace of R3 if W consists of those vectors
( a, b, c ) ∈ R3 for which :
(i) a = 2b (ii) ab = 0
(iii) a ≤ b ≤ c (iv) a = b2
(v) a2 + b2 + c2 ≤ 1 (vi) k1 a + k2 b + k3 c = 0, ki ∈ R.
Solution: (i) (0, 0, 0 ) ∈W i. e., W is non-empty.
Let α = ( a1, b1, c1 ), β = ( a2 , b2 , c2 ) ∈ W.
Then a1 = 2b1 and a2 = 2b2 .
If a, b ∈ R then aα + bβ = a ( a1, b1, c1 ) + b ( a2 , b2 , c2 )
= ( aa1 + ba2 , ab1 + bb2 , ac1 + bc2 ).
Now aa1 + ba2 = a (2b1 ) + b (2b2 ) = 2 ( ab1 + bb2 ).
Thus aα + bβ ∈ W. Hence W is a subspace of V.
(ii) Let α = (2 , 0, 0 ), β = (0, 2 , 0 ). Then α, β ∈W.
Now α + β = (2 , 0, 0 ) + (0, 2 , 0 ) = (2 , 2 , 0 ) ∉W as 2 . 2 = 4 ≠ 0.
Hence W is not a subspace of V.
V-18

(iii) W is not a subspace of V.


We have (2, 3, 4) ∈W but − 3 (2, 3, 4) ∉W.
(iv) W is not a subspace of V.
We have (4, 2, 0 ) ∈W but 2 (4, 2, 0 ) ∉W.
(v) W is not a subspace of V.
We have (1, 0, 0 ) ∈ W, (0, 1, 0 ) ∈ W.
But (1, 0, 0 ) + (0, 1, 0 ) = (1, 1, 0 ) ∉W.
(vi) We have k1 ⋅ 0 + k2 ⋅ 0 + k3 ⋅ 0 = 0
∴ (0, 0, 0 ) ∈W i. e., W is non- empty.

Let α = ( a1, b1, c1 ), β = ( a2 , b2 , c2 ) ∈ W.


Then k1a1 + k2 b1 + k3 c1 = 0, k1a2 + k2 b2 + k3 c2 = 0.
If a, b ∈ R then aα + bβ = a( a1, b1, c1 ) + b( a2 , b2 , c2 )
= ( aa1 + ba2 , ab1 + bb2 , ac1 + bc2 ).
Now k1 ( aa1 + ba2 ) + k2 ( ab1 + bb2 ) + k3 ( ac1 + bc2 )
= a( k1 a1 + k2 b1 + k3 c1 ) + b ( k1a2 + k2 b2 + k3 c2 )
= a ⋅ 0 + b ⋅ 0 = 0 + 0 = 0.
Thus aα + bβ ∈ W . Hence W is a subspace of V.
Problem 11: Let V be the (real) vector space of all functions f from R into R. Which of the
following sets of functions are sub-spaces of V ?
(i) all f such that f ( x2 ) = [ f ( x )]2 ; (ii) all f such that f (0 ) = f (1) ;
(iii) all f such that f (3) = 1 + f ( − 5) ; (iv) all f such that f ( − 1) = 0;
(v) all f which are continuous.
Solution: (i) Let W = { f : f ∈ V and f ( x2 ) = [ f ( x )]2 }.
Let f , g be any two members of W.
Then f ( x2 ) = [ f ( x )]2 and g ( x2 ) = [ g ( x )]2 .
Let a, b ∈ R.
Then ( af + bg) ( x 2 ) = ( af ) ( x 2 ) + ( bg) ( x 2 )

= af ( x 2 ) + bg ( x 2 ) = a [ f ( x )]2 + b [ g( x )]2 .

Also [( af + bg) ( x )]2 = [ af ( x ) + bg ( x )]2

= a2 [ f ( x )]2 + b2 [ g ( x )]2 + 2ab f ( x ) g ( x ).


Now ( af + bg) ( x 2 ) is not necessarily equal to [( af + bg) ( x )]2 . Thus af + bg is not
necessarily a member of W.
Hence W is not a sub-space of V.
V-19

^
(ii) 0 ∈W since ^
0 (0 ) = 0 = ^
0 (1). Let f , g ∈ W in this case.
Then f (0 ) = f (1) and g (0 ) = g (1).
Let a, b ∈ R . Then
( af + bg) (0 ) = af (0 ) + bg (0 ) = af (1) + bg (1) = ( af + bg) (1).
Therefore by definition of W, af + bg ∈ W. Hence W is a subspace of V.
(iii) Let f , g ∈ W in this case. Then f (3) = 1 + f ( − 5) and g (3) = 1 + g ( − 5). We have
( af + bg)(3) = af (3) + bg (3) = a [1 + f ( − 5)] + b [1 + g ( −5)] = a + b + ( af + bg) ( −5)
which is not necessarily equal to 1 + ( af + bg) ( − 5). Hence W is not a subspace of V.
(iv) Let W = { f : f ∈ V and f ( − 1) = 0}. ^
0 ∈W. Let f , g ∈ W. Then f ( − 1) = 0 and
g ( − 1) = 0. If a, b ∈ R , then
( af + bg) ( − 1) = ( af ) ( − 1) + ( bg) ( − 1) = af ( − 1) + bg ( − 1) = a (0 ) + b (0 ) = 0.
Therefore af + bg ∈ W. Hence W is a subspace of V.
(v) If f and g are continuous functions and a, b ∈ R , then af + bg is also a continuous
function. Hence in this case W is a subspace of V.
Problem 12: Let V be the vector space of all functions from the real field R into R. Check
whether W is a subspace of V where
(i) W = { f : f (7) = 2 + f (1) }; (ii) W consists of all bounded functions;
(iii) W consists of all integrable functions in the interval [0, 1].
Solution: (i) Let f , g ∈ W.
Then f (7) = 2 + f (1) and g (7) = 2 + g (1).
We have ( f + g) (7) = f (7) + g (7) = 2 + f (1) + 2 + g (1)
= 4 + f (1) + g (1) = 4 + ( f + g) (1)
≠ 2 + ( f + g) (1).
So f + g ∉W. Hence W is not a subspace of V.
(ii) A function f : R → R is bounded if there exists M ∈ R such that
| f ( x )|≤ M V x ∈ R.
W is non-empty as ^
0 ∈W.
Let f , g ∈ W with M f and M g bounds for f and g respectively. If a, b ∈ R then we have
|( af + bg) ( x )| = | a f ( x ) + bg ( x )|≤ | a f ( x )| + | bg ( x )|
= | a || f ( x )| + | b || g ( x )|≤ | a | M f + | b | M g .
Thus| a | M f + | b | M g is a bound for the function af + bg. So af + bg ∈ W. Hence W is a
subspace of V.
(iii) W is non-empty as ^
0 ∈W. If f and g are integrable functions in the interval [0, 1]
and a, b ∈ R then af + bg is also integrable in the interval [0, 1].
Problem 13: Let V be the vector space of infinite sequences < a1, a2 , … an, … > in a field F.
Show that W is a subspace of V , if
(i) W consists of all sequences with 0 as the first component ;
(ii) W consists of all sequences with only a finite number of non-zero components.
V-20

Solution: (i) W is non-empty since 0 = < 0, 0, … , 0 , … > ∈ W.


Let α = < 0, a2 , … , an …>, β = < 0, b2 , … , bn , …> be any two elements of W.
If a, b ∈ F then aα + bβ = < 0, aa2 + bb2 , …>.
Thus aα + bβ ∈ W. Hence W is a subspace of V.
(ii) Proceed as in part (i).
Problem 14: Let AX = B be a non-homogeneous system of linear equations in n unknowns over
a field F. Show that the solution set W of the system is not a subspace of F n .
Solution: The equation AX = B is not satisfied if we take X = On × 1. Thus the solution
set of the system does not contain On × 1 i. e., the zero vector of F n.

Hence W is not a subspace of F n.

Comprehensive Problems 3
Problem 1: For which value of k will the vector (1, − 2, k ) in R3 be a linear combination of the
vectors (3, 0, − 2) and (2, − 1, − 5) ?
Solution: Let α1 = (3, 0, − 2) and α2 = (2, − 1, − 5).
Let (1, − 2, k ) = aα1 + bα2 where a, b ∈ R.
Then (1, − 2, k ) = a (3, 0, − 2) + b (2, − 1, − 5)
= (3a, 0, − 2a) + (2b, − b, − 5b )
= (3a + 2b, 0 − b, − 2a − 5b )
∴ 3a + 2b = 1, − b = − 2 ,−2a − 5b = k.
These give b = 2, a = −1, k = − 8.
Hence the required value of k is −8.
Problem 2: In the vector space R4 determine whether or not the vector (3, 9, − 4, 2) is a linear
combination of the vectors (1, − 2, 0, 3), (2, 3, 0, − 1) and (2, − 1, 2, 1).
Solution: Proceed as in Ex. 20 of Text Book. Ans. No.
Problem 3: Is the vector (3, − 1, 0, − 1) in the subspace of R4 spanned by the vectors
(2, − 1, 3, 2), ( − 1, 1, 1, − 3) and (1, 1, 9, − 5) ?
Solution: Let α = (3, − 1, 0, − 1), α1 = (2 , − 1, 3, 2),
α2 = ( − 1, 1, 1, − 3), α3 = (1, 1, 9, − 5).
If α can be expressed as a linear combination of the vectors α1, α2 , α3 , then it will be in the
subspace of R4 spanned by these vectors otherwise it will not be.
Let α = aα1 + bα2 + cα3 where a, b, c ∈ R .
Then (3, − 1, 0, − 1) = a (2 , − 1, 3, 2) + b ( − 1, 1, 1, − 3) + c (1, 1, 9, − 5).
∴ 2a − b + c = 3, …(1)
− a + b + c = − 1, …(2)
3a + b + 9c = 0, …(3)
and 2a − 3b − 5c = − 1. …(4)
V-21

Adding the equations (1) and (2), we get a + 2c = 2 …(5)


Again adding the equations (1) and (3), we get
5a + 10 c = 3 …(6)
Multiplying the equation (5) by 5, we get
5a + 10 c = 10. …(7)
The relations (6) and (7) show that the equations (1), (2), (3) and (4) are inconsistent
i. e., do not possess a common solution. Hence the vector α cannot be expressed as a linear
combination of the vectors α1, α2 , α3 . Therefore α is not in the subspace of R4
generated by the vectors α1, α2 , α3 .
Problem 4: Is the vector (2, − 5, 3) in the subspace of R3 spanned by the vectors
(1, − 3, 2),(2, − 4, − 1),(1, − 5, 7)? (Kumaun 2009; Garhwal 13)
Solution: Let α = (2, − 5, 3), α1 = (1, − 3, 2), α2 = (2, − 4, − 1), α3 = (1, − 5, 7). If α can be
expressed as a linear combination of the vectors α1, α2 , α3 , then it will be in the
subspace of R3 spanned by these vectors otherwise it will not be.
Let α = a1 α1 + a2 α2 + a3 α3 where a1, a2 , a3 ∈ R.
Then (2, − 5, 3) = a1 (1, − 3, 2) + a2 (2, − 4, − 1) + a3 (1, − 5, 7)
or (2, − 5, 3) = ( a1 + 2a2 + a3 , − 3a1 − 4a2 − 5a3 , 2a1 − a2 + 7a3 ).
a1 + 2a2 + a3 = 2  ...(1)

∴ − 3a1 − 4a2 − 5a3 = − 5 ...(2)
2a1 − a2 + 7a3 = 3  ...(3)
Multiplying the equation (1) by 3 and adding to (2), we get
2a2 − 2a3 = 1
1
or a2 − a3 = ⋅ …(4)
2
Again multiplying the equation (1) by 2 and subtracting from (3), we get
− 5a2 + 5a3 = − 1
or a2 − a3 = 1 /5. …(5)
The relations (4) and (5) show that the above equations are inconsistent. Hence the
vector α cannot be expressed as a linear combination of the vectors α1, α2 , α3 . Therefore
α is not in the subspace of R3 generated by the vectors α1, α2 , α3 .
3 1 
Problem 5: Express the matrix E =   as a linear combination of the matrices
 1 −1 
1 1   0 0 0 2 
A=  , B =  1 1 and C =  0 −1  .
 1 0     
Solution: Let E = aA + bB + cC where a, b, c are scalars
3 1 1 1 0 0  0 2
or 1 =a + b + c
 − 1  1 0   1 1  0 − 1 
V-22

a a 0 0  0 2c   a a + 2c 
= + + =
a 0   b b   0 − c   a + b b − c 
∴ a = 3, a + 2c = 1, a + b = 1, b − c = − 1.
From the first three equations, we have a = 3, b = − 2 and c = − 1.
Since these values of a, b and c also satisfy the last equation, they form a solution of the
system i. e., E can be expressed as a linear combination of A, B and C.
Hence E = 3 A − 2B − C.
Problem 6: Write E as a linear combination of
 1 1  1 1  1 −1 
A=  ,B =   and C =  
 0 −1   −1 0  0 0 
 3 −1   2 1
where (i) E=  (ii) E =  .
 1 − 2   − 1 − 2 
Solution: Proceed as in problem 5.
Ans. (i) E = 2 A − B + 2C. (ii) Not possible.
Problem 7: Express the polynomial f = x2 + 4 x − 3 over R as a linear combination of the
polynomials
f1 = x2 − 2 x + 5, f2 = 2 x2 − 3 x and f3 = x + 3.
Solution: Let f = a f1 + b f2 + c f3 , where a, b, c ∈ R.

Then x 2 + 4 x − 3 = a ( x 2 − 2 x + 5) + b (2 x 2 − 3 x ) + c ( x + 3)

= x 2 ( a + 2b ) + x ( − 2a − 3b + c ) + (5a + 3c ).
Equating the coefficients of the like powers of x, we get
a + 2b = 1, − 2a − 3b + c = 4, 5a + 3c = − 3.
Solving these equations, we get a = − 3, b = 2 , c = 4.
Hence f = − 3 f1 + 2 f2 + 4 f3 .
Problem 8: Express f as a linear combination of the polynomials
f1 = 2 x2 + 3 x − 4, f2 = x2 − 2 x − 3
where (i) f = 3 x2 + 8 x − 5 (ii) f = 4 x2 − 6 x − 1 .
Solution: Proceed as in Problem 7.
Ans. (i) f = 2 f1 − f2 (ii) Not possible.
Problem 9: Find the condition on a, b and c so that ( a, b, c ) ∈ R3 is in the space generated by
(2, 1, 0 ), (1, − 1, 2) and (0, 3, − 4).
Solution: Let ( a, b, c ) = x (2 , 1, 0 ) + y (1, − 1, 2) + z (0, 3, − 4) where x, y, z ∈ R.
Then ( a, b, c ) = (2 x + y, x − y + 3z, 2 y − 4z ).
∴ 2x + y = a …(1)
x − y + 3z = b …(2)
2 y − 4z = c …(3)
V-23

From (2) and (3), we have


1
4 x + 2 y = 4b + 3c or 2x + y = (4b + 3c ) …(4)
2
The vector ( a, b, c ) will be in the space generated by the given vectors if and only if the
system of equations (1), (2) and (3) is consistent
1
i. e., if and only if a = (4b + 3c ), from (1) and (4)
2
i. e., if and only if 2a − 4b − 3c = 0.
Problem 10: Find one vector in R3 which generates the intersection of W1 and W2 where
W1 = {( a, b,0 ) : a, b ∈ R} and W2 is the space generated by the vectors (1, 2, 3) and (1, − 1, 1).
Solution: Let ( a, b, c ) ∈ W2 .
Then ( a, b, c ) = x (1, 2 , 3) + y (1, − 1, 1) where x, y ∈ R.
or ( a, b, c ) = ( x + y, 2 x − y, 3 x + y ).
∴ x + y = a, 2 x − y = b, 3 x + y = c.
Also ( a, b, c ) ∈ W1.
∴ c = 0 ⇒ 3 x + y = 0.
Take x = 1. Then y = − 3.
∴ a = − 2 , b = 5.
Hence one vector can be ( − 2 , 5, 0 ).
Problem 11: If α, β and γ are vectors such that α + β + γ = 0, then α and β span the same
subspace as β and γ.
Solution: We have α + β + γ = 0 …(1)
Let S = {α , β} and T = { β, γ}.
Let δ ∈ L ( S ). Then δ can be written as a linear combination of α, β. From (1),
α = − ( β + γ ). Thus δ can be written as a linear combination of β, γ so δ ∈ L (T ).
∴ L ( S ) ⊂ L (T ).
Similarly, using (1) we have L (T ) ⊂ L ( S ).
Thus L ( S ) = L (T )
i. e., α , β span the same subspace as β, γ.
Problem 12: Give an example of a subset W of a vector space V which is not a subspace of V but
for which
(i) W + W = W . (ii) W + W ⊂ W .
2
Solution: (i) Consider V = R , W = {(0, 0 ), (0, 1), (0, 2), (0, 3), …}.
Here W is a subset of R2 . But it is not a subspace of R2 as

(0, 1) =  0,  ∉W.
1 1
3  3
Also we have W + W = W.
V-24

(ii) Consider V = R2 , W = {(0, 6), (0, 7), (0, 8), …}.

Here W is a subset of R2 , but it is not a subspace of R2 as (0, 0 ) ∉W.


Every element of W + W will be in W while every element of W cannot be taken as an
element of W + W e. g., (0, 6) ∈W but (0, 6) ∉W + W.
Thus W + W ⊂ W.
Problem 13: Let V be the vector space of n × n matrices over a field F. Let W1 and W2 be the
subspaces of upper triangular matrices and lower triangular matrices respectively. Find
(i) W1 + W2 (ii) W1 ∩ W2 .
Solution: Proceed as in article 6, Illustration of Text Book.
Ans. (i) W1 + W2 = V .
(ii) W1 ∩ W2 is the space of diagonal matrices.
Problem 14: Show that the space U generated by the vectors
u1 = (1, 2, − 1, 3), u2 = (2, 4,1, − 2) and u3 = (3, 6, 3, − 7)
and the space V generated by the vectors v1 = (1, 2, − 4, 11) and v2 = (2, 4, − 5, 14) are equal.
Solution: Method 1. Show that each ui can be expressed as a linear combination of
v1 and v2 and each vi can be expressed as a linear combination of u1, u2 and u3 . Here we
have to show that six systems of linear equations are consistent.
Method 2. Form the matrix A whose rows are the generators of U and reduce A to its
row canonical form .

1 2 −1 3  1 2 −1 3 
A=2 4 1 −2 ~ 0 0 3 −8 
   
 3 6 3 −7   0 0 6 − 16 

 1 2 −1 3   1 2 0 1/3 
~ 0 0 3 −8 ~ 0 0 1 − 8 /3⋅
   
 0 0 0 0   0 0 0 0
Now form the matrix B whose rows are the generators of V and reduce B to its row
canonical form.
1 2 −4 11   1 2 −4 11 
B= ~ 
2 4 −5 14  0 0 3 − 8 

 1 2 0 1 /3 
~  ⋅
0 0 1 − 8 /3 
Since row reduced Echelon matrices of A and B have the same non-zero rows, the row
spaces of A and B are equal i. e., U = V .
V-25

Hints to Objective Type Questions

Multiple Choice Questions


1. (c). If ( a1, a2 , a3 , a4 ) is any element of R4 , then (0, 0,0, 0 ) + ( a1, a2 , a3 , a4 )
= (0 + a1, 0 + a2 , 0 + a3 , 0 + a4 ) = ( a1, a2 , a3 , a4 ).
∴ (0, 0, 0, 0 ) is the identity for addition of vectors in R4 .
Hence (0, 0, 0, 0 ) is the zero vector in the vector space R4 .
2. (a). See Ex. 8 of Text Book. 3. (c). See Ex. 19 of Text Book.
4. (c). See article 3, Remark of Text Book.
5. (c). See article 4, Theorem 3, of Text Book.
6. (a). See Ex. 18, of Text Book. 7. (b). See article 4 of Text Book.
Fill in the Blank(s)
1. See theorem 3 of article 3 of Text Book.
2. See Illustration 1 after article 6 of Text Book.
3. See article 7, Theorem 1 of Text Book.
True or False
1. T . Every subspace of a vector space contains the zero vector of the vector space.
2. F. Let W = {( x, y ) : x, y ∈ R, x2 = y2 }.Take α = (2, 2), β = (3, − 3).
Then α + β = (5, − 1). Thus α, β ∈W but α + β ∉W.
Hence W is not closed w.r.t. vector addition of V2 ( R ).
3. F. W = {( x, y, z ) : x > 0}. Let α = (1, 2, 3) ∈ W and a = − 5 ∈ R . Then
aα = − 5 (1, 2, 3) = ( − 5, − 10, − 15)
does not belong to W since − 5 < 0. Hence W is not a subspace of V3 ( R ).
4. T . Let W = {( x, y, z ) : x, y, z ∈ R and x + y = 0}.
Here 0 = (0, 0, 0 ) ∈ W. Also if a, b ∈ R and α, β ∈W then aα + bβ ∈ W.
5. F. See Note of Theorem 1 of article 4 of Text Book.
6. T . See Theorem 1 of article 4 of Text Book.
7. F. See Ex. 21 of Text Book.

❍❍❍
V-26

Chapter-2
Linear Dependence of Vectors

Comprehensive Problems 1

Problem 1: Show that the vectors (1, 1, 2, 4), (2, − 1, − 5, 2), (1, − 1, − 4, 0 ) and (2, 1, 1, 6) are
linearly dependent in R4 .
Solution: Let (1, 1, 2 , 4) = a (2 , − 1, − 5, 2) + b (1, − 1, − 4, 0 ) + c (2 , 1, 1, 6).
Then 2a + b + 2c = 1 …(1)
− a − b + c =1 …(2)
−5a − 4b + c = 2 …(3)
2a + 0 b + 6c = 4. …(4)
Now we shall solve the simultaneous equations (1), (2), (3) and (4).
Adding (1) and (2), we get a + 3c = 2 which is the same equation as (4).
If we take c = 0, we get a = 2.
Putting a = 2 and c = 0 in (1), we get b = − 3.
We see that a = 2 , b = − 3, c = 0 satisfy all the four equations (1), (2), (3) and (4).
∴ (1, 1, 2 , 4) = 2 (2 , − 1, − 5, 2) − 3 (1, − 1, − 4, 0 ) + 0 (2 , 1, 1, 6)
or 1 (1, 1, 2 , 4) − 2 (2 , − 1, − 5, 2) + 3 (1, − 1, − 4, 0 ) − 0 (2 , 1, 1, 6) = (0, 0, 0, 0 ). …(5)
Since in the linear relation (5) among the four given vectors the scalar coefficients
1, − 2 , 3, 0 are not all zero, therefore the given vectors are linearly dependent in R4 .
Problem 2: Show that the vectors (1, 1, 0, 0 ), (0, 1, − 1, 0 ), (0, 0, 0, 3) in R4 are linearly
independent.
Solution: Let a, b, c be scalars i. e., real numbers such that
a (1, 1, 0, 0 ) + b (0, 1, − 1, 0 ) + c (0, 0, 0, 3) = (0, 0, 0, 0 ) …(1)
Then a + 0 b + 0 c = 0, a + b + 0 c = 0, 0 a − b + 0 c = 0, 0 a + 0 b + 3c = 0.
The only solution of the above equations is a = 0, b = 0, c = 0.
Thus the linear relation (1) among the three given vectors is possible only if
a = 0, b = 0, c = 0.
Hence the three given vectors in R4 are linearly independent.
Problem 3: Determine whether the following set of vectors in V3 (Q ) is linearly dependent or
independent, Q being the field of rational numbers :
{( − 1, 2, 1), (3, 1, − 2)}.
V-27

Solution: Let a, b be scalars (i. e., a, b ∈Q) such that


a ( − 1, 2 , 1) + b (3, 1, − 2) = (0, 0, 0 )
i. e., ( − a + 3b, 2a + b, a − 2b ) = (0, 0, 0 ).
− a + 3b = 0, 

Then 2a + b = 0,  …(1)
a − 2b = 0. 
 −1 3
The coefficient matrix A of the system of equations (1) is A =  2 1⋅
 
 1 − 2 
− 1 3  = − 1 − 6 = − 7 ≠ 0.
We have  
2 1
Thus there exists a 2-rowed minor of the matrix A which is not zero. Also the matrix A
can have no minor of order greater than 2.
∴ rank A = 2 = the number of unknowns a and b.
Therefore the equations (1) have the only solution a = 0, b = 0. Hence the given set of
vectors is linearly independent.
Note. If we do not want to use the concept of the rank of a matrix to discuss the
solutions of the system of equations (1), we can directly say that solving the system of
equations (1) we find that the only solution of the system of equations (1) is a = 0, b = 0.
Problem 4: Find a linearly independent subset T of the set S = {α1, α2 , α3 , α4 }
where α1 = (1, 2, − 1), α2 = ( − 3, − 6, 3), α3 = (2, 1, 3), α4 = (8, 7, 7) ∈ R3
which spans the same space as S.
Solution: First we observe that α2 = − 3α1 so that the vectors α1 and α2 are linearly
dependent.
∴ if S1 = {α1, α3 , α4 }, then the subspace of R3 spanned by S1 is the same as that
spanned by S.
Now there exists no real number c such that α3 = cα1. Therefore the vectors α1 and α3 are
linearly independent.
Let us now see whether the vector α4 lies in the subspace of R3 spanned by the vectors α1
and α3 or not.
Let α4 = aα1 + bα3 , where a, b ∈ R . Then (8, 7, 7) = a (1, 2 , − 1) + b (2 , 1, 3).
∴ a + 2b = 8, 2a + b = 7, and − a + 3b = 7.
Solving the first two of these three equations, we get a = 2, b = 3. These values of a and b
also satisfy the third equation.
∴ α4 = 2α1 + 3α3 .
Thus the vector α4 has been expressed as a linear combination of α1 and α3 so that the
subspace of R3 spanned by the vectors α1, α3 and α4 is the same as that spanned by the
vectors α1 and α3 .
Hence T = {α1, α3 } is a linearly independent subset of S which spans the same subspace
of R3 as is spanned by S.
V-28

Problem 5: If the vectors (0, 1, x ),( x, 1, 0 ),(1, x, 1) of the vector space R3 are linearly dependent,
then find the value of x. (Kumaun 2012, 14)
Solution: Let a, b, c be scalars such that
a (0, 1, x ) + b ( x, 1, 0 ) + c (1, x, 1) = (0, 0, 0 )
i. e., ( a . 0 + b . x + c . 1, a . 1 + b . 1 + c . x, a . x + b . 0 + c . 1) = (0, 0, 0 )
i. e., a0 + bx + c1 = 0, a1 + b1 + cx = 0, ax + b0 + c1 = 0.
The coefficient matrix of these equations is
0 x 1
A= 1 1 x ⋅
 
 x 0 1 
If the given vectors are linearly dependent then Rank A must be less than 3 for which the
condition is | A | = 0.
Now | A | = − x (1 − x 2 ) + 1 ( − x ) = 0 ⇒ x ( x 2 − 2) = 0.
∴ x=0 or x 2 − 2 = 0 ⇒ x 2 = 2 ⇒ x = ± √ 2.
Hence, x = 0, ± √ 2 .
Problem 6: Show that the vectors (0, 2, − 4) , (1, − 2, − 1), (1, − 4, 3) in R3 are linearly
dependent. Also express (0, 2, − 4) as a linear combination of (1, − 2, − 1) and (1, − 4, 3).
Solution: Let α1 = (0, 2 , − 4), α2 = (1, − 2 , − 1), α3 = (1, − 4, 3).
Let a, b, c be scalars i. e., real numbers such that
aα1 + bα2 + cα3 = 0
i. e., a (0, 2 , − 4) + b (1, − 2, − 1) + c (1, − 4, 3) = (0, 0, 0 )
i. e., ( b + c, 2a − 2b − 4c, − 4a − b + 3c ) = (0, 0, 0 )
b+ c =0 …(1)
∴ 2a − 2b − 4c = 0 …(2)
− 4a − b + 3c = 0 …(3)
Eliminating a between (2) and (3), we get
− 5b − 5c = 0 or b + c = 0,
which is the same equation as (1).
If we choose c = 1, then b = − 1 and putting in (2) or (3), we get a = 1. Hence the given
vectors α1, α2 , α3 are L.D.
Also 1α1 − 1α2 + 1α3 = 0 i. e., α1 = α2 − α3
or (0, 2 , − 4) = (1, − 2, − 1) − (1, − 4, 3).
Problem 7: Determine if the vectors (1, − 2, 1), (2, 1, − 1), (7, − 4, 1) in R3 are linearly
independent or linearly dependent.
Solution: Let a, b, c be scalars i.e., real numbers such that
a(1, − 2, 1) + b (2, 1, − 1) + c (7, − 4, 1) = (0, 0, 0 )
i. e., ( a + 2b + 7c, − 2a + b − 4c, a − b + c ) = (0, 0, 0 )
a + 2b + 7c = 0 …(1)
−2a + b − 4c = 0 …(2)
V-29

a− b+ c =0 …(3)
Multiplying (1) by (2), we get
2a + 4b + 14c = 0 …(4)
Adding (2) and (4), we get
5b + 10 c = 0 or b + 2c = 0 …(5)
Again subtracting (3) from (1), we get
3b + 6c = 0 or b + 2c = 0 …(6)
The equations (5) and (6) are the same and give b = − 2c.
Putting b = − 2c in (1), we get a = −3c. If we take c = 1, we get b = − 2 and a = − 3 . Thus
a = − 3, b = − 2, c = 1 is non-zero solution of the equations (1), (2) and (3). Hence the
given set of vectors is linearly dependent.
Problem 8: Let α1, α2 , α3 be the vectors of V ( F ), a, b ∈ F. Show that the set {α1, α2 , α3 } is
linearly dependent if the set {α1 + aα2 + bα3 , α2 , α3 } is linearly dependent.
Solution: Since the set {α1 + aα2 + bα3 , α2 , α3 } is linearly dependent, therefore
there exist scalars, x, y, z not all zero such that
x (α1 + aα2 + bα3 ) + yα2 + zα3 = 0
i. e., xα1 + ( xa + y ) α2 + ( xb + z ) α3 = 0. …(1)
If in the relation (1), the coefficients x, xa + y, xb + z are not all zero, then the set
{α1, α2 , α3 } will also be linearly dependent.
If x ≠ 0, then the problem is at once solved whatever y and z may be. However if x = 0,
then at least one of y and z is not zero. Therefore at least one of xa + y and xb + z will
not be zero since when x = 0 then xa + y and xb + z reduce to y and z respectively.
Hence in the relation (1) the scalar coefficients of α1, α2 , α3 are not all zero. Therefore
the set {α1, α2 , α3 } is also linearly dependent.
Problem 9: If α, β, γ are linearly independent vectors of V ( F ) where F is the field of complex
numbers, then so also are α + β, α − β, α − 2β + γ.
Solution: Let a, b, c be scalars such that
a (α + β ) + b (α − β ) + c (α − 2β + γ ) = 0 …(1)
i. e., ( a + b + c ) α + ( a − b − 2c ) β + c γ = 0. …(2)
But α, β, γ are linearly independent. Therefore (2) implies
a + b + c = 0, a − b − 2c = 0, c = 0.
The only solution of these equations is c = 0, a = 0, b = 0.
Thus (1) implies a = 0, b = 0, c = 0. Therefore the vectors
α + β, α − β, α − 2β + γ
are linearly independent.
Problem 10: Show that the set {1, x, x (1 − x )} is a linearly independent set of vectors in the
space of all polynomials over the real number field.
Solution: The zero vector of the vector space of all polynomials over the real number
field is the zero polynomial.
V-30

Let a, b, c be scalars (i. e., real numbers) such that


a (1) + bx + c [ x (1 − x )] = 0 i. e., zero polynomial.
We have a (1) + bx + c ( x − x2 ) = 0 ⇒ a + ( b + c ) x − cx2 = 0. …(1)
Now two polynomials in x are said to be equal if the coefficients of like powers of x on
both sides are equal. So by the definition of the equality of two polynomials, we have
from (1),
a = 0, b + c = 0, − c = 0 ⇒ c = 0, b = 0, a = 0.
Thus a (1) + bx + c [ x (1 − x )] = 0 ⇒ a = 0, b = 0, c = 0.
∴ the vectors 1, x, x (1 − x ) are linearly independent over the field of real numbers.
Problem 11: Find whether the vectors
2 x3 + x2 + x + 1, x3 + 3 x2 + x − 2 and x3 + 2 x2 − x + 3 of R [ x ] ,
the vector space of all polynomials over the real number field, are linearly independent or not.
Solution: The zero vector of the vector space R [ x ] is the zero polynomial.
Let a, b, c be scalars (i. e., real numbers) such that
a (2 x3 + x2 + x + 1) + b ( x3 + 3 x2 + x − 2) + c ( x3 + 2 x2 − x + 3) = 0
i. e., zero polynomial.
Then (2a + b + c ) x3 + ( a + 3b + 2c ) x2 + ( a + b − c ) x + a − 2b + 3c = 0. …(1)
Equating the coefficients of like powers of x on both sides of (1), we get
2a + b + c = 0, 
a + 3b + 2c = 0, 
 …(2)
a + b − c = 0, 
and a − 2b + 3c = 0. 
The coefficient matrix A of the system of equations (2) is
2 1 1   1 3 2
1 3 2  2 1 1
A= ~  , by R1 ↔ R2
1 1 − 1   1 1 −1
 1 −2 3   1 −2 3

 1 3 2
by R2 → R2 − 2R1
0 −5 −3 
~ , R3 → R3 − R1
0 −2 −3 
0 R4 → R4 − R1
 −5 1 
 1 3 2
0 1 3 /5  1
~  , by R2 → − R2
0 −2 −3  5
0 −5 1 

 1 3 2
0 1 3 / 5  by R3 → R3 + 2R2 ,
~ ,
0 0 − 9 /5  R4 → R4 + 5R2
0 0 4 
 
V-31

 1 3 2
 0 1 3 /5  5
~  , by R3 → − R3
 0 0 1 9
 0 0 4 

 1 3 2
 0 1 3 /5 
~   , by R4 → R4 − 4R3
 0 0 1
 0 0 0 

which is in Echelon form.
∴ rank A = number of non-zero rows in its Echelon form
= 3 = number of unknowns a, b, c in the system of equations (2).
Hence the system of equations (2) has the only solution a = 0, b = 0, c = 0.
∴ the given set of vectors is linearly independent.
Problem 12: Prove that a set of vectors which contains the zero vector is linearly dependent.
Solution: Let S = {α1, α2 , … , α n} be a set of vectors of the vector space V ( F ).
Let α r be equal to zero vector where 1 ≤ r ≤ n.
To show that the set S of vectors is linearly dependent.
Obviously 0α1 + 0α2 + … + aα r + 0α r + 1 + … + 0α n = 0 i. e., zero vector …(1)
for any non-zero scalar a i. e., for any non-zero element a in the field F.
Since in the linear relation (1) among the vectors α1, α2 , … , α n , the scalar coeffi-
cient a is not zero, therefore the vectors α1, α2 , … , α n are linearly dependent.
Problem 13: Let V be the vector space of 2 × 2 matrices over R. Determine whether the matrices
A, B, C ∈ V are linearly dependent, where
1 2  3 −1   1 −5 
A=  , B =  2 2  , C =  −4 0  .
 3 1      (Garhwal 2013)
Solution: Let A = α , B = β , C = γ.
Let a, b, c be real numbers such that
aα + bβ + c γ = 0 …(1)
1 2 3 −1   1 −5  0 0 
i. e., a + b2 2 
+ c =
0   0 0 
 3 1    −4
 a + 3b + c 2a − b − 5c   0 0 
i. e.,  3a + 2b − 4c =
 a + 2b   0 0 

∴ a + 3b + c = 0 …(2)
2a − b − 5c = 0 …(3)
3a + 2b − 4c = 0 …(4)
a + 2b = 0 …(5)
V-32

From (2) and (3), we get


a b c a b c
= = or = =
− 15 + 1 2 + 5 − 1 − 6 − 14 7 − 7
a b c
or = = = k, say ⇒ a = 2 k, b = − k, c = k.
2 −1 1
Putting these values in (4), we get
6k − 2k − 4k = 0 or 6k − 6k = 0, which is true V k.
Take k = 1. Then a = 2 , b = − 1, c = 1.
These values of a, b, c also satisfy (5). From (1), we have 2α − β + γ = 0
i. e., α, β, γ are linearly dependent.
Problem 14: Prove that the three non-zero non-coplanar vectors are linearly independent.
Solution: Let α, β, γ be non-zero non-coplanar vectors and a, b, c be scalars.
Consider the equation
aα + bβ + c γ = 0 …(1)
If a ≠ 0, then (1) gives
α =  −  β +  −  γ.
b c
 a  a
This shows that α is a linear combination of β, γ and so α, β, γ are coplanar vectors,
contradicting the fact that α, β, γ are non-coplanar. Consequently a = 0. Similarly, we
can show that b = 0, c = 0.
Thus aα + bβ + c γ = 0 ⇒ a = 0, b = 0, c = 0.
Hence α, β, γ are linearly independent.
Problem 15: Show that any three non-zero coplanar vectors are linearly dependent in R3 .
Solution: Let α, β, γ be three non-zero coplanar vectors.
α , β, γ are coplanar
⇒ α can be expressed as a linear combination of β and γ i. e.,α = bβ + c γ, where b and c
are scalars not all zero.
Thus ( − 1) α + bβ + c γ = 0 i. e., α, β, γ are L.D.
Problem 16: Show that the set of functions {x, | x |} is L.I. in vector space of continuous
functions defined in ( − 1, 1).
Solution: Let a, b be scalars such that
ax + b | x | = 0 …(1)
1 1
Take two values of x as − and ⋅
2 2
a b a b
Then from (1), − + = 0 and + = 0.
2 2 2 2
These give a = 0, b = 0. Hence { x ,| x |} is L.I.
Problem 17: Show that the vectors α = (1 + i, 2i ), β = (1, 1 + i ) in C2 (C ) are L.D. but in
C2 (R) are L.I.
V-33

Solution: Let a , b be scalars such that


aα + bβ = 0 …(1)
i. e., a (1 + i, 2i ) + b (1, 1 + i ) = 0
i. e., ( a + ai + b, 2ai + b + bi ) = (0, 0 )
⇒ a + ai + b = 0 or a (1 + i ) = − b …(2)
and 2ai + b + bi = 0 or 2ai = − b (1 + i ) …(3)
2
From (2) and (3), a (1 + i ) = 2ai, which is true.
Hence (2) and (3) are consistent. Take a = 1, then b = − (1 + i ). Thus a, b ∈C and a, b ≠ 0.
From (1), we conclude that {α, β} is L.D. in C 2 (C).
From (2) and (3), we cannot find non-zero real values of a and b.
Hence {α , β} is not L.D. in C 2 (R) i. e., {α , β} is L.I. in C 2 (R).
Problem 18: If V ( F ) is a vector space and α1, α2 , … , α n ∈ V , λ2 , λ3 , … , λ n ∈ F such that
{α2 + λ2 α1, α3 + λ3 α1, … , α n + λ n α1} is L.D. then show that {α1, α2 , … , α n} is L.D.
Solution: Since {α2 + λ2α1, α3 + λ3α1, … , α n + λ nα1} is L.D. so there exist scalars
a2 , … , an ∈ F, not all zero, such that
a2 (α2 + λ2 α1 ) + a3 (α3 + λ3α1 ) + … + an (α n + λ n α1 ) = 0
n
or Σ ai α i + ( a2 λ2 + a3 λ3 + … + an λ n ) α1 = 0
i=2
n  n 
or Σ ai α i +  Σ ai λ i  α1 = 0.
i=2  i=2 
n
Taking b1 = Σ ai λ i and bi = ai for i = 2 , 3, … , n,
i=2
n n
we get Σ bi α i + b1 α1 = 0 or Σ bi α i = 0 …(1)
i=2 i=1

Here bi ’s are not all zero as ai ’s are not all zero. Hence (1) shows that {α1, α2 , … , α n} is
L.D.
Problem 19: Let V be the vector space of functions from R into R. Show that f , g, h ∈ V are
linearly independent where
(i) f ( x ) = e2 x , g ( x ) = x2 , h ( x ) = x (ii) f ( x ) = sin x, g ( x ) = cos x, h ( x ) = x.
Solution: (i) Let a, b, c be real numbers such that
af ( x ) + bg ( x ) + ch ( x ) = 0 i. e., ae 2 x + bx2 + cx = 0 …(1)
Putting x = 0, 1, 2 successively in (1), we get
a = 0, ae 2 + b + c = 0, ae4 + 4b + 2c = 0.
Solving these, we get a = 0, b = 0, c = 0.
Hence (1) shows that e 2 x, x2 , x are linearly independent.
V-34

(ii) Let a, b, c be real numbers such that


af ( x ) + bg ( x ) + ch ( x ) = 0 i. e., a sin x + b cos x + cx = 0 …(1)
π π
Putting x = 0, , π successively in (1), we get b = 0, a + c ⋅ = 0, − b + c π = 0.
2 2
Solving these, we get a = 0, b = 0, c = 0. Hence .
(1) shows that sin x, cos x, x are L.I
Problem 20: Test the vectors (0, 1, 0, 1, 1), (1, 0, 1, 0, 1), (0, 1, 0, 1, 1), (1, 1, 1, 1, 1) in V5 over the
field of rational numbers for linear independence.
Solution: Let a, b, c, d be rational numbers such that
a (0, 1, 0, 1, 1) + b (1, 0, 1, 0, 1) + c (0, 1, 0, 1, 1) + d (1, 1, 1, 1, 1) = 0
i. e., ( b + d, a + c + d, b + d, a + c + d, a + b + c + d ) = (0, 0, 0, 0, 0 )
⇒ b + d = 0, a + c + d = 0, b + d = 0, a + c + d = 0, a + b + c + d = 0
⇒ b + d = 0, a + c + d = 0, b = 0 ⇒ b = 0, d = 0, a = − c.
Taking a = 1, we have a = 1, b = 0, c = − 1, d = 0.
Thus the scalars a, b, c, d are not all zero. Hence the given vectors are L.D.
Problem 21: Prove that the four vectors (1, 0, 0 ), (0, 1, 0 ), (0, 0, 1), (1, 1, 1) in V3 (C) form a
linearly dependent set but any three of them are linearly independent.
Solution: Let α1 = (1, 0, 0 ), α2 = (0, 1, 0 ), α3 = (0, 0, 1), α4 = (1, 1, 1).

We have (1, 0, 0 ) + (0, 1, 0 ) + (0, 0, 1) − (1, 1, 1) = (0, 0, 0 )


i. e., α1 + α2 + α3 − α4 = 0
or a1α1 + a2α2 + a3α3 + a4 α4 = 0 ; a1 = a2 = a3 = 1, a4 = − 1.
Since scalars are not all zero so α1, α2 , α3 , α4 are L.D.
Now consider the vectors α1, α2 , α4 .
Let aα1 + bα2 + cα4 = 0
i. e., a (1, 0, 0 ) + b (0, 1, 0 ) + c (1, 1, 1) = 0
or ( a, 0, 0 ) + (0, b, 0 ) + ( c, c, c ) = 0 or ( a + c, b + c, c ) = (0, 0, 0 ).
∴ a + c = 0, b + c = 0, c = 0 ⇒ a = 0, b = 0, c = 0.
Hence the vectors α1, α2 , α4 are L.I.
Similarly we can show that α1, α3 , α4 ; α2 , α3 , α4 ; α1, α2 , α3 are L.I.

Hints to Objective Type Questions

Multiple Choice Questions


1. (b). See Ex. 5 of Text Book.
2. (a). See Ex. 8 (ii) of Text Book.
3. (c). See Ex. 9 of Text Book.
4. (a). See Ex. 4 of Text Book.
5. (b). See problem 15.
V-35

Fill in the Blank(s)


1. See Ex. 3 of Text Book.
2. Since any subset of a linearly independent set of vectors is also linearly
independent.
3. See Ex. 2 of Text Book.
4. See Ex. 5 of Text Book.
5. Since the vectors (1, 0, 1) and ( − 1, 0, − 1) are linearly dependent in R3 and every
superset of a linearly dependent set of vectors is linearly dependent.
True or False
1. F. The set {(1, 0, 0 ), (0, 1, 0 ),(0, 0, 1)} is L.I. but the set {(1, 0, 0 ), (0, 1, 0 ),(0, 0, 1),
(1, 1, 1)} is L.D. in V3 ( R ).
2. F. The sets {(1, 0, 0 ), (0, 1, 0 ), (0, 0, 1)} and {(1, 0, 0 ), (0, 1, 0 ), (1, 1, 1)} are L.I.
subsets of V3 ( R ) but their union i. e., the set {(1, 0, 0 ), (0, 1, 0 ),(0, 0, 1),(1, 1, 1)}
is L.D. in V3 ( R ).
3. F. Since (0, 0, 0 ) is one of the vectors, the vectors are linearly dependent.
4. T . Since any four or more vectors in R3 are linearly dependent.

❍❍❍
V-36

Chapter-3
Basis and Dimensions

Comprehensive Problems 1
Problem 1: Give a basis for each of the following vector spaces over the indicated fields.
(i) R ( √2) over R (ii) C over R (iii) Q (21 /4 ) over Q
where Q, R, C are fields of rational, real and complex numbers respectively.
Solution: (i) R ( √ 2) = {a + b √ 2 : a, b ∈ R}.
Basis for the vector space R ( √2) over R is {1, √ 2}.
(ii) C = {a + ib : a, b ∈ R}.
Basis for the vector space C over R is {1, i}.
(iii) Q (21 /4 ) = {a + b 21 /4 : a, b ∈ Q }.
Basis for the vector space Q (21 /4 ) over Q is {1, 21 /4 }.
Problem 2: Find the dimension of Q ( √ 2, √ 3) over Q.
Solution: Q ( √ 2, √ 3) = {a + b √ 2 + c √ 3 + d √ 2 √ 3 : a, b, c, d ∈ Q }.
Basis for the vector space Q ( √ 2, √ 3) over Q is
{1, √ 2, √ 3, √ 2 √ 3}.
Hence the required dimension is 4.
Problem 3: Find bases for the subspaces
W1 = {( a, b, 0 ): a, b, ∈ R}, W2 = {(0, b, c ): b, c ∈ R} of R3 .
Find the dimensions of W1, W2 and W1 ∩ W2 .
Solution: W1 = {( a, b, 0 ) : a, b ∈ R}.
We have ( a, b, 0 ) = a (1, 0, 0 ) + b (0, 1, 0 ).
Any vector ( a, b, 0 ) ∈ W can be expressed as a linear combination of the vectors
(1, 0, 0 ), (0, 1, 0 ).
Also the set {(1, 0, 0 ), (0, 1, 0 )} is L.I.
Hence {(1, 0, 0 ), (0, 1, 0 )} is a basis for W1.
∴ dim W1 = 2.
Similarly the set {(0, 1, 0 ), (0, 0, 1)} is a basis for the subspace
W2 = {(0, b, c ) : b, c ∈ R}.
∴ dim W2 = 2.
Since R3 = W1 + W2 , therefore dim (W1 + W2 ) = 3.
Now dim (W1 ∩ W2 ) = dim W1 + dim W2 − dim (W1 + W2 )
= 2 + 2 − 3 = 1.
V-37

Problem 4: Show that the real field R is a vector space of infinite dimension over the rational
field Q.
Solution: We claim that {1, π, π2 , … , π n} is linearly independent over Q, for any n.
For let a0 1 + a1 π + a2 π2 + … + an π n = 0, where ai ∈Q and not all the ai ’s are zero.
Then π is a root of the non-zero polynomial over Q namely
a0 + a1 x + a2 x2 + … + an x n.
But π is a transcendental number i. e., it cannot be a root of any non-zero polynomial
over Q. Accordingly, the ( n + 1) real numbers 1, π, π2 , … , π n are linearly independent
over Q. Hence for any finite n, R cannot be of dimension n over Q i. e., R is of infinite
dimension over Q.
Problem 5: Let V be the vector space of all 2 × 2 symmetric matrices over F. Show that
dim V = 3.
Solution: Any 2 × 2 symmetric matrix will be of the form
a b
A=  , where a, b, c ∈ F.
b c 
1 0  0 1 0 0 
Let α =  , β =  1 0 , γ =  0 be 3 elements of V.
 0 0     1 
We can show that S = {α, β, γ} is linearly independent subset of V and L ( S ) = V .
Therefore S is a basis of V. Hence dim V = 3.
Problem 6: Prove that the space of all m × n matrices over the field F has dimension mn, by
exhibiting a basis for this space.
Solution: For i = 1, … , m ; j = 1, … , n, let Eij ∈ V be the matrix with 1 as the ij-th entry
and 0 elsewhere. We shall show that {Eij } is a basis of V.
Let Σ Σ xij Eij = 0 where xij are scalars.
i j
The ij-th entry of L.H.S. is xij and the ij-th entry of R.H.S. is 0. Thus
xij = 0 V i = 1, … , m ; j = 1, … , n.
Hence the set {Eij } is linearly independent.
Now let A = [ aij ]m × n be any matrix in V.
Then A = Σ Σ aij Eij . Thus {Eij } generates V.
i j
Hence {Eij } is a basis of V.
Number of elements in the set {Eij } is m n, so dim V = mn.
Problem 7: If {α1, α2 , α3 } is a basis of V3 (R), show that {α1 + α2 , α2 + α3 , α3 + α1} is also
a basis of V3 (R).
Solution: The vector space V3 ( R ) is of dimension 3. Any subset of it having three
linearly independent vectors will form a basis. We shall show that the vectors
α1 + α2 , α2 + α3 , α3 + α1 are linearly independent in V3 ( R ).
Let a, b, c be scalars such that
a(α1 + α2 ) + b (α2 + α3 ) + c (α3 + α1 ) = 0
i. e., ( a + c ) α1 + ( a + b ) α2 + ( b + c ) α3 = 0 …(1)
But α1, α2 , α3 are linearly independent .
V-38

Therefore (1) implies


a + 0 b + c = 0, a + b + 0 c = 0, 0 a + b + c + = 0
The coefficient matrix A of these equations is
1 0 1 
A = 1 1 0 .
 
0 1 1 
We have rank A = 3 i. e., the number of unknowns a, b, c. Therefore a = 0, b = 0,
c = 0 is the only solution of the given equations.
Hence α1 + α2 , α2 + α3 , α3 + α1 are also linearly independent.
So {α1 + α2 , α2 + α3 , α3 + α1} is also a basis of V3 ( R ).
Problem 8: If W1 and W2 are finite-dimensional subspaces with the same dimension, and if
W1 ⊆ W2, then W1 = W2 .
Solution: Since W1 ⊆ W2 , therefore W1 is also a subspace of W2 .
Now dim W1 = dim W2 . Therefore we must have W1 = W2 .
Problem 9: In the vector space R3 , let α = (1, 2, 1), β = (3, 1, 5), γ = (3, − 4, 7). Show that there
exists more than one basis for the subspace spanned by the set S = {α, β, γ}.
Solution: First show that the vector γ can be expressed as a linear combination of the
vectors α and β. Therefore if T = {α , β}, then L (T ) = L ( S ). Now the set {α, β} is
linearly independent as can be easily shown. Therefore the set {α , β} is a basis for
L (T ) = L ( S ). Therefore dim L ( S ) = 2. Now {α , γ} is a linearly independent subset of
L ( S ) containing two vectors. Therefore {α , γ} is also a basis for L ( S ). Similarly { β, γ}
is also a basis for L ( S ).
Problem 10: For the 3-dimensional space R3 over the field of real numbers R, determine if the
set {(2, − 1, 0 ), (3, 5, 1), (1, 1, 2)} is a basis.
Solution: We have dim R3 = 3. If the given set containing three vectors is linearly
independent, it will form a basis of R3 otherwise not.
Let a, b, c ∈ R be such that
a (2 , − 1, 0 ) + b (3, 5, 1) + c (1, 1, 2) = (0, 0, 0 )
⇒ (2a + 3b + c, − a + 5b + c, 0 a + b + 2c ) = (0, 0, 0 ).
∴ 2a + 3b + c = 0, …(1)
− a + 5b + c = 0, …(2)
and b + 2c = 0. …(3)
Now we shall solve these equations to get the values of a, b, c.
Multiplying (2) by 2 and adding to (1), we get
13b + 3c = 0. …(4)
Multiplying (3) by 13 and then subtracting (4) from it, we get
23c = 0 or c = 0.
Putting c = 0 in (3), we get b = 0.
Putting b = 0, c = 0 in (1), we get a = 0.
Thus the only solution of the equations (1), (2) and (3) is a = 0, b = 0, c = 0. Therefore
the three given vectors are linearly independent and so they form a basis of R3 .
V-39

Problem 11: Show that the set {(1, i, 0 ), (2i, 1, 1), (0, 1 + i, 1 − i )} is a basis for V3 (C ).
(Kumaun 2011)
Solution: We know that dim V3 (C )or C3 = 3. If the given set containing three vectors is
linearly independent, it will form a basis of V3 (C ) otherwise not.
Let a, b, c ∈C be such that
a (1, i, 0 ) + b (2 i, 1, 1) + c (0, 1 + i, 1 − i) = (0, 0, 0 )
⇒ ( a + 2 ib + 0 c, ai + b + c [1 + i ], 0 a + b + c [1 − i ]) = (0, 0, 0 ).
∴ a + 2 ib = 0, …(1)
ai + b + c (1 + i ) = 0, …(2)
and b + c (1 − i ) = 0. …(3)
Now we shall solve these equations to get the values of a, b, c .
Multiplying (1) by −i and adding to (2), we get
3b + c (1 + i ) = 0. …(4)
Multiplying (3) by 3 and subtracting from (4), we get
c (1 + i ) − 3c (1 − i ) = 0
or c (1 + i − 3 + 3i ) = 0 or c ( −2 + 4i ) = 0 or c = 0.
Putting c = 0 in (3), we get b = 0.
Putting b = 0 in (1), we get a = 0.
Thus the only solution of the equations (1), (2) and (3) is a = 0, b = 0, c = 0. Therefore
the three given vectors are linearly independent and so they form a basis of V3 (C ).
Problem 12: Find three vectors in R3 which are linearly dependent, and are such that any two
of them are linearly independent.
Solution: Consider the vectors (1, 0, 0 ), (0, 1, 0 ), (1, 1, 0 ). We can verify that these 3
vectors are L.D. and any two of them are L.I.
Problem 13: Select a basis, if any, of R3 (R) from the set {α1, α2 , α3 , α4 }, where
α1 = (1, − 3, 2), α2 = (2, 4, 1), α3 = (3, 1, 3), α4 = (1, 1, 1).
Solution: Let S = {α1, α2 , α3 , α4 }.
If any three vectors in S are linearly independent, then they will form a basis of the
vector space R3 (R).
First consider the set S1 = {α1, α2 , α3 }. Let us see whether the vectors in the set S1 are
linearly independent or not.
The determinant of order 3 whose columns consist of the coordinates of the vectors
α1, α2 , α3 is
 1 2 3

= −3 4 1
 
 2 1 3
 1 0 0
by C2 − 2C1
= − 3 10 10 ,
  and C3 − 3C1
 2 −3 − 3
= − 30 + 30 = 0.
∴ the vectors α1, α2 , α3 are linearly dependent and so they do not form a basis of
R3 (R).
V-40

Now consider the set S2 = {α1, α2 , α4 }.


The determinant of order 3 whose columns consist of the coordinates of the vectors
α1, α2 , α4 is
 1 2 1  1 0 0
by C2 − 2C1
= − 3 4 1 =  − 3 10 4 ,
    and C3 − C1
 2 1 1  2 −3 − 1
= − 10 + 12 = 2 i. e., ≠ 0.
∴ the vectors α1, α2 , α4 are linearly independent.
Since S2 = {α1, α2 , α4 } is a linearly independent subset of R3 containing three vec-
tors, it is a basis of R3 .

Problem 14: If W is a subspace of a finite dimensional vector space V , prove that any basis of
W can be extended to form a basis of V .
Solution: Let V ( F ) be a finite dimensional vector space of dimension n. Let
{ β1, β2 , … , β n} be a basis of V.
Let W be a subspace of V. Then W itself is finite dimensional and dim W ≤ n. Let
dim W = m and let S = {α1, α2 , … , α m } be a basis of W.
Then S is a linearly independent subset of V. To show that S can be extended to
form a basis of V. For proof proceed as in theorem 1 of article 3 of Text Book.
Problem 15: Prove that any finite set S of vectors, not all the zero vectors, contains a linearly
independent subset T which spans the same space as S.
Solution: Let S be a finite set of vectors belonging to a vector space V ( F ). Assume
that no member of S is zero vector for if any member of S is zero vector, we can
omit it from S without affecting the subspace spanned by S.
Let S = {α1, α2 , … , α m }.
If S is linearly independent, then S itself is the required linearly independent subset T of
S which spans the same subspace of V as S.
If S is linearly dependent, then there exists, a vector α i ∈ S which can be expressed as a
linear combination of the preceding vectors α1, α2 , … , α i − 1 .
If we omit this vector α i from S, then the remaining subset S ′ of S containing m − 1
vectors spans the same subspace of V as S.
If S ′ is linearly independent, then S ′ will be the required linearly independent subset of
S which spans the same subspace of V as S. If S ′ is linearly dependent, then proceeding
as above we shall get a new subset of S containing m − 2 vectors which spans the same
space as S. Continuing this process we shall, after a finite number of steps, obtain a
linearly independent subset of S which spans the same space as S.
At the most it may happen that we shall be left with a subset of S which contains only
one non-zero vector and which spans the same space as S. We know that a set containing
a single non-zero vector is definitely linearly independent.
Hence any finite set S of vectors, not all the zero vectors, definitely contains a linearly
independent subset T which spans the same space as S.

Problem 16: Let V be a vector space. Let W be a subspace of V generated by the vectors
α1, …, α s . Prove that W is spanned by a linearly independent subset of α1, …, α s .
V-41

Solution: W is a subspace of V generated by a finite set


S = {α1, … , α s }.
To show that there exists a linearly independent subset T of S which also spans W.
Now for proof proceed as in problem 15 above.
Problem 17: If a vector space V is spanned by a finite set of m vectors, then show that any
linearly independent set of vectors in V has at most m elements.
Solution: For proof proceed as in theorem 3 of article 6 of Text Book.
Problem 18: In a vector space V over the field F, let B = {α1, α2 , …, α n} span V . Prove that the
following two statements are equivalent :
(i) B is linearly independent
n
(ii) If α ∈V, then the expression α = ∑ ai α i with ai ∈ F is unique.
i=1

Solution: (i) ⇒ (ii). For proof proceed as in theorem 8 of article 6 of Text Book.
(ii) ⇒ (i). We have that any α ∈V can be uniquely expressed as a linear combination of
elements of B. To prove that B is L.I.
For a1, a2 , … , an ∈ F, let
a1α1 + a2 α2 + … + an α n = 0. …(1)
Now 0 ∈V and 0 = 0α1 + 0α2 + … + 0α n …(2)
Since representation of 0 as a linear combination of α1, α2 , … , α n is unique so from (1)
and (2), we have a1 = 0, a2 = 0, … , an = 0.
It follows that α1, α2 , … , α n are linearly independent i. e., B is L.I.
Problem 19: Determine a basis of the subspace spanned by the vectors
α1 = (1, 2, 3), α2 = (2, 1, − 1), α3 = (1, − 1, − 4), α4 = (4, 2, − 2).
Solution: Let S = {α1, α2 , α3 , α4 }.
We shall find a linearly independent subset of the set S which spans the same space as S.
First we observe that α4 = 2α2 so that the vectors α2 and α4 are linearly dependent.
∴ if S1 = {α1, α2 , α3 } then the subspace of R3 spanned b S1 is the same as that
spanned by S.
Now there exists no real number c such that α2 = cα1. Therefore the vectors α1 and α2
are linearly independent.
Let us now see whether the vectors α3 lies in the subspace of R3 spanned by the vectors
α1 and α2 or not.
Let α3 = aα1 + bα2 where a, b ∈ R.
Then (1, − 1, − 4) = a (1, 2, 3) + b (2, 1, − 1).
∴ a + 2b = 1, 2a + b = − 1, 3a − b = − 4.
Solving the first two of these three equations, we get a = − 1, b = 1. These values of a and b
also satisfy the third equation
∴ α3 = − 1α1 + 1α2 .
V-42

Thus the vector α3 has been expressed as a linear combination of α1 and α2 so that
the subspace of R3 spanned by the vectors α1, α2 , α3 is the same as that spanned by
the vectors α1, α2 .
Hence {α1, α2 } is a basis of the subspace spanned by the vectors α1, α2 , α3 , α4 .

Problem 20: Let W be the subspace of R4 generated by the vectors (1, − 2, 5, − 3), (2, 3, 1, − 4)
and (3, 8, − 3, − 5). Find a basis and the dimension of W. Extend the basis of W to a basis of the
whole space R4 .
Solution: Form the matrix A whose rows are the generators of W and reduce it to
row-reduced Echelon matrix.
1 −2 5 −3   1 −2 5 −3 
A=2 3 1 −4 ~  0 7 −9 2
   
 3 8 −3 − 5   0 14 − 18 4 

 1 −2 5 −3 
~ 0 7 −9 2 ⋅
 
 0 0 0 0 
The non-zero rows (1, − 2, 5, − 3) and (0, 7, − 9, 2) of the last matrix form a basis of
the row space of A i. e., of W. Hence dim W = 2.
Any basis of R4 has 4 linearly independent vectors since dim R4 = 4. The vectors
(1, − 2, 5, − 3), (0, 7, − 9, 2), (0, 0, 1, 0 ) and (0, 0, 0, 1) are L.I. as they form an Echelon
matrix. Hence {(1, − 2, 5, − 3), (0, 7, − 9, 2), (0, 0, 1, 0 ), (0, 0, 0, 1)} is a basis of R4 which
is the extension of the basis of W.

Problem 21: Let E be a subfield of a field F and F a subfield of a field K i. e., E ⊂ F ⊂ K . Let K
be of dimension n over F and F is of dimension mover E. Show that K is of dimension mn over E.
Solution: Let {v1, … , vn} be a basis of K over F and {a1, … , am } be a basis of F over E.
Show that the set {ai v j : i = 1, … , m ; j = 1, … , n} is a basis of K over E. This set contains
mn elements. So dimension of K over E is mn.
Problem 22: Let W1 and W2 be the subspaces of R4 :

W1 = {( a, b, c, d) : b + c + d = 0},W2 = {( a, b, c, d): a + b = 0, c = 2d}.


Find the dimension of W1 ∩ W2 .
Solution: W1 ∩ W2 consists of those vectors ( a, b, c, d ) ∈ R4 which satisfy the condi-
tions defining both W1 and W2 i. e., we have three equations
b + c + d = 0, a + b = 0, c = 2d.
Here d is free variable. Taking d = 1, we have c = 2 , b = − 3, a = 3. Thus (3, − 3, 2, 1) is a
solution of the system of equations.
Hence {(3, − 3, 2, 1)} is a basis of W1 ≡ W2 and so dim (W1 ∩ W2 ) = 1.
V-43

Problem 23: Let W1 and W2 be subspaces of V and that dim W1 = 4, dim W2 = 5 and dim
V = 7. Find the possible dimensions of W1 ∩ W2 .
Solution: Proceed as in Ex. 13 of Text Book. Ans. dim (W1 ∩ W2 ) = 2, 3 or 4.
Problem 24: Let W1 and W2 be the subspaces of R4 generated by the sets
B1 = {(1, 1, 0, − 1), (1, 2, 3, 0 ), (2, 3, 3, − 1)}
and B2 = {(1, 2, 2, − 2),(2, 3, 2, − 3),(1, 3, 4, − 3)} respectively.
Find (i) dim (W1 + W2 ) (ii) dim (W1 ∩ W2 ).
Solution: (i) We have W1 + W2 = L (W1 ∪ W2 ) = L ( B1 ∪ B2 )i. e.,W1 + W2 is the space
spanned by all the six vectors. Now form a matrix A whose rows are the generators of
W1 + W2 and reduce it to row-reduced Echelon matrix.
1 1 0 −1   1 1 0 −1 
1 2 3 0  0 1 3 1
   
2 3 3 −1  0 1 3 1
A= ~
1 2 2 −2  0 1 2 −1 
   
2 3 2 −3  0 1 2 −1 
1 3 4 − 3  0 2 4 − 2 
 
 1 1 0 −1   1 1 0 −1 
 0 1 3 1  0 1 3 1
   
 0 1 2 −1   0 0 −1 −2 
~  ~ ⋅
0 0 0 0   0 0 0 0 
   
 0 0 0 0   0 0 0 0 
 0 0 0 0   0 0 0 0 
 
The last matrix has 3 non-zero rows. They form a basis of row space of A i. e., of W1 + W2 .
Hence dim (W1 + W2 ) = 3.
(ii) First find the dimension of W1.
Form a matrix B whose rows are the generators of W1 and reduce it to row-reduced
echelon matrix.
1 1 0 −1   1 1 0 −1 
B= 1 2 3 0 ~ 0 1 3 1
   
 2 3 3 − 1   0 1 3 1 
 1 1 0 −1 
~ 0 1 3 1⋅
 
 0 0 0 0 
The last matrix has 2 non-zero rows.
∴ dim W1 = 2.
Similarly we can show that dim W2 = 2.
We have dim (W1 + W2 ) = dim W1 + dim W2 − dim (W1 ∩ W2 )
∴ 3 = 2 + 2 − dim (W1 ∩ W2 ) or dim (W1 ∩ W2 ) = 1.
V-44

Hints to Objective Type Questions

Multiple Choice Questions


1. (b). The set {(1, 0, 0 ), (0, 1, 0 ), (0, 0, 1)} is a basis of V3 ( R ). dim V3 ( R ) = number
of elements in a basis = 3.
2. (c). See Ex. 3 of Text Book.
3. (b). The set S = {(1, 0 ), ( i, 0 ), (0, 1), (0, i )} is a basis for C2 (R).
4. (b). The set {1, i} is a basis for C (R).
5. (a). Use the result, dim (W1 + W2 ) = dim W1 + dim W2 − dim (W1 ∩ W2 ).
6. (b). The vectors (1, − 2, 3, − 1)and (1, 1, − 2, 3)are L.I. So they form a basis. Hence
the dimension of the space spanned by these vectors is 2.
7. (a). The vectors (3, − 6, 3, − 9) and ( − 2, 4, − 2, 6) are linearly dependent so the
dimension of the space spanned by these vectors is 1.
8. (b). The coordinate vector of α ∈ R3 relative to the usual basis {(1, 0, 0 ), (0, 1, 0 ),
(0, 0, 1)} is identical to α itself.
Fill in the Blank(s)
1. See Theorem 2 of article 6 of Text Book.
2. See Theorem 9 of article 6 of Text Book.
3. See Ex. 10 of Text Book.
4. See Problem 6.
5. See article 6, Theorem 6 of Text Book..
6. See article 5 of Text Book..
True or False
1. F. The given statement is false. See Theorem 4 of article 6 of Text Book.
2. F. The given statement is false. Let
A = {(0, 0, c ) : c ∈ F }, B = {( a, b, 0 ) : a, b ∈ F }.
Then both A and B are subspaces of V3 ( F ). The set {(0, 0, 1)} is a basis for A
and the set {(1, 0, 0 ),(0, 1, 0 )} is a basis for B. Thus dim A = 1and dim B = 2. We
have dim A < dim B. But A ⊄ B as (0, 0, 1) ∈ A and (0, 0, 1) ∉B.
3. F. The given statement is false. Take A = {( a, 0, 0 ) : a ∈ F },
B = {(0, b, 0 ) : b ∈ F }. Then both A and B are subspaces of V3 ( F ).
Here A ≠ B but dim A = dim B = 1.
4. T . The given statement is true. Here M is a subspace of N and dim M = dim N.
Hence M = N.
5. T . The given statement is true. See Theorem 4 of article 6 of Text Book.

❍❍❍
V-45

Chapter-4
Homomorphism of Vector Spaces

Comprehensive Problems 1
Problem 1: Define linear transformation of a vector space V ( F ) into a vector space W ( F ).
Show that the mapping
T : ( a, b ) → ( a + 2, b + 3)
of V2 ( R ) into itself is not a linear transformation.
Solution: Linear Transformation. Definition. Let V ( F ) and W ( F ) be two
vector spaces over the same field F. A mapping
T :V → W
is called a linear transformation of V into W if
T ( aα + bβ ) = aT (α ) + bT ( β ) V a, b ∈ F and V α, β ∈ V .
Now to show that the mapping
T : ( a, b ) → ( a + 2 , b + 3)
of V2 ( R ) into itself is not a linear transformation.
Take α = (1, 2)and β = (1, 3)as two vectors of V2 ( R )and a = 1, b = 1as two elements of the
field R.
Then aα + bβ = 1 (1, 2) + 1 (1, 3) = (1, 2) + (1, 3) = (2 , 5).
By the definition of the mapping T , we have
T ( aα + bβ ) = T (2, 5) = (2 + 2, 5 + 3) = (4, 8). …(1)
Also T (α ) = T (1, 2) = (1 + 2, 2 + 3) = (3, 5)
and T ( β ) = T (1, 3) = (1 + 2, 3 + 3) = (3, 6).
∴ aT (α ) + bT ( β ) = 1 (3, 5) + 1 (3, 6)
= (3, 5) + (3, 6) = (6, 11). …(2)
From (1) and (2), we see that
T ( aα + bβ ) ≠ aT (α ) + bT ( β ).
Hence T is not a linear transformation of V2 ( R ) into itself.
Problem 2: Let f be a linear transformation from a vector space U into a vector space V . If S is a
subspace of U, prove that f ( S ) will be a subspace of V .
Solution: U ( F )and V ( F )are two vector spaces over the same field F. The mapping f is
a linear transformation of U into V i. e., f : U → V such that
f ( aα + bβ ) = af (α ) + bf ( β ) V a, b ∈ F and V α, β ∈ U .
Let S be a subspace of U. Then to prove that f ( S ) is a subspace of V.
V-46

Let a, b ∈ F and f (α ), f ( β ) ∈ f ( S ) where α, β ∈ S.


Since S is a subspace of U, therefore
a, b ∈ F and α, β ∈ S ⇒ aα + bβ ∈ S
⇒ f ( aα + bβ ) ∈ f ( S )
⇒ af (α ) + bf ( β ) ∈ f ( S ). [ ∵ f ( aα + bβ ) = af (α ) + bf ( β )].
Thus a, b ∈ F and f (α ), f ( β ) ∈ f ( S )
⇒ af (α ) + bf ( β ) ∈ f ( S ).
Hence f ( S ) is a subspace of V.
Problem 3: If V is finite dimensional and f is a homomorphism of V into itself which is not onto
prove that there is some α ≠ 0 in V such that f (α ) = 0.
Solution: If f is a homomorphism of V into itself, then f (0 ) = 0. Suppose there is no
non-zero vector α in V such that f (α ) = 0. Then f is one-one. Because
f ( β ) = f (γ )
⇒ f ( β ) − f (γ ) = 0
⇒ f (β − γ) = 0 [ ∵ f is a linear transformation]
⇒ β − γ = 0 ⇒ β = γ.
Now V is finite dimensional and f is a linear transformation of V into itself. Since f is
one-one, therefore f must be onto V. But it is given that f is not onto. Therefore our
assumption is wrong. Hence there will be a non-zero vector α in V such that f (α ) = 0.

Comprehensive Problems 2
Problem 1: Let V be the vector space of square matrices of order n over the field R. Let W1 and W2
be the subspaces of symmetric and antisymmetric matrices respectively. Show that
V = W1 ⊕ W2 .
Solution: First we shall show that V = W1 + W2 .
Let A be any n × n matrix. Then
1 1
A = ( A + A ′ ) + ( A − A ′ ) where A′ denotes the transpose of A
2 2
1 1
or A = P + Q where P = ( A + A ′ ), Q = ( A − A ′ ).
2 2

We have P′=  1  1 1
( A + A ′ ) = ( A + A ′ )′ = [ A ′ + ( A ′ )′ ]
 2  2 2
1 1
= ( A′ + A) = ( A + A′ ) = P
2 2

and Q′=  1  1 1
( A − A ′ ) = ( A − A ′ )′ = [ A ′ − ( A ′ )′ ]
 2  2 2
1 1
= ( A ′ − A ) = − ( A − A ′ ) = − Q.
2 2
Thus P ∈ W1 and Q ∈ W2 . Next we show that W1 ∩ W2 = {0}.
Let E ∈ W1 ∩ W2 . Then E ∈ W1, E ∈ W2 .
E ∈ W1 ⇒ E ′ = E and E ∈ W2 ⇒ E ′ = − E.
It follows that E = − E or 2E = O or E = O.
Thus W1 ∩ W2 = {0}. Hence V = W1 ⊕ W2 .
V-47

Problem 2: Let V be the vector space of all functions from the real field R into R. Let U be the
subspace of even functions and W the subspace of odd functions. Show that V = U ⊕ W .
Solution: Show that V = U + W and U ∩ W = {0}.
See Ex. 23 of chapter 1 ‘Vector Spaces' of Text Book.

Problem 3: Let W1, W2 and W3 be the following subspaces of R3 :

W1 = {( a, b, c ) : a + b + c = 0}, W2 = {( a, b, c ) : a = c}, W3 = {(0, 0, c ) : c ∈ R}.


Show that (i) R3 = W1 + W2 (ii) R3 = W1 + W3 (iii) R3 = W2 + W3 . When is the sum
direct ?
Solution: (i) Any ( a, b, c ) ∈ R3 can be written as a sum of a vector of W1 and a vector of
W2 as shown below:

We have ( a, b, c ) =  a − , b − a − c, c −  +  , a + c, 
b b b b
 2 2  2 2

where  a − , b − a − c, c −  ∈ W1 and  , a + c,  ∈ W2 . Thus R3 = W1 + W2 .


b b b b
 2 2 2 2

Also we have ( a, b, c ) =  a − , − a − c, c −  +  , + a + c,  where


b b b b b b
 4 2 4 4 2 4
 a − b , b − a − c, c − b  ∈ W and  b , b + a + c, b  ∈ W .
   
 4 2 4
1 4 2 4
2

It follows that representation of ( a, b, c ) as a sum of a vector of W1 and a vector of


W2 is not unique. Hence R3 ≠ W1 ⊕ W2 i. e., the sum is not direct.
(ii) We claim that R3 = W1 + W3 .

For if α = ( a, b, c ) ∈ R3 then α = ( a, b, − a − b ) + (0, 0, a + b + c ) where ( a, b, − a − b ) ∈ W1


and (0, 0, a + b + c ) ∈ W3 . Also we have W1 ∩ W3 = {0}.
For α = ( a, b, c ) ∈ W1 ∩ W3 ⇒ a + b + c = 0 and a = 0, b = 0.
⇒ a = 0, b = 0, c = 0 i. e., α = 0.
Thus R3 = W1 + W3 and W1 ∩ W3 = {0}.

Hence R3 = W1 ⊕ W3 i. e., the sum is direct.

(iii) If α = ( a, b, c ) ∈ R3 then α = ( a, b, a) + (0, 0, c − a) where ( a, b, a) ∈ W2 and


(0, 0, c − a) ∈ W3 .
Thus R3 = W2 + W3 . Also We have W2 ∩ W3 = {0}.

For α = ( a, b, c ) ∈ W2 ∩ W3 ⇒ a = c and a = 0, b = 0
⇒ a = 0, b = 0, c = 0 i. e., α = 0.
Thus R = W2 + W3 and W2 ∩ W3 = {0}. Hence R3 = W2 ⊕ W3 i. e., the sum is direct.
3
V-48

Problem 4: Let W be a subspace of a vector space V over a field F. Show that α ∈ β + W iff
α − β ∈W.
Solution: Let α ∈ β + W. To show that α − β ∈W.
α ∈ β + W ⇒ ∃ γ ∈ W such that α = β + γ ⇒ α − β = γ.
Since γ ∈W so α − β ∈W.
Conversely, let α − β ∈W. To show that α ∈ β + W.
α − β ∈ W ⇒ ∃ γ ∈ W such that α − β = γ ⇒ α = β + γ.
But β + γ ∈ β + W. Hence α ∈ β + W.
Problem 5: Let W1 and W2 be two subspaces of a finite dimensional vector space V . If dim
V = dim W1 + dim W2 and W1 ∩ W2 = {0}, prove that V = W1 ⊕ W2 .
Solution: Let dim W1 = m and dim W2 = l. Also let the sets of vectors

S1 = {α1, α2 , … , α m } and S2 = {β1, β2 , … , β l }


be the bases of W1 and W2 respectively.
We have dim W1 + dim W2 = m + l. ∴ dim V = m + l.
We claim that the set S = S1 ∪ S2 = {α1, α2 , … , α m , β1, β2 , … , β l } is a basis of V .
Let a1α1 + a2 α2 + … + am α m + b1β1 + b2 β2 + … + bl β l = 0
⇒ a1α1 + a2 α2 + … + am α m = − ( b1 β1 + b2 β2 + … + bl β l ).
Now a1α1 + a2 α2 + … + am α m ∈ W1
and − ( b1 β1 + b2 β2 + … + bl β l ) ∈ W2 .
∴ a1α1 + a2 α2 + … + am α m ∈ W1 ∩ W2
and − ( b1 β1 + b2 β2 + … + bl β l ) ∈ W1 ∩ W2 .
Since W1 ∩ W2 = {0}, we have
a1α1 + a2 α2 + … + am α m = 0, b1β1 + b2 β2 + … + bl β l = 0.
Both the sets {α1, α2 , … , α m } and {β1, β2 , … , β l } are linearly independent, therefore
we have
a1 = 0, a2 = 0, … , am = 0, b1 = 0, b2 = 0, … , bl = 0.
Hence S is linearly independent.
Thus S is a L.I. set containing m + l elements. Hence S is a basis of V.
Any α ∈V can be expressed as a linear combination of the elements of S. So
α = a linear combination of elements of S
= a linear combination of elements of S1 + a linear combination of elements of S2
= an element of W1 + an element of W2
∴ V = W1 + W2 .
Combining both V = W1 + W2 and W1 ∪ W2 = {0}, we have V = W1 ⊕ W2 .
V-49

Comprehensive Problems 3
Problem 1: Show that the set S = {(1, 0, 0 ), (1, 1, 0 ), (1, 1, 1)} is a basis of C3 (C) where C is the
field of complex numbers. Hence find the coordinates of the vector (3 + 4i, 6i, 3 + 7i ) in C3 with respect
to the above basis.
Solution: The dimension of the vector space C3 (C ) is 3. If the set S is linearly
independent, then S will form a basis of C3 (C ) because S contains 3 vectors. Let a, b, c
be scalars in C such that
a (1, 0, 0 ) + b (1, 1, 0 ) + c (1, 1, 1) = (0, 0, 0 )
⇒ ( a + b + c, b + c, c ) = (0, 0, 0 ) ⇒ a + b + c = 0, b + c = 0 , c = 0
⇒ a = 0, b = 0, c = 0 ⇒ the set S is linearly independent.
∴ S is a basis of C3 (C ).
Now to find the coordinates of (3 + 4i, 6i, 3 + 7i ) with respect to the ordered basis S. Let
p, q, r be scalars in C such that (3 + 4i, 6i, 3 + 7i ) = p (1, 0, 0 ) + q (1, 1, 0 ) + r(1, 1, 1)
⇒ (3 + 4i, 6i, 3 + 7i ) = ( p + q + r, q + r, r )
⇒ p + q + r = 3 + 4i, q + r = 6i , r = 3 + 7i
⇒ r = 3 + 7i , q = − 3 − i, p = 3 − 2i.
Hence the coordinates of the vector (3 + 4i, 6i, 3 + 7i ) are ( p, q, r ) i. e.,
(3 − 2i, − 3 − i, 3 + 7i ).
Problem 2: Find the co-ordinate vector of (3, 5, − 2) relative to the basis {e1, e2 , e3 } where
e1 = (1, 1, 1), e2 = (0, 2, 3), e3 = (0, 2, − 1).
Solution: Let (3, 5, − 2) = ae1 + be2 + ce3 = a (1, 1, 1) + b (0, 2 , 3) + c (0, 2 , − 1)

= ( a, a + 2b + 2c, a + 3b − c )
∴ a = 3, a + 2b + 2c = 5, a + 3b − c = − 2.
These give a = 3, b = − 1, c = 2.
∴ Coordinate vector of (3, 5, − 2) is (3, − 1, 2).
Problem 3: Let B = {α1, α2 , α3 } be an ordered basis for R3 , where
α1 = (1, 0, − 1), α2 = (1, 1, 1), α3 = (1, 0, 0 ).
Obtain the coordinates of the vector ( a, b, c ) in the ordered basis B.
Solution: Let x, y, z be scalars in R such that
( a, b, c ) = xα1 + yα2 + zα3 = x (1, 0, − 1) + y (1, 1, 1) + z (1, 0, 0 )
= ( x + y + z , y, − x + y ).
∴ x + y + z = a, y = b, − x + y = c.
Solving these, we get y = b, x = b − c, z = a − 2b + c.
Hence the coordinates of ( a, b, c ) relative to the ordered basis B are ( x, y, z ) i. e.,
( b − c, b, a − 2b + c ).
V-50

Problem 4: Let V be the vector space of all polynomial functions of degree less than or equal to
two from the field of real numbers R into itself. For a fixed t ∈ R,
let g1 ( x ) = 1, g2 ( x ) = x + t, g3 ( x ) = ( x + t)2 .
Prove that {g1, g2 , g3 } is a basis for V and obtain the coordinates of c0 + c1 x + c2 x2 in this
ordered basis.
Solution: Let ag1 + bg2 + cg3 = 0, a, b, c ∈ R
i. e., a1 + b ( x + t) + c ( x + t)2 = 0

or ( a + bt + ct2 ) + ( b + 2ct) x + cx2 = 0 + 0 x + 0 x2

or a + bt + ct2 = 0, b + 2ct = 0, c = 0 or a = 0, b = 0, c = 0.
Hence g1, g2 , g3 are linearly independent.
Since dim V = 3 and { g1, g2 , g3 } is a L.I. subset of V so { g1, g2 , g3 } is a basis of V .
Let c0 + c1 x + c2 x2 = ag1 + bg2 + cg3 ; a, b, c ∈ R
= ( a + bt + ct2 ) + ( b + 2ct) x + cx2 .
2
∴ a + bt + ct = c0 , b + 2ct = c1, c = c2 .
Solving these, we get c = c2 , b = c1 − 2c2 t, a = c0 − c1t + c2 t2 .
Hence the coordinates of c0 + c1 x + c2 x2 in the ordered basis { g1, g2 , g3 } are
( c0 − c1t + c2 t2 , c1 − 2c2 t, c2 ).

Hints to Objective Type Questions

Multiple Choice Questions


1. (a). See dimension of a Quotient Space in article 3 of Text Book.
2. (b). See Ex. 6 of Text Book.
3. (c). See Ex. 2 of Text Book.
Fill in the Blank(s)
1. See Problem 2 of Comprehensive Problems 2.
2. See article 3, corollary of Text Book.
3. See article 5 of Text Book.
4. See article 6 of Text book.
True or False
1. T . The given statement is true. See Dimension of a Direct Sum in article 4 of
Text Book.
2. F. See Ex. 2 of Text Book.
3. F. See Ex. 3 of Text Book.

❍❍❍
V-51

Chapter-5
Matrices

Comprehensive Problems 1
 2 −2 −4
Problem 1: Show that the matrix A =  −1 3 4 is idempotent.
 
 1 −2 −3
Solution: We have
 2 −2 −4  2 −2 −4  2 −2 −4
A 2 = AA =  −1 3 4 ×  −1 3 4 =  −1 3 4 = A.
     
 1 − 2 −3  1 −2 −3  1 −2 −3
Since A 2 = A , therefore the matrix A is an idempotent matrix.
Problem 2: If AB = A and BA = B , show that A and B are idempotent.
Solution: We have AB = A
⇒ A (BA) = A [∵ BA = B]
⇒ ( AB) A = A
⇒ AA = A [∵ AB = A ]
⇒ A 2 = A ⇒ A is idempotent.
Again BA = B
⇒ B( AB) = B [∵ AB = A ]
⇒ ( BA ) B = B
⇒ BB = B
⇒ B2 = B ⇒ B is idempotent.
Problem 3: A matrix A such that A 2 = I is called involutory. Show that A is involutory if and
only if (I + A) (I − A) = O.
Solution: Let A be an involutory matrix of order n.
Then A 2 = I .
∴ I − A2 = O
∴ I2 − A 2 = O [ ∵ I2 = I ]
∴ (I + A) (I − A) = O [ ∵ AI = IA ]
Conversely if (I + A) (I − A) = O,
then I 2 − IA + AI − A 2 = O or I − A 2 + AI − AI = O [ ∵ AI = IA ]
or I − A +O =O
2
or I − A 2 = O or A 2 = I.
V-52

 ab b2  2
Problem 4: Show that the matrix A =   is nilpotent such that A = O .
2
−a − ab 
 ab b2 
Solution: If A = ,
2
 − a − ab 
 ab b2   ab b2 
then A 2 = AA =   2 
2
 − a − ab   − a − ab 
 a2 b2 − b2 a2 ab3 − ab3  0 0 
= = .
3 2 3 2 2 2 2 
 − a b + a b − a b + a b  0 0 
Hence the matrix A is nilpotent of the index 2.
 −1 2 2
1
Problem 5: Show that the matrix 2 −1 2 is orthogonal.
3 
 2 2 −1
 −1 2 2  −1 2 2
1
−1 2 . Then A ′ =  2 −1 2
1
Solution : Let A= 2
3  3 
 2 2 −1  2 2 −1
 −1 2 2  −1 2 2
1
We have AA′ = 2 −1 2  2 −1 2
9  
 2 2 −1  2 2 −1
 1 + 4 + 4 − 2 − 2 + 4 − 2 + 4 − 2
1
= −2−2+ 4 4 +1+ 4 4 − 2 − 2
9 
 − 2 + 4 − 2 4−2−2 4 + 4 + 1
9 0 0  1 0 0 
1
= 0 9 0  = 0 1 0  = I3 .
9   
0 0 9  0 0 1 

Hence the matrix A is orthogonal.


 1 1

1 
 3 6 2
 1 2 
Problem 6: Verify that the matrix.  − 0  is orthogonal.
 3 6 
 1 1 1 
 3 6 2 

 1 1

1   1 1 1 
 3 6 2  3 3 3
 1 2   1 2 1 
Solution: Let A= − 0  . Then A ′ =  − 
 3 6   6 6 6
 1 1 1  − 1 1 
0
 3 6 2   2 2 
V-53

 1 1

1   1 1 1 
 3 6 2  3 3 3
 1 2   1 2 1 
We have AA′ =  − 0  − 
 3 6   6 6 6
 1 1 1  − 1 1 
0
 3 6 2   2 2 

1 + 1 + 1 1 2
− −0
1 1 1
+ −
3 6 2 3 6 3 6 2  1 0 0 
1 2 
− + 0  = 0 1 0  = I3 .
1 4 1 2
= − + 0 + +0
3 6 3 6 3 6  
1 1 1 1 2 1 1 1  0 0 1 
 + − − +0 − + 
3 6 2 3 6 3 6 2

1 1 i
Problem 7: Show that the matrix = is unitary.

2  − i −1  (Rohilkhand 2010)

1  1 i
Solution: We have A =  
2 − i − 1

1 1 − i 1 1 i
∴ A= and A θ = (A) ′ =
2  i −1 
2 − i − 1

1 1 i  1 i
Now A θA =    
2 − i − 1 − i − 1

1 1 − i 2 i − i
=  
2  − i + i − i 2 + 1

1 2 0  1 0 
= = = I.
2 0 2  0 1 

∴ A is unitary.
1 + i − 1 + i
 2  is unitary.
Problem 8: Prove that the matrix  2
1+ i 1− i 
 
 2 2 

1 1 + i −1 + i
Solution: Let A= .
2 1 + i 1− i

1 1 − i −1 − i 1 1− i 1− i
We have (A) = ∴ A θ = (A) ′ =
2 1 − i 1 + i 2  − 1 − i 1 + i

1 1 + i −1 + i  1− i 1− i
Now AA θ =
4 1 + i 1− i  −1 − i 1 +
 i
V-54

1 (1 + i)(1 − i) − (i − 1)(i + 1) (1 + i)(1 − i) + (i − 1)(i + 1)


=
4 (1 + i)(1 − i) − (1 − i)(1 + i) (1 + i)(1 − i) + (1 − i)(1 + i)

1 (12 − i 2 ) − ( i 2 − 12 ) (12 − i 2 ) + ( i 2 − 1)


=  
4 (12 − i 2 ) − (12 − i 2 ) (12 − i 2 ) + (12 − i 2 )

1 1 + 1 + 1 + 1 0 
= [∵ i 2 = − 1]
4  0 1 + 1 + 1 + 1

1 4 0  1 0 
= = = I.
4 0 4  0 1 

∴ A is unitary.

Problem 9: If ( lr , mr , nr ), r = 1, 2, 3 be the direction cosines of three mutually perpendicular


 l1 m1 n1 
lines referred to an orthogonal cartesian co-ordinate system, then prove that  l2 m2 n2  is an
 
 l3 m3 n3 
orthogonal matrix.

 l1 m1 n1 
Solution: Let A =  l2 m2 n2  .
 
 l3 m3 n3 
 l1 l2 l3 
Then A ′ =  m1 m2 m3 
 
 n1 n2 n3 

 l1 m1 n1   l1 l2 l3 
We have AA ′ =  l2 m2 n2   m1 m2 m3 
  
 l3 m3 n3   n1 n2 n3 

 l 2 + m2 + n 2 l1l2 + m1m2 + n1n2 l1l3 + m1m3 + n1n3 


 1 1 1 
=  l2 l1 + m2 m1 + n2 n1 l22 + m22 + n22 l2 l3 + m2 m3 + n2 n3 
 
 l3 l1 + m3 m1 + n3 n1 l3 l2 + m3 m2 + n3 n2 l32 + m32 + n32 

1 0 0 
∵ l12 + m12 + n12 = 1 etc. 
= 0 1 0  = I3 .  
  + m1m2 + n1n2 = 0 etc.
0 0 1  and l1l2

Hence the matrix A is orthogonal.


V-55

Problem 10: Show that if A is an orthogonal matrix, then A ′ and A −1 are also orthogonal.
Solution: A is orthogonal
⇒ A′A = I
⇒ ( A ′ A )′ = I′
⇒ A ′ ( A ′ )′ = I
⇒ A ′ is an orthogonal.
Again A is an orthogonal ⇒ A ′ A = I
⇒ ( A ′ A )−1 = I−1
⇒ A −1 ( A ′ )−1 = I
⇒ A −1 ( A −1)′ = I [∵ (A ′ )−1 = (A −1)′ ]
⇒ A −1 is an orthogonal.
 0 1 + 2i
Problem 11: If N =  is a matrix, then show that ( I − N ) ( I + N )−1 is a
 − 1 + 2i 0 
unitary matrix, where I is an identity matrix.
 0 1 + 2i
Solution: We have N= .
 − 1 + 2 i 0 

1 0   0 1 + 2i   1 −1 − 2i
∴ I−N=  −  = ...(1)
0 1   − 1 + 2i 0  1 − 2i 1 

1 0   0 1 + 2i   1 1 + 2i
and I+ N= +  = .
0 1   − 1 + 2i 0   −1 + 2i 1 

 1 −1 − 2i
Thus | I + N | = 1 − ( − 1 − 4) = 6 and adj ( I + N ) =  ⋅
1 − 2 i 1 

adj ( I + N ) 1  1 −1 − 2i
Now ( I + N )−1 = =  ⋅
|I + N | 6 1 − 2 i 1  ...(2)
θ
For unitary matrix, we have A A = I .
From (1) and (2), we get
1 1 −1 − 2i  1 −1 − 2i
( I + N )( I + N )−1 = ⋅
6 1 − 2i 1  1 − 2i 1 

1  −4 − 2 − 4i
= 2 − 4i = B , (say).
6  − 4 

1  −4 2 + 4i
Here ( B)T = ⋅
6  − 2 − 4i − 4 

1  −4 2 + 4i  − 4 − 2 − 4i 1 36 0 


∴ ( B)T B =    =  0 36 = I ⋅
36  − 2 − 4i − 4  2 − 4i − 4  36  
Hence the result.
V-56

Hints to Objective Type Questions

Multiple Choice Questions


1. (b). See Problem 4 of Comprehensive Problems 1.
 6 9  6 9 0 0  2
2. (d). We have   =   i. e., A = 0
 −4 −6  −4 −6 0 0 
Hence A is an nilpotent matrix.
3. (b). See article 7.
4. (b). See article 12.

Fill in the Blank(s)


1. See article 1.
2. See article 1.
3. See article 12.

True or False
1. T . See Ex. 11.
2. T . See Problem 10 of Comprehensive Problems 1.
3. F. See Ex. 6.

❍❍❍
V-57
Chapter-6
Rank of a Matrix

Comprehensive Problems 1
Problem 1: Determine the rank of each of the following matrices :
1 0 0 0  1 1 1 1
0 1 0 0  1 1 1 1 0 0 
(i)   (ii)   (Meerut 2006B) (iii)  
0 0 1 0  1 1 1 1 0 0 
0 0 0 1  1 1 1 1
   
1 2 3 4   1 2 −7 5
5 10  2 4 6 8  (vi) 0 5 0 1 2 3
(iv)   (v) 8 (vii)  .
3 6      2 4 5
3 6 9 12 0 0 0 −3
1 0 0 0 
0 1 0 0 
Solution: (i) Let A =   be a unit matrix of order 4.
0 0 1 0 
0 0 0 1 
 
We have |A | = 1, Therefore A is a non-singular matrix.Hence rank A = 4.
1 1 1 1
1 1 1 1
(ii) Let A= .
1 1 1 1
1 1 1 1
 
We have|A| = 0 , since all columns are identical. Also each 3-rowed and 2-rowed minor
of A is equal to zero. But A is not a null matrix. Hence rank A = 1.
0 0 
(iii) Let A= .
0 0 
Since A is a null matrix, therefore rank A = 0.
5 10 
(iv) Let A = .
3 6 
We have |A| = 30 − 30 = 0.
Therefore the rank of A is less than 2. Hence rank A = 1.
1 2 3 4
(v) Let A = 2 4 6 8
 
3 6 9 12
Here each 3-rowed and 2-rowed minor of A is equal to zero. But A is not a null matrix.
Hence rank A = 1.
V-58

 1 2 −7 5
(vi) Let A = 0 5 0 8
 
0 0 0 −3
2 −7 5
Here we see that there is at least one minor of A of order 3 i. e., 5 0 8 which is not
0 0 −3
equal to zero.
Also there is no minor of order greater than 3. Hence the rank A = 3.
1 2 3
(vii) Let A= .
2 4 5
2 3
Here one minor of A is of order 2 i. e., which is not equal to zero.
4 5
Also there is no minor of order greater than 2. Hence the rank A = 2.
Problem 2: Show that the rank of a matrix is ≥ the rank of every sub-matrix thereof.
Solution: Let A 1 be a sub-matrix of the matrix A. Let r be the rank of the matrix A 1. Then
the matrix A 1 must have at least one r-rowed minor not equal to zero. Now every
r-rowed minor of the matrix A 1 will also be an r-rowed minor of the matrix A. Therefore
the matrix A will have at least one r-rowed minor not equal to zero. Hence the rank of
the matrix A will be ≥ r.
Problem 3: Show that the rank of a matrix does not alter on affixing any number of additional
rows or columns of zeros.
Solution: Let A be a matrix of rank r. Let M be the matrix obtained from the matrix A by
affixing some additional rows and columns of zeros.
A O 
Let M= .
O O 
Now every ( r + 1)-rowed minor of the matrix M will either also be a minor of the matrix
A or it will have at least one row or one column of zeros. Since the matrix A is of rank r,
therefore every ( r + 1)-rowed minor of the matrix A (if there is any ) will be equal to zero.
Thus every ( r + 1)-rowed minor of the matrix M is also equal to zero. Since the matrix A
has at least one minor of order r not equal to zero, therefore at least one r-rowed minor of
the matrix M will also be not equal to zero. Hence the rank of the matrix M is also equal
to r.
0 1 0 0
0 0 1 0
Problem 4: If A =   , find the rank of A and A2 .
0 0 0 1
0 0 0 0 

0 1 0 0
0 0 1 0
Solution: We have A= 
0 0 0 1
0 0 0 0 

V-59

1 0 0
Here one minor of A of order 3 i. e., 0 1 0 which is not equal to zero.
0 0 1
Also there is no minor of order greater than 3. Hence the rank A = 3.
0 1 0 0  0 1 0 0  0 0 1 0
0 0 1 0  0 0 1 0  0 0 0 1
Now A2 =    
0 0 0 1  0 0 0 1  0 0 0 0
0 0 0 0  0 0 0 0  0 0 0 0 
  
1 0
Here one minor of A of order 2 i. e., ≠ 0. Hence the rank A2 = 2.
0 1
Problem 5: Under what conditions the rank of the following matrix is 3? Is it possible for the rank to
2 4 2
be 1? Why ? A = 3 1 4 .
 
1 0 x  (Kanpur 2010)
Solution: The rank of the given matrix A = 3.
∴ The minor of order 3 of matrix A ≠ 0.
2 4 2
i. e., 3 1 4 ≠ 0 or 2( x − 0 ) − 4(3 x − 4) + 2(0 − 1) ≠ 0
 
1 0 x 
7
or 2 x − 12 x + 16 − 2 ≠ 0 or −10 x + 14 ≠ 0 or x ≠ ⋅
5
Second part. No, because one minor of order 2 of A is non-zero.
Problem 6: A is an n-rowed square matrix of rank ( n − 1), show that Adj. A is not a null matrix.
Solution: A is an n-rowed square matrix of rank (n −1). Therefore at least one
( n −1)-rowed minor of the matrix A is not equal to zero. Now every ( n −1)-rowed minor of
the matrix A is equal in magnitude to the cofactor of some element in |A|. Thus at least
one element in |A| has its cofactor not equal to zero . Therefore at least one element of
the matrix Adj. A is not equal to zero.
Hence Adj. A ≠ O.

Comprehensive Problems 2
1 2 3 0
2 4 3 2
Problem 1: Find the rank of the matrix  .
3 2 1 3
6 8 7 5 

Solution: We have the matrix
1 2 3 0
2 4 3 2
A ~  , by R 4 → R 4 − R 3 − R 2 − R 1
3 2 1 3
0 0 0 0 

V-60

1 2 3 0 
0 0 −3 2 
or A ~  ,by R 2 → R 2 − 2R 1, R 3 → R 3 − 3R 1
0 −4 −8 3 
0 0 0 0 
 
Now E-transformations do not change the rank of a matrix. We see that the
determinant of the last equivalent matrix is zero. But the leading minor of the third
order of this matrix i.e.,
1 2 3 
0 0 −3 = − 12 i. e., ≠ 0.
 
0 −4 −8
Therefore the rank of this matrix is 3. Hence rank A = 3.
Important Note. To determine the rank of a matrix, we can reduce it to Echelon form
or to normal form . But sometimes we are given such matrices that if we carefully study
their rows and columns, then we shall find that some rows or columns are linearly
dependent on some of the others. These can be reduced to zeros by E-row or column
transformations. Then we try to find some non- vanishing determinant of the highest
order in the equivalent matrix. The order of this determinant will determine the rank of
the given matrix.
 6 1 3 8
 4 2 6 −1
Problem 2: Find the rank of the matrix  
10 3 9 7
16 4 12 15
  (Bundelkhand 2008;
Rohilkhand 08; Avadh 05; Lucknow 11)
Solution: We have the matrix
6 1 3 8
4 2 6 −1
A ~  , by R3 → R3 − R2 − R1, R4 → R4 − R3 − R1.
0 0 0 0
0 0 0 0 

6 1
Since = 8 ≠ 0 and the determinant of each square submatrix of order 3 of this
4 2
equivalent matrix is zero, therefore rank(A) = 2.
2 3 −1 −1
1 −1 −2 −4
Problem 3: Find the rank of the matrix  
3 1 3 −2
6 3 0 −7

(Avadh 2006, 09; Purvanchal 06)
Solution: We have
2 3 −1 −1
1 − 1 − 2 − 4 
A ~  , by R4 → R4 − R3 − R2 − R1
3 1 3 − 2 
0 0 0 0 
 
V-61

1 − 1 − 2 − 4 
2 3 −1 −1
~  , by R1 ↔ R2
3 1 3 − 2 
0 0 0 0 
 
1 − 1 − 2 − 4 
0 5 3 7 
~  , by R2 → R2 − 2R1, R3 → R3 − 3R1.
0 4 9 10 
0 0 0 0 
 
We see that the determinant of the last equivalent matrix is zero . But there is one minor
1 −1 −2
of order 3 i.e., 0 5 3 which is equal to 45 − 12 = 33 i. e., ≠ 0.
0 4 9
Therefore the rank of the matrix A is 3.
1 a b 0
0 c d 1
Problem 4: Find the rank of the matrix  .
1 a b 0
0 c d 1 
 (Lucknow 2007)
Solution: We have the matrix
1 a b 0
0 c d 1
A ~  , by R3 → R3 − R1, R4 → R4 − R2 .
0 0 0 0
0 0 0 0 

This matrix is in Echelon form.
∴ rank A = the number of non- zero rows in this matrix = 2.
1 3 4 3
Problem 5: Find the rank of the matrix 3 9 12 9
 
1 3 4 1 (Rohilkhand 2007)
Solution: Let us denote the given matrix by A. Performing the elementary row
operations R2 → R2 − 3R1 , R3 → R3 − R1, we see that
1 3 4 3  1 3 4 3 
A ~ 0 0 0 0  ~ 0 0 0 −2 , by R2 ↔ R3 .
   
0 0 0 −2 0 0 0 0 
We see that the last equivalent matrix is in Echelon form. The number of non-zero rows
in this matrix is 2. Hence its rank is 2. Therefore rank A = 2.
 1 2 −1 4
Problem 6: Find the rank of the matrix  2 4 3 5
 
 −1 −2 6 −7
V-62

Solution: Let us denote the given matrix by A. Performing the elementary row
operations R2 → R2 − 2R1 , R3 → R3 + R1, we see that
1 2 − 1 4  1 2 − 1 4 
A ~ 0 0 5 −3 ~ 0 0 5 −3 , by R3 → R3 − R2 .
   
0 0 5 −3 0 0 0 0 
The last equivalent matrix is in Echelon form. The number of non-zero rows in this
matrix is 2. Hence its rank is 2.
Therefore rank A = 2.
 1 2 1 2
 1 3 2 2
Problem 7: Find the rank of the matrix  
2 4 3 4
3 7 4 6
  (Lucknow 2008)
Solution: Let us denote the given matrix by A. Performing the elementary operations,
R2 → R2 − R1 , R3 → R3 − 2R1 , R4 → R4 − 3R1 , we see that
1 2 1 2  1 2 1 2
0 1 1 0  0 1 1 0
A ~  ~  , by R4 → R4 − R2
0 0 1 0  0 0 1 0
0 1 1 0  0 0 0 0 

The last equivalent matrix is in Echelon form. The number of non- zero rows in this
matrix is 3. Therefore its rank is 3.
Hence rank A = 3.
 3 −2 0 −1
0 2 2 1
Problem 8: Find the rank of the matrix  
1 − 2 − 3 2 
0 1 2 1

Solution: Let us denote the given matrix by A. Performing the elementary row operation
R1 ↔ R3 , we see that
1 − 2 − 3 2  1 − 2 − 3 2 
0 2 2 1  0 2 2 1 
A ~  ~  , by R3 → R3 − 3R1
3 −2 0 −1 0 4 9 −7
0 1 2 1  0 1 2 1 
   
1 − 2 − 3 2 
0 1 2 1 
~  , by R2 ↔ R4
0 4 9 −7
0 2 2 1 
 
1 − 2 − 3 2 
0 1 2 1 
~  , by R3 → R3 − 4R2 , R4 → R4 − 2R2
0 0 1 −11
0 0 −2 −1 
 
V-63

1 − 2 − 3 2 
0 1 2 1 
~  , by R4 → R4 + 2R3 .
0 0 1 −11
0 0 0 −23
 
The last equivalent matrix is in Echelon form . The number of non- zero rows in this
matrix is 4. Therefore its rank is 4.
Hence rank A = 4.
 1 2 −4 5
Problem 9: Find the rank of the matrix 2 −1 3 6
 
8 1 9 7

Solution: Proceed as in Problem 6. Ans. 3.


 1 −1 3 6
Problem 10: Find the rank of the matrix  1 3 −3 −4
 
5 3 3 11

Solution: Proceed as in Problem 6. Ans. 3.


 8 0 0 1
 1 0 8 1
Problem 11: Find the rank of the matrix  
0 0 1 8
0 8 1 8
  (Kumaun 2008)

Solution: Proceed as in Problem 7. Ans. 4.


 0 1 −3 −1
1 0 1 1
Problem 12: Find the rank of the matrix  
 3 1 0 2
 1 1 −2 0 
 
(Garhwal 2006, 13; Avadh 10)

Solution: Proceed as in Problem 7. Ans. 2.


 1 −3 4 7
Problem 13: Find the rank of the matrix 9
 1 2 0  (Kumaun 2008)
Solution: Let us denote the given matrix by A. Then
1 − 3 4 7 
A ~ , by R2 → R2 − 9R1
0 28 −34 −63

which is in the Echelon form.


Now rank A= the number of non-zero rows in its Echelon form =2.
1 1 1
Problem 14: Find the rank of the matrix 2 2 2 .
 
3 3 3
V-64

1 1 1 
Solution: We have A~ 0 0 0  , by R2 → R2 − 2R1 , R3 → R3 − 3R1
 
0 0 0 
which is in the Echelon form.
Now rank A= the number of non-zero rows in its Echelon form = 1.
2 1 3
Problem 15: Find the rank of the matrix 4 7 13

 
4 −3 −1 (Kumaun 2012)
Solution: Proceed as in Problem 14. Ans. 2.
4 5 6
Problem 16: Find the rank of the matrix 5 6 7
 
7 8 9
Solution: Proceed as in Problem 14. Ans. 2.
 1 0 2 1
 0 1 −2 1
Problem 17: Find the rank of the matrix  
 1 −1 4 0 
 −2 2 8 0 
 
Solution: Proceed as in Problem 7. Ans. 3.
1 1 1
Problem 18: Find the rank of the matrix  a b c
 3 
 a b3 c3  (Meerut 2011; Kumaun 12)
1 1 1
Solution: We have |A|= a b c = ( b − c ) ( c − a) ( a − b )( a + b + c ).
a3 b3 c3
Now there arise the following cases :
Case1. If a, b, c are all different and a + b + c ≠ 0, then|A| ≠ 0. Hence A is non - singular
and so in this case rank A = 3.
Case II. Let a, b, c be all different but a + b + c = 0. In this case |A| = 0 and so rank
1 1
A < 3. But at least one minor of A of order 2, say , = b − a ≠ 0. Hence in this case
a b
rank A = 2 .
Case III. Suppose two of the three numbers, a, b, c are equal but are different from the
third, say a = b ≠ c.
In this case |A| = 0 and so rank A < 3. But at least one minor of A of order 2, say,
1 1
= c − b ≠ 0. Hence in this case rank A = 2.
b c
Case IV. Let a = b = c.
In this case|A| = 0 and all minors of A of order 2 are zero and so rank A < 2. But A has a
non- zero minor of order 1.
Hence rank A = 1.
V-65

Problem 19: With the help of elementary transformations find the rank of the following matrix :
1 1 2 3
1 3 0 3
 
1 −2 −3 −3
1 1 2 3
  (Rohilkhand 2010)
Solution: Let us denote the given matrix by A . Then
1 1 2 3 
0 2 −2 0 
A ~ ,
0 −3 −5 −6
0 0 0 0 
 
by R2 → R2 − R1 , R3 → R3 − R1 , R4 → R4 − R1

1 1 2 3 
0 1 −1 0  1
~  , by R2 → R2
0 − 3 − 5 − 6  2
0 0 0 0 
 
1 1 2 3
0 1 −1 0 
~  , by R3 → R3 + 3R2 ,
0 0 −8 −6
0 0 0 0 

which is in Echelon form . The number of non-zero rows in this Echelon form is 3.
Hence rank A = 3.
Problem 20: Reduce the following matrix to its Echelon form and find its rank :
 1 3 4 5
 3 9 12 9 .
 
 −1 −3 −4 −3
(Meerut 2004B; Rohilkhand 06, 10)
Solution: We have
1 3 4 5
A ~ 0 0 0 −6 , by R2 → R2 − 3R1 , R3 → R3 + R1
 
0 0 0 2 
1 3 4 5
~ 0 0 0 −6 , by R3 → R3 + R2
1
  3
0 0 0 0 

which is in the Echelon form.


Now rank A = the number of non-zero rows in its Echelon form =2.
V-66

1 2 3
Problem 21: Find the rank of the matrix A = 2 3 4 after reducing it to normal form.
 
3 5 7 (Garhwal 2015)
Solution: We have the matrix
1 2 3 
A ~ 0 −1 −2 , by R2 → R2 − 2R 1, R3 → R3 − 3R 1
 
0 −1 −2
1 0 0 
~ 0 −1 −2 , by C2 → C2 − 2C1, C3 → C3 − 3C1
 
0 −1 −2
1 0 0 
~ 0 1 2  , by R2 → − R2 ,
 
0 −1 −2
1 0 0 
~ 0 1 2  , by R3 → R3 + R2 ,
 
0 0 0 
1 0 0 
~ 0 1 0  , by C3 → C3 − 2C2
 
0 0 0 
I O 
which is in the normal form  2 .
O O 
Hence rank A = 2.
0 1 3 −1
1 0 1 1
Problem 22: Reduce the matrix   to normal form and find its rank.
3 1 0 2
 1 1 −2 0 

(Meerut 2009; Bundelkhand 11; Avadh 11)
Solution : We have
1 0 1 1
0 1 3 −1
A ~  , by R1 ↔ R2
3 1 0 2
1 1 −2 0 

1 0 1 1
0 1 3 −1
~  , by R3 → R3 − 3R1 , R4 → R4 − R1
0 1 −3 −1
0 1 −3 −1

1 0 0 0
0 1 3 −1
~  , by C3 → C3 − C1 , C4 → C4 − C1
0 1 −3 −1
0 1 −3 −1

V-67

1 0 0 0
0 1 3 −1
~  , by R3 → R3 − R2 , R4 → R4 − R2
0 0 −6 0 
0 0 −6 0 

1 0 0 0
0 1 0 0
~  , by C3 → C3 − 3C2 , C4 → C4 + C2
0 0 −6 0 
0 0 −6 0 

1 0 0 0
0 1 0 0 1
~  , by C3 → − C3
0 0 1 0 6
0 0 1 0 

1 0 0 0
0 1 0 0
~  , by R4 → R4 − R3
0 0 1 0
0 0 0 0 

I O
which is in the normal form  3 .
O O 
Hence rank A = 3.
9 7 3 6
Problem 23: (i) Reduce the matrix 5 −1 4 1 to normal form and find its rank.
 
6 8 2 4 (Agra 2007)
(ii) Reduce the following matrix A into normal form and hence find its rank:
0 1 2 −2
A =  4 0 2 6 ⋅ (Kanpur 2011)
 
 2 1 3 1
Solution: (i) We have
6 7 3 9
A ~ 1 −1 4 5 , by C1 ↔ C4
 
4 8 2 6
1 −1 4 5
~ 6 7 3 9 , by R1 ↔ R2
 
4 8 2 6
1 − 1 4 5 
~ 0 13 −21 −21,by R2 → R2 − 6R1 , R3 → R3 − 4R 1
 
0 12 −14 −14
1 0 0 0 

~ 0 13 −21 −21 ,by C2 → C2 + C1, C3 → C3 − 4C1,
 
0 12 −14 −14 C → C − 5C
4 4 1
V-68

1 0 0 0 
~ 0 13 3 3  , by C3 → − C3 , C4 → − C4
1 1
  7 7
0 12 2 2 
1 0 0 0 
~ 0 1 1 1  , by R2 → R2 − R3
 
0 12 2 2 
1 0 0 0 

~ 0 1 1 1  , by R3 → R3 − 12R2
 
0 0 −10 −10 
1 0 0 0 
~ 0 1 0 0  , by R3 → R3 − R2 , C4 → C4 − C2
 
0 0 −10 −10 
1 0 0 0 
~ 0 1 0 0  , by R3 → −
1
R3
  10
0 0 1 1 
1 0 0 0 
~ 0 1 0 0  , by C4 → C4 − C3
 
0 0 1 0 
which is in the normal form, [I3 O].
Hence rank A=3.
0 1 2 −2
(ii) We have A = 4 0 2 6 
 
2 1 3 1 
2 1 3 1 
~ 4 0 2 6  , by R1 ↔ R3
 
0 1 2 −2
2 1 3 1 
~ 0 −2 −4 4  , by R2 → R2 − 2R1
 
0 1 2 −2
2 1 3 1 
~ 0 1 2 −2 , by R2 → − R2
1
  2
0 1 2 −2
2 1 3 1 
~ 0 1 2 −2 , by R3 → R3 − R2
 
0 0 0 0 
which is in Echelon form.
Now rank A= the number of non-zero rows in its Echelon form =2.
V-69

Problem 24: Are the following pairs of matrices equivalent?


 2 −1 3 4  1 0 −5 6
0 3 4 1 3 −2 4 0 2 3 9 0 2
1 2
(i)  ,   (ii) 3 1 0  , 7 −2 0 1 ⋅
 2 3 7 5 5 −2 −9 14    
 2 5 11 5 4 −2 −4 8 5 0 0  8 1 1 5
   
Solution: (i) The two matrices are of the same size. If they have the same rank, they
are equivalent otherwise not. It can be seen that the rank of the first matrix is 4 and
that of the second matrix is 2. Hence they are not equivalent.
(ii) The two matrices are not of the same size. Therefore they cannot be equivalent.
Problem 25: Find the ranks of A, B, A + B, AB and BA where
 1 1 −1  −1 −2 −1
A = 2 −3 4, B =  6 12 6 .
   
3 −2 3  5 10 5
Solution: Do yourself.
Ans. Rank A = 2, rank B = 1, rank (A + B) = 2, rank AB = 0, rank BA = 1.
Problem 26: Show that if A and B are equivalent matrices, then there exist non-singular matrices
P and Q such that B = PAQ .
Solution: If A~B, then B can be obtained from A by a finite number of elementary
transformations of A. But elementary row (column) transformations are equivalent to
pre-(post)-multiplication by the corresponding elementary matrices . Therefore if
A~B, there exist E-matrices P1, P2 , … , Ps , Q 1 , Q 2 , … , Q t , such that

Ps Ps − 1 … P2 P1 AQ 1 Q 2 … Q t = B.
Now each elementary matrix is non- singular and the product of non-singular matrices
is also non-singular.
Therefore if P = Ps Ps − 1 … P2 P1 and Q =Q 1 Q 2 … Q t , then there exist non-singular
matrices P and Q , such that PAQ = B.
Problem 27: Show that the rank of a matrix is not altered if a column of it is multiplied by a
non-zero scalar.
Solution: Proceed as in Text Book (Ch.-Rank of a Matrix) the case II of the theorem of
article 9.
Problem 28: (i) What is the rank of a non-singular matrix of order n?
(ii) What is the rank of an elementary matrix?
Solution: (i) The rank of a non-singular matrix of order n is n .
(ii) Let E be an elementary matrix of order n. Then E is obtained from the unit matrix I n
by a single elementary transformation. Since rank I n is n and elementary
transformations do not change the rank of a matrix, therefore rank E = n. Hence the
rank of an elementary matrix is equal to the order of the matrix.
V-70

Comprehensive Problems 3
 1 2 1
Problem 1: Reduce the matrix A =  −1 0 2 to I3 by E-row transformations only.
 
 2 1 3
 1 2 1
Solution: We have A =  −1 0 2
 
 2 1 3
 1 2 1
~  −1 0 2 by R3 → R3 + 2R2
 
 0 1 7
1 2 1 
~ 0 2 3 by R2 → R2 + R 1
 
0 1 7
 1 0 −13
~ 0 0 −11 by R 1 → R 1 − 2R3 , R2 → R2 − 2R3
 
0 1 7
 1 0 −13  1 0 −13
~ 0 1 7 by R2 ↔ R3 ~ 0 1 7 by R3 → − R3
1
    11
0 0 −11 0 0 1
1 0 0 
~ 0 1 0  by R1 → R1 + 13R3 , R2 → R2 − 7R3
 
0 0 1 
Problem 2: Compute the inverse of the following matrices by using elementary
row-transformations :
 −1 −3 3 −1
 1 2 3  i −1 2i  1 1 −1 0
(i) 2 4 5 (ii)  2 0 2 (iii)  
     2 −5 2 −3
3 5 6  −1 0 1  −1 1 0
 1

1 3 3 0 1 2
(iv) 1 4 3 (v) 1 2 3 
   
1 3 4 3 1 1
(Avadh 2007; Garhwal 15) (Rohilkhand 2011; Bundelkhand 08)

1 2 3
Solution: (i) Let A = 2 4 5 .We have A = I3 A
 
3 5 6
V-71

1 2 3 1 0 0 
i. e., 2 4 5 = 0 1 0  A
   
3 5 6 0 0 1 

Applying R2 → R2 − 2R1, R3 → R3 − 3R1


1 2 3  1 0 0
0 0 −1 =  −2 1 0 A
   
0 −1 −3  −3 0 1
 1 2 3  1 0 0
Applying R2 ↔ R3 0 −1 −3 =  −3 0 1 A
   
0 0 −1  −2 1 0 
Applying R1 → R1 + 3R3 , R2 → R2 − 3R3
1 2 3  5 3 0 
0 −1 0  =  3 −3 1 A
   
0 0 −1  −2 1 0 
Applying R1 → R1 + 2R2
 1 0 0   1 −3 2
0 −1 0  =  3 −3 1 A
   
0 0 −1  −2 1 0 
Applying R2 → − R2 , R3 → − R3
1 0 0   1 − 3 2 
0 1 0  =  −3 3 −1 A
   
0 0 1   2 −1 0 
 1 −3 2
∴ A −1 =  −3 3 −1
 
 2 −1 0 
 i −1 2i
(ii) Let A =  2 0 2 .We have A = I3 A
 
 −1 0 1
 i −1 2i  1 0 0
i. e.,  2 0 2 = 0 1 0 A
   
 −1 0 1 0 0 1
1
Applying R2 → R2
2
 i −1 2i 
1 0 0 
 1 0 1 = 0 1 
 
0 A
 2
 −1 0 1 0 0 
1 

Applying R3 → R3 + R2
V-72

 
1 0 0
 i −1 2i  
 1 0 1 = 0 1 
0 A
   2
0 0 2  
 1 
0 1
 2 

 
1 0 0
 i −1 2i  
Applying R3 →
1
R3  1 0 1 = 0 1 
0 A
2    2
0 0 1  
 1 1
0 
 4 2
Applying R2 → R2 − R3
 
 i −1 2i  1 0 0
 1 0 0  = 0 1 1
− =A
   4 2
0 0 1  1 1
0 
 4 2
Applying R 1 → R 1 − iR2 − 2iR3
1 −
3i
− 
i
0 −1 0   4 2
 1 0 0  = 0 1 1
− A
   4 2
0 0 1  1 1
0 
 4 2
 −1 3i i
0 1 0   4 2
Applying R1 → − R1 1 0 0  =  0 1 1
−  A
   4 2
0 0 1   1 1
 0 
 4 2
 0 1
− 
1
1 0 0   4 2
Applying R1 ↔ R2 0 1 0  =  −1 3i i
   4 2
A
0 0 1   1 1
 0 
 4 2
 0 1 − 1
 4 2
−1  3i i
∴ A =  −1 
 4 2
1 1
 0 
 4 2
V-73

 −1 −3 3 −1 1 0 0 0 
 1 1 −1 0  0 1 0 0 
(iii) We write  =  A.
 2 −5 2 −3 0 0 1 0 
 −1 1 0 1  0 0 0 1 

Applying R 1 → − R 1 , we get
 1 3 −3 1   −1 0 0 0 
 1 1 −1 0   0 1 0 0 
 =  A.
 2 −5 2 −3  0 0 1 0 
 −1 1 0 1   0 0 0 1 

Applying R2 → R2 − R 1 , R3 → R3 − 2R 1 , R4 → R4 + R 1 , we get
1 3 −3 1   −1 0 0 0
0 − 2 2 −1  1 1 0 0
 =  A.
0 −11 8 −5  2 0 1 0
0 4 −3 2   −1 0 0 1 

1
Applying R2 → − R2 , we get
2
1 3 −3 1   −1 0 0 0
 1  1 1 
0 1 −1 − − 0 0
2 =  2 2 A.
0 −11 8 −5   2 0 1 0
   
0 4 −3 2  −1 0 0 1
Applying R 1 → R 1 − 3R2 , R3 → R3 + 11R2 , R4 → R4 − 4R2 , we get
1 0 0 − 1   1 3
0 0
 2  2 2 
 1  1 1 
0 1 −1  = − − 0 0
A.
 2 2 2
1  7 11 
0 0 −3  − − 1 0
 2  2 2 
0 0 1 0  1 2 0 1
Applying R3 ↔ R4 , we get
1 0 0−  
1 1 3
0 0
 2   2 2 
 1  1 1 
0 1 −1  = − − 0 0
2 2 2 A.
0 0 1 0  1 2 0 1
   
0 1  7 11
0 −3 − − 1 0
 2  2 2 
Applying R2 → R2 + R3 , R4 → R4 + 3R3 , we get
1 0  1 0 0
0 −  
1 3
 2  2 2 
 1   1 3 
0 1
0 1 0  
2 = 2 2 A.
0 0 1 0  1 2 0 1
 1   1 1

0 0 0  − 1 3
 2   2 2 
V-74

Applying R2 → R2 − R4 , R1 → R1 + R4 , we get
1 0 0 0  0 2 1 3
0  −1 −2
1 0 0  1 1

 =
0 0 1 0  1 2 0 1  A.
0  1 1
1 3
 0 0 1 / 2  − 
 2 2
Applying R4 → 2R4 , we get
1 0 0 0   0 2 1 3 
0 1 0 0   1 1 −1 −2
 =  A.
0 0 1 0   1 2 0 1 
0 0 0 1   −1 1 2 6 
   
0 2 1 3 
 1 1 −1 −2
∴ A −1 =  
1 2 0 1 
 −1 1 2 6 
 
1 3 3
(iv) Let A = 1 4 3 . We have A = I3 A
 
1 3 4
1 3 3 1 0 0 
i. e., 1 4 3 = 0 1 0  A
   
1 3 4 0 0 1 
Applying R2 → R2 − R 1, R3 → R3 − R 1
1 3 3   1 0 0
0 1 0  =  −1 1 0  A
   
0 0 1   −1 0 1
Applying R1 → R1 − 3R3
1 3 0   4 0 −3
0 1 0  =  −1 1 0 A
   
0 0 1   −1 0 1 
1 0 0   − 7 −3 −3
Applying R1 → R1 − 3R2 0 1 0  =  −1 1 0 A
   
0 0 1   −1 0 1
 −7 −3 −3
∴ A − 1 =  −1 1 0 .
 
 −1 0 1
0 1 2
(v) Let A = 1 2 3 
 
3 1 1
We have A = I3 A
V-75

0 1 2 1 0 0  0 1 2
i. e., 1 2 3 = 0 1 0  1 2 3
    
3 1 1 0 0 1  3 1 1
Applying R1 ↔ R2
1 2 3 0 1 0 
0 1 2 = 1 0 0  A
   
3 1 1 0 0 1 
Applying R3 → R3 − 3R 1
 1 2 3 0 1 0
0  
1 2 = 1 0 0 A
   
0 −5 −8 0 −3 1
Applying R3 → R3 + 5R2
1 2 3 0 1 0
0 1 2 =  1 0 0  A
   
0 0 2  5 −3 1
 1 0
1 2 3   0 
1
Applying R3 → R3  
0 1 2 = 1 0 0 A
2   5 − 3 1 
0 0 1
 2 2 2 
Applying R2 → R2 − 2R3 , R 1 → R1 − 3R3
 15 11 − 3 
1 2 0   − 2 2 2
0 1 0  =  −4 3 −1 A
   5 3 1
0 0 1   −
 2 2 2 
 1 1 1
1 0 0   2 − 2 2 
Applying R 1 → R 1 − 2R2 0 1 0  =  −4 3 −1 A.
   5 3 1
0 0 1   −
 2 2 2 
 1 − 1 1
 2 2 2
∴ A − 1 =  −4 3 −1.
 5 3 1
 2 − 2 2
 

Hints to Objective Type Questions

Multiple Choice Questions


1. (b). See article 2 Definition of rank of a matrix.
2. (b). See Problem 1(ii) of Comprehensive Problems 1.
3. (c). See Problem 1 of Comprehensive Problems 2.
V-76

4. (d). We know that the rank of a product of two matrices cannot exceed the
rank of either matrix. See article 15.
So, we must have rank ( AB) ≤ min (rank A, rank B).
5. (b). We know that the rank of a matrix does not alter by pre-multiplication
or post-multiplication with a non-singular matrix. See Corollary 3 of
article 16.
Since A is non-singular, therefore rank (AB) = rank B.
1 2
6. (c). Obviously, every 3-rowed minor of A is zero. But, = 3 − 4 = − 1 ≠ 0.
2 3
Thus, there is a 2-rowed minor of A which is not zero. Therefore, rank
A=2.

Fill in the Blank(s)


1. See article 2 Definition of rank of a matrix.
2. Let A be a non-singular matrix of order n. Then |A|≠ 0. Since |A| is a
non-zero minor of A of order n and A has no minor of order n + 1, therefore
rank A = n.
3. See article 3 Definition of a matrix in Echelon form.
4. See Ex. 6.
5. See article 6, Important remark.
6. See article 7, Definition of an elementary matrix.
7. See Corollary 1 of article 10.
8. See Corollary 2 of article 10.
9. See article 11 definition of equivalent matrices.
10. See Ex. 8.
11. The matrix is unit matrix of order 2, therefore rank is 2.

True or False
1. T . See article 4, theorem.
2. F. See article 8, theorem. The correct statement is that every elementary
row transformation of a matrix can be obtained by pre-multiplication
with the corresponding elementary matrix.
3. F. We know that elementary transformations do not change the rank of a
matrix. See article 9.
4. T . See Ex. 7.
5. T . See Corollary 3 of article 16.

❍❍❍
V-77
Chapter-7
Applications of Matrices

Comprehensive Problems 1
Problem 1: Find all the solutions of the following system of linear homogeneous equations :
2 x − 3 y + z = 0, x + 2 y − 3z = 0, 4 x − y − 2z = 0.
Solution: The given system of equations is equivalent to the single matrix equation
2 −3 1  x 
A X =  1 2 −3  y  = O.
  
4 −1 −2  z 
We shall first find the rank of the coefficient matrix A by reducing it to Echelon form by
applying elementary row transformations only.
Applying R1 ↔ R2 , we get
 1 2 −3  x 
2 −3 1  y  = O.
  
4 −1 −2  z 
Performing R2 → R2 − 2R1 , R3 → R3 − 4R1, we get
 1 2 −3  x 
0 −7 7  y  = O.
  
0 −9 10   z 
Here we find that the determinant of the matrix on left hand side of this equation is not
equal to zero. Therefore the rank of this matrix is 3. So there is no need of further
applying E-row transformations on the coefficient matrix. The rank of the coefficient
matrix A is 3 i. e., equal to the number of unknowns. Therefore the given system of
equations does not possess any linearly independent solution. The zero solution i. e.,
x = y = z = 0 is the only solution of the given system of equations.
Problem 2: Find all the solutions of the following system of linear homogeneous equations :
x + y − 3z + 2w = 0, 2 x − y + 2z − 3w = 0, 3 x − 2 y + z − 4w = 0,
− 4 x + y − 3z + w = 0 .
Solution: The given system of equations is equivalent to the single matrix equation
 1 1 −3 2  x 
 2 −1 2 −3  y 
A X=     = O.
 3 −2 1 −4  z 
 −4 1 −3 1  w 

V-78

We shall reduce the coefficient matrix A to Echelon form by applying only E-row
operations on it. Performing R2 → R2 − 2R 1, R 3→ R3 − 3R 1, R4 → R4 + 4R4 , we
have
 1 1 −3 2  x 
0 −3 8 −7  y 
    = O.
0 −5 10 −10   z 
0 5 9 9  w 

1
Performing R3 → − R3 , we have
5
 1 1 −3 2  x 
0 −3 8 −7  y 
    = O.
0 1 −2 2  z 
0 5 9 9  w 

Performing R2 ↔ R1, we have
 1 1 −3 2  x 
0 1 −2 2  y 
    O.
0 −3 8 −7  z 
0 5 9 9  w 

Performing R3 → R3`+ 3R2 , R4 → R4 − 5R2 , we have
 1 1 −3 2  x 
0 1 −2 2  y 
    = O.
0 0 2 −1  z 
0 0 19 −1  w 
  
Here we find that the determinant of the matrix on left hand side of this equation is not
equal to zero. Therefore the rank of this matrix is 4. So there is no need of further
applying E-row transformations on the coefficient matrix.The rank of the coefficient
matrix A is 4, i. e., equal to the number of unknowns. Therefore the given system of
equations does not possess any linearly independent solution. The zero solution i. e.,
x = y = z = w = 0 is the only solution of the given system of equations.
Problem 3: Find all the solutions of the following system of linear homogeneous equations :
x + y + z = 0, 2 x + 5 y + 7z = 0, 2 x − 5 y + 3z = 0 .
Solution: The given system of equations is equivalent to the single matrix equation
 1 1 1  x 
A X = 2 5 7  y  = O.
  
2 −5 3  z 
We shall first find the rank of the coefficient matrix A by reducing it to Echelon form by
applying elementary row transformations only.
V-79

Applying R2 → R2 − 2R 1, R3 → R3 − 2R 1, we get
 1 1 1  x 
0 3 5  y  = O.
  
0 −7 1  z 
Here we find that the determinant of the matrix on left hand side of this equation is not
equal to zero. Therefore the rank of this matrix is 3. So there is no need of further
applying E-row transformations on the coefficient matrix. The rank of the coefficient
matrix A is 3, i. e., equal to the number of unknowns. Therefore the given system of
equations does not possess any linearly independent solution. The zero solution i. e.,
x = y = z = 0 is the only solution of the given system of equations.
Problem 4: Find all the solutions of the following system of linear homogeneous equations :
x + 2 y + 3z = 0, 2 x + 3 y + 4z = 0, 7 x + 13 y + 19z = 0.
Solution: The given system of equations is equivalent to the single matrix equation
 1 2 3  x 
AX = 2 3 4  y  = O.
  
7 13 19  z 
We shall first find the rank of the coefficient matrix A by reducing it to Echelon form by
applying E-row transformations only.
Applying R2 → R2 − 2R 1 , R3 → R3 − 7R 1, we get
 1 2 3  1 2 3
A ~ 0 −1 −2 ~ 0 −1 −2 by R3 → R3 − R2
   
0 −1 −2 0 0 0 
The rank of A is 2 which is less than the number of unknowns 3. Therefore the given
system of equations possesses 3 − 2 i. e., 1 linearly independent solution. The given
system of equations is equivalent to the equation
 1 2 3  x  0 
0 −1 −2  y  = 0 
    
0 0 0   z  0 
Thus the given system of three equations is equivalent to the system of one equation
x + 2 y + 3z = 0 
i.e., 
− y − 2z = 0 
From these equations, we get y = − 2z , x = − 2 y − 2z = 2z
Hence x = 2c, y = − 2c, z = c constitute the general solution of the given system of
equations, where c is arbitrary number.
Problem 5: Find all the solutions of the following system of linear homogeneous equations :
4 x + 2 y + z + 3u = 0, 6 x + 3 y + 4z + 7u = 0, 2 x + y + u = 0.
V-80

 x
4 2 1 3  
y
Solution : The matrix form of the given system is 6 3 4 7   = O
  z 
2 1 0 1  
 u
 
z
1 2 4 3   
  y
or 4 3 6 7   = O, interchanging the variables x and z.
   x
0 1 2 1  
 u
Performing R2 → R2 − 4R 1, we get
z 
1 2 4 3 
0 −5 −10 −5  y  = O.
   x
0 1 2 1   
 u
1
Performing R2 → − R2 , we get
5
z 
1 2 4 3   y
0 1 2 1   = O.
   x
0 1 2 1  u
 
Performing R3 → R3 − R2 , we get
z 
1 2 4 3   y
0 1 2 1    = O.
  x
0 0 0 0  u
 
The coefficient matrix is of rank 2 and therefore the given system will have 4 − 2 i.e., 2
linearly independent solutions. The given system of equations is equivalent to
z + 2 y + 4 x + 3u = 0, y + 2 x + u = 0 .
∴ y = − 2 x − u , z = − 4 x − 3u + 4 x + 2u = − u.
∴ x = c1 , u = c2 , y = − 2c1 − c2, z = − c2
constitute the general solution where c1 and c2 are arbitrary constants.
Problem 6: Find all the solutions of the following system of linear homogeneous equations :
2 x − 2 y + 5z + 3w = 0, 4 x − y + z + w = 0, 3 x − 2 y + 3z + 4w = 0,
x − 3 y + 7z + 6w = 0.
Solution: The given system of equations is equivalent to the single matrix equation
2 −2 5 3  x
4 −1 1 1  y
AX =     = O.
3 −2 3 4 z 
1 −3 7 6 w
  
V-81

We shall first find the rank of the coefficient matrix A by reducing it to Echelon form by
applying elementary row transformations only. Applying R1 ↔ R4 , we get
1 −3 7 6 1 −3 7 6 
4 −1 1 1 0 11 −27 −23
A ~  ~ ,
3 −2 3 4 0 7 −18 −14
2 −2 5 3 0 4 −9 −9 
  
by R2 → R2 − 4R 1, R3 → R3 − 3R 1 , R4 → R4 − 2R 1
1 − 3 7 6 
0 4 −9 −9 
~  , by R2 → R2 − R3
0 7 −18 −14
0 4 −9 −9 

 1 − 3 7 6 
0 4 −9 −9 
~  , by R3 → 4R3
0 28 −72 −56
0 4 −9 −9 

1 − 3 7 7 
0 4 −9 −9
~  , by R3 → R3 − 7R2 , R4 → R4 − R2 .
0 0 −9 7 
0 0 0 0 
 
Above is the Echelon form of the coefficient matrix A. We have rank A = the number of
non-zero rows in this Echelon form = 3. The number of unknowns is 4. Since rank A is
less than the number of unknowns, therefore the given system of equations possesses
non-zero solutions. The given system of equations will have 4 − 3 = 1 linearly
independent solution. We shall assign arbitrary values to n − r = 4 − 3 = 1 variable and
the remaining r = 3 variables shall be found in terms of these. The given system of
equations is equivalent to
1 −3 7 6   x  0 
0 4 −9 −9  y  0 
    =  .
0 0 −9 7   z  0 
0 0 0 0   w  0 
    
This gives the equations
x − 3 y + 7z + 6w = 0, 4 y − 9z − 9w = 0, − 9z + 7w = 0.
From these, we get
7 9 9 9 7 9 7 9
z= w, y = z + w = ( w ) + w = w + w = 4w,
9 4 4 4 9 4 4 4
7
x = 3 y − 7z − 6w = 3 (4w ) − 3 ( w ) − 6w
9
49 49 5
= 12w − w − 6w = 6w − w = w.
9 9 9
V-82

5 7
Taking w = c , we see that x = c , y = 4c , z = c , w = c constitute the general
9 9
solution of the given system, where c is an arbitrary number. Since we can give any
arbitrary values to c, therefore the given system of equations has an infinite number of
solutions.
Problem 7: Find all the solutions of the following system of linear homogeneous equations :
x − 2 y + z − w = 0, x + y − 2z + 3w = 0, 4 x + y − 5z + 8w = 0,
5 x − 7 y + 2z − w = 0 .
Solution: The given system of equations is equivalent to the single matrix equation
 1 −2 1 −1  x 
 1 1 −2 3  y 
A X=     = O.
4 1 −5 8  z 
5 −7 2 −1  w 
  
We shall first find the rank of the coefficient matrix A by reducing it to Echelon form by
applying E-row transformations only.
Applying R2 → R2 − R 1 , R3 → R3 − 4R 1, R4 → R4 − 5R 1, we get
 1 −2 1 −1
0 3 −3 4
A~  
0 9 −9 12
0 3 −3 4

 1 −2 1 −1
0 3 −3 4
~  by R3 → R3 − 3R2 , R4 → R4 − R2
0 0 0 0 
0 0 0 0 
 
The rank of A is obviously 2 which is less than the number of unknowns 4. Therefore the
given system of equations possesses 4 – 2, i.e., 2 linearly independent solutions. The
given system of equations is equivalent to the equation
 1 −2 1 −1  x  0 
0 3 −3 4  y  0 
  = 
0 0 0 0   z  0 
0 0 0 0   w  0 
    
Thus the given system of four equations is equivalent to the system of two equations, i. e.,
x − 2 y + z − w = 0, 3 y − 3z + 4w = 0
From these equations, we get
4 8 5
y = z − w , x = 2 y − z + w = 2z − w − z + w = z − w
3 3 3
5 4
Here x = c1 − c2, y = c1 − c2 , z = c1, w = c2 .
3 3
constitute the general solution of the given system of equations, where c1 and c2 are
arbitrary numbers.
V-83

Comprehensive Problems 2
Problem 1: Use the test of rank to show that the following equations are not consistent :
2 x − y + z = 4, 3 x − y + z = 6, 4 x − y + 2z = 7, − x + y − z = 9.
(Rohilkhand 2009; Kumaun 11)

Solution: The given system of equations is equivalent to the single matrix equation
 2 −1 1 4
 3 −1 1  x  6
AX =    y =   = B .
 4 −1 2   7
 −1 1 −1  z  9
   
The augmented matrix
 2 −1 1 : 4  −1 1 −1 : 9
 3 −1 1 : 6  3 −1 1 : 6
[ A : B] =  ~  by R1 ↔ R4
 4 −1 2 : 7  4 −1 2 : 7
 −1 1 −1 : 9  2 −1 −1 : 4

 −1 1 −1 : 9
 0 2 −2 : 33
~ 
 0 3 −2 : 43
 0 1 −1 : 22

by R2 → R2 + 3R1, R3 → R4 + 4R1, R4 → R4 + 2R1
 −1 1 −1 : 9
 0 1 −1 : 22
~  by R2 ↔ R4
 0 3 −2 : 43
 0 2 −2 : 33

 −1 1 −1 : 9
 0 1 1 : 22
~  by R3 → R3 − 3R2 , R4 → R4 − 2R2
 0 0 1 : −23
 0 0 0 : −11

which is in Echelon form.
We have rank [ A : B ] = 4 and rank A = 3.
Since rank [A :B] ≠ rank A, therefore the given equations are inconsistent.
Problem 2 : Show that the equations −2 x + y + z = a, x − 2 y + z = b, x + y − 2z = c have
no solution unless a + b + c ≠ 0 (Kanpur 2009)
Solution : The given equations are equivalent to the single matrix equation
 −2 1 1  x   a
A X =  1 −2 1  y  =  b  = B
    
 1 1 −2  z   c 
V-84

The augmented matrix


 −2 1 1 : a  1 1 − 2 : c 
[ A : B ] =  1 −2 1 : b  ~  1 −2 1 : b  , by R1 ↔ R3
   
 1 1 −2 : c   −2 1 1 : a

 1 1 −2 : c 
~ 0 −3 3 : b − c  , by R2 → R2 − R1, R3 → R3 + 2R1
 
0 3 −3 : a + 2c 

 1 1 −2 : c 
~ 0 −3 3 : b−c 
 
0 0 0 : a + b + c 
If a + b + c ≠ 0, then rank [ A: B] = 3 ≠ rank A. Hence the given system of equations is
inconsistent i. e., possesses no solution.
Problem 3: Apply the test of rank to examine if the following system of equations is consistent and if
consistent, find the complete solution : x + y + z = 6, x + 2 y + 3z = 10, x + 2 y + 4z = 1.
Solution: The given system of equations is equivalent to the single matrix equation.
1 1 1  6 
A X = 1 2 3 = 10  = B
   
1 2 4  1 
The augmented matrix
1 1 1 : 6
[ A: B] = 1 2 3 : 10 
 
1 2 4 : 1
 1 1 1 : 6
~ 0 1 2 : 4 by R2 → R2 − R 1, R3 → R3 − R 1
 
0 1 3 : −5
 1 1 1 : 6
~ 0 1 2 : 4 by R3 → R3 − R2
 
0 0 1 : −9
which is in Echelon form.
∴ rank [ A : B] = the number of non-zero rows in its Echelon form = 3
1 1 1 
Also A ~ 0 1 2 so that rank A = 3
 
0 0 1
Since rank A = rank B, therefore the given equations are consistent. Here the number of
unknowns is 3. Since rank A is equal to the number of unknowns, therefore the given
equations have a unique solution.
The given equations are equivalent to the equations x + y + z = 6 , y + 2z = 4 , z = − 9
These give z = − 9 , y = 22 , x = − 7. Hence x = − 7 , y = 22 , z = − 9.
V-85

Problem 4: Use matrix method to solve the equations :


2 x − y + 3z = 9, x + y + z = 6, x − y + z = 2.
(Meerut 2005; Avadh 06, 11; Garhwal 12)
Solution: The given system of equations is equivalent to the single matrix equation
2 −1 3  x  9
AX =  1 1 1  y  = 6 = B.
    
 1 −1 1  z  2
The augmented matrix
2 −1 3 ⋮ 9  1 1 1 ⋮ 6
[A B] =  1 1 1 ⋮ 6 ~ 2 −1 3 ⋮ 9 , by R1 ↔ R2
   
 1 −1 1 ⋮ 2  1 −1 1 ⋮ 2
 1 1 1 ⋮ 6
~ 0 −3 1 ⋮ −3 , by R2 → R2 − 2R1 , R3 → R3 − R 1
 
0 −2 0 ⋮ −4
1 1 1 ⋮ 6

~ 0 −3 1 ⋮ −3 , by R3 → R3 − R2
2
  3
0 0 −2 / 3 ⋮ −2
which is in Echelon form.
∴ rank [A B] = the number of non-zero rows in its Echelon form = 3.
By the same elementary transformations, we get
1 1 1 
A = 0 −3 1 ,
 
0 0 −2 / 3
which is in Echelon form. Obviously rank A = 3.
Since rank A = rank [A B], therefore the given equations are consistent. Here the
number of unknowns is 3. Since rank A is equal to the number of unknowns, therefore
the given equations have a unique solution.
The given equations are equivalent to the equations
2
x + y + z = 6 , − 3 y + z = − 3 , − z = − 2.
3
These give z = 3 , y = 2 , x = 1. Hence x = 1 , y = 2 , z = 3 is the required solution.
Problem 5: Solve completely the equations
x + 2 y + 3z = 6, 2 x + 4 y + z = 7, 3 x + 2 y + 9z = 4. (Kumaun 2014)
Solution: The given equations are equivalent to the single matrix equation
 1 2 3  x  6
AX = 2 4 1  y  = 7 = B.
    
3 2 9  z  4
The augmented matrix
 1 2 3 ⋮ 6
[A B] = 2 4 1 ⋮ 7
 
3 2 9 ⋮ 4
V-86

 1 2 3 ⋮ 6
~ 0 0 −5 ⋮ −5 , by R2 → R2 − 2R 1 , R3 → R3 − 3R 1
 
0 −4 0 ⋮ −4
 1 2 3 ⋮ 6
~ 0 0 1 ⋮ 1 , by R2 → − R2 , R3 → − R3
1 1
  5 4
0 1 0 ⋮ 1
 1 2 3 ⋮ 6
~ 0 1 0 ⋮ 1 , by R2 ↔ R3
 
0 0 1 ⋮ 1
which is in Echelon form.
∴ rank [A B] = the number of non-zero rows in its Echelon form = 3.
 1 2 3
Also A ~ 0 1 0  so that rank A = 3.
 
 0 0 1
Since rank A = rank [A B], the given equations are consistent. Also rank A = 3 = the
number of unknowns x, y, z. So the given equations have a unique number. The given
equations are equivalent to the equations
x + 2 y + 3z = 6 , y = 1 , z = 1.
These give x = 1 , y = 1 , z = 1.
The give equations are consistent and have a unique solution x = y = z = 1.
Problem 6: Show that the equations x + 2 y − 5z = − 9, 3 x − y + 2z = 5, 2 x + 3 y − z = 3,
4 x − 5 y + z = − 3 are consistent and solve the same.
Solution: The given system of equations is equivalent to the single matrix equation
 1 2 −5   −9 
 3 −1 2   x   5 
AX =    y =   = B.
 2 3 −1     3 
 
z
 4 −5
 1     −3 

 1 2 −5 ⋮ −9 
 3 −1 2 ⋮ 5 
The augmented matrix [ A B] =  .
 2 3 −1 ⋮ 3 
 4 −5 1 ⋮ −3 
 
We shall reduce the augmented matrix [A B] to triangular form by applying E-row
transformations only. Performing R2 → R2 − 3R 1, R3 → R3 − 2R 1 , R4 → R4 − 4R 1 ,
we get
1 2 −5 ⋮ −9  1 2 −5 ⋮ −9
0 −7 17 ⋮ 32  0 −1 9 ⋮ 21
[ A B] ~  ~  , applying R2 ↔ R3
0 −1 9 ⋮ 21 0 −7 17 ⋮ 32
0 −13 21 ⋮ 33 0 −13 21 ⋮ 33
  
V-87

1 2 −5 ⋮ −9
0 −1 9 ⋮ 21
~  , applying R3 → R3 − 7R2 , R4 → R4 − 13 R2
0 0 −46 ⋮ −115
0 0 −96 ⋮ −224

 1 2 −5 ⋮ −9
0 −1 9 ⋮ 21 1 1
~  , applying R3 → R3 , R4 → R4
0 0 −2 ⋮ −5 23 48
0 0 −2 ⋮ −5
 
 1 2 −5 ⋮ −9
0 −1 9 ⋮ 21
~  , applying R4 → R4 − R3 .
0 0 −2 ⋮ −5
0 0 0 ⋮ 0 
 
Thus the matrix [A B] has been reduced to Echelon form. We have rank [A B] = the
number of non-zero rows in this Echelon form = 3.
Also by the same E-row transformations, we get
 1 2 −5
0 −1 9
A ~ .
0 0 −2
0 0 0 
 
The rank of A is also 3.
Since rank A = rank [A B], therefore the given equations are consistent. Also rank A =
3 and the number of unknowns is also 3. Hence the given equations will have a unique
solution. To find the solution, we see that the given system of equations is equivalent to
the matrix equation
1 2 − 5   −9
0 −1 9   x   21
   y =   .
0 0 −2    −5
0 0 0   z   0 
   
This matrix equation is equivalent to the system of equations
x + 2 y − 5z = − 9 , − y + 9z = 21 , − 2z = − 5 .
5 3 1
Solving these, we get z = , y = , x = .
2 2 2
Problem 7: Show that the equations
x − 3 y − 8z + 10 = 0, 3 x + y − 4z = 0, 2 x + 5 y + 6z − 13 = 0
are consistent and solve the same. (Bundelkhand 2007)
Solution: The given system of equations is equivalent to the single matrix equation
 1 −3 −8  x   −10 
AX = 3 1 −4  y  =  0  = B.
    
2 5 6  z   13
V-88

The augmented matrix


 1 −3 −8 ⋮ −10 
[A B] = 3 1 −4 ⋮ 0
 
2 5 6 ⋮ 13
 1 −3 −8 ⋮ −10 
~ 0 10 20 ⋮ 30  , by R2 → R2 − 3R 1 , R3 → R3 − 2R1
 
0 11 22 ⋮ 33
 1 −3 −8 ⋮ −10 
~ 0 3 , by R2 →
1 1
1 2 ⋮ R2 , R3 → R3
  10 11
0 1 2 ⋮ 3
 1 −3 −8 ⋮ −10 
~ 0 1 2 ⋮ 3 , by R3 → R3 − R2
 
0 0 0 ⋮ 0 
which is in Echelon form.
∴ rank [A B] = the number of non-zero rows in its Echelon form = 2.
 1 −3 −8
Also A ~ 0 1 2 so that rank A = 2.
 
 0 0 0 
Since rank A = rank [A B], therefore the given equations are consistent.
Also rank A = 2 which is less than the number of unknowns x, y, z. Hence the given
equations have an infinite number of solutions. The given equations are equivalent to
the equations
x − 3 y − 8z = − 10 
.
y + 2z = 3
∴ y = 3 − 2 z , x = − 10 + 3 y + 8z = − 10 + 9 − 6z + 8z = 2z − 1.
Taking z = c, we see that x = 2c − 1 , y = 3 − 2c , z = c constitute the general solution of
the given system, where c is an arbitrary constant.
Problem 8: Solve completely the equations
2 x + 3 y + z = 9, x + 2 y + 3z = 6, 3 x + y + 2z = 8.
Solution: The given system of equations is equivalent to the single matrix equation
2 3 1  x  9
A X = 1 2 3  y  = 6 = B .
    
3 1 2  z  8
The augmented matrix
2 3 1 : 9
[ A : B ] =  1 2 3 : 6
 
3 1 2 : 8
 1 2 3 : 6
~ 2 3 1 : 9 , by R 1 ↔ R2
 
3 1 2 : 8
V-89

1 2 3 : 6
~ 0 −1 −5 : −3 , by R2 → R2 − 2R 1, R3 → R3 − 3R 1
 
0 −5 −7 : −10 
1 2 3 : 6
~ 0 −1 −5 : −3 , by R3 → R3 − 5R2
 
0 0 18 : 5
which is in Echelon form.
∴ rank [ A : B] = the number of non-zero rows in its Echelon form = 3.
 1 2 3
Also A~ 0 −1 −5
 
0 0 18
so that rank A = 3.
Since rank A = rank [ A : B] , the given equations are consistent. Also rank A = 3 = the
number of unknowns x, y, z. So the given equations have a unique solution. The given
equations are equivalent to the equations
x + 2 y + 3z = 6 , − y − 5z = − 3, 18z = 5.
5 29 35
These give z= , y= ,x= .
18 18 18
Hence the given equations are consistent and have a unique solution
35 29 5
x= , y= ,z = .
18 18 18
Problem 9 : Solve completely the equations
x + y + z = 1, x + 2 y + 3z = 4, x + 3 y + 5z = 7, x + 4 y + 7z = 10.
(Lucknow 2009)
Solution : The given system of equations is equivalent to the single matrix equation.
1 1 1  1
1 2 3  x   4
A X=    y =   = B
1 3 5    7
1 4 7  z  10 
   
The augmented matrix
1 1 1 : 1
1 2 3 : 4
[ A : B] =  
1 3 5 : 7
1 4 7 : 10 

1 1 1 : 1
0 1 2 : 3
~  , by R2 → R2 − R 1 , R3 → R3 − R2 , R4 → R4 − R 1
0 2 4 : 6
0 3 6 : 9

V-90

1 1 1 : 1
0 1 2 : 3 1 1
~  , by R3 → R3, R4 → R4
0 1 2 : 3 2 3
0 1 2 : 3

1 1 1 : 1
0 1 2 : 3
~  , by R3 → R3 − R2 , R4 → R4 − R2
0 0 0 : 0
0 0 0 : 0 

which is in Echelon form.
∴ rank [ A B] = the number of non-zero rows in its Echelon form = 2.
 1 1 1
0 1 2
Also A~   so rank A = 2.
0 0 0 
0 0 0 
 
Since rank A = rank [ A B] , therefore the given equations are consistent.
Also rank A = 2 which is lass than the number of unknowns x, y, z. Hence the given
equations have an infinite number of solutions. The given equations are equivalent to
the equations
x + y + z = 1 , y + 2z = 3
∴ y = 3 − 2z , x = 1 − y − z = 1 − (3 − 2z ) − z = − 2 + z
Taking z = c, we see that x = − 2 + c , y = 3 − 2c , z = c, constitute the general solution of
the given matrix system, where c is an arbitrary constant.
Problem 10: Express the following system of equations into the matrix equation form AX = B :
x + 2 y + z = − 1, 6 x + y + z = − 4, 2 x − 3 y − z = 0 , − x − 7 y − 2z = 7, x − y = 1.
Determine if this system of equations is consistent and if so find its solution.
Solution: The given equations are equivalent to the single matrix equation
 1 2 1  −1
 6 
1 1    −4
x
   
AX =  2 −3 −1  y  =  0  = B .
 
 −1 −7 −2  z   7
    
 1 −1 0   1
The augmented matrix
 1 2 1 ⋮ −1
 6 1 1 ⋮ −4
 
[A B] =  2 −3 −1 ⋮ 0
 −1 −7 −2 ⋮ 7
 
 1 −1 0 ⋮ 1
V-91

1 2 1 ⋮ −1
0 −11 −5 ⋮ 2
  by R → R − 6R , R → R − 2R ,
~ 0 −7 −3 ⋮ 2 , 2 2 1 3 3 1
0 R → R + R , R → R −`R
−5 −1 ⋮ 6 4 4 1 5 5 1
 
0 −3 −1 ⋮ 2
1 2 1 ⋮ −1
0 −1 −3 ⋮ −10 
 
~ 0 −7 −3 ⋮ 2 , by R2 → R2 − 2 R4
0 −5 −1 ⋮ 6
 
0 −3 −1 ⋮ 2
1 2 1 ⋮ −1
0 −1 −3 ⋮ −10 
 
~ 0 0 18 ⋮ 72 ,
0 0 14 ⋮ 56
 
0 0 8 ⋮ 32
by R3 → R3 − 7R2 , R4 → R4 − 5R2 , R5 →`R5 − 3R2
1 2 1 ⋮ −1
0 −1 −3 ⋮ −10 
  1 1 1
~ 0 0 1 ⋮ 4 , by R3 → R3 , R4 → R4 , R5 → R5
0 0 18 14 8
1 ⋮ 4
 
0 0 1 ⋮ 4
1 2 1 ⋮ −1
0 −1 −3 ⋮ −10 
 
~ 0 0 1 ⋮ 4 , by R4 → R4 − R3 , R5 → R5 − R3
0 0 0 ⋮ 0
 
0 0 0 ⋮ 0 
which is in Echelon form.
∴ rank [A B] = the number of non-zero rows in its Echelon form = 3.
1 2 1
0 −1 −3
 
Also A ~ 0 0 1 so that rank A = 3.
0 0 0 
 
0 0 0 
Since rank A = rank [A B], the given equations are consistent. Also rank A = 3 = the
number of unknowns x, y, z. So the given equations have a unique solution. The given
equations are equivalent to the equations
x + 2 y + z = − 1 , − y − 3z = − 10 , z = 4.
These give z = 4 , y = − 2 , x = − 1.
Hence the given equations are consistent and have a unique solution
x = − 1 , y = − 2 , z = 4.
V-92

Problem 11: Examine if the system of equations :


x + y + 4z = 6, 3 x + 2 y − 2z = 9, 5 x + y + 2z = 13
is consistent. Find also the solution if it is consistent.
Solution: The given equations are equivalent to
 1 1 4  x   6
AX = 3 2 −2  y  =  9 = B.
    
5 1 2  z  13
The augmented matrix
1 1 4 ⋮ 6
[A B] = 3 2 −2 ⋮ 9
 
5 1 2 ⋮ 13
1 1 4 ⋮ 6
~ 0 −1 −14 ⋮ −9 , by R2 → R2 − 3R 1 , R3 → R3 − 5R 1
 
0 −4 −18 ⋮ −17
1 1 4 ⋮ 6

~ 0 −1 −14 ⋮ −9 , by R3 → R3 − 4R2
 
0 0 38 ⋮ 19
which is in Echelon form.
∴ rank [A B] = the number of non-zero rows in its Echelon form = 3.
1 1 4
Also A ~ 0 −1 −14 so that rank A = 3.
 
 0 0 38
Since rank A = rank [A B], the given equations are consistent. Also rank A = 3 = the
number of unknowns x, y, z. So the given equations have a unique solution. The given
equations are equivalent to the equations
x + y + 4z = 6, − y − 14z = − 9 , 38z = 19.
1
These give z = , y = 2 , x = 2.
2
Hence the given equations are consistent and have a unique solution
x = 2 , y = 2 , z = 1 / 2.
Problem 12: Prove, without actually solving, that the following system of equations has a unique
solution :
5 x + 3 y + 14z = 4, y + 2z = 1, x − y + 2z = 0.
Solution: The given system of equations is equivalent to the single matrix equation
 5 3 14  x   4
AX = 0 1 2  y  =  1 = B.
    
 1 −1 2  z  0 
Here we have three equations in three unknowns x , y and z. These equations will have a
unique solution if and only if the coefficient matrix A is non-singular i. e., if and only if
|A| ≠ 0.
V-93

 5 3 14
We have |A| = 0 1 2 = 5 (2 + 2) + 0 + 1 (6 − 14) = 20 − 8 = 12 ≠ 0.
 
 1 −1 2
Hence the given equations have a unique solution.
Problem 13: For what values of the parameter λ will the following equations fail to have a unique
solution 3 x − y + λz = 1, 2 x + y + z = 2, x + 2 y − λz = − 1?
Will the equations have any solutions for these values of λ ? (Garhwal 2008)
Solution: The matrix form of the given system of equations is
3 −1 λ   x   1
2 1 1  y  =  2 .
    
 1 2 −λ   z   −1
The given system of equations will have a unique solution if and only if the coefficient
matrix is non-singular.
Performing R 1 ↔ R3 , we get
 1 2 − λ   x   −1
2 1 1  y  =  2 .
    
3 −1 λ   z   1
Performing R2 → R2 − 2R 1 , R3 → R3 − 3R 1, we get
1 2 − λ   x   −1
0 −3 1 + 2λ   y  =  4 ...(1)
    
0 −7 4λ   z   4
Therefore the coefficient matrix will be non-singular if and only if
− 12 λ + 7 + 14 λ ≠ 0
7
i. e., if and only if λ ≠ − .
2
7
Thus the given system will have a unique solution if λ ≠ − .
2
7
In case λ = − , the equation (1) becomes
2
1 2 7
−1
2    
 x
0 −3 −6  y  =  4 .
    
0 −7 −14  z   4
 
7
Performing R3 → R3 − `R2 , we get
3
 
 1 2 7 / 2  x   −1
0 −3   
−6 y =  4 ,
  
0   z   − 
16
0 0
 3 
showing that the given equations are inconsistent in this case.
7
Thus if λ = − , no solution exists.
2
V-94

Problem 14: Solve the equations

λx + 2 y − 2z − 1 = 0, 4 x + 2λy − z − 2 = 0, 6 x + 6 y + λz − 3 = 0,
considering specially the case when λ = 2.
Solution: The matrix form of the given system is
 λ 2 −2  x   1
 4 2λ −1  y  = 2 .
    
 6 6 λ   z  3 ...(1)
The given system of equations will have a unique solution if and only if the coefficient
matrix is non-singular, i. e., iff
 λ 2 −2
 4 2λ −1 ≠ 0
 
 6 6 λ 
i. e., iff λ3 + 11λ − 30 ≠ 0 i. e., iff ( λ − 2) ( λ2 + 2λ + 15) ≠ 0.
Now the only real root of the equation
( λ − 2) ( λ2 + 2λ + 15) = 0 is λ = 2.
Therefore if λ ≠ 2 , the given system of equations will have a unique solution given by
x y z 1
= = =
 1 2 −2 λ 1 −2 λ 2 1 λ 2 −2
2 2 λ −1  4 2 −1  4 2 λ 2  4 2 λ −1
       
3 6 λ   6 3 λ   6 6 3  6 6 λ 
In case λ = 2, the equation (1) becomes
2 2 −2  x   1
4 4 −1  y  = 2 .
    
6 6 2  z  3
Performing R2 → R2 − 2R 1, R3 → R3 − 3R 1 , we get
2 2 −2  x   1
0 0 3  y  = 0  .
    
0 0 8  z  0 
The above system of equations is equivalent to
8z = 0 , 3z = 0 , 2 x + 2 y − 2z = 1.
1
∴ x= − c, y = c,z =0
2
constitute the general solution of the given system of equations in case λ = 2.
Problem 15: Discuss for all values of λ, the system of equations
x + y + 4z = 6, x + 2 y − 2z = 6, λx + y + z = 6,
as regards existence and nature of solutions.
V-95

Solution: The matrix form of the given system is


 1 1 4  x  6
 1 2 −2  y  = 6 .
    
 λ 1 1  z  6
The given system of equations will have a unique solution if and only if the coefficient
matrix is non-singular.
Performing R2 → R2 − R 1 , R3 → R3 − λR 1 , we get
1 1 4   x  6 
0 1 −6   y  =  0  .
    
0 1 − λ 1 − 4λ   z  6 − 6λ  ...(1)
Therefore the coefficient matrix will be non-singular if and only if
1 − 4λ + 6 − 6λ ≠ 0 i. e., λ ≠ 7 / 10
Thus the given system will have a unique solution if λ ≠ 7 / 10.
In case λ = 7 / 10, the equation (1) becomes
1 1 4  x   6
   
0 1 −6  y  =  0  .
0 3 − 18  z   18 
 
 10 10  10 
3
Performing R3 → R3 − R2 , we get
10
 
 1 1 4  x   6
0 1 −6  y  =  0  ,
    18 
0 0 0   z 
10 
showing that the equations are not consistent in this case.
Problem 16: Solve the following equations by matrix method :
5 x + 3 y + 7z = 4, 3 x + 26 y + 2z = 9, 7 x + 2 y + 10 z = 5.
Solution: The given system of equations is equivalent to the single matrix equation
5 3 7  x  4
AX = 3 26 2  y  = 9 = B .
    
7 2 10   z  5
The augmented matrix
5 3 7 ⋮ 4  −1 −49 3 ⋮ −14
[A B] = 3 26 2 ⋮ 9 ~  3 26 2 ⋮ 9 , by R 1 → R 1 − 2R2
   
7 2 10 ⋮ 5  7 2 10 ⋮ 5
 −1 −49 3 ⋮ −14
~  0 −121 11 ⋮ −33 , by R2 → R2 + 3R 1 , R3 → R3 + 7R 1
 
 0 −341 31 ⋮ −93
 −1 −49 3 ⋮ −14
~ 0 3 , by R2 → − R2 , R3 → −
1 1
11 −1 ⋮ R3
  11 31
 0 11 −1 ⋮ 3
V-96

 −1 −49 3 ⋮ −14
~ 0 11 −1 ⋮ 3 , by R3 → R3 − R2
 
 0 0 0 ⋮ 0 
which is in Echelon form.
∴ rank [A B] = the number of non-zero rows in its Echelon form = 2.
 −1 −49 3
Also A ~ 0 11 −1 so that rank A = 2.
 
 0 0 0 
Since rank A = rank [A B], therefore the given equations are consistent.
Also rank A = 2 which is less than the number of unknowns x, y, z. Hence the given
equations have an infinite number of solutions. The given equations are equivalent to
the equations
− x − 49 y + 3z = − 14
.
11 y − z = 3
3 1 147 49 7 16
∴ y= + z , x = − 49 y + 3z + 14 = − − z + 3z + 14 = − z.
11 11 11 11 11 11
Taking z = c, we see that the solution of the given equations is given by
7 16 3 1
x= − c, y= + c , z = c,
11 11 11 11
where c is a parameter i.e., c is an arbitrary constant.
Problem 17: Solve the following system of linear equations by matrix method :
5 x − 6 y + 4z = 15, 7 x + 4 y − 3z = 19, 2 x + y + 6z = 46 .
Solution: The given system of equations is equivalent to the single matrix equation
 −6 5 4  y  15
AX =  4 7 −3  x  = 19 = B. [Note]
    
 1 2 6     
z 46
The augmented matrix
 −6 5 4 ⋮ 15  1 2 6 ⋮ 46
[A B] =  4 7 −3 ⋮ 19 ~  4 7 −3 ⋮ 19 , by R1 ↔ R3
   
 1 2 6 ⋮ 46  −6 5 4 ⋮ 15
1 2 6 ⋮ 46
~ 0 −1 −27 ⋮ −165 , by R2 → R2 − 4R 1 , R3 → R3 + 6R 1
 
0 17 40 ⋮ 291
1 2 6 ⋮ 46

~ 0 −1 −27 ⋮ −165 , by R3 → R3 + 17R2
 
0 0 −419 ⋮ −2514
which is in Echelon form.
∴ rank [A B] = the number of non-zero rows in its Echelon form = 3.
1 2 6
Also A ~ 0 −1 −27 so that rank A = 3.
 
 0 0 −419
V-97

Since rank A = rank [A B], therefore the given equations are consistent.
Also rank A = 3 = the number of unknowns x, y, z. So the given equations have a unique
solution.
The given equations are equivalent to the equations
y + 2 x + 6z = 46 , − x − 27z = − 165 , − 419z = − 2514.
These give z = 6 , x = 3 , y = 4.
Hence x = 3 , y = 4 , z = 6 is the required solution.
Problem 18: Solve the following equations by matrix method :
x − y + 2z = 4, 3 x + y + 4z = 6, x + y + z = 1. (Garhwal 2014)

Solution: The given system of equations is equivalent to the single matrix equation
 1 −1 2  x  4
AX = 3 1 4  y  = 6 = B.
    
 1 1 1  z   1

The augmented matrix


 1 −1 2 ⋮ 4
[A B] = 3 1 4 ⋮ 6
 
 1 1 1 ⋮ 1
 1 −1 2 ⋮ 4
~ 0 4 −2 ⋮ −6 , by R2 → R2 − 3R 1 , R3 → R3 − R 1
 
0 2 −1 ⋮ −3
 1 −1 2 ⋮ 4
~ 0 2 −1 ⋮ −3 , by R2 → R2
1
  2
0 2 −1 ⋮ −3
 1 −1 2 ⋮ 4
~ 0 2 −1 ⋮ −3 , by R3 → R3 − R2
 
0 0 0 ⋮ 0 

which is in Echelon form.


∴ rank [A B] = the number of non-zero rows in its Echelon form = 2.
 1 −1 2
Also A ~ 0 2 −1 so that rank A = 2.
 
 0 0 0 

Since rank A = rank [A B], therefore the given equations are consistent.
Also rank A = 2 which is less than the number of unknowns x, y, z. Hence the given
equations have an infinite number of solutions.
V-98

The given equations are equivalent to the equations


x − y + 2z = 4
.
2 y − z = − 3
3 1 3 1 5 3
∴ y=− + z , x = y − 2z + 4 = − + z − 2z + 4 = − z .
2 2 2 2 2 2
Taking z = c, we see that the solution of the given equations is given by
5 3 3 1
x= − c , y = − + c , z = c,
2 2 2 2
where c is an arbitrary constant.
Problem 19: Solve the following equations by matrix method :
x − 2 y + 3z = 6, 3 x + y − 4z = − 7, 5 x − 3 y + 2z = 5.
(Meerut 2010; Garhwal 13)

Solution: The given equations are equivalent to the single matrix equation
 1 −2 3  x   6
AX = 3 1 −4  y  =  −7 = B.
    
5 −3 2  z   5

The augmented matrix


 1 −2 3 ⋮ 6
[A B] = 3 −1 −4 ⋮ −7
 
5 −3 2 ⋮ 5
 1 −2 3 ⋮ 6
~ 0 7 −13 ⋮ −25 , by R2 → R2 − 3R 1 , R3 → R3 − 5R 1
 
0 7 −13 ⋮ −25

 1 −2 3 ⋮ 6

~ 0 7 −13 ⋮ −25 , by R3 → R3 − R2
 
0 0 0 ⋮ 0 

which is in Echelon form.


∴ rank [A B] = the number of non-zero rows in its Echelon form = 2.
 1 −2 3
Also A ~ 0 7 −13 so that rank A = 2.
 
 0 0 0 

Since rank A = rank [A B], therefore the given equations are consistent.
Also rank A = 2 which is less than the number of unknowns x, y, z. So the given
equations have an infinite number of solutions.
V-99

The given equations are equivalent to the equations


x − 2 y + 3z = 6
.
7 y − 13z = −25
25 13 50 26 8 5
∴ y=− + z , x = 2 y − 3z + 6 = − + z − 3z + 6 = − + z .
7 7 7 7 7 7
Taking z = c , we see that the solution of the given equations is given by
8 5 25 13
x=− + c, y=− + c , z = c,
7 7 7 7
where c is an arbitrary constant.
V-100

Hints to Objective Type Questions

Multiple Choice Questions


1. (b). See article 2, definition of linearly independent set of vectors.
 a1 b1 c1 
2. (a). The coefficient matrix A =  a2 b2 c2 
 
 a3 b3 c3 

of the given system of linear homogeneous equations is of the type 3 × 3. If


|A| = 0, then rank A < 3 i.e., A < the number of unknowns x, y and z. So, in
this case the given system of equations will have an infinite number of
solutions. See article 16, Case II.
3. (a). The system of linear equations x + y + z = 2, 2 x + y − z = 3,

3 x + 2 y + k z = 4 has a unique solution i and only if


1 1 1
2 1 −1 ≠ 0
3 2 k

1 0 0
i.e., if and only if 2 −1 −3 ≠ 0
3 −1 k − 3

i.e., if and only if ( − k + 3 − 3) ≠ 0


i.e., if and only k ≠ 0.

4. (b). The augmented matrix of the given system of equations is


 1 2 3 ⋮ 1
[A B] = 2 1 3 ⋮ 2
 
5 1 9 ⋮ 4

 1 2 3 ⋮ 1
~ 0 −3 −3 ⋮ 0 ,
 
0 −9 −6 ⋮ −1

by R2 → R2 − 2R 1 , R3 → R3 − 5R 1
V-101

 1 2 3 ⋮ 1
~ 0 −3 −3 ⋮ 0 , by R3 → R3 − 3R2 .
 
0 0 3 ⋮ −1

This is in Echelon form.


∴ rank [A B] = the number of non-zero rows in the Echelon form = 3.
Also, the coefficient matrix
 1 2 3
A ~ 0 −3 −3 .
 
0 0 3

We have rank A = 3.
Since rank [A B] = rank A, therefore the given equations are consistent.
Since rank A = 3 = the number of unknowns x, y and z, therefore the given
equations have a unique solution.

Fill in the Blank(s)


1. See Ex. 2.
2. See Ex. 4.
3. See Theorem after article 15.
4. See article 19 Consistent and Inconsistent equations.
5. See article 20.
6. See Theorem of article 21.
7. See article 22.

True or False
1. T . The given statement is true, See article 2, linearly dependent set of
vectors.
2. F. The given statement is false. If one member of the set can be expressed as
a linear combination of the remaining members, then obviously the vectors
are linearly dependent.
1 2 3 1 0 0
3. T . We have 3 4 4 = 3 −2 −5 = 18 − 20 = −2 ≠ 0.
7 10 12 7 −4 −9

Therefore, the given equations have a unique solution i.e., x = y = z = 0 is the


only solution of the given system of equations.
Hence, the given statement is true.
V-102

4. T . The given statement is true. If the matrices A and [A B] are not of the
same rank, then the given equations are inconsistent i.e., they have no
common solution.

5. F. The given statement is false. If rank [A B] = rank A but is less than the
number of unknowns, then the equations are consistent and have infinite
solutions.

❍❍❍
Krishna's

REAL ANALYSIS
C hapters

1. Continuity

2. Differentiability

3. Sequences
Uniform Convergence of Sequences
4.
and Series of Functions

5. Infinite Series

6. The Riemann Integral

7. Convergence of Improper Integrals


R-3

Chapter-1
Continuity

Comprehensive Problems 1
Problem 1: Discuss the continuity and discontinuity of the following functions :
( i) f ( x ) = x3 − 3 x .

( ii) f ( x ) = x + x − 1.

( iii) f ( x ) = e − 1 / x .
( iv ) f ( x ) = sin x .

(v) f ( x ) = cos   when x ≠ 0; f (0 ) = 0.


1
 x (Lucknow 2005)
( vi) f ( x ) = sin (1 / x ) when x ≠ 0, f (0 ) = 0. (Lucknow 2011)
sin x
( vii) f ( x ) = when x ≠ 0 and f (0 ) = 1 .
x (Kanpur 2007; Avadh 08)
e1 / x − 1
( viii) f ( x ) = when x ≠ 0 and f (0 ) = 1 .
e1 / x + 1 (Meerut 2004B; Kumaun 10)
e1 / x
( ix ) f ( x ) = when x ≠ 0, f (0 ) = 0 .
1 + e1 / x (Bundelkhand 2011; Lucknow 11)
xe1 / x 1
( x) f ( x) = + sin ( ) when x ≠ 0, f (0 ) = 0 .
1 + e1 / x x

( xi) f ( x ) = sin x cos (1 / x ) when x ≠ 0, f (0 ) = 0 .


Solution: (i) Here f ( x ) = x3 − 3 x. The domain f is the whole R. Let c ∈ R.
lim lim lim lim
Then f ( x) = ( x3 − 3 x ) = x3 − 3 x
x→ c x→ c x→ c x→ c
= c3 − 3c = f ( c ).
∴ f ( x ) is continuous at every point c of R and thus f ( x ) is continuous on the whole
real line.
(ii) Here f ( x ) = x + x − 1 = x + (1 / x ).
The function f ( x ) is not defined at x = 0 and is defined at every other real number.
Thus domain f = R − {0 } i. e., R0 .
R-4

Let c ∈ R and c ≠ 0.
lim lim  x + 1  = c + 1 = f ( c ).
We have f ( x) =  
x→ c x→ c  x c
∴ f ( x ) is continuous at every point c of R if c ≠ 0.
lim lim
Now f (0 + 0 ) = f ( x) = f (0 + h), h > 0
x→0 + h→0
lim lim  h + 1 = 0 + ∞ = ∞
= f ( h) =  
h→0 h→0  h
lim lim
and f (0 − 0 ) = f ( x) = f (0 − h), h > 0
x→0 − h→0
lim lim  1
= f ( − h) =  − h −  = − 0 − ∞ = − ∞.
h→0 h→0  h
Thus f ( x ) has a discontinuity of the second kind at x = 0. In fact it is an infinite
discontinuity.
(iii) Here f ( x ) = e − 1 / x .
The function f ( x ) is not defined at x = 0 and is defined at every other real number.
Thus domain f = R − {0 } i. e., R0 .
Let c ∈ R and c ≠ 0.
lim lim
We have f ( x) = e − 1 / x = e − 1 / c = f ( c ).
x→ c x→ c
∴ f ( x ) is continuous at every point c of R if c ≠ 0.
lim lim
Now f (0 + 0 ) = f ( x) = f (0 + h), h > 0
x→0 + h→0
lim lim lim
e− 1/ h =
1 1
= f ( h) = = =0
h→0 h→0 h→0 e1 / h ∞
lim lim
and f (0 − 0 ) = f ( x) = f (0 − h), h > 0
x→0 − h→0
lim lim lim
= f ( − h) = e − 1 /(− h) = e1 / h = ∞.
h→0 h→0 h→0
Thus f ( x ) has an infinite discontinuity at x = 0. The discontinuity is not ordinary but
lim
is of the second kind since f ( x ) does not exist .
x→0 −
(iv) f ( x ) = sin x
We have f ( x ) = sin x V x ∈ R and and domain f = R
Let c ∈ R . Then lim lim
x → c f ( x ) = x → c sin x = sin c = f
Hence, f ( x ) = sin x is continuous at x = c
But c is an arbitrary point of R. Hence f ( x ) is continuous at all points of R i.e. the
function f ( x ) = c is continuous on the whole real line.
R-5

(v) Here f ( x ) = cos (1 / x ) when x ≠ 0, f (0 ) .


Let c ∈ R and c ≠ 0.
lim lim
cos   = cos   = f ( c ).
1 1
We have f ( x) =
x→ c x→ c  x  c
∴ f ( x ) is continuous at every point c of R if c ≠ 0.
Now to check the continuity of f ( x ) at x = 0.
lim lim
We have f (0 + 0 ) = f (0 + h) = f ( h), h > 0
h→0 h→0
lim 1
= cos , h > 0 which does not exist.
h→0 h
As h → 0, the value of cos (1 / h) oscillates between +1 and −1, passing through zero
and intermediate values an infinite number of times. Hence there is no definite real
number l to which cos(1 / h) tends as h tends to zero. Therefore the right hand limit
f(0 + 0 ) does not exist.
lim lim
Again f (0 − 0 ) = f (0 − h) = f ( − h), h > 0
h→0 h→0
lim  1  lim
cos   , which does not exist.
1
= cos   =−
h→0  − h h→0  h
Thus both f (0 − 0 ) and f (0 + 0 ) do not exist and so f ( x ) has a discontinuity of the
second kind at x = 0.
(vi) Here f ( x ) = sin (1 / x ) when x ≠ 0, f (0 ) = 0.
Let c ∈ R and c ≠ 0.
lim lim 1 1
We have f ( x) = sin = sin = f ( c ).
x→ c x→ c x c
∴ f ( x ) is continuous at every point c of R if c ≠ 0.
Now to check the continuity of f ( x ) at x = 0.
lim lim
We have f (0 + 0 ) = f (0 + h) = f ( h), h > 0
h→0 h→0
lim 1
= sin , which does not exist.
h→0 h
lim lim
Again f (0 − 0 ) = f (0 − h) = f ( − h), h > 0
h→0 h→0
lim  1  lim 1
= sin   =− sin , which does not exist.
h→0  − h h→0 h
Thus both f (0 − 0 ) and f (0 + 0 ) do not exist and so f ( x ) has a discontinuity of the
second kind at x = 0.
sin x
(vii) Here f ( x ) = when x ≠ 0 and f (0 ) = 1 .
x
Let c ∈ R and c ≠ 0.
lim lim sin x sin c
We have f ( x) = = = f ( c ).
x→ c x→ c x c
∴ f ( x ) is continuous at every point c of R if c ≠ 0.
R-6

Now to check the continuity of f ( x ) at x = 0.


lim lim lim sin h
We have f (0 + 0 ) = f (0 + h) = f ( h), h > 0 = = 1.
h→0 h→0 h→0 h
lim lim
Again f (0 − 0 ) = f (0 − h) = f ( − h), h > 0
h→0 h→0
lim sin ( − h) lim sin h
= = = 1.
h → 0 ( − h) h→0 h
Also f (0 ) = 1 .
Since f (0 − 0 ) = f (0 ) = f (0 + 0 ), therefore f ( x ) is continuous at x = 0.
Hence f ( x ) is continuous on the whole real line.
e1 / x − 1
(viii) Here f ( x ) = when x ≠ 0 and f (0 ) = 1 .
e1 / x + 1
Let c ∈ R and c ≠ 0.
lim lim e1 / x − 1 e1 / c − 1
We have f ( x) = = = f ( c ).
x→ c x→ c e1 / x + 1 e1 / c + 1
∴ f ( x ) is continuous at every point c of R if c ≠ 0.
Now to check the continuity of f ( x ) at x = 0 .
lim lim
We have f (0 + 0 ) = f (0 + h) = f ( h) , h > 0
h→0 h→0
lim e1 / h − 1 lim 1 − (1 / e1 / h )  lim 1 1 
= = = 1. ∵ h → 0 = = 0
h→0 e1 / h + 1 h → 0 1 + (1 / e1 / h )  e1 / h ∞ 
lim lim
Again f (0 − 0 ) = f (0 − h) = f ( − h), h > 0
h→0 h→0
lim e− 1/ h − 1 0 −1  lim lim 
e− 1/ h =
1
= = = − 1. ∵ h → 0 = 0
h→0 e− 1/ h + 1 0 +1  h→0 e1 / h 
Also f (0 ) = 1.
Thus f (0 − 0 ) ≠ f (0 ) = f (0 + 0 ).
∴ at x = 0, f ( x ) is discontinuous from the left and is continuous from the right.
Thus f ( x ) is discontinuous at x = 0 and the discontinuity is of the first kind because
both the limits f (0 − 0 ) and f (0 + 0 ) exist. The jump of the function at x = 0 is
f (0 + 0 ) − f (0 − 0 ) i. e., 1 − ( − 1) i. e., 2.
e1 / x
(ix) Here f ( x ) = when x ≠ 0, f (0 ) = 0.
1 + e1 / x
Let c ∈ R and c ≠ 0.
lim lim e1 / x e1 / c
We have f ( x) = = = f ( c ).
x→ c x → c 1 + e1 / x 1 + e1 / c
∴ f ( x ) is continuous at every point c of R if c ≠ 0.
R-7

Now to check the continuity of f ( x ) at x = 0.


lim lim e1 / h
We have f (0 + 0 ) = f ( h) =
h→0 h→0 e1 / h + 1
lim 1 1
= = =1
h → 0 1+ (1 / e1 / h ) 1+ 0

lim lim e −1 / h 0
and f (0 − 0 ) = f ( − h) = = = 0.
h→0 h→0 e −1 / h + 1 0 +1

Also f (0 ) = 0.
Thus f (0 − 0 ) = f (0 ) ≠ f (0 + 0 ).
∴ at x = 0, f ( x ) is continuous from the left and is discontinuous from the right.
Thus f ( x ) has an ordinary discontinuity at x = 0. The jump of the function at x = 0 is
f (0 + 0 ) − f (0 − 0 ) i. e., 1 − 0 i. e., 1 .
xe1 / x 1
(x) Here f ( x ) = + sin ( ) when x ≠ 0, f (0 ) = 0.
1 + e1 / x x
Let c ∈ R and c ≠ 0.
lim  xe1 / x
lim 1 c e1 / c 1
We have f ( x) =  + sin  = + sin = f ( c ).
x→ c x→ c
1 + e
1 / x x  1 + e1 / c c
∴ f ( x ) is continuous at every point c of R if c ≠ 0.
Now to check the continuity of f ( x ) at x = 0 .
We have f (0 ) = 0.
lim lim
Also f (0 + 0 ) = f (0 + h) = f ( h) , h > 0
h→0 h→0
lim  he1 / h 1 lim  h 1
=  + sin  =  − 1/ h + sin 
h→0 1 + e
1 / h h  h → 0  e +1 h 

0 lim 1 lim 1
= + sin = 0 + sin ,
0 +1 h→0 h h→0 h
lim
which does not exist because sin (1 / h) does not exist.
h→0
lim lim
Again f (0 − 0 ) = f (0 − h) = f ( − h), h > 0
h→0 h→0
lim ( − h) e − 1 / h  1 
=  + sin  
h→0 − 1 / h  − h  
 1 + e
0 lim 1 lim 1
= − sin = 0 − sin , which does not exist.
1+ 0 h→0 h h→0 h
Thus both f (0 − 0 ) and f (0 + 0 ) do not exist. It follows that f ( x ) is discontinuous
at x = 0 and the discontinuity is of the second kind.
R-8

(xi) Here f ( x ) = sin x cos (1 / x ) when x ≠ 0, f (0 ) = 0.


Let c ∈ R and c ≠ 0.
lim lim
We have f ( x) = sin x cos (1 / x ) = sin c cos (1 / c ) = f ( c ).
x→ c x→ c
∴ f ( x ) is continuous at every point c of R if c ≠ 0.
Now to check the continuity of f ( x ) at x = 0.
lim lim
We have f (0 + 0 ) = f (0 + h) = f ( h), h > 0
h→0 h→0
lim
= sin h cos (1 / h) = 0.
h→0
 lim
∵ h → 0 sin h = 0 and|cos (1 / h)|≤ 1 when h ≠ 0


i. e., cos (1 / h) is bounded in some deleted neighbourhood of zero

lim lim
Again f (0 − 0 ) = f (0 − h) = f ( − h), h > 0
h→0 h→0
lim lim
= sin ( − h) cos ( − 1 / h) = − sin h cos (1 / h)
h→0 h→0
= − 0 = 0.
Also f (0 ) = 0.
Thus f (0 − 0 ) = f (0 ) = f (0 + 0 ).
Hence the function f ( x ) is continuous at x = 0.
 1 / x2
 e , when x ≠ 0
Problem 2 (i) : Examine at x = 0, the continuity of f ( x) =  2
1 − e1 / x

 1, when x = 0.
(Meerut 2008)

Solution: We have f (0 ) = 1 .
lim
Also f (0 + 0 ) = f (0 + h), where h is + ive and sufficiently small
h→0
2
lim lim e1 / h lim 1
= f ( h) = = ,
h→0 h→0 2 h→0 2
1 − e1 / h e− 1/ h − 1
2
dividing the Nr. and Dr. by e1 / h
 lim 2 lim 
e− 1/ h =
1 1 1
= = − 1. ∵ = = 0
0 −1  h →0 h→0 2 ∞ 
e1 / h
R-9

lim
Again f (0 − 0 ) = f (0 − h), where h is + ive and sufficiently small
h→0
2
lim lim e1 / h
= f ( − h) = = −1.
h→0 h→0 2
1 − e1 / h
Thus f (0 − 0 ) = − 1, f (0 ) = 1, f (0 + 0 ) = − 1 .
Since f (0 − 0 ) = f (0 + 0 ) ≠ f (0 ), therefore f ( x ) is not continuous at x = 0. In fact
f ( x ) has a removable discontinuity at x = 0.
1 1
Problem 2(ii): If f ( x ) = sin , find f ( a + 0 ) and f ( a − 0 ). Is the function
x−a x−a
continuous at x = a ?
lim
Solution: We have f ( a + 0 ) = f ( a + h), h > 0
h→0
lim 1 1
= ⋅ sin
h→0 ( a + h) − a ( a + h) − a
lim 1 1
= sin , which does not exist.
h→0 h h
lim
Again f (a − 0 ) = f ( a − h) , h > 0
h→0
lim 1 1 lim  1  1 
= sin = − sin  −  
h→0 ( a − h) − a ( a − h) − a h → 0  h h 
lim 1 1
= sin , which does not exist.
h→0 h h
Thus both f ( a + 0 ) and f ( a − 0 ) do not exist and so f ( x ) has a discontinuity of the
second kind at x = a.

Problem 3: Find out the points of discontinuity of the following functions :


( i) f ( x ) = (2 + e1 / x )− 1 + cos e1 / x for x ≠ 0, f (0 ) = 0.

( ii) f ( x ) = 1 / 2n for 1 / 2n + 1 < x ≤ 1 / 2n, n = 0, 1, 2, … and f (0 ) = 0.


Solution: (i) Let c ∈ R and c ≠ 0. We have
lim lim  1  1
f ( x) =  + cos e1 / x  = + cos e1 / c = f ( c ).
x→ c x→ c  2 + e
1 / x
 2 + e
1/ c

∴ f ( x ) is continuous at every point c of R if c ≠ 0.


Now to check the continuity of f ( x ) at x = 0.
We have f (0 ) = 0.
lim lim
Now f (0 + 0 ) = f (0 + h) = f ( h), h > 0
h→0 h→0
R-10

lim  1  1 lim
=  + cos e1 / h  = + cos e1 / h
h→0  2 + e1 / h
 2 + ∞ h → 0
lim 1 / lim
=0 + cos e h = cos e1 / h.
h→0 h→0
lim
As h → 0, cos e1 / h oscillates between − 1 and 1 and so cos e1 / h does not exist.
h→0
Hence f (0 + 0 ) does not exist.
lim lim
Again f (0 − 0 ) = f (0 − h) = f ( − h), h > 0
h→0 h→0
lim  
+ cos e − 1 / h 
1
= 
h→0 − 1 / h
 2 + e 

+ cos e − ∞ =
1 1 1 3
= + cos 0 = + 1 = ⋅
2 + e− ∞ 2+0 2 2
Since f (0 + 0 ) does not exist, f ( x ) is discontinuous at x = 0 and discontinuity is of
second kind. It is a kind of mixed discontinuity since the limit on the right does not
exist whereas the limit on the left exists.
1
(ii) We have f ( x ) = 1 for < x ≤ 1,
2
1 1 1
f ( x) = for < x≤
2 22 2
1 1 1
f ( x) = for < x≤
22 23 22
.........................................
.........................................
1 1 1
f ( x) = for < x≤
2 n − 1 2 n 2 −1
n
1 1 1
f ( x) = for < x≤
2 n 2 n + 1 2n
and so on. Also f (0 ) = 0. The domain f ( x ) = [0, 1].
Obviously f ( x ) is continuous at x = 1 and at all points x where
1 1
< x< , n = 0, 1, 2, .... because in each such interval f ( x ) is constant.
2n + 1 2n
1
Now it remains to check the continuity of f ( x ) at x = , n = 1, 2, 3, … . We consider
2n
x = 1 / 2n.
1 1 1  1 1  1
We have f   = , f  − 0 = and f  + 0 = ⋅
 2n  2n  2n  2n  2n  2n − 1
1  1 
Since f  − 0 ≠ f  + 0  , the function f is discontinuous at x = 1 / 2n ,
 2n   2n 
n = 1, 2, 3, … At each such point f ( x ) is continuous from the left and is discontinuous
from the right and the discontinuity is of the first kind.
R-11

lim
A little consideration shows that f ( x ) = 0.
x→0 +
Also f (0 ) = 0. Since f (0 + 0 ) = f (0 ), therefore f ( x ) is continuous at x = 0. The
question of finding f (0 − 0 ) does not arise because f ( x ) is not defined for x < 0.
1
Hence f ( x ) is discontinuous at x = , n = 1, 2, 3, …
2n
1 1
Problem 4: If f ( x ) = sin for x ≠ 0 and f (0 ) = 0, show that f ( x ) is finite for every
x x
value of x in the interval [ − 1, 1] but is not bounded. Determine the points of discontinuity of the
function if any.
Solution: Let c ∈ [ − 1, 1] and c ≠ 0.
1 1
Then f ( c ) = sin which is finite because both 1 / c and sin (1 / c ) are definite real
c c
numbers.
Also f (0 ) = 0 which is also finite.
Thus f ( x ) is finite for every value of x in the interval [ − 1, 1].
However f ( x ) is not bounded in [ − 1, 1]. Take any positive real number k, however
large. Then there exists a real number x lying in ] 0, 1 [ such that
1 1 1 1
sin = 1 and > k so that sin > k.
x x x x
∴ f ( x ) is not bounded in [ − 1, 1].
Obviously f ( x ) is continuous at every real number c if c ≠ 0.
Also f (0 − 0 ) and f (0 + 0 ) both do not exist and so f ( x ) has a discontinuity of the
second kind at x = 0.

 x, if x is rational
Problem 5: A function f defined on [0, 1] is given by f ( x ) = 
1 − x, if x is irrational.
Show that f takes every value between 0 and 1 (both inclusive), but it is continuous only at the
1
point x = ⋅
2 (Rohilkhand 2012B)
Solution: Let c ∈[0, 1].
If c is rational, then f ( c ) = c.
If c is irrational, then 1 − c is also irrational
and 0 < 1 − c < 1 i. e., 1 − c ∈ [0, 1].
We have f (1 − c ) = 1 − (1 − c ) = c.
Thus f takes every value c in [0, 1].
1
Now to show that f is continuous only at the point x = ⋅
2
Let x 0 be any point of [0, 1]. For each positive integer n we select a rational number an
and an irrational number bn, both in [0, 1], such that
|an − x 0|< 1 / n,|bn − x 0|< 1 / n.
lim lim
∴ an = x 0 = bn .
n→ ∞ n→ ∞
R-12

If f is to be continuous at x 0 , then we must have


lim lim
f ( an) = f ( x 0 ) = f ( bn).
n→ ∞ n→ ∞
Now f ( an) = an for all n and f ( bn) = 1 − bn for all n.
lim lim
∴ f ( an) = an = x 0
n→ ∞ n→ ∞
lim lim
and f ( bn) = (1 − bn) = 1 − x 0 .
n→ ∞ n→ ∞
So for f to be continuous at x 0 , we must have
1
x0 = f ( x0 ) = 1 − x0 i. e., x 0 = ⋅
2
1
Thus x = is the only possible point of [0, 1] where f can be continuous.
2
Now we shall show that f is actually continuous at the point x = 1 / 2 .
1
We have f (1 / 2) = ⋅
2
Let ε > 0 be given.
1
Take a positive real number δ = ε . Then if x is rational, we have
2
 f ( x ) − f  1  
x − 1< δ ⇒  = x − 1< δ = 1 ε < ε
 2   2    2 2
and if x is irrational, we have
 f ( x) − f  1  
 x − 1 < δ ⇒  1 1 1
  = (1 − x ) −  = x − < δ = ε < ε.
 2   2   2  2 2
Thus, we have
x − 1< δ ⇒ f ( x) − f  1  

  < ε,
 2   2 
1
so that f is continuous at x = ⋅
2
Hence f is continuous only at the point x = 1 / 2.

1 , if x is rational

Problem 6: Prove that the function f defined by f ( x ) = 2
1 , if x is irrational
3
is discontinuous everywhere.
Solution: We shall show that f ( x ) is discontinuous at every point a of R .
Take any δ > 0. Then a − δ and a + δ are two distinct real numbers and between two
distinct real numbers there lie infinite rational and infinite irrational numbers. Thus
for every δ > 0, there exist infinite rational and infinite irrational numbers in the open
interval ] a − δ, a + δ [.
R-13

∴ for every δ > 0, there exists a point x in ] a − δ, a + δ [ at which


1 1 1
| f ( x ) − f ( a)| =  −  = , taking x as rational if a is irrational
2 3 6
1 1 1
or at which | f ( x ) − f ( a)| =  −  = , taking x as irrational if a is rational.
3 2 6
Thus whatever the point a in R may be, for every δ > 0 there exists a point x in
1
] a − δ, a + δ [ at which | f ( x ) − f ( a)| = ⋅
6
1
Thus for ε = > 0, there exists no δ > 0 such that
6
| f ( x ) − f ( a)|< ε, whenever |x − a|< δ i. e., whenever x ∈ ] a, − δ, a + δ [ .
∴ f ( x ) is not continuous at x = a.
Hence f ( x ) is discontinuous at every point a of R .
x e1 / x
Problem 7(i): Show that the function f defined by f ( x) = , x ≠ 0, f (0 ) = 1
1 + e1 / x
is not continuous at x = 0 and also show how the discontinuity can be removed.
(Rohilkhand 2006; Lucknow 08; Meerut 11)
lim lim
Solution: We have f (0 + 0 ) = f (0 + h) = f ( h), h > 0
h→0 h→0
lim  e1 / h  lim  1 
= h ⋅  = h ⋅ 
h→0  1 / h  h→0  − 1 / h 
 1+ e   e + 1
1
=0⋅ = 0 ⋅1 = 0
0 +1
lim lim
and f (0 − 0 ) = f (0 − h) = f ( − h), h > 0
h→0 h→0
lim  e − 1 / h  0
= ( − h) ⋅ =0⋅ = 0 ⋅ 0 = 0.
h→0  1 + e − 1 / h  1+ 0
lim
Since f (0 − 0 ) = 0 = f (0 + 0 ), therefore f ( x ) = 0.
x→0
lim
But f (0 ) = 1 ≠ f ( x ). Hence f ( x ) is discontinuous at x = 0 and the
x→0
discontinuity is removable and can be removed by defining f ( x ) as follows :
x e1 / x
f ( x) = , x ≠ 0, f (0 ) = 0.
1 + e1 / x
Problem 7(ii): Show that the function f ( x ) = 3 x2 + 2 x − 1 is continuous for x = 2.
Solution: We have
lim lim lim lim lim
f ( x) = (3 x2 + 2 x − 1) = 3 x2 + 2 x− 1
x→2 x→2 x→2 x→2 x→2
R-14

 lim lim 
= 3 ⋅ 22 + 2 ⋅ 2 − 1 ∵ x → c x = c and x2 = c2 
 x→ c 
= f (2).
lim
Since f ( x ) = f (2), therefore f ( x ) is continuous at x = 2.
x→2

Problem 7(iii): Show that the function f ( x ) = (1 + 2 x )1 / x , x ≠ 0 and f ( x ) = e2 , x = 0 is


continuous at x = 0.
lim
Solution: We know that (1 + ax )1 / x = e a .
h→0
lim lim
∴ f ( x) = (1 + 2 x )1 / x = e = f (0 ) and so f ( x ) is continuous at x = 0
x→0 x→0

− x2 if x ≤ 0

5 x − 4 if 0 < x ≤ 1
Problem 8: Examine the continuity of the function f ( x) = 
2
4 x − 3 x if 1 < x < 2
3 x + 4 if x ≥ 2

at x = 0, 1 and 2.
(Meerut 2004, 06B, 07B; Purvanchal 06; 10; Avadh 06; Lucknow 06;
Gorakhpur 15)
Solution: (i) Continuity at x = 0. We have f (0 ) = − 02 = 0;
lim lim
f (0 − 0 ) = f (0 − h) = f ( − h), h > 0 and is sufficiently small
h→0 h→0
lim lim
= {− ( − h)2 } = ( − h2 ) = 0;
h→0 h→0
lim lim
and f (0 + 0 ) = f (0 + h) = f ( h), h > 0 and is sufficiently small
h→0 h→0
lim
= (5h − 4) [∵ 0 < h < 1]
h→0
= − 4.
Since f (0 − 0 ) ≠ f (0 + 0 ), the function f ( x ) is discontinuous at x = 0. It is
continuous at x = 0 from the left but is discontinuous from the right and the
discontinuity is ordinary.
(ii) Continuity at x = 1. We have f (1) = 5 ⋅1 − 4 = 1 ;
lim lim
f (1 − 0 ) = f (1 − h), h > 0 = { 5 (1 − h) − 4 } [∵ 0 < 1 − h < 1]
h→0 h→0
lim
= (1 − 5h) = 1 ;
h→0
R-15

lim lim
and f (1 + 0 ) = f (1 + h), h > 0 = [4 (1 + h)2 − 3 (1 + h)]
h→0 h→0
[∵ 1 < 1 + h < 2]
lim
= (4h2 + 5h + 1) = 1 .
h→0
Thus f (1 − 0 ) = f (1) = f (1 + 0 ) and so f ( x ) is continuous at x = 1 .
(iii) Continuity at x = 2. We have f (2) = 3 ⋅ 2 + 4 = 10;
lim lim
f (2 − 0 ) = f (2 − h), h > 0 = [4 (2 − h)2 − 3 (2 − h)]
h→0 h→0
[∵ 1 < 2 − h < 2]
lim
= (4h2 − 13h + 10 ) = 10;
h→0
lim lim
and f (2 + 0 ) = f (2 + h), h > 0 = { 3 (2 + h) + 4 } [∵ 2 + h > 2]
h→0 h→0
lim
= (3h + 10 ) = 10.
h→0
Since f (2 − 0 ) = f (2) = f (2 + 0 ), f ( x ) is continuous at x = 2 .

e1 / x − 1
Problem 9(i): Show that the function f ( x) = , x ≠ 0 and f (0 ) = 0 is
e1 / x + 1
discontinuous at x = 0.
Solution: (i) Proceeding as in part (viii) of problem 1, we have f (0 + 0 ) = 1 and
f (0 − 0 ) = − 1.
Also f (0 ) = 0. Thus f (0 − 0 ) ≠ f (0 ) and f (0 + 0 ) ≠ f (0 ).
∴ f ( x ) is discontinuous at x = 0 both from the left and from the right.
Hence f ( x ) is discontinuous at x = 0 and the discontinuity is ordinary because both
lim
the limits f (0 − 0 ) and f (0 + 0 ) exist. Here f ( x ) does not exist because
x→0
f (0 − 0 ) ≠ f (0 + 0 ).
Problem 9(ii): Show that the following function is continuous at x = 0.
sin− 1 x
f ( x) = , x ≠ 0, f (0 ) = 1 .
x (Agra 2003)
lim lim sin− 1 x
Solution: We have f ( x) =
x→0 x→0 x
lim θ
= ,
θ→ 0 sin θ
putting sin− 1 x = θ so that x = sin θ and θ → 0 as x → 0
= 1 = f (0 ).
∴ f ( x ) is continuous at x = 0.
R-16

1
Problem 10: Discuss the continuity of the function f ( x ) = , when x ≠ 0 and
1 − e1 / x
f (0 ) = 0 for all values of x. (Meerut 2004; Rohilkhand 10B; Lucknow 10)
Solution: Let c ∈ R and c ≠ 0.
lim lim 1 1
We have f ( x) = = = f ( c ).
x→ c x → c 1 − e1 / x 1 − e1 / c
∴ f ( x ) is continuous at every point c of R if c ≠ 0.
Now to check the continuity of f ( x ) at x = 0.
lim lim
We have f (0 + 0 ) = f (0 + h) = f ( h), h > 0
h→0 h→0
lim 1 1 1
= = =− = − 0 = 0.
h → 0 1 − e1 / h 1 − ∞ ∞
lim lim
Again f (0 − 0 ) = f (0 − h) = f ( − h), h > 0
h→0 h→0
lim 1 1
= = = 1.
h → 0 1 − e− 1/ h 1 − 0

Also f (0 ) = 0. Thus f (0 − 0 ) ≠ f (0 ) = f (0 + 0 ).
∴ f ( x ) is discontinuous at x = 0 and the discontinuity is ordinary. The jump of the
function at x = 0 is f (0 − 0 ) − f (0 + 0 ) i. e., 1 − 0 i. e., 1. The function f ( x ) is
continuous at x = 0 from the right but is discontinuous from the left.
|x|
Problem 11: Prove that the function f ( x ) = for x ≠ 0, f (0 ) = 0 is continuous at all
x
points except x = 0. (Kanpur 2008, 09; Meerut 09; Gorakhpur 11)
x
Solution: If x > 0, then f ( x ) = = 1 . [∵ x > 0 ⇒ |x | = x ]
x
− x
If x < 0, then f ( x) = = − 1. [∵ x < 0 ⇒ | x | = − x ]
x
− 1, if x < 0

Thus f ( x ) =  0, at x = 0
 1, if x > 0.

If x < 0, f ( x ) = − 1 i. e., f ( x ) is a constant function and a constant function is
continuous at each point of its domain.
∴ f ( x ) is continuous at each point x where x < 0. Similarly f ( x ) is continuous at each
point x where x > 0.
Now to check the continuity of f ( x ) at x = 0.
lim lim
We have f (0 − 0 ) = f (0 − h) = f ( − h), h > 0
h→0 h→0
lim
= −1= −1
h→0
and similarly f (0 + 0 ) = 1.
R-17

Also f (0 ) = 0.
lim
Since f (0 − 0 ) ≠ f (0 + 0 ), therefore f ( x ) does not exist and so f ( x ) is
x→0
discontinuous at x = 0.
Since both f (0 − 0 ) and f (0 + 0 ) exist, therefore the discontinuity is ordinary and
the jump of the function at x = 0 is f (0 + 0 ) − f (0 − 0 ) i. e.,1 − ( − 1) i. e., 2.

Problem 12: Test the continuity of the function f ( x ) at x = 0 if

e1 / x sin (1 / x )
f ( x) = , x ≠ 0 and f (0 ) = 0.
1 + e1 / x (Meerut 2005)
lim lim
Solution: We have f (0 − 0 ) = f (0 − h) = f ( − h), h > 0
h→0 h→0
lim  e − 1 / h 1 
=  ⋅ sin  −   = 0.
h→0 − 1 / h  h  
1 + e
 lim e− 1/ h 0
∵ = = 0 and |sin ( − 1 / h)|≤ 1 when h ≠ 0
h → 0 1+ e − 1 / h 1 + 0


i. e., sin ( − 1 / h) is bounded in some deleted neighbourhood of zero

lim lim
Again f (0 + 0 ) = f (0 + h) = f ( h), h > 0
h→0 h→0

 e1 / h   1 
= lim  ⋅ sin (1 / h) = lim  ⋅ sin(1 / h)
h → 0 1 + e1 / h  h → 0  e −1 / h + 1 
lim 1 1 lim
which does not exist because = = 1 but sin (1 / h) does
h→0 e− 1/ h + 1 0 +1 h→0
not exist.
Also f (0 ) = 0.
Thus f (0 − 0 ) = f (0 ) and f (0 + 0 ) does not exist.
lim
∴ f ( x ) does not exist and f ( x ) has discontinuity of the second kind at x = 0
x→0
from the right.
We observe that f ( x ) is continuous at x = 0 from the left.

Problem 13: Examine the following function for continuity at x = 0 and x = 1 :


 2
x for x≤0

f ( x ) = 1 for 0 < x ≤ 1
1 for x > 1
 x (Meerut 2001, 03, 04B, 05)
Solution: (i) Continuity at x = 0
we have f (0 ) = (0 )2 = 0
R-18

f (0 − 0 ) = lim f (0 − h) = lim f ( − h) = lim ( − h)2 = 0


h→0 h→0 h→0
and f (0 + 0 ) = lim f (0 + h) = lim f ( h) = lim 1 = 1
h→0 h→0 h→0
Since f (0 − 0 ) ≠ f (0 + 0 ), the function f ( x ) is discontinuous at x = 0.
(ii) Continuity at x = 0
We have f (1) = 1 = 1
f (1 − 0 ) = lim f (1 − h), h > 0 = lim 1 = 1
h→0 h→0
lim 1
and f (1 + 0 ) = f (1 + h), h > 0 = lim =1
h→0 h→0 1+ h

Thus f (1 − 0 ) = f (1) = f (1 + 0 ) and so f ( x ) is continuous at x = 1.


Problem 14: Discuss the continuity of the following function at x = 0 :
 cos x , x ≥ 0
f ( x) = 
 − cos x , x < 0 .
Solution: We have f (0 ) = cos 0 = 1
f (0 + 0 ) = lim f (0 + h) = lim f ( h) = lim cos h = cos 0 = 1
h→0 h→0 h→0
and f (0 − 0 ) = lim f (0 − h) = lim f ( − h) = lim − cos( − h)
h→0 h→0 h→0
= lim − cos h = − cos 0 = − 1
h→0
Since f (0 − 0 ) ≠ f (0 + 0 ), the function f ( x ) is discontinuous at x = 0 .
Problem 15: Test the continuity of the following functions at x = 0 :
( i) f ( x ) = x cos (1 / x ), when x ≠ 0, f (0 ) = 0. (Meerut 2007)
( ii) f ( x ) = x log x, for x > 0, f (0 ) = 0.
Solution: (i) We have
lim lim
f (0 + 0 ) = f (0 + h) = f ( h), h > 0
h→0 h→0
lim
= h cos (1 / h) = 0.
h→0
lim
[∵ h = 0 and |cos (1 / h)|≤ 1 when h ≠ 0 i. e., cos (1 / h)
h→0
is bounded in some deleted neighbourhood of zero]
lim lim
Again f (0 − 0 ) = f (0 − h) = f ( − h), h > 0
h→0 h→0
lim lim
= ( − h) cos ( − 1 / h) = − h cos (1 / h) = − 0 = 0.
h→0 h→0
Also f (0 ) = 0.
Thus f (0 − 0 ) = f (0 ) = f (0 + 0 ) and so f ( x ) is continuous at x = 0.
R-19

lim lim
(ii) We have f (0 + 0 ) = f (0 + h) = f ( h), h > 0
h→0 h→0
lim
= h log h [From 0 × ∞ ]
h→0

=
lim log h From ∞ 
h→0 1/h  ∞ 
lim 1/h lim
= =− h = − 0 = 0.
h→0 − 1 / h2 h→0

Also f (0 ) = 0.
lim
Since f ( x ) is not defined for x < 0, therefore the question of finding f ( x ) does
x→0 −
not arise.
Since f (0 ) = f (0 + 0 ) and f ( x ) is not defined for x < 0, i. e., domain f ( x ) is [ 0, ∞ [,
therefore f ( x ) is continuous at x = 0.

Problem 16: Discuss the nature of discontinuity at x = 0 of the function f ( x ) = [ x ] − [ − x ]


where [ x ] denotes the integral part of x.
Solution: We have f (0 ) = [0 ] − [ − 0 ] = [0 ] − [0 ] = 0 − 0 = 0.
lim lim
Also f (0 + 0 ) = f (0 + h) = f ( h), h > 0 and is sufficiently small
h→0 h→0
lim lim lim
= ([ h] − [ − h]) = { 0 − ( − 1) } = 1 = 1.
h→0 h→0 h→0
[∵ 0 < h < 1 ⇒ [ h] = 0 and − 1 < − h < 0 ⇒ [ − h] = − 1]
lim lim
Again f (0 − 0 ) = f (0 − h) = f ( − h), h > 0 and is sufficiently small
h→0 h→0
lim lim
= ([ − h] − [ − ( − h)]) = ([ − h] − [ h])
h→0 h→0
lim
= ( − 1 − 0 ) = − 1.
h→0
lim
Since f (0 − 0 ) ≠ f (0 + 0 ), therefore f ( x ) does not exist and so f ( x ) is
x→0
discontinuous at x = 0. Also f (0 ) ≠ f (0 − 0 ) and f (0 ) ≠ f (0 + 0 ) implies that f ( x ) is
discontinuous at x = 0 both from the left and from the right . Since f(0 − 0 ) and
f(0 + 0 ) both exist, therefore the discontinuity is of the first kind and the jump of the
function at x = 0 is f (0 + 0 ) − f (0 − 0 ) i. e., 1 − ( −1) i. e., 2.
Problem 17: Discuss the continuity of f ( x ) = (1 / x ) cos (1 / x ).
Solution: Let c ∈ R and c ≠ 0.
lim lim
1 1 1 1
We have f ( x) = cos = cos = f ( c ).
x→ c x→ c
x x c c
∴ f ( x ) is continuous at every point c of R if c ≠ 0.
R-20

Now to check the continuity of f ( x ) at x = 0 .


lim lim
We have f (0 + 0 ) = f (0 + h) = f ( h), h > 0
h→0 h→0
lim 1 1
= cos which does not exist.
h→0 h h
lim lim
Again f (0 − 0 ) = f (0 − h) = f ( − h), h > 0
h→0 h→0
lim  1  1 
= − cos  −  
h→0  h h 
lim  1  1 
= − cos    , which does not exist.
h→0  h h 
Thus both f (0 − 0 ) and f (0 + 0 ) do not exist and so f ( x ) has a discontinuity of the
second kind at x = 0.

Problem 18: Give an example of each of the following types of functions :


( i) The function which possesses a limit at x = 1 but is not defined at x = 1 .
( ii) The function which is neither defined at x = 1 nor has a limit at x = 1 .
( iii) The function which is defined at two points but is nevertheless discontinuous at both the
points.
Solution: (i) f ( x ) = x2 for x > 1 , f ( x ) = x3 for x < 1 .
(ii) f ( x ) = − x2 for x < 1 , f ( x ) = x2 for x > 1 .
3 1 3 1
(iii) f ( x ) = 0 for x ≤ 0, f ( x ) = − x for 0 < x ≤ , f ( x ) = + x for x > ⋅
2 2 2 2
Problem 19: In the closed interval [ − 1, 1] let f be defined by

f ( x ) = x2 sin (1 / x2 ) for x ≠ 0 and f (0 ) = 0.


In the given interval ( i) Is the function bounded ? ( ii) Is it continuous ?
Solution: (i) If x ∈ [ − 1, 1] and x ≠ 0, we have
| f ( x )| = |x2 sin (1 / x2 )| = |x2|⋅ |sin (1 / x2 )|

= | x|2 ⋅ |sin (1 / x2 )|≤ 1⋅1 = 1 .

[∵|sin (1 / x2 )|≤ 1 and − 1 ≤ x ≤ 1 ⇒ |x|≤ 1]


Also f (0 ) = 0 ⇒ | f (0 )| = 0 < 1 .
Thus | f ( x )|≤ 1, ∀ x ∈ [ − 1, 1] and so f is bounded in [ − 1, 1].
(ii)Let c ∈ [ − 1, 1] and c ≠ 0.
lim lim 1 1
We have f ( x) = x2 sin = c2 sin = f ( c ).
x→ c x→ c x2 c2
∴ f ( x ) is continuous at every point c of [ − 1, 1] if c ≠ 0.
Now to check the continuity of f ( x ) at x = 0.
R-21
lim lim
We have f (0 − 0 ) = f (0 − h) = f ( − h), h > 0
h→0 h→0
lim  1 
= ( − h)2 sin  
h→0 ( − h)2 
lim 1
= h2 sin = 0.
h→0 h2
 lim 2 
 1 
 
 ∵ h → 0 h = 0 and sin 2 ≤ 1 if h ≠ 0 
  h  
lim lim
Again f (0 + 0 ) = f (0 + h) = f ( h)
h→0 h→0
lim 1
= h2 sin = 0.
h→0 h2
Also f (0 ) = 0.
Since f (0 − 0 ) = f (0 ) = f (0 + 0 ), therefore f ( x ) is continuous at x = 0.
Thus f ( x ) is continuous at each point of [ − 1, 1] and so it is continuous in [ − 1, 1].

Comprehensive Problems 2
Problem 1: Let f be the function defined on [ −1, 1] by
f ( x ) = x, if x is irrational,
f ( x ) = 0, if x is rational.
Show that f is continuous only at x = 0.
Solution: Let x0 be any point of [ −1, 1]. For each positive integer n, we select a rational
number an, and an irrational number bn, both in [ − 1, 1], such that
|an − x0|< 1 / n, |bn − x0|< 1 / n.
∴ lim an = x0 = lim bn.
n→ ∞ n→ ∞
If f is continuous at x0 , then we must have
lim f ( an) = f ( x0 ) = lim f ( bn).
n→ ∞ n→ ∞
Since f ( an) = 0 for all n, and f ( bn) = bn for all n, therefore, we must have
0 = f ( x0 ) = lim bn ,
n→ ∞
i.e., 0 = f ( x0 ) = x0 .
Thus 0 is the only possible point where f can be continuous. Now, we shall show that f is
actually continuous at 0.
1
For any ε > 0, if we take δ = ε, then we have
2
|x|< δ ⇒ | f ( x ) − f (0 )| = 0 < ε, if x is rational,
and |x|< δ ⇒ | f ( x ) − f (0 )| = |x|< δ < ε, if x is irrational.
Thus, we have |x|< δ ⇒ | f ( x ) − f (0 )|< ε, so that f is continuous at 0.
R-22

Problem 2: Let f : R → R be such that


f ( x ) = x when x is irrational
= − x when x is rational.
Show that f ( x ) is continuous only at x = 0.
Solution: First we show that f ( x ) is continuous at x = 0.
Let ε > 0 be given.
1
Take a positive real number δ = ε. Then if x is rational, we have
2
|x − 0|< δ ⇒ | f ( x ) − f (0 )| = |− x − 0| = |− x|
1
= |x|< δ = ε < ε
2
and if x is irrational, we have
|x − 0|< δ ⇒ | f ( x ) − f (0 )| = |x − 0| = |x|< δ < ε .
Thus, we have |x|< δ ⇒ | f ( x ) − f (0 )|< ε, so that f is continuous at 0.
Now we show that f is not continuous at any x ≠ 0. Let x be any rational number. For
each positive integer n, let xn be an irrational number such that |xn − x|< 1 / n.
Then < xn > is a sequence of irrational numbers such that lim xn = x.
n→ ∞
Now by the definition of the function f , f ( xn) = xn for all n, so that
lim f ( xn ) = lim xn = x ≠ − x = f ( x ).
n→ ∞ n→ ∞
∴ the function f is not continuous at any non-zero rational number.
Similarly we can show that f is not continuous at any irrational number.
Now we give two important characterizations of continuous functions defined on R.
Problem 3: Show that the function f defined on R by
f ( x) = 1 when x is rational
f ( x) = − 1 when x is irrational
is discontinuous at every point of R.
Solution: First, suppose that x is rational. For each positive integer n, let xn be an
irrational number such that |xn − x|< 1 / n. Then < xn > converges to x.
Now by the definition of the function f , f ( xn) = − 1 for all n, so that
lim f ( xn) = − 1 ≠ f ( x ).
n→ ∞
Hence f is discontinuous at each rational point.
Next, suppose that x is an irrational number. For each positive integer n, let us choose a
rational number xn such that |xn − x|< 1 / n. Then the sequence < xn > converges to x.
Now f ( xn) = 1 for all n, so that
lim f ( xn) = 1 ≠ f ( x ).
n→ ∞
Hence f is discontinuous at each irrational point also.
∴ f is discontinuous at every point of R.
This function is known as Dirichlet’s function.
R-23

Alternative Solution: We shall show that f ( x )is discontinuous at every point a of R.


Take any δ > 0. Then a − δ and a + δ are two distinct real numbers and between two
distinct real numbers there lie infinite rational and infinite irrational numbers. Thus for
all δ > 0, there exist infinite rational and infinite irrational numbers in the open interval
] a − δ, a + δ [.
∴ for all δ > 0, there exists a point x in ] a − δ, a + δ [ at which
| f ( x ) − f ( a)| = |− 1 − 1| = 2, taking x as irrational if a is rational
or at which | f ( x ) − f ( a)| = |1 − ( − 1)| = 2, taking x as rational if a is irrational.
Thus whatever the point a in R may be, for all δ > 0 there exists a point x in ] a − δ, a + δ [
at which | f ( x ) − f ( a)| = 2.
1
Thus for ε = > 0, there exists no δ > 0 such that | f ( x ) − f ( a)|< ε , whenever
2
|x − a|< δ i. e., whenever x ∈ ] a − δ, a + δ [.
∴ f ( x ) is not continuous at x = a.
Hence f ( x ) is discontinuous at every point a of R.
Problem 4: Show that f : R → R given by f ( x ) = x2 is continuous but not uniformly
continuous on R.
Solution: We have f ( x ) = x2 , V x ∈ R.
Let c ∈ R . Then lim f ( x ) = lim x2 = c2 = f ( c ).
x→ c x→ c
∴ f ( x ) is continuous at c.
Now c was an arbitrary point of R. Therefore f ( x ) is continuous at each point of R and
hence f ( x ) is continuous on R.
In order to show that f ( x ) = x2 is not uniformly continuous on R, Refer to Text
Book solved example 17.
Problem 5: Explain fully uniform continuity and discuss the uniform continuity of the
function f ( x ) = x2 , V x ∈ R in ] 0, 1 [.
Solution: Uniform continuity. Definition.
A function f defined on an interval I is said to be uniformly continuous on I if for any given ε > 0,
there exists a δ > 0 such that
| f ( x ) − f ( y )|< ε whenever |x − y|< δ where x, y are in I.
A function is continuous in an interval I if it is continuous at each point of I. The
concept of continuity is local in character i. e., continuity is first defined at a point and a
function is continuous on an interval I if it is continuous at each point of I. The concept
of uniform continuity is global in character i. e., uniform continuity is a property
associated with an interval and not with a single point. Uniform continuity is defined
only on an interval and not at a point.
If a function f is uniformly continuous on an interval I, then it is continuous on I i. e., it
is continuous at each point of I. Thus uniform continuity always implies continuity.
But the converse is not true. A function may be continuous in an open interval I but it
may fail to be uniformly continuous in I.
R-24

For example, the function f ( x ) = 1 / x, V x ∈ ] 0, 1 [ is continuous in ] 0, 1 [ but it is not


uniformly continuous in ] 0, 1 [ .
However, if a function f is continuous in a closed and bounded interval [ a, b ], then it is
uniformly continuous in [ a, b ].
Second part: We shall show that the function f ( x ) = x2 , V x ∈ R is uniformly
continuous in ] 0, 1 [ .
Take any given ε > 0.
Let x1, x2 ∈ ] 0, 1 [ . We have
| f ( x2 ) − f ( x1 )| = |x22 − x12| = |( x2 − x1 ) ( x2 + x1 )|
= |x2 − x1|.|x2 + x1|
≤ |x2 − x1|. (|x2| + |x1|) [∵|x2 + x1|≤ |x2| + |x1|]
≤ 2 .|x2 − x1|.
[∵ x1, x2 ∈ ] 0, 1 [ ⇒ |x1|< 1 and |x2|< 1 ⇒ |x1| + |x2|< 2]
1
Then | f ( x2 ) − f ( x1 )|< ε whenever |x2 − x1|< ε.
2
1
Thus given ε > 0, there exists δ = ε > 0 such that
2
| f ( x2 ) − f ( x1 )|< ε whenever |x2 − x1|< δ, V x1, x2 ∈ ] 0, 1 [ .
Hence f ( x ) = x2 is uniformly continuous on ] 0, 1 [ .

Problem 6: Let f : ] − 1, 1 [ → [0, 1] be a function defined by f ( x ) = x2 . Using definition


show that f is uniformly continuous on its domain.
Solution: Proceed exactly as in problem 5.

Problem 7: Show that the function f ( x ) = 1 / x, x > 0 is continuous in (0, 1)but not uniformly
continuous.
Solution: For complete solution of this problem Refer to Text Book, example 14.

Problem 8: Define uniform continuity and show that the function f ( x ) = x2 +


3 x, x ∈ [ − 1, 1] is uniformly continuous in [ − 1, 1].
Solution: Take any given ε > 0.
Let x1, x2 ∈ [ − 1, 1]. We have | f ( x2 ) − f ( x1 )| = |( x22 + 3 x2 ) − ( x12 + 3 x1 )|
= |( x22 − x12 ) + 3 ( x2 − x1 )| = |( x2 − x1 ) ( x2 + x1 + 3)|
= |x2 − x1|⋅ |x2 + x1 + 3|
≤ |x2 − x1|⋅ (|x2| + |x1| + 3) [∵ |x1 + x2|≤ |x1| + |x2|]

≤ 5 |x2 − x1|. [∵ x1, x2 ∈ [ − 1, 1] ⇒ |x1|≤ 1 and |x2|≤ 1]


∴ | f ( x2 ) − f ( x1 )|< ε whenever |x2 − x1|< ε / 5.
Thus for any given ε > 0, there exists δ = ε / 5 > 0 such that
| f ( x2 ) − f ( x1 )|< ε whenever |x2 − x1|< δ, V x1, x2 ∈ [ − 1, 1].
Hence f ( x ) is uniformly continuous in [– 1, 1].
R-25

Alternative Solution: The function f ( x ) = x2 + 3 x defined on [ − 1, 1]is a polynomial


in x of degree 2. We know that a polynomial function is always continuous at each point
of its domain. Therefore f ( x ) = x2 + 3 x is continuous in [ − 1, 1].
Now a function which is continuous in a closed interval [ a, b ] is uniformly continuous in
[ a, b ]. Therefore the function f ( x ) = x2 + 3 x, x ∈ [ − 1, 1] is uniformly continuous in
[ − 1, 1].

Problem 9: Show that a function which is continuous in a closed interval is bounded in that
interval. Verify the theorem for
1 1
f ( x ) = cos x in [ − π, π ] .
2 2
Solution: For the first part of the question refer to Text Book theorem 2 of article 6.
1 1
Verification of the theorem for f ( x ) = cos x in [ − π, π ].
2 2
1 1
The function f ( x ) = cos x is continuous in the closed interval [ − π, π ] .
2 2
 1 1 
For all x ∈ − π, π , we have | f ( x )| = |cos x|≤ 1
 2 2 

− 1 ≤ f ( x ) ≤ 1 for all x ∈  − π, π  ⋅
1 1
i. e.,
 2 2 

Thus f ( x ) = cos x is bounded in  − 1 π, 1 π  ⋅


 2 2 

Problem 10: Give an example to show that a function continuous on an open interval need not
be bounded on that interval.
Solution: Consider the function f ( x ) = tan x defined on the open interval
] − π / 2, π / 2 [ .
If c ∈ ] − π / 2, π / 2 [, then f ( c ) = tan c
lim lim
and f ( x) = tan x = tan c.
x→ c x→ c
Thus at each point c ∈ ] − π / 2, π / 2 [,
lim
f (c ) = f ( x)
x→ c
so that f ( x ) is continuous at each point c ∈ ] − π / 2, π / 2 [.
Hence f ( x ) = tan x is continuous on the open interval ] − π / 2, π / 2 [.
But f ( x ) = tan x is not bounded on ] − π / 2, π / 2 [ . If k is any positive real number,
however large, there exists x ∈ ] − π / 2, π / 2 [ such that f ( x ) = tan x > k and so
f ( x ) = tan x is not bounded on ] − π / 2, π / 2 [.
Thus the function f ( x ) = tan x is continuous in the open interval ] − π / 2, π / 2 [ but is
not bounded in that interval.
Problem 11: If the function f is continuous in the closed interval [ a, b ] , prove that it attains its
least upper bound and greatest lower bound in [ a, b ]. Verify the theorem for the function
f ( x ) = sin x in [0, 2 π ].
Solution: For the first part of the question refer to Text Book theorem 3 of article 6.
R-26

Verification of the theorem for f ( x ) = sin x in [0, 2 π ]


The function f ( x ) = sin x is continuous in the closed interval [0, 2 π ].
In the interval [0, 2 π ], inf f ( x ) = − 1 and sup f ( x ) = 1.
3
We have f (3 π / 2) = sin (3 π / 2) = − 1 and π ∈ [0, 2 π ].
2
Thus f ( x ) = sin x attains its infimum − 1 in [0, 2 π ].
1
Again f ( π / 2) = sin ( π / 2) = 1 and π ∈ [0, 2 π ].
2
Thus f ( x ) = sin x attains its supremum in [0, 2 π ].
Problem 12: If f is continuous in [ a, b ] and f ( a). f ( b ) < 0, show that f ( c ) = 0 for at least
one c ∈[ a, b ].
Solution: Since f ( a) . f ( b ) < 0, therefore f ( a) and f ( b ) are of opposite signs. So now
proceed as in Text Book theorem 5 of article 6.
Problem 13: Let f be continuous on [ a, b ] and suppose that f ( x ) = 0 for every rational x in
[ a, b ]. Prove that f ( x ) = 0 for all x in [ a, b ].
Solution: Suppose f ( x0 ) ≠ 0 for some x0 in [ a, b ].
Since f is continuous at x = x0 and f ( x0 ) ≠ 0, therefore a positive number δ can be
found such that f ( x ) has the same sign as f ( x0 ) for every value of x in
] x0 − δ, x0 + δ [ . [See theorem 4 of article 6 ]
Consequently f ( x0 ) ≠ 0 for every value of x in ] x0 − δ, x0 + δ [ ⋅
Now between the two distinct real numbers x0 − δ and x0 + δ there lie infinite rational
numbers and according to hypothesis f ( x ) = 0 for every rational x in [ a, b ]. But as
mentioned above f ( x ) ≠ 0 for every value of x in ] x0 − δ, x0 + δ [ , and so we get a
contradiction. Therefore our initial assumption that f ( x0 ) ≠ 0 for some x0 in [ a, b ] is
wrong and we must have f ( x ) = 0 for all x in [ a, b ].

Problem 14: If a function is continuous on a closed interval [ a, b ] , then it attains its bounds at
least once in [ a, b ]. Give an example of a function which is continuous and bounded, and attains its
supremum but does not attain its infimum.
Solution: For the first part of the question refer to Text Book theorem 3 of article 6.
Second part: Consider the function f defined on R by the formula
1
f ( x) = , V x ∈R .
1 + |x|
We have 0 < f ( x ) ≤ 1, V x ∈ R and so f is bounded. Also f is continuous on the whole R.
Here sup f = 1 = f (0 ) and inf f = 0. There exists no real number x for which f ( x ) = 0.
Thus the function f attains its supremum for x = 0 but does not attain its infimum 0.
Remark: The function
1
f ( x) = − , V x ∈R
1 + | x|
is continuous and bounded, attains its infimum − 1 for x = 0 but does not attain its
supremum 0.
The function f ( x ) = x, for all x ∈ ] 0, 1] is continuous and bounded, attains its
supremum 1 but does not attain its infimum 0.
R-27

The function f ( x ) = x, for all x ∈[0, 1 [ is continuous and bounded, attains its infimum
0 but does not attain its supremum 1.
The function f ( x ) = x, for all x ∈ ] 0, 1[ is continuous and bounded but attains neither its
supremum 1 nor its infimum 0.
Problem 15: Prove that the identity mapping of any interval I is uniformly continuous on I.
Solution: Let f be the identity mapping of any interval I. Then
f : I → I is such that f ( x ) = x, V x ∈ I .
To prove that f is uniformly continuous on I.
Take any given ε > 0.
Then there exists δ = ε > 0 such that
| f ( x ) − f ( y )| = |x − y|< ε,
whenever |x − y|< δ, where x, y ∈ I .
Hence f is uniformly continuous on I.
Problem 16: Show that the function f : R → R defined by
1
f ( x) =
1 + e1 /sin (n ! πx)
can be made discontinuous at any rational point in the interval [0, 1] by a proper choice of n.
Solution: Let x be rational say p / q, where p and q are integers prime to each other. If we
take n = q, we have n ! π x = q ! . π. ( p / q ), which is an integral multiple of π and hence
sin ( n ! π x ) = 0.
But cos ( n ! πx ) = 1 or − 1 according as n ! x is an even or an odd integer.
Now sin { n ! π ( x ± h)} = sin ( n ! πx ) cos ( n ! π h) ± cos ( n ! πx ) sin ( n ! πh)
= ± sin ( n ! πh) or + sin ( n ! πh) …(1)
according as n ! x is an even or an odd integer.
(i) Let n ! x be an even integer.
1
Then f ( x + 0 ) = lim
h → 0 1 + e1 /sin {n ! π ( x + h)}
1 1
= lim = = 0, using (1),
h → 0 1+ e 1 /sin (n ! π h) 1 + e∞
1
and f ( x − 0 ) = lim
h→0 1+ e 1 /sin { n ! π ( x − h)}
1
= lim , using (1)
h → 0 1 + e − 1 /sin (n ! πh)
1 1
= = = 1.
1 + e− ∞ 1 + 0
(ii) Let n ! x be an odd integer.
1
Then f ( x + 0 ) = lim = 1,
h → 0 1 + e − 1 /sin (n ! π h)
1
and f ( x − 0 ) = lim = 0.
h → 0 1 + e1 /sin (n ! πh)
Since f ( x + 0 ) ≠ f ( x − 0 ) at any rational point x, therefore f can be made
discontinuous at any rational point by a proper choice of n.
R-28

Problem 17: If f :R→ R is a continuous function and satisfies the relation


f ( x + y ) = f ( x ) f ( y ), V x, y ∈ R , then either
f ( x) = 0 V x ∈ R
or there exists an a > 0 such that
f ( x ) = a x , V x ∈ R.
Solution: Since f ( x + y ) = f ( x ) f ( y ) V x, y ∈ R, therefore taking x = y = 0, we
have
f (0 + 0 ) = f (0 ) f (0 )
or f (0 ) = f (0 ) f (0 ).
∴ either f (0 ) = 0 or f (0 ) = 1.
Case I: Let f (0 ) = 0. Then for any x ∈ R, we have
f ( x ) = f ( x + 0 ) = f ( x ) f (0 ) = f ( x ) . 0 = 0.
Case II: Let f (0 ) = 1. Then f (1) ≠ 0. For if f (1) = 0, then
1 = f (0 ) = f (1 − 1) = f (1) f ( − 1) = 0 which is absurd.
2
Also, we have f (1) = f  +  = f   f   =  f    , from which it follows
1 1 1 1 1
 2 2  2  2   2 
that f (1) > 0.
Now we shall prove that in this case
f ( x ) = a x V x ∈ R, where a > 0.
(i) First let x be a positive integer. We have
f ( x ) = f (1 + 1 + … x times ) = f (1) f (1) … x times
= a x , where a = f (1) > 0.
(ii) Let x be any negative integer. Let x = − y so that y is a positive integer.
Then 1 = f (0 ) = f ( y − y ) = f ( y ) f ( − y ) so that
1 1
f (− y ) = = , by (i)
f ( y) a y
= a− y.
Hence, in this case also f ( x ) = a x .
(iii) Let x be any rational number. Put x = p / q where q is a positive integer and p is any
integer.
 p p p 
Now f ( p) = f  q .  = f  + + … q times 
 q q q 
q
 p  p   p 
= f   f   … q times =  f    .
 q  q   q
p
But f ( p) = a , by (i) and (ii).
q
  p  p
Hence  f    = f ( p) = a .
  q
p 
∴ f   = a p / q or f ( x) = a x .
 q
R-29

(iv) Finally let x be any real number. Let < xn > be a sequence of rational numbers
converging to x.
Since f is continuous, the sequence < f ( xn) > converges to f ( x ). Thus, we have
lim xn = x and lim f ( xn) = f ( x ).
n→ ∞ n→ ∞
Since xn is a rational number, we have by (iii)
f ( xn) = a xn .
∴ lim f ( xn) = lim a xn , or f ( x ) = a x .
n→ ∞ n→ ∞
The proof is now complete.
Note: Since a x = e x log a = e cx , we have f ( x ) = e cx where c is a constant.
Problem 18: Show that a function f is continuous at a iff for ε > 0 there exists δ > 0 such that
x1, x2 ∈ ]a − δ, a + δ [ ⇒ | f ( x1 ) − f ( x2 )|< ε.
Solution: Let f be continuous at a. Then given ε > 0, there exists a δ > 0 such that
1
|x − a|< δ ⇒ | f ( x ) − f ( a)|< ε.
2
Now | f ( x1 ) − f ( x2 )| = | f ( x1 ) − f ( a) + f ( a) − f ( x2 )|
≤ | f ( x1 ) − f ( a)| + | f ( a) − f ( x2 )|
1 1
< ε + ε = ε V x1, x2 ∈ ]a − δ, a + δ[.
2 2
Conversely, let
x1, x2 ∈] a − δ, a + δ[ ∈| f ( x1 ) − f ( x2 )|< ε.
Then to show that f is continuous at a.
Replacing x2 by a and x1 by x, we see that for every ε > 0, there exists δ > 0 such that
x, a ∈] a − δ, a + δ[ ⇒ | f ( x ) − f ( a)|< ε
i. e., |x − a|< δ ⇒ | f ( x ) − f ( a)|< ε.
Consequently f is continuous at a.
Problem 19: Discuss the nature of discontinuity of the function f, defined by
log (2 + x ) − x 2 n sin x
f ( x) = lim
n→ ∞ 1 + x 2n
at x = 1 . Show that f (0 ) and f ( π / 2) differ in sign.
 x2 < 1,
0 if

Solution: We have lim x2 n = lim ( x2 )n =  1 if x2 = 1,
n→ ∞ n→ ∞ ∞ if
 x2 > 1.

∴ if x2 < 1 i.e., − 1 < x < 1, then we have


log (2 + x ) − x2 n sin x
f ( x) = lim
n→ ∞ 1 + x2 n
R-30

log (2 + x ) − lim x2 n . sin x


n→ ∞
=
1+ lim x2 n
n→ ∞
log (2 + x ) − 0  2 2n 
=  ∵ x < 1 ⇒ lim x = 0 
1+ 0  n→ ∞ 
= log (2 + x ).
log (2 + 1) − 1. sin 1
If x = 1, f ( x ) = lim
n→ ∞ 1+1

log 3 − sin 1  2 n = 1
= ⋅ ∵ x = 1 ⇒ lim x 
2  n → ∞ 
Finally, if x > 1, then
log (2 + x ) − x2 n sin x
f ( x) = lim
n→ ∞ 1 + x2 n
1
log (2 + x ) − sin x
x 2n
= lim
n→ ∞ 1
+1
2
x n
 1 
lim   . log (2 + x ) − sin x
n → ∞  x2 n 
=
 1 
lim   +1
n → ∞  x2 n 
0 − sin x
= = − sin x.
0 +1  lim 2 n 
∵ x > 1 ⇒ n → ∞ x = ∞ 
 
Thus we see that
 log (2 + x ) , if − 1< x < 1
 log 3 − sin 1
f ( x) =  , if x =1
 2
 − sin x , if x > 1.
Now f (1 − 0 ) = lim f (1 − h) = lim log (2 + 1 − h) = log 3
, h→0 h→0
f (1 + 0 ) = lim f (1 + h) = lim [ − sin (1 + h) ] = − sin 1.
n→ ∞ n→ ∞
Since f (1 − 0 ) ≠ f (1 + 0 ), f is discontinuous at x = 1and the discontinuity is of the first
kind.
Now since π / 2 > 1, we have from the definition of f,
f ( π / 2) = − sin ( π / 2) = − 1 < 0.
Again, − 1 < 0 < 1 so that f (0 ) = log (2 + 0 ) = log 2 > 0.
Thus f (0 ) and f ( π / 2) are of opposite signs.
R-31

Note: Here, we see that though f (0 ) and f ( π / 2) are of opposite signs yet the
function f does not vanish anywhere in [0, π / 2]. The reason is that1 ∈[0, π / 2]is a point
where f is not continuous.
Problem 20: Let f and g be continuous on [ a, b ] and let f ( a) < g ( a) but f ( b ) > g ( b ) . Prove
that f ( c ) = g ( c ) for some c ∈ ] a, b [ .
Solution: Since both the functions f and g are continuous on [ a, b ], therefore the
function f − g is also continuous on [ a, b ].
Now according to question ( f − g) ( a) = f ( a) − g ( a) < 0
and ( f − g ) ( b ) = f ( b ) − g ( b ) > 0.
Since f − g is continuous on [ a, b ] and ( f − g) ( a) and ( f − g) ( b ) are of opposite signs,
therefore ( f − g) ( x ) vanishes for some c ∈ ] a, b [ i. e., there exists some c ∈ ] a, b [ such
that
( f − g) ( c ) = 0 i. e., f ( c ) − g ( c ) = 0
i. e., f ( c ) = g ( c ).
Problem 21: Let f : R → R be continuous and let f be zero on a dense set ( i. e., a set whose
intersection with every interval is non-empty). Then f is identically zero.
Solution: Let x ∈ R. Then there is a sequence < xn > such that lim xn = x and let
f ( xn) = 0, n = 1, 2, 3, .....
Then lim f ( xn) = 0. Since f is continuous, we have
n→ ∞
lim f ( xn) = f ( x ). Hence f ( x ) = 0, V x ∈ R.
n→ ∞
Problem 22: Determine the discontinuities of the function f : R → R defined by
{1 + sin ( π / x )}n − 1
f ( x) = lim , 0 < x ≤ 1.
n → ∞ {1 + sin ( π / x )}n + 1
Solution: For x = 1, 1 / 2, 1 / 3, 1 / 4, … , 1 / n, … , we have sin ( π / x ) = 0, so that at these
values of x,
(1 + 0 )n − 1 0
f ( x) = lim = lim
n → ∞ (1 + 0 )n + 1 n → ∞ 2
= lim 0 = 0.
n→ ∞
1 1
Now if < x< (m being a + ive integer), then
2m 2m − 1
π
(2m − 1) π < < 2mπ and so sin ( π / x ) is negative and consequently
x
0 < 1 + sin ( π / x ) < 1.
∴ for such values of x, lim { 1 + sin ( π / x )}n = 0.
n→ ∞
1 1
Hence f ( x ) = − 1, if < x< , m being a + ive integer.
2m 2m − 1
R-32
1 1
Again if < x< ( m being a + ive integer), then
2m + 1 2m
π
2mπ < < (2m + 1) π
x
and so sin ( π / x ) is positive and consequently 1 + sin ( π / x ) > 1.
Hence for such values of x, we have
1
1−
{1 + sin ( π / x )}n
f ( x ) = lim
n→ ∞ 1+ 1
{1 + sin ( π / x )}n
1
1−
= ∞ = 1.
1
1+

1 1
Thus f ( x ) = 1, if < x< , m being a + ive integer.
2m + 1 2m
Now f (1) = 0 and f (1 − 0 ) = − 1 and so f ( x ) has discontinuity of first kind at x = 1.
1
When x = , m = 1, 2, 3, … , we have f ( x ) = 0, f ( x − 0 ) = 1, f ( x + 0 ) = − 1.
2m
1
Again when x = , m = 1, 2, 3, … , we have
2m + 1
f ( x ) = 0, f ( x − 0 ) = − 1, f ( x + 0 ) = 1.
Hence f has discontinuities of the first kind at x = 1, 1 / 2, 1 / 3, 1 / 4, …
At x = 0, f has discontinuity of the second kind because f (0 + 0 ) does not exist.
Problem 23: Show that the function
x2 n + 2 − cos x
φ ( x) = lim
n→ ∞ x2 n + 1
does not vanish anywhere in the interval [0, 2] though φ (0 ) and φ (2) differ in sign. Discuss the
continuity of the function at x = 1.
0 − cos 0
Solution: We have φ (0 ) = lim = − 1,
n→ ∞ 0 + 1
1 − cos 1 1 − cos 1
and φ (1) = lim = ⋅
n→ ∞ 1+ 1 2
When 0 < x < 1,
x 2 n + 2 − cos x 0 − cos x
φ ( x) = lim = = − cos x
n→ ∞ x 2n + 1 0 +1
x2 [1 − {(cos x ) / x 2 n + 2 }]
and when 1 < x ≤ 2, φ ( x ) = lim = x 2.
n→ ∞ 1 + (1 / x 2 n)
− cos x, when 0 ≤ x < 1

Thus φ ( x ) = (1 − cos 1) / 2, when x = 1
 2
 x , when 1 < x ≤ 2.
R-33

We have φ (0 ) = − 1 and φ (2) = 22 = 4 so φ (0 ) and φ (2) differ in sign. But from the
above formula for φ ( x ) we observe that φ ( x ) does not vanish anywhere in the interval
[0, 2]. The reason is that φ ( x ) is discontinuous at x = 1. We have
φ (1 − 0 ) = − cos 1, φ (1) = (1 − cos 1) / 2, φ (1 + 0 ) = 1,
showing that φ ( x ) has discontinuity of the first kind at x = 1.
Problem 24: Examine for continuity the function f defined by
e x − x n sin x  π
f ( x) = lim 0 ≤ x ≤ 
n→ ∞ 1 + xn  2

at x = 1. Explain why the function f does not vanish anywhere in [0, π / 2] although
f (0 ) . f ( π / 2) < 0.
Solution: When 0 ≤ x < 1, we have
e x − x n sin x
f ( x) = lim
n→ ∞ 1 + xn
e x − 0 . sin x
= = e x,
1+ 0

e − sin 1
at x = 1, f ( x) = lim
n→ ∞ 1+1

e − sin 1
=
2
π ( e x / x n) − sin x
and when 1 < x ≤ , f ( x ) = lim = − sin x.
2 n → ∞ 1 + (1 / x n)

e x , when 0 ≤ x < 1

Thus f ( x ) = ( e − sin 1) / 2, when x = 1
− sin x, when 1 < x ≤ π / 2.

We have f (1 − 0 ) = lim f ( x ) = lim e x = e,


x → 1− x →1
f (1 + 0 ) = lim f ( x )
x → 1+

= lim ( − sin x ) = − sin 1


x →1

1
and f (1) = ( e − sin 1).
2
We have f (1 − 0 ) ≠ f (1) and f (1) ≠ f (1 + 0 )
and so f ( x ) has discontinuity of the first kind at x = 1. The jump of the function at this
point is f (1 − 0 ) − f (1 + 0 ) = e − ( − sin 1) = e + sin 1.

We have f (0 ) = e0 = 1 and f ( π / 2) = − sin ( π / 2) = − 1.


R-34

Thus f (0 ) . f ( π / 2) = − 1 < 0 i. e., f (0 ) and f ( π / 2) are of opposite signs. From the


formula obtained for f ( x ) we observe that f ( x ) does not vanish anywhere in [0, π / 2]
although f (0 ) and f ( π / 2) are of opposite signs. The reason is that the function f is
not continuous in the interval [0, π / 2] because it is discontinuous at the point x = 1 of
this interval.

Hints to Objective Type Questions


Multiple Choice Questions:
1. (d). Here f (0 + 0 )
lim lim lim sin 5h lim  sin 5h  5 5
= f (0 + h) = f ( h) = =  ⋅ =
h→0 h→0 h → 0 3h h → 0  5h  3 3

and f (0 − 0 )
lim lim lim sin 5( − h) lim  sin 5h  5 5
= f (0 − h) = f ( − h) = =  ⋅ = ⋅
h→0 h→0 h → 0 3( − h) h → 0  5h  3 3
For the function to be continuous at x = 0, we have
f (0 + 0 ) = f (0 − 0 ) = f (0 ).
∴ K = 5 / 3.
2. (c). Theorem 1, article 5.
3. (a). See Problem 1 of Comprehensive Problems 2.
4. (b). See Problem 22 of Comprehensive Problems 2.
5. (a), (d). See Problem 4 of Comprehensive Problems 2.
6. (a), (b). See Ex. 18.

Fill in the Blanks:


1. See article 1 an alternative definition of continuity.
2. If 3 < x < 4, then [ x ] = 3. So, if 3 < x < 4, then f ( x ) = x − 3.
3. See article 2, part (i).
sin x
4. The function f ( x ) = is defined for all real numbers x except x = 0. So,
x
domain f = R − { 0}.
5. The given function f ( x ) is defined for all real numbers x. So, domain f = R.
6. Bounded. See Theorem 2, of article 6.
7. Uniformly continuous. See Theorem 2, of article 7
8. Discontinuous, See Ex. 22.
9. Uniformly. See Ex, 16.
10. Continuous, uniformly continuous. See Ex. 15.
11. Discontinuous. See Problem 24 of Comprehensive Problems 2.
R-35

True or False:
lim lim sin x
1. F. We have f ( x) = =1
x→0 x→0 x
But, f(0 ) = 2
lim
Since, f ( x ) ≠ f (0 ), therefore f ( x ) is discontinuous at x = 0.
x→0
lim lim
2. T . We have f ( x) = ( − sin x ) [∵ f ( x ) = − sin x, when x < 0]
x→0 − x→0 −
=0
lim lim
Again, f ( x) = sin x [∵ f ( x ) = sin x, when x > 0]
x→0 + x→0 +
lim lim lim
Since f ( x) = f ( x ) = 0, therefore f ( x ) = 0.
x→0 − x→0 + x→0
Also, f(0 ) = sin0 = 0. [∵ f ( x ) = sin x, when x = 0]
lim
Since f ( x ) = f (0 ), therefore f ( x ) is continuous at x = 0.
x→0
3. F. See problem 15, part (i) of Comprehensive Problems 1.
4. T . See theorem 5 of article 4.
lim lim sin x
5. T . The given statement is true. We have f ( x) = = 1.
x→0 x→0 x
Also, f(0 ) = 1.
lim
Since, f ( x ) = f (0 ), therefore f ( x ) is continuous at x = 0.
x→0
lim lim
6. F. We have f ( x) = 1 [∵ f ( x ) = 1, when x < 1 ]
x → 1− x → 1−
lim lim
Again, f ( x) = (2 − x ) [∵ f ( x ) = 2 − x, when 1 ≤ x < 2]
x → 1+ x → 1+
= 2 − 1 = 1.
Also, f(1) = 2 − 1 = 1.
lim lim
Since, f ( x ) = f (1) f ( x ), therefore f ( x ) is continuous at x = 1.
x → 1− x → 1+
7. T. See Theorem 2, article 6.
8. T. See Theorem 1 of article 7.
9. F. See Ex.14.
10. T. See Theorem 2 of article 7.
11. F. Refer Problem 10 of Comprehensive Problems 2.
12. T. See Problem 8 of Comprehensive Problems 2.
13. T. See Problem 5 of Comprehensive Problems 2.
14. F. See Ex. 12.
❍❍❍
R-36

Chapter-2
Differentiability

Comprehensive Problems 1
Problem 1: Show that f ( x ) = | x − 1|, 0 ≤ x ≤ 2 is not derivable at x = 1. Is it continuous in
[0, 2] ?
1 − x, when 0 ≤ x ≤ 1
Solution: We have f ( x ) = 
 x − 1, when 1 ≤ x ≤ 2.
To test f ( x ) for differentiability at x = 1 .
We have
lim f (1 + h) − f (1) lim (1 + h − 1) − 0
R f ′ (1) = =
h→0 h h →0 h
lim h lim
= = 1 = 1;
h→0 h h→0
lim f (1 − h) − f (1) lim 1 − (1 − h) − 0
and L f ′ (1) = =
h→0 −h h → 0 −h
lim h lim
= = − 1 = − 1.
h→0 −h h→0
Since R f ′ (1) ≠ L f ′ (1), the function f ( x ) is not differentiable at x = 1.
To test f ( x ) for continuity in [0, 2].
When 0 ≤ x < 1, f ( x ) = 1 − x which is a polynomial and when 1 < x ≤ 2, f ( x ) = x − 1
which is also a polynomial.
Now a polynomial function is continuous at each point of its domain. Therefore f ( x )
is continuous when x ≤ 0 < 1 and also when 1 < x ≤ 2 .
Now to check f ( x ) for continuity at x = 1.
lim lim lim
We have f (1 − 0 ) = f (1 − h) = { 1 − (1 − h) } = h = 0,
h→0 h→0 h→0
lim lim lim
f (1 + 0 ) = f (1 + h) = { (1 + h) − 1 } = h=0
h→0 h→0 h→0
and f (1) = 0.
Since f (1 − 0 ) = f (1) = f (1 + 0 ), f ( x ) is continuous at x = 1 .
Thus f ( x ) is continuous at each point of [0, 2] and so f ( x ) is continuous in [0, 2] .
R-37
x
Problem 2 (a): If f ( x ) = , x ≠ 0, f (0 ) = 0, show that f is continuous at x = 0 , but
1 + e1 / x
f ′ (0 ) does not exist. (Lucknow 2005, 10; Gorakhpur 13; Purvanchal 14)
lim lim
Solution: We have f (0 − 0 ) = f (0 − h) = f ( − h), h > 0
h→0 h→0
lim −h 0
= = = 0,
h → 0 1 + e− 1/ h 1 + 0
lim lim
f (0 + 0 ) = f (0 + h) = f ( h), h > 0
h→0 h→0
lim h 1
= =0⋅ =0 ⋅0 =0
h → 0 1 + e1 / h 1+ ∞
and f (0 ) = 0.
Since f (0 − 0 ) = f (0 ) = f (0 + 0 ), therefore f ( x ) is continuous at x = 0.
We now proceed to find the derivative of f ( x ) at x = 0. We have
lim f (0 + h) − f (0 ) lim f ( h) − f (0 )
R f ′ (0 ) = = ,h>0
h→0 h h→0 h
h
−0
lim 1 + e1 / h lim 1 1
= = = =0
h→0 h h → 0 1 + e1 / h 1 + ∞
lim f (0 − h) − f (0 ) lim f ( − h) − f (0 )
and L f ′ (0 ) = = ,h>0
h→0 −h h→0 −h
−h
−0
lim 1 + e− 1/ h
=
h→0 −h
lim 1 1 1
= = = = 1.
h → 0 1 + e− 1/ h 1 + e− ∞ 1 + 0
Since R f ′ (0 ) ≠ L f ′ (0 ), the derivative of f ( x ) at x = 0 does not exist.

x e1 / x
Problem 2 (b): If f ( x ) = for x ≠ 0 and f (0 ) = 0, show that f ( x ) is continuous
1 + e1 / x
at x = 0, but f ′ (0 ) does not exist. (Lucknow 2006)
lim lim
Solution: We have f (0 − 0 ) = f (0 − h) = f ( − h), h > 0
h→0 h→0
lim − he − 1 / h 0
= = = 0,
h → 0 1 + e− 1/ h 1 + 0
lim lim
f (0 + 0 ) = f (0 + h) = f ( h), h > 0
h→0 h→0
lim he1 / h lim h 0
= = = =0
h → 0 1 + e1 / h h→0 e− 1/ h + 1 0 +1
and f (0 ) = 0.
Since f (0 − 0 ) = f (0 ) = f (0 + 0 ), f ( x ) is continuous at x = 0.
R-38

We now proceed to find the derivative of f ( x ) at x = 0.


lim f (0 + h) − f (0 ) lim f ( h) − f (0 )
We have R f ′ (0 ) = =
h→0 h h →0 h
he1 / h
−0
lim 1 + e1 / h lim e1 / h
= =
h→0 h h → 0 1 + e1 / h
lim 1 1
= = =1
h→0 e− 1/ h + 1 0 +1
lim f (0 − h) − f (0 ) lim f ( − h) − f (0 )
and L f ′ (0 ) = =
h→0 −h h→0 −h
− h e− 1/ h
−0
lim 1 + e− 1/ h lim e− 1/ h 0
= = = = 0.
h→0 −h h → 0 1 + e− 1/ h 1 + 0

Since R f ′ (0 ) ≠ L f ′ (0 ), the derivative of f ( x ) at x = 0 does not exist.

Problem 3: A function φ is defined as follows : φ ( x ) = − x for x ≤ 0, φ ( x ) = x for x ≥ 0.


Test the character of the function at x = 0 as regards continuity and differentiability.
lim lim
Solution: We have φ (0 − 0 ) = φ (0 − h) = φ ( − h), h > 0
h→0 h→0
lim lim
= − ( − h) = h=0
h→0 h→0
lim lim lim
φ (0 + 0 ) = φ (0 + h) = φ ( h) = h=0
h→0 h→0 h→0
and φ (0 ) = 0.
Since φ (0 − 0 ) = φ (0 ) = φ (0 + 0 ), φ ( x ) is continuous at x = 0.
lim φ (0 + h) − φ (0 ) lim φ ( h) − φ (0 )
Again R φ′ (0 ) = =
h→0 h h→0 h
limh−0 lim
= = 1=1
h→0 h h→0
lim φ (0 − h) − φ (0 ) lim φ ( − h) − φ (0 )
and L φ′ (0 ) = = ,h>0
h→0 −h h→0 −h
lim − ( − h) − 0 lim h lim
= = = − 1 = − 1.
h→0 −h h→0 −h h→0
Since Rφ′ (0 ) ≠ Lφ′ (0 ), φ ( x ) is not differentiable at x = 0.

Problem 4: Show that the function f defined on R by

f ( x ) = | x − 1| + 2 | x − 2 | + 3 | x − 3 |
is continuous but not differentiable at the points 1, 2, and 3. (Bundelkhand 2009)
R-39

Solution: From the definition of the function f, we have


f ( x ) = 1 − x + 2 (2 − x ) + 3 (3 − x ) = 14 − 6 x when x ≤ 1
f ( x ) = x − 1 + 2 (2 − x ) + 3 (3 − x ) = 12 − 4 x when 1 ≤ x ≤ 2
f ( x ) = x − 1 + 2 ( x − 2) + 3 (3 − x ) = 4 when 2 ≤ x ≤ 3
and f ( x ) = x − 1 + 2 ( x − 2) + 3 ( x − 3) = 6 x − 14 when x ≥ 3.
Continuity of f ( x ) at x = 1 .
We have f (1) = 14 − 6 = 8,
lim lim
f (1 − 0 ) = f ( x) = (14 − 6 x ) = 8
x →1− x →1
lim lim
and f (1 + 0 ) = f ( x) = (12 − 4 x ) = 8.
x →1+ x →1
Since f (1 − 0 ) = f (1) = f (1 + 0 ), f ( x ) is continuous at x = 1.
Differentiability of f ( x ) at x = 1.
lim f (1 + h) − f (1)
We have Rf ′ (1) =
h→0 h
lim 12 − 4 (1 + h) − 8 lim − 4h lim
= = = −4= −4
h→0 h h→0 h h→0
lim f (1 − h) − f (1)
and L f ′ (1) = ,h>0
h→0 −h
lim 14 − 6 (1 − h) − 8 lim 6h lim
= = = − 6 = − 6.
h→0 −h h→0 −h h→0
Since R f ′ (1) ≠ L f ′ (1), the function f is not differentiable at x = 1.
Similarly check f ( x ) for continuity and differentiability at x = 2 and at x = 3.

Problem 5: Show that the function f ( x ) = x, 0 < x ≤ 1


= x − 1, 1 < x ≤ 2
has no derivative at x = 1.
Solution: We have
lim f (1 + h) − f (1)
R f ′ (1) =
h→0 h
lim (1 + h) − 1 − 1 lim h − 1
= = =−∞
h→0 h h→0 h
lim f (1 − h) − f (1)
and L f ′ (1) = ,h>0
h→0 −h
lim (1 − h) − 1 lim
= = 1 =1.
h→0 −h h→0
Since R f ′ (1) ≠ L f ′ (1), the function f ( x ) has no derivative at x = 1 .
Remark: Here f (1 − 0 ) = 1, f (1) = 1, f (1 + 0 ) = 0 and so f ( x ) is not continuous at
x = 1 and consequently f ( x ) is not differentiable at x = 1.
R-40

Problem 6: Show that the function f ( x ) = x2 − 1, x ≥ 1 = 1 − x, x < 1


has no derivative at x = 1.
lim f (1 − h) − f (1)
Solution: We have L f ′ (1) = ,h>0
h→0 −h
lim 1 − (1 − h) − (1 − 1) lim h
= =
h→0 −h h→0 −h
lim
= −1= −1
h→0
lim f (1 + h) − f (1) lim (1 + h)2 − 1 − (1 − 1)
and R f ′ (1) = =
h→0 h h →0 h
lim h2 + 2h lim h ( h + 2) lim
= = = ( h + 2) = 2.
h→0 h h → 0 h h →0
Since L f ′ (1) ≠ R f ′ (1), f ( x ) is not differentiable at x = 1 .

Problem 7: The following limits are derivatives of certain functions at a certain point.
Determine these functions and the points.
lim log x − log 2 lim √ ( a + h) − √ a
( i) ( ii) ⋅
x→2 x−2 h→0 h
Solution: We know that
lim f ( x ) − f ( a) lim f ( a + h) − f ( a )
f ′ ( a) = = ⋅
x→ a x−a h→0 h
(i) The function is f ( x ) = log x and the point is x = 2 .
(ii) The function is f ( x ) = √ x and the point is x = a .

Problem 8: Let f ( x ) = x2 sin ( x − 4 /3 ) except when x = 0 and f (0 ) = 0. Prove that f ( x )


has zero as a derivative at x = 0.
lim f (0 + h) − f (0 )
Solution: We have R f ′ (0 ) = ,h>0
h→0 h
lim f ( h) − f (0 ) lim h2 sin ( h− 4 /3 ) − 0
= =
h→0 h h→0 h
lim
= h sin ( h− 4 /3 ) = 0.
h→0
 lim − 4 /3 )| ≤ 1 when h ≠ 0 
∵ h → 0 h = 0 and |sin ( h 
 
lim f (0 − h) − f (0 )
Again L f ′ (0 ) =
h→0 −h
lim f ( − h) − f (0 ) lim ( − h)2 sin { ( − h)− 4 /3 } − 0
= =
h→0 −h h → 0 −h
lim − 4 / 3
= [( − h) . sin { ( − h) }] = 0, as before.
h→0
Since R f ′ (0 ) = L f ′ (0 ) = 0, the function f has zero as a derivative at x = 0.
R-41

Problem 9: A function φ is defined as follows :


φ ( x ) = 1 + x if x ≤ 2 , φ ( x ) = 5 − x if x > 2.
Test the character of the function at x = 2 as regards its continuity and differentiability.
(Avadh 2007)
Solution: We have φ (2) = 1 + 2 = 3,
lim lim
φ (2 − 0 ) = φ ( x) = (1 + x ) = 3
x→2 − x→2
lim lim
and φ (2 + 0 ) = φ ( x) = (5 − x ) = 3.
x→2 + x→2
Since φ (2 − 0 ) = φ (2) = φ (2 + 0 ), the function φ is continuous at x = 2.
lim φ (2 + h) − φ (2) lim 5 − (2 + h) − 3
Again R φ′ (2) = =
h→0 h h→0 h
lim −h lim
= = −1= −1
h→0 h h →0
lim φ (2 − h) − φ (2) lim 1 + (2 − h) − 3
and L φ′ (2) = =
h→0 −h h→0 −h
lim −h lim
= = 1 = 1.
h→0 −h h→0
Since R φ′ (2) ≠ L φ′ (2), the function φ is not differentiable at x = 2.
Problem 10: Examine the following curve for continuity and differentiability at x = 0 and
x = 1:
y = x2 for x≤0
y =1 for 0 < x ≤1
y = 1 / x for x > 1.
Also draw the graph of the function. (Meerut 2003, 04B, 09B)
Solution: Let y = f ( x ). We need to test f ( x ) for continuity and differentiability at
the points x = 0 and 1. It is obviously continuous and differentiable at all other points.
At x = 0. We have f (0 ) = 02 = 0;
lim lim lim
f (0 + 0 ) = f (0 + h) = f ( h) = 1 = 1,
h→0 h→0 h→0
lim lim lim
and f (0 − 0 ) = f (0 − h) = f ( − h) = ( − h)2
h→0 h→0 h→0
lim
= h2 = 0.
h→0
Thus f (0 − 0 ) = f (0 ) ≠ f (0 + 0 ) and so the function f ( x ) is not continuous at x = 0.
Consequently it is also not differentiable at x = 0.
Here f ( x ) is continuous at x = 0 from the left.
R-42

lim lim
At x = 1 . We have f (1 − 0 ) = f ( x) = 1 = 1, f (1) = 1
x →1− x →1
lim lim 1
and f (1 + 0 ) = f ( x) = = 1.
x →1+ x →1 x
Since f (1 − 0 ) = f (1) = f (1 + 0 ), f ( x ) is continuous at x = 1.
lim f (1 − h) − f (1)
Now L f ′ (1) =
h→0 −h
lim 1−1 lim
= = 0 =0
h→0 − h h→0
1
−1
lim f (1 + h) − f (1) lim 1+ h
and R f ′ (1) = =
h→0 h h→0 h
lim 1−1− h lim −1
= = = − 1.
h→0 h (1 + h) h→0 1+ h
Since L f ′ (1) ≠ R f ′ (1), f ( x ) is not
differentiable at x = 1.
The graph of the function consists of the
following curves :
y = x2 for x ≤ 0, (parabola)
y = 1 for 0 < x ≤ 1, (straight line)
y = 1 / x for x > 1, (rectangular hyperbola).
Problem 11: A function f ( x ) is defined as follows :
f ( x) = 1 + x for x ≤ 0,
f ( x) = x for 0 < x < 1,
f ( x) = 2 − x for 1 ≤ x ≤ 2,
f ( x ) = 3 x − x2 for x > 2 .
Discuss the continuity of f ( x ) and the existence of f ′ ( x ) at x = 0, 1 and 2.
Solution: At x = 0. We have f (0 ) = 1 + 0 = 1,
lim lim
f (0 − 0 ) = f ( x) = (1 + x ) = 1
x→0 − x→0
lim lim
f (0 + 0 ) = f ( x) = x = 0.
x→0 + x→0
lim
Since f (0 − 0 ) ≠ f (0 + 0 ), therefore f ( x ) does not exist and so f ( x ) is not
x→0
continuous at x = 0. Consequently f ( x ) is also not differentiable at x = 0.
At x = 1 . We have f (1) = 2 − 1 = 1,
lim lim
f (1 − 0 ) = f ( x) = x =1
x →1− x →1
lim lim
and f (1 + 0 ) = f ( x) = (2 − x ) = 1 .
x →1+ x →1
R-43

Since f (1 − 0 ) = f (1) = f (1 + 0 ), f ( x ) is continuous at x = 1.


lim f (1 − h) − f (1) lim (1 − h) − 1
Now L f ′ (1) = =
h→0 −h h →0 −h
lim −h lim
= = 1=1
h→0 −h h→0
lim f (1 + h) − f (1) lim 2 − (1 + h) − 1
and R f ′ (1) = =
h→0 h h→0 h
lim −h lim
= = − 1 = − 1.
h→0 h h→0
Since L f ′ (1) ≠ R f ′ (1), f ( x ) is not differentiable at x = 1 and so f ′ (1) does not exist.
At x = 2. We have f (2) = 2 − 2 = 0,
lim lim
f (2 − 0 ) = f ( x) = (2 − x ) = 0
x→2 − x→2
lim lim
and f (2 + 0 ) = f ( x) = (3 x − x2 ) = 2 .
x→2 + x→2
Since f (2 − 0 ) ≠ f (2 + 0 ), f ( x ) is not continuous at x = 2 and consequently it is also
not differentiable at x = 2.
Problem 12: Discuss the continuity and differentiability of the following function :
f ( x ) = x2 for x < − 2
f ( x) = 4 for − 2 ≤ x ≤ 2
f ( x ) = x2 for x > 2 .
Also draw the graph. (Meerut 2007, 10B)
Solution: When x < − 2, f ( x ) = x and when x > 2, f ( x ) = x2 . Thus f ( x ) is a
2

polynomial when x < − 2 or when x > 2 and a polynomial function is continuous as


well as differentiable at each point of its domain. So f ( x ) is continuous as well as
differentiable at every point where x < − 2 or x > 2.
Again when − 2 < x < 2, f ( x ) = 4 i. e., f ( x ) is a constant function. So f ( x ) is
continuous as well as differentiable at every point where − 2 < x < 2 .
Now it remains to check the continuity and differentiability of f ( x ) at x = − 2 and
2.
At x = − 2. We have f ( − 2) = 4,
lim lim
f (− 2 − 0 ) = f ( x) = x2 = ( − 2)2 = 4
x→ −2 − x→ −2
lim lim
and f (− 2 + 0 ) = f ( x) = 4 = 4.
x→ −2 + x→ −2
Since f ( − 2 − 0 ) = f ( − 2) = f ( − 2 + 0 ), f ( x ) is continuous at x = − 2 .
R-44

lim f ( − 2 − h) − f ( − 2 )
Now Lf ′ ( − 2) = ,h>0
h→0 −h
lim ( − 2 − h)2 − 4 lim 4h + h2 lim h (4 + h)
= = =
h→0 −h h→0 −h h→0 −h
lim
= − (4 + h) = − 4
h→0
lim f ( − 2 + h) − f ( − 2 )
and R f ′ ( − 2) = ,h>0
h→0 h
lim 4−4 lim
= = 0 = 0.
h→0 h h→0
Since L f ′ ( − 2) ≠ R f ′ ( − 2), f ( x ) is not differentiable at x = − 2.
At x = 2. Here proceeding as above,
f (2 − 0 ) = 4 = f (2) = f (2 + 0 ) and so f ( x ) is continuous at x = 2.
Again Lf ′ (2) = 0, Rf ′ (2) = 4 and thus L f ′ (2) ≠ R f ′ (2)
and so f ( x ) is not differentiable at x = 2.
To draw the graph of the function, put y = f ( x ).
Then the graph of the function consists of the
following curves :
y = x2 when x < − 2 (a parabola)
y = 4 when − 2 ≤ x ≤ 2 (a straight
line)
y = x2 when x > 2 (a parabola).

Problem 13: A function f ( x ) is defined as follows :


f ( x ) = x for 0 ≤ x ≤ 1, f ( x ) = 2 − x for x ≥ 1.
Test the character of the function at x = 1 as regards the continuity and differentiability.
(Meerut 2003)
Solution: Proceed yourself.
Here f (1 − 0 ) = 1 = f (1) = f (1 + 0 ) and so f ( x ) is continuous at x = 1.
Again L f ′ (1) = 1 and R f ′ (1) = − 1 and thus L f ′ (1) ≠ R f ′ (1) and so f ( x ) is not
differentiable at x = 1.

Problem 14: Examine the function defined by


f ( x ) = x2 cos ( e1 / x ), x ≠ 0, f (0 ) = 0

with regard to ( i) continuity ( ii) differentiability in the interval ] − 1, 1 [.


Solution: If c ∈ ] − 1, 1 [ and c ≠ 0, then
lim lim
f ( x) = x2 cos ( e1 / x ) = c2 cos ( e1 / c ) = f ( c ).
x→ c x→ c
∴ f ( x ) is continuous at every point x = c if c ≠ 0.
R-45

Again f ′ ( x ) = 2 x cos ( e1 / x ) − x2 { sin ( e1 / x ) } . e1 / x . ( − 1 / x2 )


= 2 x cos ( e1 / x ) + e1 / x sin ( e1 / x )
which exists at every point x = c if c ≠ 0. ∴ f ( x ) is differentiable at every point
x = c if c ≠ 0.
Continuity of f ( x ) at x = 0. We have f (0 ) = 0,
lim lim
f (0 + 0 ) = f (0 + h) = f ( h), h > 0
h→0 h→0
lim  lim
= h2 cos ( e1 / h ) = 0 ∵ h→0 h2 = 0 and
h→0 
| cos ( e1 / h )|≤ 1 when h ≠ 0 i. e., cos ( e1 / h )

is bounded in some deleted neighbourhood of zero

lim lim
and f (0 − 0 ) = f (0 − h) = f ( − h), h > 0
h→0 h→0
lim lim
= ( − h)2 cos ( e − 1 / h ) = h2 cos ( e − 1 / h ) = 0,
h→0 h→0
as discussed above.
Since f (0 − 0 ) = f (0 ) = f (0 + 0 ), f ( x ) is continuous at x = 0.
Differentiability of f ( x ) at x = 0. We have
lim f (0 + h) − f (0 ) lim f ( h) − f (0 )
Rf ′ (0 ) = = ,h>0
h→0 h h→0 h
lim h2 cos ( e1 / h ) − 0 lim
= = h cos ( e1 / h ) = 0
h→0 h h→0
lim f (0 − h) − f (0 ) lim f ( − h) − f (0 )
and Lf ′ (0 ) = = ,h>0
h→0 −h h→0 −h
lim ( − h)2 cos ( e − 1 / h ) − 0 lim
= = − h cos ( e − 1 / h ) = 0.
h→0 −h h →0
Since L f ′ (0 ) = R f ′ (0 ), f ( x ) is differentiable at x = 0 and f ′ (0 ) = 0.
Thus f ( x ) is continuous as well as differentiable throughout R.

Problem 15(a): Define continuity and differentiability of a function at a given point. If


a function possesses a finite differential coefficient at a point, show that it is continuous at
this point. Is the converse true ? Give example in support of your answer.
Solution: A function f ( x ) is said to be continuous at a point a of its domain if
lim
f ( x ) = f ( a).
x→ a
For definition of differentiability of a function at a point see article 1.
R-46

If a function f ( x ) possesses a finite differential coefficient f ′ ( x0 ) at a point x = x0 , it


is continuous at x = x0 . For proof refer article 4. The converse is not true.
Consider the function f ( x ) = | x |, x ∈ R.

 x, x ≥ 0
Then f ( x) = 
− x, x ≤ 0.
This function is continuous at x = 0 but is not differentiable at x = 0. For complete
solution see solved example 1 after article 7.
Remark: Consider the function f ( x ) = x sin (1 / x ), x ≠ 0 and f (0 ) = 0.
This function is continuous at x = 0 but is not differentiable at x = 0. For complete
solution see article 4.

Problem 15(b): What do you understand by the derivative of a real valued function at the
point ? Show that f ( x ) = x sin   , x ≠ 0, f (0 ) = 0 is not derivable at x = 0.
1
 x
Solution: Refer remark of Problem 15(a).

Problem 15(c): Prove that if a function f ( x ) possesses a finite derivative in a closed interval
[ a, b ], then f ( x ) is continuous in [ a, b ].
Solution: It is given that f ( x ) possesses a finite derivative at each point of [ a, b ] and
to prove that f ( x ) is continuous in [ a, b ] i. e., f ( x ) is continuous at each point c of
[ a, b ].
lim f ( x) − f (c )
By hypothesis, f ′ ( c ) = exists and is finite.
x→ c x−c
f ( x) − f (c )
We can write f ( x) − f (c ) = ⋅ ( x − c ), if x ≠ c. ...(1)
x−c
Taking limit of both sides of (1) as x → c, we get
lim lim  f ( x) − f (c ) 
{ f ( x) − f (c ) } =  ⋅ ( x − c )
x→ c x→ c  x − c 
lim f ( x ) − f ( c ) lim
= ⋅ ( x − c ) = f ′ ( c ) . 0 = 0.
x→ c x−c x→ c
lim lim lim
∴ { f ( x) − f (c ) } = 0 ⇒ f ( x) − f (c ) = 0
x→ c x→ c x→ c
lim lim
⇒ f ( x) − f (c ) = 0 ⇒ f ( x) = f (c )
x→ c x→ c
⇒ f ( x ) is continuous at x = c.
R-47

Comprehensive Problems 2
Problem 1: (i) State Rolle’s theorem. (Kanpur 2005, 08; Lucknow 07)
(ii) Verify Rolle’s theorem when f ( x ) = e x sin x, a = 0, b = π.
(Gorakhpur 2012)
Solution: (i) See article 8.
(ii) The function f ( x ) = e x sin x is continuous as well as differentiable on the whole
R. So f ( x ) is continuous in [0, π ] and differentiable in ] 0, π [.
We have f (0 ) = e 0 sin 0 = 0 and f ( π ) = e π sin π = 0.
∴ f (0 ) = f ( π ).
Thus f ( x ) satisfies all the three conditions of Rolle’s theorem in [ 0, π ]. Therefore
there must exist at least one number, say c, in the open interval ] 0, π [ for which
f ′ ( c ) = 0.
Now f ′ ( x ) = e x sin x + e x cos x = e x (sin x + cos x )

= e x . √ 2 .  cos x 
1 1
sin x +
 √2 √2 

= √ 2 . e x sin  x + π  ⋅
1
 4 

f ′ ( x ) = 0 we get sin  x + π  = 0
1
From [∵ e x ≠ 0, V x ∈ R]
 4 
1
⇒ x + π = 0, ± π, ± 2 π, … .
4
1 3
Out of these values x = π − π = π lies in the open interval ] 0, π [. Thus the Rolle’s
4 4
theorem is verified.
Problem 2: Verify Rolle’s theorem for the following functions :
( i) f ( x ) = ( x − 4)5 ( x − 3)4 in the interval [3, 4].
( ii) f ( x ) = x3 − 6 x2 + 11x − 6.
( iii) f ( x ) = x3 − 4 x in [ − 2, 2]. (Kanpur 2007)
( iv ) f ( x ) = e x ( sin x − cos x ) in [ π / 4, 5 π / 4]. (Meerut 2013B)
(v) f ( x ) = 10 x − x2 in [ 0, 10 ]. (Kanpur 2006)
Solution: (i) We have f ( x ) = ( x − 4)5 ( x − 3)4 which is a polynomial in x of degree 9
and a polynomial function is continuous as well as differentiable on the whole R. So
f ( x ) is continuous in [3, 4] and differentiable in ] 3, 4 [ .
Also f (3) = 0 and f (4) = 0 so that f (3) = f (4).
Thus all the three conditions of Rolle’s theorem are satisfied so that there is at least
one value of x in the open interval ] 3, 4 [ where f ′ ( x ) = 0.
R-48

Now f ′ ( x ) = 5 ( x − 4)4 ( x − 3)4 + 4 ( x − 3)3 ( x − 4)5


= ( x − 3)3 ( x − 4)4 [5 ( x − 3) + 4 ( x − 4)]
= ( x − 3)3 ( x − 4)4 (9 x − 31).
Solving the equation f ′ ( x ) = 0, we get x = 3, 4, 31 / 9.
4
Out of these values the value 31 / 9 i. e., 3 is a point which lies in the open interval
9
4
] 3, 4 [ since 3 < 3 < 4. Hence the Rolle’s theorem is verified.
9
(ii) We have f ( x ) = x3 − 6 x2 + 11x − 6 which is a polynomial in x of degree 3 and
so it is continuous as well as differentiable for all real values of x.
Now f ( x ) = 0 gives x3 − 6 x2 + 11x − 6 = 0
or x2 ( x − 1) − 5 x ( x − 1) + 6 ( x − 1) = 0
or ( x − 1) ( x2 − 5 x + 6) = 0
or ( x − 1) ( x − 2) ( x − 3) = 0 i. e., x = 1, 2, 3.
Thus f (1) = 0 = f (2) = f (3).
If we take the interval [1, 3], then all the three conditions of Rolle’s theorem are
satisfied in this interval. Consequently there is at least one value of x in the open
interval ] 1, 3 [ for which f ′ ( x ) = 0.
Now f ′ ( x ) = 0 ⇒ 3 x2 − 12 x + 11 = 0
12 ± √ (144 − 12 . 11) 6 ± √ 3 1
⇒ x= = =2± ⋅
6 3 √3
1 1
Since both the points x = 2 + and x = 2 −
√3 √3
lie in the open interval ] 1, 3 [, Rolle’s theorem is verified.
If we take the interval [1, 2], then the point x = 2 − (1 / √ 3) lies in the open interval
] 1, 2 [ and f ′ ( x ) = 0 at this point. If we take the interval [2, 3], then the point
x = 2 + (1 / √ 3) lies in the open interval ] 2, 3 [ and f ′ ( x ) = 0 at this point.
(iii) Here f ( x ) = x3 − 4 x which is a polynomial in x of degree 3 and so it is
continuous and differentiable for every real value of x. Also f ( − 2) = 0 = f (2).
Thus f ( x ) satisfies all the three conditions of Rolle’s theorem in [ − 2, 2].
∴ there must exist at least one number, say c, in the open interval ] − 2, 2 [ for which
f ′ ( c ) = 0.
Now f ′ ( x ) = 0 gives 3 x2 − 4 = 0
2
or x=± = ± 1⋅ 55 (approx).
√3
Both these values lie in the open interval ] − 2, 2 [ and thus the Rolle’s theorem is
verified.
R-49

(iv) Here f ( x ) = e x (sin x − cos x ).


We have f ( π / 4) = e π /4 { sin ( π / 4) − cos ( π / 4) } = e π /4 { (1 / √ 2) − (1 / √ 2) } = 0
5π  sin 5 π − cos 5 π  = e5 π /4  − 1 + 1  = 0.
and f   =e
5 π /4
 4   4 4   √ 2 √ 2 

∴ f ( π / 4) = f (5 π / 4).
The function f ( x ) is continuous as well as differentiable for all real values of x and so
f ( x ) is continuous in [ π / 4, 5 π / 4] and differentiable in ] π / 4, 5 π / 4 [ . Thus f ( x )
satisfies all the three conditions of Rolle’s theorem in [ π / 4, 5 π / 4].
∴ there must exist at least one real number x in the open interval ] π / 4, 5 π / 4 [ at
which f ′ ( x ) = 0.
Now f ′ ( x ) = e x (cos x + sin x ) + e x (sin x − cos x ) = 2e x sin x.
From f ′ ( x ) = 0 we get 2e x sin x = 0
or sin x = 0 [∵ e x ≠ 0 V x ∈ R]
or x = 0, ± π, ± 2 π, ± 3 π, …
Out of these values x = π lies in the open interval ] π / 4, 5 π / 4 [ . Thus the Rolle’s
theorem is verified.
(v) Here f ( x ) = 10 x − x2 which is a polynomial in x of degree 2 and so it is continuous
and differentiable for every real value of x
Also f (0 ) = 0 = f (10 )
Thus f ( x ) satisfies all, the three conditions of Rolle’s theorem in [0, 10].
∴ There must exist at least one number say c in the open interval ] 0, 10 [ for which
f ′ ( c ) = 0.
10
Now f ′( x ) = 0 gives 10 − 2 x = 0 or x = = 5 this value lies in the open interval
2
] 0, 10 [ and thus the Rolle’s theorem is verified.
Problem 3: Discuss the applicability of Rolle’s theorem to the function
f ( x ) = x2 + 1, when 0 ≤ x ≤ 1
= 3 − x, when 1 < x ≤ 2.
Solution: Here the function f ( x ) is defined in the closed interval [0, 2].
We have f (0 ) = 02 + 1 = 1, f (2) = 3 − 2 = 1 so that f (0 ) = f (2).
When 0 ≤ x < 1, f ( x ) = x2 + 1 and when 1 < x ≤ 2, f ( x ) = 3 − x each of which is a
polynomial and a polynomial function is continuous as well as differentiable at each
point of its domain.
So f ( x ) is continuous and differentiable at each point x when 0 ≤ x < 1 or when
1 < x ≤ 2.
Now to check the continuity and differentiability of f ( x ) at x = 1.
We have f (1) = 12 + 1 = 2,
R-50

lim lim
f (1 − 0 ) = f ( x) = ( x2 + 1) = 2
x →1− x →1
lim lim
and f (1 + 0 ) = f ( x) = (3 − x ) = 2.
x →1+ x →1
Since f (1 − 0 ) = f (1) = f (1 + 0 ), f ( x ) is continuous at x = 1. Thus f ( x ) is continuous
in the closed interval [0, 2].
lim f (1 + h) − f (1) lim { 3 − (1 + h) } − 2
Now R f ′ (1) = =
h→0 h h→0 h
lim −h lim
= = −1= −1
h→0 h h→0
lim f (1 − h) − f (1) lim { (1 − h)2 + 1 } − 2
and L f ′ (1) = =
h→0 −h h→0 −h
lim − 2h + h2 lim − h (2 − h) lim
= = = (2 − h) = 2.
h→0 −h h→0 −h h→0
Since R f ′ (1) ≠ L f ′ (1), f ( x ) is not differentiable at x = 1 which is a point of the open
interval ] 0, 2 [ .
Thus f ( x ) is not differentiable in ] 0, 2 [ .
Hence Rolle’s theorem is not applicable to f ( x ).

Problem 4: Show that between any two roots of e x cos x = 1 there exists at least one root of
e x sin x − 1 = 0.
Solution: If x = a and x = b are two distinct roots of e x cos x = 1, then
e a cos a = 1 and e b cos b = 1 . …(1)
Let f be the function defined as follows :
f ( x ) = e − x − cos x .
We observe that
(i) f is continuous in [ a, b ] as both e − x and cos x are continuous.
(ii) f ′ ( x ) = − e − x + sin x , which exists V x ∈ ] a , b [ ; so f is differentiable in
] a, b [.
(iii) f ( a) = e − a − cos a = e − a − e − a = 0 [By (1)]
f ( b ) = e − b − cos b = e − b − e − b = 0 [By (1)]
i.e., f ( a) = f ( b ) = 0 .
Thus f satisfies all the three conditions of Rolle’s theorem in [ a, b ]. Hence there is at
least one value of x in the open interval ]a, b[, say c, such that f ′ ( c ) = 0.
Now f ′ ( c ) = 0 ⇒ − e − c + sin c = 0 ⇒ e c sin c − 1 = 0
⇒ c is a root of the equation e x sin x − 1 = 0.
R-51

Hence between any two roots of the equation e x cos x = 1 there is at least one root of
the equation e x sin x − 1 = 0.

Problem 5: State and prove Rolle’s theorem. Interpret it geometrically. Verify Rolle’s theorem
for the function
f ( x ) = x2 in [ − 1, 1]. (Lucknow 2010)
Solution: For the first part of this question refer article 8.
The function f ( x ) = x2 is continuous as well as differentiable on the whole R. So f ( x )
is continuous in [ − 1, 1] and differentiable in ] − 1, 1 [.
We have f (1) = 12 = 1 and f ( − 1) = ( − 1)2 = 1 so that f ( − 1) = f (1).
Thus f ( x ) satisfies all the three conditions of Rolle’s theorem in [ − 1, 1]. Therefore
there must exist at least one number, say c, in ] − 1, 1 [ for which f ′ ( c ) = 0.
Now f ′ ( x ) = 2 x.
We have f ′ ( x ) = 0 ⇒ 2 x = 0 ⇒ x = 0.
We observe that the root x = 0 of the equation f ′ ( x ) = 0 lies in the open interval
] − 1, 1 [. Hence the Rolle’s theorem is verified.

Problem 6: Verify the truth of Rolle’s theorem for the function f ( x ) = x2 − 3 x + 2 on the
interval [1, 2].
Solution: The function f ( x ) = x2 − 3 x + 2 is a polynomial in x of degree 2 and so it
is continuous as well as differentiable on the whole R.
∴ f ( x ) is continuous in the closed interval [1, 2] and differentiable in the open
interval ] 1, 2 [ .
We have f (1) = 12 − 3 .1 + 2 = 0 and f (2) = 22 − 3 . 2 + 2 = 0 so that f (1) = f (2).
Thus f ( x ) satisfies all the three conditions of Rolle’s theorem in [1, 2]. Therefore there
must exist at least one point, say c, in ] 1, 2 [ at which f ′( c ) = 0.
Now f ′( x ) = 2 x − 3. We have f ′ ( x ) = 0 ⇒ 2 x − 3 = 0 ⇒ x = 3 / 2 . We observe
that the root x = 3 / 2 of the equation f ′ ( x ) = 0 lies in the open interval ] 1, 2 [ because
1 < 3 / 2 < 2. Hence the Rolle’s theorem is verified.
Problem 7: Does the function f ( x ) = | x − 2 |satisfy the conditions of Rolle’s theorem in the
interval [1, 3] ? Justify your answer with correct reasoning.
Solution: In the interval [1, 3], the function f ( x ) = | x − 2 | is defined as follows :
 2 − x, 1 ≤ x ≤ 2
f ( x) = 
 x − 2, 2 ≤ x ≤ 3.
We have f (1) = 2 − 1 = 1 and f (3) = 3 − 2 = 1 and so f (1) = f (3).
Obviously the function f ( x ) is continuous in [ 1, 2 [ and in ] 2, 3] because in each of
these intervals it is represented by polynomials. Let us test the continuity of f ( x ) at
x = 2.
R-52

We have f (2) = 0,
lim
f (2 − 0 ) = f (2 − h), h > 0
h→0
lim lim
= { 2 − (2 − h) } = h=0
h→0 h→0
lim
and f (2 + 0 ) = f (2 + h), h > 0
h→0
lim lim
= { (2 + h) − 2 } = h = 0.
h→0 h→0
Thus f (2 − 0 ) = f (2) = f (2 + 0 ) and so f ( x ) is continuous at x = 2.
∴ f ( x ) is continuous in the closed interval [1, 3].
Now f ( x ) is obviously differentiable in [ 1, 2 [ and in ] 2, 3 ]. Let us test its
differentiability at x = 2.
lim f (2 + h) − f (2)
We have Rf ′ (2) = ,h>0
h→0 h
lim { (2 + h) − 2 } − 0 lim h lim
= = = 1=1
h→0 h h→0 h h→0
lim f (2 − h) − f (2)
and Lf ′ (2) = ,h>0
h→0 −h
lim { 2 − (2 − h) } − 0
=
h→0 −h
lim h lim
= = − 1 = − 1.
h→0 −h h→0
Since R f ′ (2) ≠ L f ′ (2), therefore f ( x ) is not differentiable at x = 2.
Thus f ( x ) is not differentiable at x = 2 which is a point in the open interval ] 1, 3 [ .
Hence out of the three conditions of Rolle’s theorem, f ( x ) satisfies the two
conditions that f (1) = f (3) and f ( x ) is continuous in the closed interval [ 1, 3 ] . But
it does not satisfy the third condition that f ( x ) must be differentiable in the open
interval ] 1, 3 [ .

Problem 8: The function f is defined in [0, 1] as follows :


1
f ( x) = 1 for 0 ≤ x <
2
1
=2 for ≤ x ≤ 1.
2
Show that f ( x ) satisfies none of the conditions of Rolle’s theorem, yet f ′ ( x ) = 0 for many
points in [0, 1] .
f  + 0  = 2, f  − 0  = 1.
1 1
Solution: Here
2  2 
R-53

Since f  + 0  ≠ f  − 0  ,
1 1 1
f is discontinuous at x = and so it is not
2  2  2
1
differentiable at x = ⋅ Also f (0 ) = 1, f (1) = 2 so that f (0 ) ≠ f (1).
2
Thus all the three conditions of Rolle’s theorem are not satisfied by f in [0, 1]. But f is a
1 1
constant function in [0, [ and in [ , 1].
2 2
Hence f ′ ( x ) = 0 for many points in [0, 1].

Problem 9: If a + b + c = 0, then show that the quadratic equation 3ax2 + 2bx + c = 0 has
at least one root in ] 0, 1 [ .
Solution: Consider the function f, defined by f ( x ) = ax3 + bx2 + cx + d.
We have f (0 ) = d and f (1) = a + b + c + d = d,
because it is given that a + b + c = 0.
Since the function f ( x ) is a polynomial, therefore it is continuous and differentiable
for all real x. Consequently f ( x ) is continuous in the closed interval [0, 1] and
differentiable in the open interval ] 0, 1 [ . Also f (0 ) = d = f (1).
Thus f ( x ) satisfies all the three conditions of Rolle’s theorem in [0, 1]. Hence there is
at least one value of x in the open interval ] 0, 1 [ where f ′ ( x ) = 0 i. e.,
3ax2 + 2bx + c = 0.

Hence the equation 3ax2 + 2bx + c = 0 has at least one root in ] 0, 1 [ .

a0 a a an − 1
Problem 10: Let + 1 + 2 + …+ + an = 0. Show that there exists at
n+1 n n −1 2
least one real x between 0 and 1 such that
a0 x n + a1 x n − 1 + … + an = 0. (Lucknow 2009)
Solution: Consider the function, f, defined by
x n +1 xn x2
f ( x ) = a0 + a1 + … + an − 1 + an x.
n+1 n 2
Since f ( x ) is a polynomial, it is continuous and differentiable for all x. Consequently
f ( x ) is continuous in the closed interval [0, 1] and differentiable in the open interval
] 0, 1 [.
Also f (0 ) = 0 and
a0 a an − 1
f (1) = + 1 + …+ + an = 0 (given)
n+1 n 2
i.e., f (0 ) = f (1).
Thus all the three conditions of Rolle’s theorem are satisfied. Hence there is at least
one value of x in the open interval ] 0, 1 [ where f ′ ( x ) = 0
i.e., a0 x n + a1 x n − 1 + … + an − 1 x + an = 0.
R-54

sin x sin α sin β


Problem 11: If f ( x ) = cos x cos α cos β where 0 < α < β < π / 2,
 
tan x tan α tan β

show that f ′ ( ξ) = 0, where α < ξ < β.

Solution: We have
f ( x ) = (cos α tan β − cos β tan α ) sin x − (sin α tan β − sin β tan α ) cos x
+ (sin α cos β − sin β cos α ) tan x.
Obviously f ′ ( x ) exists at each point of ] 0, π / 2 [ . So f ( x ) is continuous in ] 0, π / 2 [ ⋅
Since 0 < α < β < π / 2, therefore f ( x ) is continuous in [α, β ] and f ( x ) is differentiable
in ] α, β [ .
sin α sin α sin β
Also f (α ) = cos α cos α cos β = 0
 
tan α tan α tan β

sin β sin α sin β


and f ( β ) = cos β cos α cos β = 0
 
tan β tan α tan β
.

∴ f (α ) = f ( β ).
Thus f ( x ) satisfies all the three conditions of Rolle’s theorem in [α, β ]. Hence there
exists at least one real number x in the open interval ] α, β [ at which f ′ ( x ) = 0.
∴ f ′ ( ξ) = 0 where α < ξ < β.

Problem 12: Show that there is no real number k for which the equation x3 − 3 x + k = 0 ,
has two distinct roots in ] 0, 1[.

Solution: Suppose, if possible, there are two distinct roots a, b of the given equation
in ]0, 1[ such that 0 < a < b < 1.

Let f ( x ) = x3 − 3 x + k.
Since f ( x ) is a polynomial, so it is continuous and differentiable for all values of x i.e,
f ( x ) is continuous in [ a, b ] and differentiable in ]a, b[. Also, we have f ( a) = f ( b ) = 0.
Thus f satisfies all the three conditions of Rolle’s theorem in [ a, b ]. Hence there is a
value c of x in ]a, b[ such that f ′ ( c ) = 0.
Now f ′ ( x ) = 0 ⇒ 3 x2 − 3 = 0 ⇒ x = ± 1, which contradicts the fact that a < c < b, as
0 < a < b < 1.
Hence our assumption is wrong. So there cannot be two distinct roots of f ( x ) = 0 in
] 0, 1[ for any value of k.
R-55

Comprehensive Problems 3
Problem 1: State Lagrange’s mean value theorem.
Test if Lagrange’s mean value theorem holds for the function f ( x ) = |x| in the interval [ −1, 1] .
(Kanpur 2010; Rohilkhand 13B)
Solution: See article 9.
The function f ( x ) is continuous throughout the closed interval [ − 1, 1] but it is not
differentiable at x = 0 which is a point of the open interval ] − 1, 1 [ . Thus f ( x ) is not
differentiable in ] − 1, 1 [ . Hence Lagrange’s mean value theorem does not hold for the
function f ( x ) = | x | in the interval [ − 1, 1].

Problem 2: If f ( x ) = 1 / x in [ − 1, 1], will the Lagrange’s mean value theorem be applicable


to f ( x ) ?
(Meerut 2012B)
Solution: The function f ( x ) = 1 / x is continuous and differentiable on R − {0 }. Thus
it is not continuous and differentiable at x = 0.
Since 0 ∈ [ − 1, 1], therefore f ( x ) is not continuous on [ − 1, 1]. Hence Lagrange’s mean
value theorem is not applicable to f ( x ) = 1 / x in [ − 1, 1].

Problem 3: Verify Lagrange’s mean value theorem for the function


f : [ − 1, 1] → R given by f ( x ) = x3 .

Solution: The function f ( x ) = x3 is a polynomial and so it is continuous and


differentiable at all x ∈ R. In particular it is continuous in the closed interval [ − 1, 1]
and differentiable in the open interval ] − 1, 1 [ as is required for the application of
Lagrange’s mean value theorem.
By Lagrange’s mean value theorem, there must exist at least one value ‘c’ of x lying in
the open interval ] − 1, 1 [ such that
f (1) − f ( − 1)
= f ′ ( c ). ...(1)
1 − ( − 1)
Let us verify it.
We have f (1) = 13 = 1, f ( − 1) = ( − 1)3 = − 1.

Also f ′ ( x ) = 3 x2 gives f ′ ( c ) = 3c2 .


Putting these values in (1), we have
1 − ( − 1) 1 1
= 3c2 or c2 = or c = ± ⋅
1 − ( − 1) 3 √3
As both of these values of c lie in the open interval ] − 1, 1 [, hence both of these are the
required values of c and this verifies Lagrange’s mean value theorem.
R-56

1
Problem 4: Find ‘c’ of the mean value theorem, if f ( x ) = x ( x − 1) ( x − 2) ; a = 0, b = ⋅
2
(Kumaun 2012)
Solution: We have f ( a) = f (0 ) = 0,

f ( b ) = f   =  − 1  − 2 = ⋅
1 1 1 1 3
 2 2  2  2  8
3
−0
f ( b ) − f ( a) 8 3
∴ = = ⋅
b−a 1
−0 4
2
Now f ( x ) = x3 − 3 x2 + 2 x.

∴ f ′ ( x ) = 3 x2 − 6 x + 2 gives f ′ ( c ) = 3c2 − 6c + 2.
Putting these values in Lagrange’s mean value theorem
f ( b ) − f ( a)
= f ′ ( c ), ( a < c < b ), we get
b−a
3
= 3c2 − 6c + 2 or 12c2 − 24c + 5 = 0.
4
24 ± √ (24 × 24 − 4 × 12 × 5) 24 ± 4 √ (36 − 15) √ 21
∴ c= = =1± ⋅
24 24 6
√ 21 1
Out of these two values of c only 1 − lies in the open interval ] 0, [ which is
6 2
therefore the required value of c.
Problem 5: Find ‘c’ of Mean value theorem when
( i) f ( x ) = x3 − 3 x − 2 in [ − 2, 3] ( ii) f ( x ) = 2 x2 + 3 x + 4 in [1, 2]

(Meerut 2013B) ( iv ) f ( x ) = x2 − 3 x − 1 in  −
11 13 
( iii) f ( x ) = x ( x − 1) in [1, 2] , ⋅
 7 7 
Solution: (i) We have f ( x ) = x3 − 3 x − 2, a = − 2, b = 3.
∴ f ( a) = f ( − 2) = ( − 2)3 − 3. ( − 2) − 2 = − 8 + 6 − 2 = − 4
and f ( b ) = f (3) = 33 − 3 . 3 − 2 = 16.
f ( b ) − f ( a) 16 − ( − 4) 20
∴ = = = 4.
b−a 3 − ( − 2) 5
Also f ′ ( x ) = 3 x2 − 3 gives f ′ ( c ) = 3c2 − 3.
Putting these values in Lagrange’s mean value theorem
f ( b ) − f ( a)
= f ′ ( c ), ( a < c < b ), we get
b−a
4 = 4 c2 − 3
or 3 c2 − 7 = 0 or c = ± √(7 / 3).
As both these values of c lie in the open interval ] − 2, 3 [, hence both of these are the
required values of c.
R-57

(ii) We have f ( x ) = 2 x2 + 3 x + 4, a = 1, b = 2.
∴ f ( a) = 2 ⋅12 + 3 ⋅1 + 4 = 9 and f ( b ) = 2 ⋅ 22 + 3 ⋅ 2 + 4 = 18.
f ( b ) − f ( a) 18 − 9
∴ = = 9.
b−a 2 −1
Also f ′ ( x ) = 4 x + 3 gives f ′ ( c ) = 4c + 3.
Putting these values in Lagrange’s mean value theorem
f ( b ) − f ( a)
= f ′ ( c ), ( a < c < b ), we get
b−a
9 = 4c + 3 or c = 3 / 2 which lies in the open interval ] 1, 2 [ .
(iii) We have f ( x ) = x2 − x, a = 1, b = 2.
∴ f ( a) = 0 and f ( b ) = 2.
f ( b ) − f ( a) 2 − 0
∴ = = 2.
b−a 2 −1
Also f ′ ( x ) = 2 x − 1 gives f ′ ( c ) = 2c − 1.
Putting these values in Lagrange’s mean value theorem
f ( b ) − f ( a)
= f ′ ( c ), ( a < c < b ), we get 2 = 2c − 1 or c = 3 / 2
b−a
which lies in the open interval ] 1, 2 [ .
(iv) We have f ( x ) = x2 − 3 x − 1, a = − 11 / 7, b = 13 / 7.

f ( a) = f  −  =
11 121 33
∴ + −1
 7 49 7
303
=
49
f ( b ) = f  
13 169 39 153
and = − −1= − ⋅
7 49 7 49
f ( b ) − f ( a) − 456 / 49 19
∴ = =− ⋅
b−a 24 / 7 7
Now f ′ ( x ) = 2 x − 3;
∴ f ′ ( c ) = 2c − 3.
From Lagrange’s mean value theorem, we have 2c − 3 = − 19 / 7 or c = 1 / 7.
Problem 6: Show that any chord of the parabola y = Ax2 + Bx + C is parallel to the
tangent at the point whose abscissa is same as that of the middle point of the chord.
Solution: Let a and b ( a < b ) be the abscissae of the ends of the chord and let
f ( x ) = Ax2 + Bx + C. Since f is a polynomial function, f is continuous on [ a, b ] and
differentiable in ]a, b[. Consequently by Lagrange’s mean value theorem there exists
c ∈ ]a, b [ such that
f ( b ) − f ( a)
= f ′ (c )
b−a
R-58

i.e., Ab2 + Bb + C − Aa2 − Ba − C = ( b − a) (2 Ac + B).


[∵ f ′ ( c ) = 2 Ac + B]
1
On simplification it gives c = ( a + b ) i.e., the abscissa of the point at which the tangent is
2
parallel to the chord is the same as that of the middle point of the chord.

Problem 7: If f ′ ′ ( x ) exists for all points in [ a, b ] and


f ( c ) − f ( a) f (b) − f (c )
=
c−a b−c
where a < c < b, then there is a number ξ such that a < ξ < b and f ′ ′ ( ξ) = 0.
Solution: Since f ′ ′ ( x ) exists for all points in [ a, b ], f ′ ( x ) is continuous in [ a, b ] and
so also is f ( x ).
Applying Lagrange’s mean value theorem to f ( x ) in the intervals [ a, c ] and [ c, b ]
respectively, we get
f ( c ) − f ( a)
= f ′ ( ξ1 ), a < ξ1 < c ...(1)
c−a
f (b) − f (c )
and = f ′ ( ξ2 ), c < ξ2 < b. ...(2)
b−c
Then from (1) and (2), we get on using the relation given in the question
f ′ ( ξ1 ) = f ′ ( ξ2 ).
Now the function f ′ ( x ) satisfies all the three conditions of Rolle’s theorem in the
interval [ ξ1, ξ2 ].
Hence f ′ ′ ( ξ) = 0 where ξ1 < ξ < ξ2 so that a < ξ < b.
Problem 8: State the conditions for the validity of the formula
f ( x + h) = f ( x ) + h f ′ ( x + θ h)
and investigate how far these conditions are satisfied and whether the result is true, when
f ( x ) = x sin (1 / x ) (being defined to be zero at x = 0 ) and x < 0 < x + h.
Solution: The conditions for the validity of the given formula are :
(i) The function f ( x ) must be continuous in the closed interval [ x, x + h].
(ii) The function f ( x ) must be differentiable in the open interval ] x, x + h [ .
and (iii) θ is a real number such that 0 < θ < 1.
Now consider the function f ( x ) defined as :
f ( x ) = x sin (1 / x ) for x ≠ 0, f (0 ) = 0.
The first condition is satisfied because f ( x ) is continuous in the closed interval
[ x, x + h] for x < 0 < x + h. Obviously f ( x ) is continuous at every point x = c if c ≠ 0
and it can be easily shown that f ( x ) is continuous at x = 0.
But the second condition is not satisfied because f ( x ) is not differentiable at x = 0
which is a point lying in the open interval ] x, x + h [ for x < 0 < x + h. [Show here that
f ( x ) is not differentiable at x = 0 ].
Hence the result of the given formula is not true for this function f ( x ).
R-59

Problem 9(a): Show that x3 − 3 x2 + 3 x + 2 is monotonically increasing in every interval.


Solution: Let f ( x ) = x3 − 3 x2 + 3 x + 2.
Then f ′ ( x ) = 3 x2 − 6 x + 3 = 3 ( x − 1)2 .
We observe that f ′ ( x ) > 0 for every real value of x except 1 where its value is zero.
Hence f ( x ) is monotonically increasing in every interval.
2x
Problem 9(b): Show that log (1 + x ) − is increasing when x > 0.
2+ x
2x
Solution: Let f ( x ) = log (1 + x ) − ⋅
2+ x
1 2.(2 + x ) − 2 x . 1 1 4 (2 + x )2 − 4 (1 + x )
We have f ′ ( x ) = − = − =
1+ x (2 + x ) 2 1 + x (2 + x )2 (1 + x ) (2 + x )2
4 + 4 x + x2 − 4 − 4 x x2
= = ⋅
(1 + x ) (2 + x )2 (1 + x ) (2 + x )2
We observe that f ′ ( x ) > 0 for all x > 0.
∴ the function f ( x ) is monotonically increasing in the interval [ 0, ∞ [ .
2x
Hence the function log (1 + x ) − is increasing when x > 0.
2+ x

Problem 10: Determine the intervals in which the function


( x4 + 6 x3 + 17 x2 + 32 x + 32) e − x
is increasing or decreasing.
Solution: Let f ( x ) = ( x4 + 6 x3 + 17 x2 + 32 x + 32) e − x .
Then f ′ ( x ) = ( x4 + 6 x3 + 17 x2 + 32 x + 32) ( − e − x ) + (4 x3 + 18 x2 + 34 x + 32) e − x
= − e − x ( x4 + 2 x3 − x2 − 2 x ) = − x e − x ( x3 + 2 x2 − x − 2)
= − x ( x + 2) ( x − 1) ( x + 1) e − x = x (1 − x ) (1 + x ) (2 + x ) e − x .
Now f ′ ( x ) is positive in the intervals ] − 2, − 1 [ and ] 0, 1 [ and negative in
] − ∞, − 2 [, ] − 1, 0 [ and ] 1, ∞ [ . Hence the function f ( x ) is monotonically
increasing in the intervals [ − 2, − 1] and [0, 1] and monotonically decreasing in the
intervals ] − ∞, − 2 ], [ − 1, 0 ] and [ 1, ∞ [ .

Problem 11: Use the function f ( x ) = x1 / x , x > 0 to determine the bigger of the two numbers
e π and π e.
1
Solution: Let f ( x ) = x1 / x . Then log f ( x ) = log e x.
x
1 1 1 1
Differentiating w.r.t. x, we get f ′ ( x) = ⋅ − log e x
f ( x) x x x2
x1 / x
or f ′ ( x) =
x2
[
1 − log e x ⋅ ]
R-60

For x > e, f ′ ( x ) < 0. [∵ log e x > 1 for x > e ]


∴ f ( x ) is a decreasing function of x for x > e.
Hence π > e ⇒ f ( π ) < f ( e ) ⇒ π1 / π < e1 / e

⇒ ( π1 / π )e π < ( e1 / e )e π ⇒ π e < e π ⇒ e π is bigger than π e.

Problem 12: If a = − 1, b ≥ 1 and f ( x ) = 1/| x | , show that the conditions of Lagrange’s


mean value theorem are not satisfied in the interval [ a, b ] , but the conclusion of the theorem is
true if and only if b > 1 + √ 2 .
Solution: This function is not defined at x = 0. So we take f (0 ) = A, where A is
some definite real number.
1
− A
lim |h| lim 1  1
Now R f ′ (0 ) = =  − A  = ∞ × ∞ = ∞
h→0 h h→0 h h 
1
− A
lim |− h| lim 1  1 − A  = − ∞ × ∞ = − ∞.
and Lf ′ (0 ) = = −  
h→0 −h h→0 h h 
Since R f ′ (0 ) ≠ L f ′ (0 ) so f is not differentiable at x = 0. Thus the conditions of the
mean value theorem are not satisfied in the interval [ a, b ] which includes the origin.
Again, the conclusion of the mean value theorem is
f ( b ) − f ( a)
= f ′ ( c ) where a < c < b.
b−a
If this result is true, we have
1 1 d 1  1 
− = ( b − a)   = ( b − a)  −
2
|b| |a|  dx | x|x = c  |c| 
1  1  b+1
or − 1 = ( b + 1)  −  =−
 
b  | c |2  c2
b2 + b b2 + b
or c2 = or < b2 [∵ b2 > c2 ]
b −1 b −1
or b + 1 < b2 − b or b2 − 2b − 1 > 0
or ( b − 1)2 > 2 or ( b − 1) > √ 2 or b > 1 + √ 2.
Hence the conclusion of the mean value theorem is true iff b > 1 + √ 2.

Problem 13: (a) State Cauchy’s mean value theorem. (Kanpur 2007)
(b) Verify Cauchy’s mean value theorem for f ( x ) = sin x, g ( x ) = cos x in [ − π / 2, 0 ].
(Lucknow 2007)
Solution: (a) Cauchy’s Mean Value Theorem : If two functions f ( x ) and g ( x ) are
(i) continuous in a closed interval [ a, b ] ,
(ii) differentiable in the open interval ]a, b[ ,
R-61

(iii) g′ ( x ) ≠ 0 for any point of the open interval ]a, b[, then there exists at least one value c of x in
the open interval ] a, b [, such that
f ( b ) − f ( a) f ′ (c )
= , a < c < b.
g ( b ) − g ( a) g′ (c )

(b) Here both the functions f ( x ) = sin x and g ( x ) = cos x are continuous in the closed
interval [ − π / 2, 0 ] and differentiable in the open interval ] − π / 2, 0 [ . Also
g′ ( x ) = − sin x ≠ 0 for any point in the open interval ] − π / 2, 0 [ . Hence by Cauchy’s
mean value theorem there exists at least one real number c in the open interval
] − π / 2, 0 [ such that
f (0 ) − f ( − π / 2) f ′ (c )
= ⋅ ...(1)
g (0 ) − g ( − π / 2) g′ ( c )
Let us verify it.
f (0 ) − f ( − π / 2) sin 0 − sin ( − π / 2) 1
We have = = = 1.
g (0 ) − g ( − π / 2) cos 0 − cos ( − π / 2) 1
Also f ′ ( x ) = cos x, g′ ( x ) = − sin x.
f ′ (c ) cos c
∴ = = − cot c.
g′ ( c ) − sin c
Putting these values in (1), we get − cot c = 1 or cot c = − 1 whose solution c = − π / 4
lies in the open interval ] − π / 2, 0 [ . Hence Cauchy’s mean value theorem is verified.

Problem 14: If f ( x ) = x2 , g ( x ) = cos x, then find the point c ∈ ] 0, π / 2 [ which gives the
result of Cauchy’s mean value theorem in the interval [0, π / 2] for the functions f ( x ) and
g ( x ).
Solution: Both the functions f ( x ) = x2 and g ( x ) = cos x are continuous in the
closed interval [0, π / 2] and differentiable in the open interval ] 0, π / 2 [ . Also
g′ ( x ) = − sin x ≠ 0 for any point in the open interval ] 0, π / 2 [ . Hence by Cauchy’s
mean value theorem there exists at least one real number c in the open interval
] 0, π / 2 [, such that
f ( π / 2) − f (0 ) f ′ (c )
= ⋅ ...(1)
g ( π / 2) − g (0 ) g′ ( c )
1 2
π −0
f ( π / 2) − f (0 ) 4 π2
We have = =− ⋅
g ( π / 2) − g (0 ) cos π − cos 0
1 4
2
Also f ′ ( x ) = 2 x, g′ ( x ) = − sin x.
f ′ (c ) 2c
∴ = ⋅
g′ ( c ) − sin c
Putting these values in (1), we get
2c π2 8c
− =− or sin c − = 0.
sin c 4 π2
R-62

8c
Let F ( c ) = sin c −

π2
π 8 π 1 π 8 π
F   = − > 0 and F   = 1 −
1 4 4
We have ⋅ = − ⋅ = 1 − < 0.
 6  2 π2 6 2 3 π 2 π2 2 π
Since F ( π / 6) and F ( π / 2) are of opposite signs, therefore the equation F ( c ) = 0 has a
root lying in the open interval ] π / 6, π / 2 [ and this root of the equation
sin c − (8c / π2 ) = 0 is the required value of c.

sin α − sin β π
Problem 15: Show that = cot θ, where 0 < α < θ < β < ⋅
cos β − cos α 2
Solution: Let f ( x ) = sin x and g ( x ) = cos x, for x ∈[α, β ] where 0 < α < β < π / 2.
∴ f ′( x ) = cos x and g′( x ) = − sin x.
Here both the functions f ( x ) and g ( x ) are continuous in the closed interval [α, β ] and
differentiable in the open interval ] α, β[ . Also g′( x ) = − sin x ≠ 0 for any point in the
open interval ] α, β [ . Hence by Cauchy’s mean value theorem there exists at least one
real number, say θ, in the open interval ] α, β [ such that
f (β ) − f (α ) f ′(θ) sin β − sin α cos θ
= ⇒ = = − cot θ
g (β ) − g (α ) g′(θ) cos β − cos α − sin θ

sin α − sin β π
⇒ = cot θ, where 0 < α < θ < β < ⋅
cos β − cos α 2
Problem 16: Use Cauchy’s mean value theorem to evaluate
 1 
lim  cos 2 π x 

x → 1  log (1 / x ) 
 
 
 1 
Solution: Let f ( x ) = cos  πx  ; g ( x ) = log x, a = x, b = 1.
2 
Putting these values in Cauchy’s mean value theorem,
f ( b ) − f ( a) f ′ (c )
= , a < c < b, we get
g ( b ) − g ( a) g′ ( c )
1 1 
π − cos πx − 2 π sin  2 πc 
1 1
cos
2 2 = , x < c < 1.
log 1 − log x 1/ c
Taking limits as x → 1 which implies that c → 1, we get
   1 
0 − cos  πx  − π sin  πc 
1 1
lim  2  
 lim  2  2  
 =  
x →1 log (1 / x )  c →1 (1 / c ) 
   
R-63

 
− cos  πx 
1
lim  2  
 1 1 
or   = − π as sin  π c  → 1 as c → 1
x → 1  log (1 / x )  2 2
 
 
cos  πx 
1
lim  2  
 π
or  = ⋅
x → 1  log (1 / x )  2
 

Hints to Objective Type Questions

Multiple Choice Questions:


 x − 1, x ≥ 1
1. (c). The function f ( x ) = |x − 1| = 
1 − x, x < 1.
This function is not differentiable at x = 1.
2. (b). The function f ( x ) = |x + 3| is not differentiable at x = −3.
3. (a). A function f ( x ) is differentiable at x = a if Rf ′( a) = Lf ′( a). See article 1.
4. (a). See problem 9 of Comprehensive Problems 1.
5. (c). For the function f ( x ) = sin x in [0, π ] , we have f(0 ) = 0 and f( π ) = 0 so
that f (0 ) = f ( π ). Also f ( x ) is continuous in [0, π] and differentiable in ]0, π[.
Thus f ( x ) = sin x satisfies all the three conditions of Rolle’s theorem in [0, π].
6. (b). The function f ( x ) = sin x is continuous on the whole R .
We have f ′( x ) = cos x. Out of the given four intervals f ′( x ) = cos x is positive
π
only in 0,  and is positive as well as negative in each of the intervals [0, π ],
 2 
 π , 3 π  and  π , π . So, out of the four given intervals the function
 4 4   2 
π
f ( x ) = sin x is increasing only in 0,  .
 2 
7. (b). See problem 5(iii) of Comprehensive Problems 3.
8. (a). See problem 1(ii) of Comprehensive Problems 2.
9. (b). See Ex. 1.

Fill in the Blanks:


1. f ( a). See article 1, definition of derivative of a function at a point.
2. h. See article 1, definition of right hand derivative of a function at a point.
3. −h. See article 1, definition of left hand derivative of a function at a point.
R-64

4. ]a, b[. See article 1, definition of differentiability of a function in an open in-


terval.
5. the x-axis. See article 2.
6. sufficient. See article 4.
7. x = 0. See Ex. 1.
8. Rolle’s theorem. See article 8
9. f ′( c ). See article 9, Lagrange’s mean value theorem.
10. g( b ) − g( a). See article 11, Cauchy’s mean value theorem.
11. increasing. See article 10, theorem 4.
f ( x + h) − f ( x )
12. cos x, We have lim = f ′( x ).
h→ 0 h
If f ( x ) = sin x, then f ′( x ) = cos x .

True or False:
1. F. For example, the function f ( x ) = |x| is continuous at x = 0 but it is not
differentiable at x = 0.
2. T . If a function f ( x ) possesses a finite derivative f ′( a) at a point x = a, we
know that f ( x ) must be continuous at x = a .
3. F. If a function f ( x ) is differentiable at x = a , we know that it must be con-
tinuous at x = a.
4. F. The function f ( x ) = |x| is not differentiable at x = 0.
5. T . For the function f ( x ) = sin x in [0, 2 π] ,
we have f(0 ) = 0, f(2 π ) = 0 so that f (0 ) = f (2 π ).
Also f ( x ) is continuous in [0, 2 π ] and differentiable in ] 0, 2 π [. Thus
f ( x ) = sin x satisfies all the three conditions of Rolle’ theorem in [0, 2 π]. So
Rolle’s theorem is applicable for f ( x ) = sin x [0, 2 π ] .
6. F. Rolle’s theorem is not applicable for f ( x ) = |x| in [ −1, 1] because f ( x ) = |x| is
not differentiable at x = 0 which is a point in ] −1, 1[.
7. F. Lagrange’s mean value theorem is not applicable for f ( x ) = |x| in [ −1, 1] be-
cause f ( x ) = |x| is not differentiable at x = 0 which is a point in [ −1, 1[.
8. T . The function f ( x ) = sin x is continuous on the whole R and so also in
 − π , π  . We have f ′( x ) = cos x which is positive in  − π , π . So the func-
 2 2   2 2 
π π
tion f ( x ) = sin x is increasing in  − ,  .
 2 2 

9. F. If a + b + c = 0, then the quadratic equation 3ax2 + 2bx + c = 0 has at least


one root in ] 0, 1 [. See problem 9 of Comprehensive Problems 2.
R-65

10. F. If f is continuous on [ a, b ] and f ′( x ) ≤ 0 in ] a, b [ , then f is decreasing in


[ a, b ] . See article 10, Theorem 4.
11. T . If f ( x ) = 2 x3 − 15 x2 + 36 x + 1, then f ′( x ) = 6 x2 − 30 x + 36 = 6( x − 2) ( x − 3)
.
Now, f ( x ) is continuous in (2, 3) and f ′( x ) < 0 in ] 2, 3 [. so f ( x ) is decreasing
in [2, 3].
12. T . If f ( x ) = |x| + |x −1| , then R f ′(0 ) = 0. See Ex. 2.
13. F. For the function f ( x ) = x( x + 2) e − x /2 , we have f( −2) = 0 and f(0 ) = 0, so
that f ( −2) = f (0 ). Also f ( x ) is continuous in [ −2, 0 ] and differentiable in
[ −2, 0[.
Thus f ( x ) satisfies all the three conditions of Rolle’s theorem in [ −2, 0 ] and so
Rolle’s theorem is applicable for f ( x ) in [ −2, 0 ] .
14. F. See problem 5 part (ii) of Comprehensive Problems 3. We note that the
3
value of c is given by c = .
2

f ( x + h) − f ( x )
15. T . If f ( x ) = x n, then lim = f ′( x ) = nx n − 1.
h→0 h
f ( x ) − f ( a)
16. T . We have lim = f ′( a).
h→0 x−a
If f ( x ) = cos x, then f ′( x ) = − sin x so that f ′( a) = − sin a
f ( x ) − f ( a)
So if f ( x ) = cos x, then lim = − sin a.
x→ a x−a
f ( x) − f ( x 0 )
17. F. We have lim = f ′( x 0 )
x→ x0 x − x0

If f ( x ) = e x , then f ′( x ) = e x so that f ′( x0 ) = e x 0.
f ( x ) − f ( x0 )
So if f ( x ) = e x , then lim = e x0 and not e x .
x→ x0 x − x0

❍❍❍
R-66
Chapter-3
Sequences

Comprehensive Problems 1
Problem 1: Write a formula or formulae for the nth term s n for each of the following sequences:
(a) 1, − 4, 9, − 16, 25, − 36, … (b) 1, 0, 1, 0, 1, 0, ...
(c) 1, 3, 6, 10, 15, ...
Solution: (a) Here s n = ( − 1)n − 1 n2 .
(b) Here s n = 1 if n is odd, s n = 0 if n is even.
n ( n + 1)
(c) Here s n = as explained below.
2
Let S = 1 + 3 + 6 + 10 + 15 + … + s n ...(1)
Then S = 1 + 3 + 6 + 10 + … + s n − 1 + s n ...(2)
Subtracting (2) from (1), we get
0 = (1 + 2 + 3 + 4 + … upto n terms) − s n
n ( n + 1)
⇒ s n = 1 + 2 + 3 + 4 + … upto n terms = ⋅
2
Problem 2: Which of the sequences ( a), ( b ), ( c ) in the above problem are subsequences of the
sequence < s n > defined by s n = n ?
Solution: Only the sequence given in (c).
Problem 3: Find whether the following sequences are bounded above or below :
( − 1)n
(i ) < > ( ii ) < 2n > ( iii ) < n ! >.
n
( − 1)n
Solution: (i) Here s n = ⋅
n
( − 1)n 1
We have |s n| =   = ≤ 1, V n ∈ N.
 n  n
∴ the sequence < s n > is a bounded sequence i. e., it is bounded above as well as bounded
below.
1 1 1 1 1
In fact < s n > = < − 1, , − , , − , , … > .
2 3 4 5 6
1
We observe that is the greatest term of the sequence < s n > and − 1is its smallest term.
2
∴ sup < s n > = 1 / 2 and inf < s n > = − 1.
R-67

(ii) Here s n = 2n.


We have s n ≥ 2, V n ∈ N.
∴ the sequence < s n > is bounded below and 2 = inf < s n >.
But the sequence < s n > is not bounded above . If x be any real number, however large,
there exists n ∈ N such that 2n > x as shown below.
First we shall show by induction that 2n > n, V n ≥ 2.
If n = 2, then 2n = 22 = 4 > 2 = n, showing that the result is true for n = 2.
Let us assume that 2n > n, for some given n ≥ 2.
Then 2n > n
⇒ 2n.2 > 2n ⇒ 2n + 1 > n + n ⇒ 2n + 1 > n + 1, [∵ 1 < n]
showing that the result is true for n + 1 if it is true for n.
Hence by induction 2n > n, V n ≥ 2.
Now if x is any real number, then by Archimedean property of real numbers, there exists
n ∈ N such that n > x.
But 2n > n, V n ≥ 2.
∴ there exists n ∈ N such that 2n > x.
Thus if x be any real number, however large, there exists s n = 2n such that s n > x.
Therefore the sequence < 2n > possesses no upper bound and so it is not bounded above.
Hence the sequence < s n > is also not bounded.
(iii) Here s n = n ! .
We have s n ≥ 1, V n ∈ N.
∴ the sequence < s n > is bounded below and 1! = 1 = inf < s n > .
If x be any real number, however large, there exists n ∈ N such that n ! > x. Therefore the
sequence < n! > possesses no upper bound and so it is not bounded above. Hence the
sequence < s n > is also not bounded.

Problem 4: Are the sequences < s n > defined as follows, bounded ?


n
( − 1)n
( ii ) s n = 1 + 
1
(i ) sn = 1 +
n  n
1 1 1
( iii ) sn = 1 + + + …+ ⋅( iv ) s n = 1, if n is divisible by 3 and s n = 0, otherwise.
1! 2 ! n!
Solution: (i) The given sequence
3 2 5 4 7 6 9 8
< s n > = < 0,
, , , , , , , ,…>.
2 3 4 5 6 7 8 9
We observe that 0 is the smallest term and 3/2 is the greatest term of the sequence < s n >.
So the sequence < s n > is bounded below as well as bounded above and hence it is a
bounded sequence.
In fact inf < s n > = 0 and sup < s n > = 3 / 2.
R-68
n
(ii) Here s n = 1 +  .
1
 n
We have 2 ≤ s n < 3, V n ∈ N. [See in Text Book Example 19.]
So the sequence < s n > is bounded below as well as bounded above and hence it is a
bounded sequence.
Here inf < s n > = 2 and sup < s n > = e.
1 1 1
(iii) Here s n = 1 + + + …+ ⋅
1! 2 ! n!
1 1 1 1 1 1
We have s1 = 1 + = 2, s2 = 1 + + , s3 = 1 + + + ,….
1! 1! 2 ! 1! 2 ! 3 !
We have s n ≥ 2, V n ∈ N.
∴ the sequence < s n > is bounded below.
Also s n < 3, V n ∈ N. [See in Text Book Example 18.]
∴ the sequence < s n > is also bounded above.
Hence the sequence < s n > is a bounded sequence.
In fact here, inf < s n > = 2, sup < s n > = e.
Note that the sequence < s n > is a bounded monotonic increasing sequence and so it
converges to its supremum.
Since lim s n = e, therefore sup < s n > = e.
(iv) Here < s n > = < 0, 0, 1, 0, 0, 1, 0, 0, 1,… > .
We observe that 0 is the smallest term of the sequence < s n > and 1 is its greatest term.
∴ the sequence < s n > is a bounded sequence.
Here 0 = inf < s n > and 1 = sup < s n >.
Problem 5: Use the definition of the limit of a sequence to show that the limit of the sequence
< s n > where s n = 2 n / ( n + 3), is 2.
Solution: Take any given ε > 0.
 2n  2n − 2n − 6
We have |s n − 2| =  − 2 =  
n + 3   n+3 
6 6 6
= < < ε, provided n > .
n+3 n ε
Now by Archimedean property of real numbers, there exists m ∈ N such that m > 6 / ε. So
take m ∈ N such that m > 6 / ε. Then V n ≥ m, we have n > 6 / ε and consequently
| s n − 2 | < ε.
Thus for any given ε > 0, there exists a positive integer m such that
|s n − 2|< ε, V n ≥ m.
Hence by the definition of the limit of a sequence, lim s n = 2.
Problem 6: Show that the sequence < s n > where s n = n / ( n + 1) converges to 1.
Solution: Take any given ε > 0.
 n   n − n − 1
 = 1 < 1 < ε,
We have |s n − 1| =  − 1 = 
n + 1   n + 1  n + 1 n
provided n > 1 / ε.
R-69

So if we take m ∈ N such that m > 1 / ε, then


| s n − 1|< ε, V n ≥ m.
∴ lim s n = 1, by the definition of the limit of a sequence.
Problem 7: If the sequence < s n > converges to l , then prove that the sequence | s n|converges to
| l |.
Solution: Take any given ε > 0.
Since the sequence < s n > converges to l, therefore there exists m ∈ N such that
|s n − l|< ε, V n ≥ m. ...(1)
But if x, y are any real numbers, then
||x| − | y||≤ | x − y |.
∴ ||s n| − |l|| ≤ | s n − l |. ...(2)
Hence from (1) and (2), we have
||s n| − |l||< ε, V n ≥ m.
lim
Hence |s n| = |l|, by the definition of the limit of a sequence.
n→ ∞

Problem 8: Show by considering the sequence < s n = ( − 1)n > that < | s n |> may converge but
< s n > may not.
Solution: Let s n = ( − 1)n. Then |s n| = 1, V n ∈ N.
∴ the sequence < |s n|> converges to 1.
But the sequence < s n > = < − 1, 1, − 1, 1, − 1, 1, … > is not convergent because its two
subsequences
< s2 n − 1 > = < − 1, − 1, − 1, … > and < s2 n > = < 1, 1, 1, … >
converge to different limits − 1 and 1 while every subsequence of a convergent sequence
converges to the same limit to which the sequence converges.
Thus < |s n|> is convergent while < s n > is not convergent.
Problem 9: If lim s n = l and s n ≤ m for all n ∈ N , prove that l ≤ m.
Solution: Suppose, if possible l > m.
Then l − m > 0. Since lim s n = l, therefore for ε = l − m > 0, there exists p ∈ N such that
|s n − l|< l − m = ε, V n ≥ p.
In particular, | s p − l |< l − m
i. e., l − ( l − m) < s p < l + ( l − m) i. e., s p > m.
But by hypothesis s n ≤ m V n ∈ N.
Hence our initial assumption that l > m is wrong.
So we must have l ≤ m.
Problem 10: Let < s n > be a sequence such that < s n2 > converges to zero. Is it necessary that
< s n > should converge to zero ?
Solution: Yes, < s n > must also converge to zero as shown below.
Take any given ε > 0.
Since lim s n2 = 0, therefore, for a given positive real number ε2 , there exists m ∈ N such
that
R-70

|s n2 − 0| = |s n2| = |s n|2 < ε2 , V n ≥ m


⇒ |s n| = |s n − 0|< ε, V n ≥ m.
Hence lim s n = 0, by the definition of the limit of a sequence.
2n
Problem 11: If s n = , prove that < s n > is convergent.
n + 4n1 /2
2n 2
Solution: We have s n = = , dividing the Nr and Dr by n.
n + 4n1 /2 1 + (4 / √ n)
lim lim 2 2
∴ s = = = 2.
n → ∞ n n → ∞ 1 + (4 / √ n) 1 + 4 ⋅ 0
Hence the given sequence < s n > converges to 2.
Alternative solution: We shall show by applying the definition of the limit of a
sequence that the given sequence converges to 2.
Take any given ε > 0.
 2n  8 n1 /2 8n1 /2
We have |s n − 2| =  − 2 = <
1 /2 1 /2
 n + 4n  n + 4n n
8 8 64
= < ε, provided n1 /2 > i. e, n > ⋅
n1 /2 ε ε2
If we choose a positive integer m > 64 / ε2 , then
| s n − 2|< ε for all n ≥ m.
lim
Hence s = 2.
n→ ∞ n
n
Problem 12: If s n = , prove that < s n > → 0.
2n
Solution: Here s n ≠ 0, V n ∈ N.
s n + 1 ( n + 1) 2n 1 n + 1 1  1
Also = ⋅ = . = 1 + ⋅
sn 2n + 1 n 2 n 2 n
lim sn + 1 lim 1  1  1 1
∴ = 1 +   = (1 + 0 ) =
n→ ∞ sn n → ∞  2  n  2 2

and 1 = 1 < 1.


2 2
∴ by the result mentioned in the solution of problem 30 i. e., the result proved in Text
Book Example 5, we have lim s n = 0.
Problem 13: If < s n > converges to l ( ≠ 0 ), prove that < ( − 1)n s n > oscillates.
Solution: Since the sequence < s n > converges to l, therefore each of its subsequences
< s1, s3 , s5 , … > and < s2 , s4 , s6 , … > converges to l.
∴ the sequence < − s1, − s3 , − s5 , … > converges to − l.
Now l ≠ 0 ⇒ l and − l are two different real numbers.
∴ the sequence < ( − 1)n s n > is not convergent because its two subsequences
< − s1, − s3 , − s5 , … > and < s2 , s4 , s6 , … >
converge to different limits.
R-71

Also the sequence < s n > is convergent


⇒ < s n > is bounded ⇒ < ( − 1)n s n > is also bounded
⇒ < ( − 1)n s n > cannot diverge to ∞ or − ∞.
Hence the sequence < ( − 1)n s n > oscillates.
Problem 14: If < s n > diverges and c ( ≠ 0 ) ∈ R , prove that < cs n > diverges.
Solution: Let lim s n = ∞.
If c > 0, we shall prove that lim ( cs n) = ∞
and if c < 0, we shall prove that lim ( cs n) = − ∞.
Let c > 0.
Take any given k > 0. Then k / c > 0.
Since lim s n = ∞, therefore there exists m ∈ N such that
k
sn > , V n ≥ m
c
⇒ c s n > c . ( k / c ) V n ≥ m ⇒ c s n > k V n ≥ m.
∴ lim ( c s n) = ∞.
Again let c < 0.
Take any given k < 0.
Then k / c > 0.
Since lim s n = ∞, there exists m ∈ N such that
k
sn > V n ≥ m
c
⇒ c s n < c. ( k / c ) V n ≥ m [∵ c < 0 ]
⇒ c s n < k V n ≥ m.
∴ lim ( c s n) = − ∞.
Thus if < s n > diverges to ∞ and c ( ≠ 0 ) ∈ R, then < cs n > also diverges.
Similarly if < s n > diverges to − ∞, we can prove that < cs n > diverges to − ∞ or ∞
according as c > or < 0.
Problem 15: Show that the sequence < n + ( − 1)n n > oscillates infinitely.
Solution: Let s n = n + ( − 1)n n.
The subsequence < s2 n − 1 > of the sequence < s n > is given by
< s2 n − 1 > = < 0, 0, 0, … > which converges to zero.
Again the subsequence < s2 n > of the sequence < s n > is given by
< s2 n > = < 4, 8, 12, 16, … > which diverges to ∞.
Since every subsequence of a convergent sequence is also convergent and every
subsequence of a divergent sequence is also divergent, therefore the sequence < s n > is
neither convergent nor divergent and hence it is an oscillatory sequence.
Also the sequence < s n > is not bounded above. If k is any positive real number, there
exists n ∈ N such that
n> k ⇒ 4n > k
R-72

⇒ s2 n = 2n + ( − 1)2 n . 2n = 4n > k.
∴ the sequence < s n > is not bounded above and hence not bounded also.
Since the sequence < s n > oscillates and it is not bounded, therefore it oscillates
infinitely.
Problem 16: Show that < s n > converges to e, where s n is
n +1 
n −n
1 
( i ) 1 +  (iii) 1 − 
1 1
( ii ) 1 + 
 n  n + 1  n
n +1 n
Solution: (i) We have s n = 1 +  = 1 +  . 1 +  ⋅
1 1 1
 n  n  n
lim lim  1 
n
1  
∴ sn = 1 +  . 1 +   ⋅ ...(1)
n→ ∞ n→ ∞

n n 

lim n lim
1 + 1  = e 1 + 1  = 1 + 0 = 1.
But   and  
n→ ∞  n n→ ∞  n
Now we know that if lim s n = l and lim tn = l ′, then lim ( s ntn) = l l ′.
Hence from (1), we conclude that lim s n = e.1 = e.
n +1
 1 
n 1 + 
 1   n + 1
(ii) We have s n = 1 +  = ⋅
 n + 1 1+
1
n+1
n +1
 1 
1 + 
lim lim  n + 1
∴ sn = ⋅ ...(2)
n→ ∞ n→ ∞  1 
1 + 
 n + 1
n +1
lim  1  lim  1 
Now 1 +  = e and 1 +  = 1 + 0 = 1.
n→ ∞  n + 1 n→ ∞  n + 1
Also we know that if lim s n = l and lim tn = l ′, where l ′ ≠ 0, then lim ( s n / tn) = l / l ′.
Hence from (1), we conclude that lim s n = e / 1 = e.
−n −n n
n − 1  n 
(iii) We have s n = 1 −  = 
1
 = 
 n  n   n − 1
n n n −1
 ( n − 1) + 1  1   1   1 
=  = 1 + n − 1 = 1 + n − 1 1 + ⋅
 n − 1       n − 1
n −1
lim  1  lim  1 
But 1 +  = e and 1 +  = 1.
n→ ∞  n − 1 n→ ∞  n − 1
lim
∴ s n = e .1 = e.
n→ ∞
R-73
n
Show that the sequence < s n >, where s n = 1 +  , converges to e2 .
2
Problem 17:
 n
2 n+ 2 n+ 2 n+1
Solution: We have 1 + = = ⋅
n n n+1 n
( n + 1) + 1 n + 1  1   1
= ⋅ = 1 +  1 +  ⋅
n+1 n  n + 1  n
n +1 n
 1 
⋅ 1 + 
1
1 + 
n  n n  n + 1  n
1  
s n = 1 +  = 1 +
2 1
∴  ⋅ 1 +  = ⋅
 n   n + 1  n   1 
1 + 
 n + 1
n +1 n
 1 
⋅ 1 + 
1
1 + 
lim lim  n + 1  n e. e
∴ sn = = = e2 .
n→ ∞ n→ ∞  1  1
1 + 
 n + 1
n3 − 2n + 1
Problem 18: If s n = , prove that lim s n = 1.
n3 + 2n2 − 1
lim lim n3 − 2n + 1
Solution: We have s =
n→ ∞ n n→ ∞ n3 + 2n2 − 1
1 1
1−2⋅ +
lim n2 n3 , dividing the Nr and Dr by n3
=
n→ ∞ 1 1
1+ 2⋅ −
n n3
1 − 2⋅0 + 0
= = 1.
1 + 2⋅0 − 0
1
[∵ if s n = c, then lim s n = c; lim = 0; if lim s n = l and lim tn = l ′, then
n
lim ( cs n) = cl, lim ( s ntn) = ll ′ and lim ( s n / tn) = l / l ′, provided l ′ ≠ 0]
(3n − 1) ( n4 − n)
Problem 19: If s n = , prove that lim s n = 3.
( n2 + 2) ( n3 + 1)
lim lim (3n − 1) ( n4 − n)
Solution: We have sn =
n→ ∞ n→ ∞ ( n2 + 2) ( n3 + 1)
 3 − 1  1 − 1 
   
lim  n  n3 
= , dividing the Nr and Dr by n5
n→ ∞  1  1
1 + 2 . 2  1 + 3 
 n   n 
(3 − 0 ) (1 − 0 )
= = 3.
(1 + 2 ⋅ 0 ) (1 + 0 )
R-74

n2 + 3n + 5 1
Problem 20: Prove that the sequence < > converges to ⋅
2n2 + 5n + 7 2
n2 + 3n + 5
Solution: Let s n = ⋅
2n2 + 5n + 7
lim lim n2 + 3n + 5
Then sn =
n→ ∞ n→ ∞ 2n2 + 5n + 7
1 1
1 + 3⋅ + 5⋅
lim n 2
= n , dividing the Nr and Dr by n2
n → ∞ 2 + 5⋅ + 7⋅
1 1
n n2
1 + 3 .0 + 5 .0 1  lim 1 lim 1 
= = ⋅ ∵ =0 =
2 + 5 .0 + 7 .0 2  n → ∞ n n → ∞ n2 
Hence the sequence < s n > converges to 1 / 2.
n
Problem 21: Prove that the sequence < s n > where s n = is convergent.
n2 + 1
Solution: We have
lim lim n lim (1 / n)
sn = = ,
n→ ∞ n→ ∞ n2 + 1 n → ∞ 1 + (1 / n2 )
dividing the Nr and Dr by n2
0
= = 0.
1+ 0
Another solution: Using the definition of the limit of a sequence we shall show that the
given sequence converges to zero.
Take any given ε > 0.
n n 1 1
We have |s n − 0| = |s n| = < = < ε, provided n > ⋅
n2 + 1 n2 n ε
If we take m ∈ N such that m > 1 / ε, then
|s n − 0|< ε, V n ≥ m.
∴ lim s n = 0, by the definition of the limit of a convergent sequence.
Another solution by using monotone convergence theorem
First we shall show that the given sequence is monotonic decreasing.
We have
n+1 n
sn + 1 − sn = −
( n + 1)2 + 1 n2 + 1
( n + 1) ( n2 + 1) − n ( n2 + 2n + 2)
=
[( n + 1)2 + 1] ( n2 + 1)
− n2 − n + 1
= < 0, V n ∈ N.
[( n + 1)2 + 1] ( n2 + 1)
∴ s n + 1 < s n V n ∈ N and so < s n > is a decreasing sequence.
Also s n > 0, V n ∈ N.
R-75

Thus < s n > is a monotonic decreasing sequence and is bounded below. Therefore by
monotone convergence theorem < s n > converges to its infimum.
Since lim s n = 0, therefore inf < s n > = 0.
Since < s n > is a decreasing sequence, therefore
1
s n ≤ s1 = , V n ∈ N.
2
1
∴ sup < s n > = s1 = ⋅
2
Another solution by using Sandwich theorem
n n 1
We have sn = < = ⋅
n2 + 1 n2 n
1
Thus 0 < sn < , V n ∈ N.
n
lim lim 1
Since 0 = 0 and = 0, therefore by Sandwich theorem
n→ ∞ n→ ∞ n
lim
s n = 0.
n→ ∞
Problem 22: Show that the sequence < s n > defined by
1 1 1
sn = + + …+
√ ( n2 + 1) √ ( n2 + 2) √ ( n2 + n)
converges to 1.
Solution: For all n > 1, we have
1 1 1
sn > + + …+ [upto n terms]
2 2
√ ( n + n) √ ( n + n) 2
√ ( n + n)
n 1
= =
√ ( n2 + n) 1 + 1 
 
 n
1 1 1
and sn < + + …+ [upto n terms]
√ ( n2 + 1) √ ( n2 + 1) √ ( n2 + 1)
n 1
= = ⋅
√ ( n2 + 1)  1
1 + 2 
 n 
1 1
Thus < sn < , V n > 1.
1 + 1   1
  1 + 2 
 n  n 
lim 1 lim 1
Since = 1 and = 1,
n→ ∞ 1 + 1  n→ ∞  1
  1 + 2 
 n  n 
lim
therefore by Sandwich theorem s n = 1.
n→ ∞
R-76

Problem 23: Prove that


 1 1 1  1
lim  + + …+ = ⋅
2 2
 √ (2n + 1) √ (2n + 2)
2
√ (2n + n) √ 2
1 1 1
Solution: Let s n = + + …+ ⋅
√ (2n2 + 1) √ (2n2 + 2) √ (2n2 + n)
Then for all n > 1, we have
n n
< sn <
√ (2n2 + n) √ (2n2 + 1)
1 1
⇒ < sn < , V n > 1.
2 + 1   1
  2 + 2 
 n  n 
lim 1 1 lim 1
Since = = ,
n→ ∞ 2 + 1  √2 n → ∞  1
  2 + 2 
 n  n 
lim 1
therefore by Sandwich theorem sn = ⋅
n→ ∞ √2
Problem 24: Show that
 1 1 1 
lim  + + …+  = 0, λ > 1.
λ λ (2n) λ 
 ( n + 1) ( n + 2)
1 1 1
Solution: Let s n = + + …+ ⋅
( n + 1) λ ( n + 2) λ ( n + n) λ
Then for all n > 1, we have
n n n 1
< sn < < =
( n + n) λ ( n + 1) λ nλ n λ −1
1 1
⇒ < sn < , V n > 1.
2 λ n λ −1 n λ −1
lim 1 lim 1
Since λ > 1, therefore =0 = ⋅
n → ∞ 2 λ n λ −1 n→ ∞ n λ −1
lim
Hence by Sandwich theorem, s n = 0.
n→ ∞
Problem 25: A sequence < s n > is defined as follows :
s1 = a > 0, s n + 1 = √ {( ab2 + s n2 ) / ( a + 1)}, b > a, n ≥ 1.
Show that < s n > is a bounded monotonically increasing sequence and lim s n = b.
Solution: Since a > 0, therefore s n > 0, V n ∈ N.
First by mathematical induction we shall show that s n < b, V n ∈ N.
We have s1 = a < b. [∵ b > a]
Now assume that s n < b, for some n ∈ N.
Then s n2 < b2 . [∵ s n > 0 and b > 0 ]
R-77

ab2 + s n2 s 2 − b2
We have s n 2+ 1 − b2 = − b2 = n < 0.
a+1 a+1
∴ s n2+ 1 < b2 or s n + 1 < b.

Thus s1 < b and if s n < b, then s n + 1 < b.


∴ by induction s n < b, V n ∈ N.
Since 0 < s n < b, V n ∈ N, therefore the sequence < s n > is bounded.
ab2 + s2 2
n − s 2 = ab − a s n
2
Now s n2+ 1 − s n2 = n
a+1 a+1
a ( b2 − s2
n ) > 0, V n ∈ N.
= [∵ s n2 < b2 , V n ∈ N]
a+1
∴ s n2+ 1 > s n2 , V n ∈ N

⇒ s n + 1 > s n , V n ∈ N. [∵ s n > 0, V n ∈ N]
∴ the sequence < s n > is a monotonic increasing sequence and so s n ≥ s1 = a ,
V n ∈ N.
Since the sequence < s n > is monotonic increasing and bounded above, therefore by
monotone convergence theorem < s n > converges to its supremum.
lim
Let s = l.
n→ ∞ n

 ab2 + s n2 
We have sn + 1 =  ⋅
 a+1 
 
ab2 + s n2
∴ s n2+ 1 = ⋅ ...(1)
a+1
Taking limit of both sides of (1) as n → ∞, we get
lim lim ab2 + s n2
s 2 = ⋅
n → ∞ n +1 n → ∞ a+1
ab2 + l2
∴ l2 = or l2 a + l2 = ab2 + l2
a+1
or al2 = ab2 or l2 = b2 [∵ a ≠ 0 ]
or l = ± b.
But l ≠ − b because s n ≥ a > 0, V n ∈ N.
Hence l = b.
1/ n
 (3n) ! 
Problem 26: Prove that lim   = 27.
n→ ∞ ( n !)3 
(3n) !
Solution: Let s n = ⋅ Then s n > 0, V n ∈ N.
( n !)3
sn + 1
Now we know that if s n > 0 V n ∈ N, then lim ( s n)1 / n = lim , provided the latter
sn
limit exists.
R-78
sn + 1 (3n + 3) ! ( n !)3
We have = ⋅
sn {( n + 1) !}3 (3n) !
3 + 3  3 + 2  3 + 1 
     
(3n + 3) (3n + 2) (3n + 1)  n  n  n
= = ⋅
( n + 1)3 1 + 1 
3
 
 n
lim sn + 1 (3 + 0 ) (3 + 0 ) (3 + 0 )
∴ = = 27.
n→ ∞ sn (1 + 0 )3
1/ n
 (3n) ! 
∴ lim ( s n)1 / n = lim   = 27.
( n !)3 

 ( n !)1 / n  1
Problem 27: Prove that lim  = ⋅
 n  e
n!
Solution: Let s n = ⋅ Then s n > 0, V n ∈ N.
nn
sn + 1
Now we know that if s n > 0 for all n ∈ N, then lim ( s n)1 / n = lim , provided the latter
sn
limit exists. [See Theorem 12 of article 9]
sn + 1 ( n + 1) ! nn nn 1
We have = ⋅ = = ⋅
sn ( n + 1)n + 1 n ! ( n + 1)n  1
n
1 + 
 n
sn + 1 1 1
∴ lim = lim = ⋅
sn n e
1 + 1
 
 n
 ( n !)1 / n  1
∴ lim ( s n)1 / n = lim  = ⋅
 n  e
Problem 28: (i) Let < s n > be a sequence defined as follows :
3 1
s1 = ; s n + 1 = 2 − , n ≥ 1.
2 sn
Show that < s n > is monotonic and bounded. Find the limit of the sequence.
(ii) Show that the sequence < s n > defined by the formula s1 = 1, s n + 1 = √ (3s n)converges to 3.
Solution: (i) First we shall show by induction that s n > 1, V n ∈ N.
3
To start the induction we observe that s1 = > 1.
2
Now assume as our induction hypothesis that s n > 1, for some positive integer n.
Then sn > 1
1
⇒ <1
sn
1
⇒ − > −1
sn
R-79
1
⇒ 2− > 2 −1
sn
 1
⇒ s n + 1 > 1. ∵ s n + 1 = 2 − s 
 n
Thus s1 > 1 and if s n > 1, then s n + 1 > 1.
∴ by induction s n > 1, V n ∈ N.
3 1 1 2 4 1
Now s1 = = 1 , s2 = 2 − = 2 − = =1 ⋅
2 2 s1 3 3 3
∴ s2 < s1.
Assume as our induction hypothesis that s n + 1 < s n , for some positive integer n.
Then sn + 1 < sn
1 1
⇒ >
sn + 1 sn [ ∵ s > 0, V n ∈ N]
n
1 1 1 1
⇒ − <− ⇒ 2− <2−
sn + 1 sn sn + 1 sn
⇒ s n + 2 < s n + 1.
Thus s2 < s1 and if s n + 1 < s n , then s n + 2 < s n + 1 .
∴ by induction s n + 1 < s n , V n ∈ N.
∴ the sequence < s n > is a monotonic decreasing sequence.
As shown above, s n > 1, V n ∈ N.
∴ 1 is a lower bound for < s n > and so < s n > is bounded below.
Also since < s n > is a decreasing sequence, therefore
3
s n ≤ s1 = , V n ∈ N.
2
3
∴ is an upper bound for < s n > and so < s n > is bounded above.
2
Also sup < s n > = s1 = 3 / 2.
Since the sequence < s n > is monotonic decreasing and bounded below, therefore by
monotone convergence theorem < s n > converges to its infimum.
lim
Let s n = l.
n→ ∞
1
We have sn + 1 = 2 − ⋅ ...(1)
sn
Taking limit of both sides of (1) as n → ∞, we get
lim lim 1
sn + 1 = 2 − ⋅
n→ ∞ n → ∞ sn
1
∴ l =2− or l2 − 2l + 1 = 0
l
or ( l − 1)2 = 0 or l = 1.
lim
Hence s n = 1 = inf < s n >.
n→ ∞
R-80

(ii) We have s2 = √ (3s1 ) = √ (3 .1) = √ 3.


Since 1 < √ 3, therefore s1 < s2 .
Let us suppose that s m < s m + 1 .
Then √ (3s m ) < √ (3s m + 1 ) ⇒ s m + 1 < s m + 2 .
Hence, by mathematical induction, we have s n < s n + 1 for all n ∈ N i. e., < s n > is
monotonically increasing.
Again, s n + 1 > s n ⇒ √ (3s n) > s n
⇒ 3s n > s n2 [ ∵ s n > 0 for all n ∈ N ]
⇒ 3s n − s n2 > 0
⇒ s n (3 − s n) > 0
⇒ 3 − sn > 0 [ ∵ s n > 0 for all n ∈ N ]
⇒ s n < 3, V n ∈ N .
Hence < s n > is bounded above by 3.
Thus < s n > is a monotonically increasing sequence bounded above by 3; consequently it
converges.
Let lim s n = l . Then lim s n + 1 = l.
Now s n + 1 = √ (3s n) ⇒ lim s n + 1 = lim √ (3s n) ⇒ l = √ (3l )

⇒ l2 = 3l ⇒ l ( l − 3) = 0 ⇒ l = 0, 3.
But s n ≥ s1 = 1 V n ∈ N ⇒ s n − 1 ≥ 0 V n ∈ N.
∴ lim ( s n − 1) ≥ 0 i. e., lim s n ≥ 1.
Hence l cannot be zero. Therefore l = 3.

Problem 29: If < s n > is a sequence such that s n > 0 and s n + 1 ≤ k s n for all n ≥ m and
0 < k < 1, m being a fixed positive integer, then lim s n = 0.
Solution: We are given that
sn + 1
s n + 1 ≤ k s n i. e., ≤ k, V n ≥ m. ...(1)
sn
Putting n = m, m + 1, m + 2, ..., n − 1 in succession in (1) and multiplying the
corresponding sides of the n − m inequalities thus obtained, we get
sm + 1 sm + 2 sm + 3 s
⋅ ⋅ ⋅ .... ⋅ n ≤
sm sm + 1 sm + 2 sn − 1

≤ k n− m
sn sm
or or sn ≤ k n.
sm km
Thus 0 < s n ≤ Mk n, for all n > m ...(2)
sm
where M = is a fixed positive real number.
km
Since 0 < k < 1, therefore
lim lim
( Mk n) = M . k n = M .0 = 0.
n→ ∞ n→ ∞
lim
Also 0 = 0.
n→ ∞
R-81
lim
Hence from (2) by Sandwich theorem, = 0.
n → ∞ sn
lim xn
Problem 30: Prove that if x be any real number, then = 0.
n→ ∞ n!
xn
Solution: Let sn = ⋅
n!
If x = 0, then s n = 0, V n ∈ N and so lim s n = 0. So let x ≠ 0.
sn + 1
Now we know that if s n ≠ 0 for any n and lim = l where | l|< 1, then lim s n = 0.
sn
[See Example 5]
Now x ≠ 0 ⇒ s n ≠ 0, V n ∈ N.
sn + 1 x n +1 n ! x
We have = ⋅ = ⋅
sn ( n + 1) ! x n n + 1
lim sn + 1 lim x
∴ = = 0 and |0| = 0 < 1.
n→ ∞ s n→ ∞ n + 1
n
∴ by the above mentioned result, lim s n = 0.
lim xn
Hence = 0, V x ∈ R.
n→ ∞ n!

1 1 1
Problem 31: If s n = 1 + + + …+ , prove that < s n > converges. Also find sup < s n >
3 32 3n
and inf < s n >.
1 1 1
Solution: Here s n = 1 + + + …+
3 3 2 3n
1 1 1 1
and sn + 1 = 1 + + + …+ + ⋅
3 32 3 n 3 +1
n
1
∴ sn + 1 − sn = > 0, V n ∈ N.
3n + 1
∴ s n + 1 > s n , V n ∈ N and so the sequence < s n > is monotonic increasing.
1 1 1
Now sn = 1 + + + …+ ...(1)
3 32 3n
 n + 1
1 1 −  1
  
  3 
= , summing up the G.P. (1) upto ( n + 1) terms
1 − (1 / 3)
[1 − (1 / 3)n + 1 ] < , V n ∈ N.
3 3
=
2 2
Thus s n < 3 / 2, V n ∈ N i. e., 3/2 is an upper bound for < s n >.
∴ the sequence < s n > is monotonic increasing and bounded above.
Hence by monotone convergence theorem, the sequence < s n > converges to its
supremum.
R-82

[1 − (1 / 3)n + 1 ], therefore
3
Since s n =
2
lim 3 3  lim 1 
s = [1 − 0 ] = ⋅ ∵ n → ∞ = 0
n→ ∞ n 2 2  3n + 1 
Thus the sequence < s n > converges to 3/2 and so sup < s n > = 3 / 2.
Again < s n > is a monotonic increasing sequence and so
1 4
s n ≥ s1 = 1 + = , V n ∈ N.
3 3
∴ s1 = 4 / 3 is a lower bound for < s n > and inf < s n > = 4 / 3.
Problem 32: What do you understand by a monotonic sequence ? Prove that the sequence
2n − 7
< s n >, where s n = , is :
3n + 1
(i ) monotonic increasing ( ii ) bounded above. ( iii ) bounded below.
Also show that the sequence < s n > is convergent and find sup < s n > and inf < s n >.
Solution: Monotonic sequence: A sequence < s n > is said to be a monotonic sequence
if it is either an increasing sequence or a decreasing sequence i. e., if either s n + 1 ≥ s n or
s n + 1 ≤ s n, V n ∈ N.
For the given sequence,
2 4
(3n + 2) − − 7
2n − 7 3 3
sn = =
3n + 2 3n + 2
2 25 1
= − ⋅ ⋅ ...(1)
3 3 3n + 2
2 25 1
∴ sn + 1 = − ⋅ ⋅
3 3 3n + 5
25  1 1  25
∴ sn + 1 − sn =  3n + 2 − 3n + 5  = (3n + 2) (3n + 5) > 0, V n ∈ N.
3  
∴ s n + 1 > s n , V n ∈ N and so the sequence < s n > is a monotonic increasing sequence.
Also from (1), we observe that
2
s n < , V n ∈ N i. e., 2 / 3 is an upper bound for < s n > and so < s n > is bounded above.
3
Again < s n > is a monotonic increasing sequence and so
2−7
s n ≥ s1 = = − 1, V n ∈ N.
3+2
∴ s1 = − 1 is a lower bound for < s n > and < s n > is bounded below.
Since the sequence < s n > is monotonic increasing and bounded above, therefore by
monotone convergence theorem < s n > converges to its supremum.
lim 2 2
From (1), we observe that s = −0 = ⋅
n→ ∞ n 3 3
2
Thus the sequence < s n > converges to 2/3 and so sup < s n > = ⋅
3
Also inf < s n > = s1 = − 1.
R-83

1 1 1 1
Problem 33: If s n = + + + …+ , prove that < s n > is increasing and
1⋅ 2 2 ⋅ 3 3 ⋅ 4 n ( n + 1)
convergent. Find also sup < s n > and inf < s n >.
1 1 1 1
Solution: Here s n = + + + …+
1⋅ 2 2 ⋅ 3 3 ⋅ 4 n ( n + 1)
2 −1 3 − 2 4 − 3 ( n + 1) − n
= + + + …+
1⋅ 2 2⋅3 3⋅4 n ( n + 1)
1 1 
= 1 −  +  −  +  −  + … +  −
1 1 1 1 1

 2  2 3  3 4  n n + 1
1
=1− ⋅ ...(1)
n+1
1
∴ sn + 1 = 1 − ⋅
n+2
1 1 1
∴ sn + 1 − sn = − = > 0, V n ∈ N.
n + 1 n + 2 ( n + 1) ( n + 2)
∴ s n + 1 > s n , V n ∈ N and so the sequence < s n > is monotonic increasing.
Also from (1) we observe that
s n < 1, V n ∈ N i. e.,1 is an upper bound for < s n >.
Thus the sequence < s n > is monotonic increasing and bounded above. Hence by
monotone convergence theorem < s n > converges to its supremum.
1 lim
Since sn = 1 − , therefore s n = 1 − 0 = 1.
n+1 n→ ∞
Thus the sequence < s n > converges to 1 and so sup < s n > = 1.
Again < s n > is a monotonic increasing sequence and so
1 1
s n ≥ s1 = 1 − = , V n ∈ N.
2 2
1
∴ s1 = is a lower bound for < s n >.
2
1
Since is the value of the first term s1 of the sequence < s n >, therefore the sequence
2
1 1
< s n > attains its lower bound and so inf < s n > = ⋅
2 2
1 1
Problem 34: If s n = 1 + + … + − log n, prove that < s n > is decreasing and bounded.
2 n
1 1
Solution: Here s n = 1 + + … + − log n.
2 n
1 1 1
∴ sn + 1 = 1 + + … + + − log ( n + 1).
2 n n+1
1
∴ s n − s n + 1 = − log n − + log ( n + 1)
n+1
n + 1
= log 
1
 − ⋅
 n  n+1
R-84
x
We know that < log (1 + x ) < x for x > 0.
1+ x

< log 1 +  < for all n ∈ N


1/n 1 1

1 + (1 / n)  n n
n + 1 1
< log 
1
i. e.,  < for all n ∈ N.
n+1  n  n
n + 1
s n − s n + 1 = log 
1
∴  − > 0 for all n ∈ N.
 n  n+1
∴ < s n > is a decreasing sequence.
Now let f ( x ) = 1 / x. Then f ( x ) ≥ 0 for all x ∈ [1, ∞ ] and the function f ( x ) is
monotonically decreasing on [1, ∞ ].
Let n ∈ N.
1 1
If n ≤ x ≤ n + 1, then ≥ ⋅
n x
n +1 1 n +1 1

∫ n n
dx ≥
∫ n x
dx.

1 n +1 1

n

∫ n x
dx for all n ∈ N. ...(1)

Putting n = 1, 2, 3, … , n − 1 in succession in (1) and then adding all the results, we


get
1 1 1 2 1 3 1 n 1
1+ + + …+
2 3 n −1

∫1 x
dx +

2 x
dx + … +

n −1 x
dx.

1 1 1 n 1
∴ 1+ + + …+
2 3 n −1

1 ∫x
dx = log n.

1 1 1 1 1
∴ 1+ + + …+ + ≥ + log n
2 3 n −1 n n
1 1 1 1
or 1 + + + … + − log n ≥ > 0 for all n ∈ N.
2 3 n n
∴ s n > 0 for all n ∈ N.
∴ the sequence < s n > is bounded below and hence bounded also because it is a
decreasing sequence.

Problem 35: Test the sequence < s n >, defined by s n = ( − 1)n n, for limit points.
Solution: The sequence < s n > = < − 1, 2, − 3, 4, − 5, 6, … >.
Obviously each s n is some integer.
We shall show that no real number can be a limit point of the sequence < s n >.
Let l be any real number.
1 1
Then ] l − , l + [ is a nbd of l and it contains at most one term of the sequence < s n >.
4 4
1
We observe that the length of this interval is and so it can contain at most one integer
2
i. e., it can contain at most one term of the sequence < s n > .
R-85
1 1
Thus if l ∈ R, there exists a nbd ] l − , l + [ of l which does not contain infinite terms
4 4
of the sequence < s n > . Hence l is not a limit point of < s n >.
Thus no real number can be a limit point of < s n > and so < s n > has no limit points.

Problem 36: Find the limit points of the sequence < s n > defined by
s n = ( − 1)n 1 +  ⋅
1
 n
Solution: We know that a real number p is a limit point of a sequence < s n > iff there
exists a subsequence of < s n > converging to p.
Here the subsequence < s2 n − 1 > of < s n > converges to − 1 and the subsequence < s2 n >
converges to 1.
lim lim  1 
We have s = ( − 1)2 n − 1 1 + 
n → ∞ 2n − 1 n→ ∞  2 n − 1

lim  1 
= −1 −  = −1
n→ ∞  2 n − 1

lim lim lim 


( − 1)2 n 1 +
1 1
and s =  = n → ∞ 1 +  = 1.
n → ∞ 2n n → ∞  2n   2n 
∴ both 1 and − 1are limit points of < s n > . Also if l is any real number other than 1 or − 1
then no subsequence of < s n > can converge to l.
Hence 1 and −1 are the only limit points of < s n > .

Problem 37: State and prove Cauchy’s general principle of convergence for real sequences.
n+1
Hence prove that the sequence < s n >, where s n = converges.
n
Solution: For the first part of this problem refer to Text Book Theorem 2 of article 12.
n+1 1
Second part: Let s n = =1+ ⋅
n n
We shall show that the sequence < s n > is a Cauchy sequence.
Take any given ε > 0.
If n ≥ m, then
 = 1 − 1 = m − n
1 + 1  − 1 + 1  
|s n − s m| = 
 n  m  n m  nm 
n−m n−m 1 1  n−m 
= = ⋅ < ⋅ ∵ 0 ≤ < 1
nm n m m  n 
1 1
If we take m ∈ N such that m > i. e., if < ε, then
ε m
1 1
|s n − s m| = − < ε, V n ≥ m.
n m
Thus for any given ε > 0, there exists a positive integer m such that
|s n − s m|< ε, V n ≥ m.
Hence the given sequence < s n > is a Cauchy sequence and so by Cauchy’s general
principle of convergence for real sequences it is a convergent sequence.
R-86

Problem 38: Show, by applying Cauchy’s convergence criterion, that the sequence < s n > defined
by
1 1 1
sn = 1 + + + …+ does not converge.
3 5 2n − 1
Solution: We shall show that the given sequence is not a Cauchy sequence. For this we
1
shall show that if we take ε = > 0, then there exists no positive integer m such that
4
| s n − s m |< ε, V n ≥ m.
Whatever positive integer m may be, if we take n = 2m + 1, then n > m and we have
 1 1 1 1 1 
|s n − s m| = 1 + + + … + + + …+ 
 3 5 2m − 1 2m + 1 4 m + 1

 1 1 
− 1 + + … + 
 3 2 m − 1 
1 1 1
= + + …+ ,
2m + 1 2m + 3 4m + 1
the number of terms in this sum which is an A.P. is m + 1
1 1
> + + … upto m + 1 terms
4m + 1 4m + 1
1 3
(4m + 1) +
m+1 4 4 =1+ 3 1
= = > ⋅
4m + 1 4m + 1 4 4 (4m + 1) 4
1
Thus if we take ε = , then whatever positive integer m we take, we have
4
1
n = 2m + 1 > m and |s n − s m| = s2 m + 1 − s m > i. e.,|s n − s m|> ε.
4
1
In this way for ε = > 0, there exists no positive integer m such that
4
|s n − s m|< ε , V n ≥ m.
∴ the given sequence is not a Cauchy sequence.
Hence by Cauchy convergence criterion < s n > is not convergent.
Problem 39: If the sequence < s n > converges and if < tn > is a sequence such that
|tn − tm|≤ |s n − s m|,
for all positive integers m and n, then prove that < tn > converges.
Solution: It is given that the sequence < s n > is convergent and
|tn − tm|≤ |s n − s m| ...(1)
for all positive integers m and n.
We are to prove that the sequence < tn > converges.
Take any given ε > 0.
R-87

Since the sequence < s n > is convergent, therefore it is a Cauchy sequence and so there
exists m ∈ N such that
|s n − s m|< ε, V n ≥ m
⇒ |tn − tm|< ε, V n ≥ m. [∵ from (1), | tn − tm |≤ |s n − s m|]
∴ the sequence < tn > is also a Cauchy sequence and hence < tn > converges by Cauchy
convergence criterion.
Problem 40: If < s n > is a Cauchy sequence of real numbers which has a subsequence converging
to l, prove that < s n > itself converges to l.
Solution: Since the sequence < s n > is a Cauchy sequence, therefore it is a convergent
sequence.
Now we know that every subsequence of a convergent sequence < s n > is convergent and
converges to the same limit to which the sequence < s n > converges.
Now it is given that < s n > has a subsequence converging to l.
lim
∴ s = l i. e., < s n > itself converges to l.
n→ ∞ n

Problem 41: Show that lim [( n !) ( a / n)n] = 0 or + ∞ according as a < e or a > e, where a is any
non-negative real number.
Solution: Let s n = ( n !) ( a / n)n.
If a = 0, then a < e. Also then s n = 0, V n ∈ N and so lim s n = 0.
So let a > 0. Then s n > 0, V n ∈ N.
n +1
sn + 1  a  1
Now = ( n + 1) ! ⋅   ⋅
sn  n + 1 n
n ! .  
a
 n
nn 1
= a. = a. ⋅
( n + 1)n 1 + 1 
n
 
 n
sn + 1a
∴ lim = ⋅
sn e
 a a
We have = < 1 if a < e.
e e
∴ by the result mentioned in Text Book Example 5 we have lim s n = 0.
1
Again let tn = 1 / s n = ⋅
( n !) ( a / n)n
tn + 1 n
1 
⋅ 1 +  ⋅
1
Then =
tn a  n
tn + 1 e
∴ lim = ⋅
tn a
We have  e  = e < 1 if a > e.
 a a
R-88

∴ if a > e, then lim tn = 0 and consequently lim s n = ∞.


Note that from Text Book Theorem 2 of article 8 if s n > 0 for all n ∈ N, then
1
s n → ∞ as n → ∞ ⇔ → 0 as n → ∞.
sn
Here ( n !) ( a / n)n > 0, V n ∈ N because a > 0.
2un − 1 un − 2
Problem 42: If 0 < u1 < u2 and un = ( i. e., un is the harmonic mean of un − 1
un − 1 + un − 2
and un − 2 ), show that
lim un = 3u1 u2 / (2 u1 + u2 ).
2un − 1 un − 2
Solution: Here u1 > 0, u2 > 0 and un = for all n > 2.
un − 1 + un − 2
∴ un > 0 for all n ∈ N.
1 1
Let sn = i. e., un = ⋅
un sn
1
Then s n = ( s n − 1 + s n − 2 ) for all n > 2.
2
Now proceed as in Text Book Example 30.
1
We find that lim s n = ( s1 + 2s2 ).
3
1 1 3 3u1 u2
∴ lim un = = = = ⋅
lim s n 1 ( s + 2s )  1 2  2u1 + u2
1 2  + 
3  u1 u2 
Problem 43: If the sequences < s n > and < tn > converge to zero and if < tn > is a strictly
decreasing sequence so that tn + 1 < tn V n ∈ N , then
s sn − sn + 1
lim n = lim
tn tn − tn + 1
provided that the limit on the right exists, whether finite or infinite.
sn − sn + 1
Solution: Case 1: Let lim = l, where l is finite.
tn − tn + 1 ...(1)

Take any given ε > 0.


By virtue of (1), there exists a positive integer m such that for all n ≥ m, we have
1 sn − sn + 1 1
l − ε< <l+ ε
2 tn − tn + 1 2

⇒  l − 1 ε ( t − t  1 
  n n + 1 ) < s n − s n + 1 <  l + ε ( tn − tn + 1 ), ...(2)
 2   2 
because tn − tn + 1 is positive.
Changing n to n, n + 1, n + 2, … , n + p − 1, in succession in (2) and adding, we see
that for all n ≥ m and for all p ≥ 0
 l − 1 ε ( t − t  1 
  n n + p ) < ( s n − s n + p ) <  l + ε ( tn − tn + p ).
 2   2 
R-89

Keep n fixed and let p → ∞. Since s n + p → 0 and tn + p → 0, we obtain

 l − 1 ε t ≤ s ≤  l + 1 ε t , V n ≥ m.
  n n   n ...(3)
 2   2 
Since the sequence < tn > is strictly decreasing and converges to zero, therefore it
converges to its infimum.
∴ inf < tn > = 0 and so tn > 0, V n ∈ N.
Hence from (3), we have
1 s 1
l − ε ≤ n ≤ l + ε, V n ≥ m
2 tn 2

 s n − l
≤ 1 ε < ε, V n ≥ m.

 n
t  2
lim sn
∴ = l.
n→ ∞ t
n
sn − sn + 1
Case 2: Let lim = ∞. ...(4)
tn − tn + 1
Take any given k > 0. By virtue of (4), there exists a positive integer m such that for all
n ≥ m, we have
sn − sn + 1
> 2k
tn − tn + 1
⇒ s n − s n + 1 > 2k ( tn − tn + 1 ) because tn − tn + 1 is positive.
As in case 1, we obtain
s n − s n + p > 2k ( tn − tn + p )
⇒ s n ≥ 2k tn V n ≥ m
sn
⇒ ≥ 2k > k V n ≥ m
tn
lim sn
⇒ = ∞.
n→ ∞ tn
a
Problem 44: If s n = , where a, s n are positive, show that the sequence < s n > tends to a
1 + sn − 1
definite limit l, the positive root of the equation x2 + x = a.
Solution: It is given that s n > 0, V n ∈ N.
Also for all n ≥ 2, we have
a 1  1 
sn = = a. < a. ∵ < 1
1 + sn − 1 1 + sn − 1  1 + s n − 1 
∴ 0 < s n < a, for all n ≥ 2 .
Hence < s n > is a bounded sequence.
a a − a (sn − 1 − sn − 3 )
Now s n − s n − 2 = − = ...(1)
1 + s n − 1 1 + s n − 3 (1 + s n − 1 ) (1 + s n − 3 )
R-90

Replacing n by n − 1 in (1), we get


− a (sn − 2 − sn − 4 )
sn − 1 − sn − 3 = ⋅
(1 + s n − 2 ) (1 + s n − 4 )
Substituting this value of s n − 1 − s n − 3 in (1), we get
a2 ( s n − 2 − s n − 4 )
sn − sn − 2 = ...(2)
(1 + s n − 1 ) (1 + s n − 2 ) (1 + s n − 3 ) (1 + s n − 4 )
The result (2) shows that the even and odd terms of < s n > form separate monotonic
subsequences. Again (1) shows that if odd terms form a monotonic decreasing
subsequence, even terms will form a monotonic increasing subsequence and vice versa.
Also both these subsequences are bounded because < s n > is bounded. Hence by
monotone convergence theorem both these subsequences are convergent.
Let the odd terms subsequence converge to l and the even terms subsequence converge
to l′.
a
By hypothesis s n = ⋅ ...(3)
1 + sn − 1
Taking limit of both sides of (3) as n → ∞, we get
a
l= , when n is odd
1+ l′
a
and l′ = , when n is even.
1+ l
From these, we get
a = l + ll ′ = l ′ + ll ′
⇒ l = l ′.
Thus both the even and odd terms subsequences of < s n > converge to the same limit l
and hence < s n > also converges to l.
Now taking limit of both sides of (3) as n → ∞, we get
a
l= ⇒ l2 + l − a = 0.
1+ l
∴ l is a root of the equation x2 + x − a = 0 which has one positive and one negative
root.
Since s n > 0, V n ∈ N, therefore l cannot be negative.
Hence l is the positive root of the equation x2 + x − a = 0.

Problem 45: Prove that the set of limit points of every sequence is a closed set.
Solution: Closed set. Definition. A subset S of R is said to be closed if it contains all
its limit points.
Let S be the set of all limit points of a sequence < s n > .
Then to prove that S is a closed set.
S will be closed if D ( S ) ⊆ S.
R-91

Let x be a limit point of S. Then for every ε > 0, the open interval ] x − ε, x + ε [ contains
infinite points of S.
Let y ∈ S and x − ε < y < x + ε.
Let ε1 = minimum{ x + ε − y, y − ( x − ε)}.
Then ] y − ε1, y + ε1 [ ⊆ ] x − ε, x + ε [ .
Since y ∈ S, so y is a limit point of < s n > .
∴ ] y − ε1, y + ε1 [ contains infinite terms of the sequence < s n >.
But ] y − ε1, y + ε1 [ ⊆ ] x − ε, x + ε [ .
∴ for every ε > 0, ] x − ε, x + ε [ contains infinite terms of the sequence < s n > and
so x is a limit point of < s n > i. e., x ∈ S.
Thus x ∈ D ( S ) ⇒ x ∈ S.
∴ D ( S ) ⊆ S i. e., S contains all its limit points and so S is a closed set.

Hints to Objective Type Questions

Multiple Choice Questions


1. (a). See article 10, Theorem 1.
2. (c). See article 6, Theorem 1.
3. (b). See article 6, Theorem 3.
5n
4. (d). We have lim s n = lim
n→ ∞ n→ ∞ n + 3 n1 /2
lim 5n lim 5
= =
n→ ∞ n [1 + 3 / n1 /2 ] n → ∞ 1 + 3 / n1 /2
5
= = 5.
1+ 0
5. (c). The sequence < 1, − 1, 1, − 1, 1, − 1, … > has exactly two limit points namely 1
and −1. See article 11, Illustration 1.
n+3
lim s = lim 1 + 2 
6. (b). We have n  
n→ ∞ n→ ∞  n
n 3
= lim 1 + 2  1 + 2  = e 2 .1 = e 2.
   
n→ ∞  n  n
lim s = lim  1 1  1
7. (b). We have n 1 + +  + …+
n→ ∞ n→ ∞  2 2 n22
1 1 1 1
=1+ + +
+ …∞ = =2.
2 22 23 1
1−
2
[Summing the infinite geometric series.]
Thus the given sequence converges to 2.
R-92
2
8. (a). We have lim s = lim 3n + 1
n
n→ ∞ n → ∞ 3n2 − 1
2 2 2
= lim n [3 + (1 / n )] = lim 3 + (1 / n )
n → ∞ n2 [3 − (1 / n2 )] n → ∞ 3 − (1 / n2 )
3+0
= = 1.
3−0
∴ the given sequence converges to 1.
9. (d). We know that every Cauchy sequence is convergent. See article 12,
Theorem 2.
lim
10. (a). We have n1 / n = 1.
n→ ∞
∴ by Cauchy’s first theorem on limits,
lim 1 ( s + s + … + s ) = lim 1 (1 + 21 /2 + 31 /3 + … + n1 / n) = 1.
1 2 n
n→ ∞ n n→ ∞ n
See Ex. 35.
11. (c). See article 6, Theorem 5.

Fill in the Blank(s)


1. { −1, 1}. See article 2, definition of the range of a sequence.
lim 1
2. 0. We have = 0. See Example 1.
n→ ∞ n
1 1 1 1
3. We have < s n > = < , , , … >. Obviously sup < s n > = and inf < s n > = 0.
2 22 23 2
n 1 2 3 4
4. The given sequence < > = < , , , , … >.
n+1 2 3 4 5
n
Obviously sup < > = 1.
n+1
5. The nth term of the given sequence < 1, − 1, 1, − 1, … > is ( −1)n − 1.
6. l. See Theorem 4 of article 6.
7. 1. See Example 11.
lim
8. − ∞. We have ( −2 n) = − ∞.
n→ ∞
9. ll ′. See Theorem 5 of article 9.
10. l. See article 9, Theorem 10, Cauchy’s first theorem on limits.
11. Convergent. See article 10, Theorem 1, Corollary 2.
1 lim n2 + 4 n + 7 lim n2 [1 + (4 / n) + (7 / n2 )]
12. ⋅ We have =
3 n→ ∞ 3n2 + 5n − 9 n→ ∞ n2 [3 + (5 / n) − (9 / n2 )]
lim 1 + (4 / n) + (7 / n2 ) 1 + 0 + 0 1
= = = ⋅
n → ∞ 3 + (5 / n) − (9 / n2 ) 3 + 0 + 0 3
R-93

lim n+2 lim n 2


1 + 1  = 1 + 1  1 + 1  = e . 12 = e .
13. We have      

n→ ∞ n n→ ∞  n  n
lim lim
14. We have sn = ( n + 1 − n)
n→ ∞ n→ ∞

lim ( n+1− n) ( n + 1 + n)
=
n→ ∞
n+1+ n

lim ( n + 1) − n lim 1
= = = 0.
n→ ∞ n→ ∞
n+1+ n n+1+ n

n
We have lim s n = lim 1 +  = e .
1
15.
n→ ∞ n→ ∞  n

∴ by Cauchy’s second theorem on limits, lim ( s1 s2 … s n)1 / n = e .


n→ ∞
See article 9, Theorem 11.
True or False
1. T . We have |s n| = 2 . Thus, |s n|≤ 2 for all n ∈ N and so the given sequence is
bounded.
2. F. The sequence < 1, − 1, 1, − 1, 1, − 1, … > is an oscillatory sequence and so it is not
convergent.
3. F. The sequence < 3, − 3, 3, − 3, … > is not a convergent sequence and so it cannot
be a Cauchy sequence because we know that every Cauchy sequence is
convergent.
4. T . See article 12, Theorem 2.
5. T . See article 8, Theorem 1.
6. F. See article 6, definition of a convergent sequence.
7. T . See article 6, Theorem 2.
8. T . See article 6, Theorem 5.
9. F. For example, the sequence < 1, − 1, 1, − 1, … > is bounded but it is not
convergent.
1 1 1 1
10. T . We have lim = 0. So, the sequence < 1, , , … , , … > converges to
n→ ∞ n 2 3 n
zero.We know that every convergent sequence is a Cauchy sequence.
11. T . See Theorem 1 of article 9.
12. T . See article 10, Theorem 1, Corollary 1.
13. T . See article 10, Theorem 3.
14. F. We know that a monotonically increasing sequence which is not bounded
above diverges to infinity.
15. T . See article 11, Theorem 3, Bolzano-Weierstrass theorem for sequences.
R-94

16. F. Both 1 and −1 are limit points of the sequence < 1, − 1, 1, − 1, … >.
17. F. We know that every convergent sequence is also a Cauchy sequence.
1 1 1
18. F. The sequence < s n >, where s n = 1 + + + … + , is not a Cauchy
2 3 n
sequence. See Ex. 29.
19. T . The given sequence is a convergent sequence. See Ex. 18.
20. T . We know that if a sequence < s n > converges to l, then the sequence < |s n|>
converges to |l |. See problem 7 of Comprehensive Problems1.
21. F. Consider the sequence < s n > = < 1, − 1, 1, − 1, … >.
We have <|s n|> = < 1, 1, 1, … > which converges to 1 while the sequence
< 1, − 1, 1, − 1, 1, −1, … > itself is not convergent.
n
22. T . We have lim s n = lim
n→ ∞ n → ∞ n3 + 1

lim n 1
= = lim = 0.
n → ∞ n3 [1 + (1 / n3 )] n → ∞ n2 [1 + (1 / n3 )]
Thus, the sequence < s n > converges to zero.

23. T . For all n ∈ N, we have 2 ≤ s n < 3. See Ex. 19.

24. F. The sequence < 1, − 1, 1, − 1, 1, − 1, … > is neither an increasing sequence nor a


decreasing sequence. So it is not a monotonic sequence.

 1 
25. F. We have lim s n = lim  3 − n − 1  = 3 − 0 = 3.
n→ ∞ n→ ∞  3 
Thus the given sequence converges to 3.

❍❍❍
R-95

Chapter-4
Uniform Convergence of
Sequences and Series of Functions

Comprehensive Problems 1
x
Problem1: Prove that the sequence < fn > defined by fn ( x ) = (0 ≤ x < ∞ ) is uniformly
1 + nx
convergent to 0 on [0, ∞[.
Solution: We have f ( x ) = lim fn ( x ) = 0 V x ∈ [0, ∞[ .
n→ ∞
 x  x
∴ | fn ( x ) − f ( x )| =  − 0 = [∵ x ≥ 0]
1 + nx  1 + nx
1
< for all x ∈ [0, ∞[.
n
1
∴ M n = sup {| fn ( x ) − f ( x )|: x ∈ [0, ∞ [} =

n
It follows that M n → 0 as n → ∞. Hence the sequence converges uniformly to 0 by
M n -test.
Problem 2: Show that the sequence < fn > where
fn ( x ) = nx (1 − x )n
does not converge uniformly on [0, 1]. (Garhwal 2007)
Solution: We have fn ( x ) = nx (1 − x )n .
Now fn ( x ) = 0 when x = 0 or 1 and when 0 < x < 1, we have
lim fn ( x ) = lim
nx  Form ∞ 
n→ ∞ n → ∞ (1 − x )
−n  ∞ 
x
= lim −n
n → ∞ − (1 − x ) log (1 − x )
x (1 − x )n
= lim −
n → ∞ log (1 − x )
= 0 since (1 − x )n → 0 as n → ∞.
∴ f ( x) =
lim fn ( x ) = 0 V x ∈ [0, 1].
n→ ∞
Here M n = sup {| fn ( x ) − f ( x )|: x ∈ [0, 1]}
= sup{ nx (1 − x )n : x ∈ [0, 1]}
n
1 1  taking x = 1 ∈ [0, 1]
≥ n. 1 − 
n n  n 
R-96
n
= 1 −  → as n → ∞.
1 1
 n e
Since M n cannot tend to zero as n → ∞ hence by M n -test the sequence < fn > does not
converge uniformly on [0, 1]. Here 0 is a point of non-uniform convergence since x → 0
as n → ∞.
Problem 3: Show that 0 is a point of non-uniform convergence of the sequence < fn ( x ) >, where
− n x2
fn ( x ) = nxe , x ∈ R. (Rohilkhand 2007)
2
Solution: Here f ( x ) = lim fn ( x ) = lim nxe − nx
n→ ∞ n→ ∞

= lim
nx  Form ∞ 
n → ∞ e nx
2  ∞ 
x
= lim 2
= 0.
n → ∞ x2 e nx

Now M n = sup {| fn ( x ) − f ( x )|: x ∈ R }


2
= sup { n |x| e − n x : x ∈ R }
1 − n . (1 / n) 1
≥n e (taking x = ∈ R)
√n √n
√n
= → ∞ as n → ∞.
e
Thus M n cannot tend to zero as n → ∞.
It follows by M n -test that the sequence is non-uniformly convergent. Also as
n → ∞, x → 0 and hence 0 is a point of non-uniform convergence.
Problem 4: Show that the sequence < fn > where fn ( x ) = x n − 1 (1 − x ) converges uniformly in
the interval [0, 1].
Solution: We have f ( x) = lim x n − 1 (1 − x ) = 0 V x ∈ [0, 1]
n→ ∞
Let y = | fn ( x ) − f ( x )| = x n − 1 (1 − x ).
For maximum or minimum of y, we have
dy
= ( n − 1) x n − 2 (1 − x ) − x n − 1 = 0
dx
or x n − 2 [( n − 1) (1 − x ) − x ] = 0
n −1
or x = 0 or ⋅
n
2
d y n −1
It can be easily seen that is −ive when x = ⋅
2 n
dx
n−1
∴ M n = max. y = 1 − 
1 1 − n − 1 → 1 × 0 = 0 as n → ∞.
 
 n   n  e
Hence by M n -test the sequence is uniformly convergent on [0, 1].
R-97
∞ ∞
Problem 5: If Σ an converges absolutely prove that Σ an x n converges uniformly on
n=0 n=0
[0, 1].
Solution: Let un ( x ) = an x n. Then
|un ( x )| = |an x n|≤ |an| for all x ∈[0, 1].
∴ |un ( x )|≤ |an| V x ∈ [0, 1] and V n ∈ N.
But Σ M n = Σ |an| converges and hence by Weierstrass’s M-test Σ un ( x ) converges
uniformly on [0, 1].
∞ 1
Problem 6: Show that Σ (0 ≤ x < ∞) is uniformly convergent on [0, ∞[.
n = 1 n2 + x2

Solution: We have, V x ∈ [0, ∞ [ and V n ∈ N


 1  1 1
2 2
 = 2 2
≤ ⋅
n + x  n + x n2
1
But Σ M n = Σ is a series of positive numbers which is convergent.
n2
1
Hence, by Weierstrass’s M-test the series Σ is uniformly convergent on [0, ∞[.
n2 + x2
∞ x
Problem 7: Prove that Σ is uniformly convergent on ]0, k [, k > 0 but not on
n=1 n ( n + 1)
]0, ∞[.
x
Solution: Here un ( x ) = > 0 V x ∈ ]0, k [.
n ( n + 1)
x k k
∴ |un ( x )| = ≤ < , for all x ∈ ]0, k [ and for all n ∈ N.
n ( n + 1) n ( n + 1) n2
k 1
Since Σ M n = Σ =kΣ is a series of positive terms known to be convergent hence
2
n n2
by Weierstrass’s M-test Σ un ( x ) is uniformly convergent on ]0, k [.
Let, if possible, Σ un ( x ) be uniformly convergent on ]0, ∞ [.
1
Take ε = ⋅ There exists m ∈ N such that
4

 x x x 
< 1 for n = p = m
+ + …+
( m + 1) ( m + 2) ( m + 2) ( m + 3) 2m (2m + 1) 4
mx 1
i. e., < V x ∈ ] 0, ∞ [
2m (2m + 1) 4
x 1
i. e., < V x ∈ ]0, ∞ [.
2 (2m + 1) 4
1 1
If we take x = 2m + 1 then < which is a contradiction.
2 4
∴ Σ un ( x ) is not uniformly convergent on ]0, ∞[.
R-98

xn
Problem 8: Prove that if δ is any fixed positive number less than unity, the series Σ is
n+1
uniformly convergent in [ − δ, δ].
1
Solution: Take un ( x ) = x n, vn ( x ) = ⋅
n+1
Since V x in [ − δ, δ], we have |x|≤ δ < 1, therefore
| fn ( x )| = |x + x2 + … + x n|≤ |x| + |x|2 + … + |x|n
δ (1 − δ n ) δ
≤ δ + δ2 + … + δ n = < ⋅
1−δ 1−δ
Also < vn > is a positive monotonic decreasing sequence converging to zero.
Hence, by Dirichlet’s test the given series is uniformly convergent.
1 1
Problem 9: Show that the series sin x + sin 2 x + sin 3 x + … converges uniformly in
2 3
0 < a ≤ x ≤ b < 2 π.
1
Solution: Take un ( x ) = sin nx, vn ( x ) = ⋅
n
Here fn ( x ) = sin x + sin 2 x + sin 3 x + … + sin nx
n −1 
sin  x +
nx n+1 nx
x  sin sin x sin
 2  2
= = 2 2 ⋅
x x
sin sin
2 2
sin n + 1 xsin nx
∴ | fn ( x )| =  2  2 ≤ 1
sin x sin x
 2  2
x
i. e., | fn ( x )|≤ cosec .
 2
x
But cosec is bounded for all values of x in 0 < a ≤ x ≤ b < 2 π. If k be its least upper
2
bound in this interval then | fn ( x )|< k for all values of x in this interval.
1
Also the sequence < > is a positive monotonic decreasing sequence converging to zero.
n
Hence by Dirichlet’s test the given series is uniformly convergent in
0 < a ≤ x ≤ b < 2 π.
Problem 10: Show that the series
x2  2 x2 x2   nx2 ( n − 1) x2 
+  −  + …+  −  + …
1 + x  1 + 2 x 1 + x   1 + nx 1 + ( n − 1) x 
converges uniformly on [0, 1].
x2
Solution: We have u1 ( x ) =
1+ x
R-99

2 x2 x2
u2 ( x ) = −
1 + 2x 1 + x
… … … … …
… … … … …
nx2 ( n − 1) x2
un ( x ) = − ⋅
1 + nx 1 + ( n − 1) x
nx2
Adding, fn ( x ) = ⋅
1 + nx
lim lim nx2
Now f ( x) = n → ∞ fn ( x ) = = x.
n → ∞ 1 + nx

Let ε > 0 be given. Then


 nx2  x
| fn ( x ) − f ( x )|< ε ⇒ − x< ε ⇒ <ε
1 + nx  1 + nx
x 1 1
⇒ 1 + nx > ⇒ n > − ⋅ ...(1)
ε ε x
Evidently the inequality (1) holds for all n ≥ m where m is a positive integer just greater
than 1 / ε. In particular, for x = 0, m = 1. Hence the series is uniformly convergent on
[0, 1].

( − 1)n − 1 n
Problem 11: Consider the series Σ x for uniform convergence in [0, 1].
n
n ( − 1)n − 1
Solution: Take vn ( x ) = x and un ( x ) = ⋅
n
The sequence {vn ( x )} is clearly uniformly bounded and monotonic non- increasing on
[0, 1].
( − 1)n − 1
Also the series Σ is convergent.
n
Hence by Abel’s test the given series is uniformly convergent on [0, 1].

( − 1)n − 1
Problem 12: Consider the series Σ for uniform convergence for all values of x.
( n + x2 )
1
Solution: Let un = ( − 1)n − 1, vn ( x ) = ⋅
n + x2
n
Since fn ( x ) = Σ ur = 0 or 1 according as n is even or odd, fn ( x )is bounded for all n.
r =1

Also vn ( x ) is a positive monotonic decreasing sequence converging to zero for all real
values of x.
Hence by Dirichlet’s test the given series is uniformly convergent for all real values of x.
R-100

Problem 13: Show that the series Σ ( − 1)n − 1 x n
converges uniformly in 0 ≤ x ≤ k < 1.
n=1

Solution: Let un = ( − 1)n − 1, vn ( x ) = x n.


n
Since fn ( x ) = Σ ur = 0 or 1 according as n is even or odd, fn ( x ) is bounded for all n.
r =1
Also {vn ( x )} is a positive monotonic decreasing sequence, converging to zero for all
values of x in 0 ≤ x ≤ k < 1. Hence by Dirichlet’s test, the given series is uniformly
convergent in 0 ≤ x ≤ k < 1.
∞ 1
Problem 14: Show that the series Σ converges uniformly in [1, ∞[.
n = 1 1 + n2 x

1
Solution: Here un ( x ) = ⋅
1 + n2 x
1 1
∴ | un ( x )|≤ V x ∈ [1, ∞ [ < ⋅
2
1+ n n2
∞1
But Σ is known to be convergent.
n2
n=1

Hence by Weierstrass’s M-test the given series is uniformly convergent on [1, ∞ [.


∞ ∞ 1
∴ f ′ ( x) = Σ un ′ ( x ) = − 2 x Σ ⋅
n=1 1 n2 (1 + nx2 )2

Comprehensive Problems 2
Problem 1: Examine for term by term integration the series for which
2
fn ( x ) = nxe − n x
indicating the interval over which your conclusion holds. (Purvanchal 2009)
Solution: As shown in problem 3 of Comprehensive Problems 1, f ( x ) = 0 for all finite
values of x.
Consider the interval 0 ≤ x ≤ 1.
1 1
We have
∫0 f ( x ) dx =
∫0 0. dx = 0

1 1 2
and
∫0 fn ( x ) dx =
∫0 nxe − n x dx
1
=  − e − n x  = [1 − e − n ] → as n → ∞,
1 2 1 1
 2 0 2 2
so that term by term integration is not justified in 0 ≤ x ≤ 1. As a matter of fact, the series
is non-uniformly convergent in 0 ≤ x ≤ 1 and 0 is a point of non-uniform convergence as
we have seen earlier. However term by term integration is justified over the interval [ k, 1]
where 0 < k < 1. For we have
R-101
1 1 2
∫k fn ( x ) dx =
∫k nxe − n x dx

=
1  e − n k2 − e − n  → 0 as n → ∞.
2  
1 1
Thus, in this case
∫k f ( x ) dx = lim
n→ ∞ ∫k fn ( x ) dx.

Problem 2: Show that the series for which


1
(i ) fn ( x ) = , ( ii ) fn ( x ) = nx (1 − x )n
1 + nx
can be integrated term by term in 0 ≤ x ≤ 1, although they are not uniformly convergent in this
interval.
(Purvanchal 2008; Rohilkhand 10)
1
Solution: (i) Here fn ( x ) =
1 + nx
0, when 0 < x ≤ 1
so that f ( x ) = lim fn ( x ) = 
n→ ∞ 1, when x = 0.
1

∫0 f ( x ) dx = 0,
1 1 1
and lim
n→ ∞ ∫0 fn ( x ) dx = lim
n→ ∞ ∫0 1 + nx
dx

= lim
1
log (1 + n)  Form ∞ 
n→ ∞ n  ∞ 
1
1+ n
= lim = 0.
n→ ∞ 1
Hence the given series can be integrated term by term in 0 ≤ x ≤ 1. But we have seen in
Ex. 9 (i) of Text Book that the series is non-uniformly convergent in [0, 1]. In fact zero is a
point of non-uniform convergence of the series.
(ii) Here f ( x ) = 0 V x ∈ [0, 1], as can be easily seen.
[See problem 2 of Comprehensive Problems 1]
1

∫0 f ( x ) dx = 0,

1 1
and
∫0 fn ( x ) dx =
∫0 nx (1 − x )n dx

1
 x (1 − x )n + 1 (1 − x )n + 2  n
= n − −  = → 0 as n → ∞.
 n+1 ( n + 1) ( n + 2) ( n + 1) ( n + 2)
0

Hence the given series can be integrated term by term in 0 ≤ x ≤ 1. But the series is not
uniformly convergent on [0, 1]. [See Problem 2, of Comprehensive Problems 1]
R-102

Problem 3: Examine for the continuity the sum function and for term by term integration the
series whose nth term is
2 x2 2 x2
n2 xe − n − ( n − 1)2 xe −( n − 1)
x having all values in the interval [0, 1].
2 x2 2 x2
Solution: We have un ( x ) = n2 xe − n − ( n − 1)2 xe − ( n − 1) .
− x2
∴ u1 ( x ) = xe − 0,
2 x2 x2
u2 ( x ) = 22 xe − 2 − xe − ,
2 − 32 x2 2 − 22 x2
u3 ( x ) = 3 xe − 2 xe ,
… .... … … … … … … … …
… .... … … … … … … … …
2 x2
Now fn ( x ) = u1 ( x ) + u2 ( x ) + … + un ( x ) = n2 xe − n .

∴ f ( x ) = lim fn ( x ) = 0 for 0 ≤ x ≤ 1.
n→ ∞
Thus the sum function f ( x ) is continuous for all values of x in [0, 1].
1
Now
∫0 f ( x ) dx = 0,
1
fn ( x ) dx =  e − n x  = 1 − e − n2  → 1 as n → ∞,
1 1 2 2 1
and
∫0  2 
0 2   2
so that the series cannot be integrated term by term in [0, 1]. In fact the series is
non-uniformly convergent in [0, 1], zero is a point of non-uniform convergence. For we
have
2 2
| fn ( x ) − f ( x )| = n2 xe − n x ,
1
which → ∞ as n → ∞, when x = ⋅
n
Problem 4: Show that near x = 0 , the series u1 ( x ) + u2 ( x ) + u3 ( x ) + … where
1 /(2 n − 1) 1 /(2 n − 3)
u1 ( x ) = x, un ( x ) = x − x and real values of x are concerned, is discontinuous
and non-uniformly convergent. Can the series be integrated term by term ?
Solution: We have u1 ( x ) = x,
u2 ( x ) = x1 /3 − x,
u3 ( x ) = x1 /5 − x1 /3 ,
……………………
un ( x ) = x1 /(2 n − 1) − x1 /(2 n − 3) .
∴ fn ( x ) = u1 ( x ) + u2 ( x ) + … + un ( x ) = x1 /(2 n − 1) ,
0, for x = 0
so that f ( x ) = lim fn ( x ) = 
n→ ∞ 1 , for all other values of x.
The function f is discontinuous at x = 0 and hence zero is a point of non-uniform
convergence of the series.
R-103

Now if 0 ≤ x < c < ∞, we have


c c
∫0 f ( x ) dx =
∫0 dx = c
c c 2n − 1 2 n /(2 n − 1)
and
∫0 fn ( x ) dx =
∫0 x1 /(2 n − 1) dx =
2n
c → c as n → ∞,

so that in the above interval the series can be integrated term by term although 0 is a
point of non-uniform convergence of the series.
n−1
Problem 5: Examine for term by term integration the series Σ x (1 − 2 x n ) in the interval 0
≤ x ≤ 1.
Solution: Here un ( x ) = x n − 1 − 2 x2 n − 1.
n n 1 − x n 2 x (1 − x2 n )
∴ fn ( x ) = Σ x n − 1 − 2 Σ x2 n − 1 = − ⋅
1 1 1− x (1 − x2 )
1 2x
Then f ( x ) = lim fn ( x ) = −
n→ ∞ 1 − x 1 − x2
 1 , for 0 < x < 1

= 1 + x
 1, for x = 0

and f ( x ) = − ∞ when x = 1 since un ( x ) = − 1.
Hence the series diverges to − ∞ at x = 1.
Consider the interval 0 < x < 1.
1 1 1
We have
∫0 f ( x ) dx =
∫0 1+ x
dx = log 2.

( x n − 1 − 2 x2 n − 1 ) dx = Σ  −
1 1 1 2
But Σ
∫0 un ( x ) dx = Σ
∫0  = 0.
 n 2n 
Hence the term by term integration is not justified in the interval [0, 1].
Problem 6: Show that the series, for which
nx
fn ( x ) = ,0 ≤ x ≤1
1 + n2 x2
cannot be differentiated term by term at x = 0.
Solution: Here
nx
f ( x ) = lim fn ( x ) = lim = 0 for 0 ≤ x ≤ 1.
n→ ∞ n → ∞ 1 + n2 x2

∴ f ′ (0 ) = 0.
fn (0 + h) − fn (0 )
Also fn ′ (0 ) = lim
h→0 h
nh
−0
lim 1 + n2 h2 n
= = lim = n → ∞ as n → ∞.
h→0 h h → 0 1 + n2 h2
R-104

∴ f ′ (0 ) ≠ lim fn ′ (0 ),
n→ ∞
so that the given series cannot be differentiated term by term at x = 0.
∞ sin nx
Problem 7: Show that the function represented by Σ is differentiable for every x and its
n=1 n3
∞ cos nx
derivative is Σ ⋅
1 n2
∞ sin nx sin nx
Solution: Let f ( x) = Σ and un ( x ) = ⋅
1 n3 n3
cos nx ∞ ∞ cos nx
Then un ′ ( x ) = , so that Σ un ′ ( x ) = Σ ⋅
2
n 1 1 n2
 cos nx
Since  ≤ 1 V x and Σ 1 is convergent,
 n  n2
2
n2
hence the series Σ un ′ ( x ) converges uniformly for all values of x, by Weierstrass’s M-test.
It follows that the series Σ un ( x ) can be differentiated term by term.
∞ ∞ cos nx
Consequently f ′ ( x ) = Σ un ′ ( x ) = Σ ⋅
1 1 n2
1
Problem 8: Let un ( x ) = x2 [ x1 /(2 n − 1) − x1 /(2 n − 3) ] sin for x ≠ 0, un (0 ) = 0
x
for any positive integer greater than unity and
u1 ( x ) = x3 sin (1 / x ) for x ≠ 0, u1 (0 ) = 0.

Show that Σ un ( x ) converges for all values of x to f ( x ) , where f ( x ) = x3 sin (1 / x ) for x ≠ 0 and
1

f(0 ) = 0. Also that f ′ is discontinuous at x = 0 , that Σ un ′ ( x ) is not uniformly convergent in any
1
interval including the origin, and that

f ′ ( x ) = Σ un ′ ( x ) for all values of x.
1

Solution: For x ≠ 0, we have

u1 ( x ) = x3 sin (1 / x ) = x2 ( x − 0 ) sin (1 / x ),

u2 ( x ) = x2 [ x1 /3 − x ] sin (1 / x ),

u3 ( x ) = x2 [ x1 /5 − x1 /3 ] sin (1 / x ),

… … … … … … … ………………
un ( x ) = x2 [ x1 /(2 n − 1) − x1 /(2 n − 3) ] sin (1 / x ),

so that fn ( x ) = u1 ( x ) + … + un ( x ) = x2 sin (1 / x ) . x1 /(2 n − 1) .

Then f ( x ) = lim fn ( x ) = x2 sin (1 / x ), when x ≠ 0


n→ ∞
R-105

and f ( x) = 0 when x = 0.
We have, when x ≠ 0
f ′ ( x ) = 2 x sin (1 / x ) − cos (1 / x ).

Also f ′ is discontinuous at x = 0 as lim cos (1 / x ) does not exist.


x→0
Since the sum function f ′ ( x ) of the series Σ un ′ ( x ) is discontinuous at x = 0, the series
Σ un ′ ( x ) is non-uniformly convergent near x = 0.
∞ n
Also Σ un ′ ( x ) = lim Σ un ′ ( x )
1 n→ ∞ 1

d  2 
x sin ⋅ x1 /(2 n − 1) 
1
= lim
n→ ∞  dx  x 

 4n − 1 2 n /(2 n − 1) 1 1
= lim  2n − 1 x sin − x1 /(2 n − 1) cos 
n→ ∞  x x
1 1
= 2 x sin − cos ⋅
x x
∞ ∞
∴ f ′ ( x) = Σ un ′ ( x ) when x ≠ 0. Obviously f ′ (0 ) = Σ un ′ (0 ).
n=1 1

Hence f ′ ( x ) = Σ un ′ ( x ) for all values of x.
n=1

Hints to Objective Type Questions

Multiple Choice Questions


1. (a). See Example 4.
2. (a). See Example 5.
3. (a). See Example 8 (ii).
4. (b). See Example 9 (ii).
5. (b). See Example 14.

Fill in the Blanks


1. uniformly. See Example 6.
2. 0. Refer Problem 1, Comprehensive Problems 1.
3. uniformly. See Problem 6. Comprehensive Problems 1.
4. uniformly. See Problem 9, Comprehensive Problems 1.
5. uniformly. See Problem 14, Comprehensive Problems 1.
6. sufficient, necessary. Refer note 1, after Theorem 1, article 5.
R-106

True or False
1. F. See Definition, article 2.
2. F. See Example 3.
3. T . See Example 7.
4. T . See Problem 3, Comprehensive Problems 1.
5. F. See Problem 7, Comprehensive Problems 1.
6. F. See Example 11.
7. T . See Problem 6, Comprehensive Problems 2.
8. T . See Example 16.
9. F. See Example 12.

❍❍❍
R-107

Chapter-5
Infinite Series

Comprehensive Problems 1
Test for convergence the following series :
1 1 1 1
Problem 1: (i) + + + + .....
2 ⋅3 3 ⋅4 4 ⋅5 5 ⋅6 (Kumaun 2002)
1⋅2 3 ⋅4 5 ⋅6
(ii) + + + .....
32 ⋅ 42 52 ⋅ 62 72 ⋅ 82 (Kumaun 2002; Meerut 12B)
1 3 5
(iii) + + + .....
1⋅2 ⋅3 2 ⋅3 ⋅4 3 ⋅4 ⋅5 (Avadh 2011; Meerut 12)
(1 + a) (1 + b ) (2 + a) (2 + b ) (3 + a) (3 + b )
(iv) + + + .....
1. 2 . 3 2.3.4 3.4.5
Solution: (i) Here
1 1 1
un = = ⋅ ⋅
( n + 1) ( n + 2) n2 [1 + (1 / n)] [1 + (2 / n)]
1
Take vn = ⋅
n2
un 1
Then = ⋅
vn [1 + (1 / n)] [1 + (2 / n)]
lim un 1
∴ = = 1 which is finite and non-zero.
n → ∞ vn (1 + 0 ) (1 + 0 )
∴ by comparison test Σun and Σvn are either both convergent or both divergent.
1 1
But for vn = = , p = 2 > 1.
n2
np
∴ Σvn is convergent.
Hence Σun is also convergent.
(2n − 1) (2n) 1 2 [2 − (1 / n)]
(ii) Here un = = ⋅ ⋅
2 2
(2n + 1) (2n + 2) n [2 + (1 / n)]2 [2 + (2 / n)]2
2

1
Take vn = ⋅
n2
lim un 2 (2 − 0 ) 1
Then = = which is finite and non-zero.
n → ∞ vn (2 + 0 )2 (2 + 0 )2 4
R-108

∴ by comparison test Σun and Σvn are either both convergent or both divergent.
1 1
But for vn = = , p = 2 > 1.
n2 np
∴ Σ vn is convergent and hence Σ un is also convergent.
2n − 1 1 [2 − (1 / n)]
(iii) Here un = = ⋅ ⋅
n ( n + 1) ( n + 2) n2 [1 + (1 / n)] [1 + (2 / n)]
1
Take vn = ⋅
n2
lim un 2−0
Then = = 2 which is finite and non-zero.
n → ∞ vn (1 + 0 ) (1 + 0 )
∴ by comparison test Σun and Σvn are either both convergent or both divergent.
1 1
But for vn = = , p = 2 > 1.
n2 n p
∴ Σvn is convergent and hence Σun is also convergent.

( n + a) ( n + b ) 1 [1 + ( a / n)] [1 + ( b / n)]
(iv) Here un = = ⋅ ⋅
n ( n + 1) ( n + 2) n [1 + (1 / n)] [1 + (2 / n)]
1
Take vn = ⋅
n
lim un
Then = 1 which is finite and non-zero.
n→ ∞ vn
∴ by comparison test Σun and Σvn are either both convergent or both divergent.
1 1
But for vn = = , p = 1.
n np
∴ Σvn is divergent and hence Σun is also divergent.
1 1 1
Problem 2: (i) + + + .....
√1 + √ 2 √ 2 + √ 3 √ 3 + √ 4
√2 − 1 √3 − 1 √4 − 1 √5 − 1
(ii) + + + + .....
33 − 1 43 − 1 53 − 1 63 − 1
1 1 1
(iii) 1 + + + + .....
2 ⋅ 21 /100 3 ⋅ 31 /100 4 ⋅ 41 /100
1 1 1
Solution: Here un = = ⋅ ⋅
√ n + √ ( n + 1) n1 /2 1 + √ [1 + (1 / n)]
1 un 1
Take vn = ⋅ Then = ⋅
n1 /2 v n 1 + √ [1 + (1 / n)]
lim un 1
∴ = which is finite and non-zero.
n → ∞ vn 2
∴ by comparison test Σun and Σvn are either both convergent or both divergent.
1 1 1
But for vn = = , p = < 1.
n1 /2 n p 2
∴ Σvn is divergent and hence Σun is also divergent.
R-109

√ ( n + 1) − 1 n1 /2 √ [1 + (1 / n)] − (1 / √ n)
(ii) Here un = = ⋅
3
( n + 2) − 1 n3 [1 + (2 / n)]3 − (1 / n3 )
1 √ [1 + (1 / n)] − (1 / √ n)
= ⋅ ⋅
5 /2
n [1 + (2 / n)]3 − (1 / n3 )
1
Take vn = ⋅
n5 /2
lim un
Then = 1 which is finite and non-zero.
n→ ∞ vn
∴ by comparison test Σ un and Σvn are either both convergent or both divergent.
1 1 5
But for vn = = , p = > 1.
n5 /2 np 2
∴ Σvn is convergent and hence Σun is also convergent.
1 1
(iii) Here un = = ⋅
n ⋅ n1 /100 n101 /100
1
Now we know that the series Σ is convergent if p > 1.
np
1 1 101
Here = and so p = which is > 1.
n p n101 /100 100
∴ the given series is convergent.
 n 
∑ ∑
1
Problem 3: (i)   . (ii) .
 n5 + 2  (2n − 1) p

 1 sin 1  .
∑ ∑
1
(iii)   (iv) cos .
 √n n n (Kanpur 2007)
 n  1  1 
Solution: (i) Here un =   = ⋅  ⋅
5 
 n + 2  n2
5
1 + (2 / n )
1
Take vn = ⋅ Apply comparison test in the usual manner. The series comes out to
n2
be
convergent.
1 1 1
(ii) Here un = = ⋅ ⋅
(2n − 1)p np [2 − (1 / n)] p
1 un 1
Take vn = ⋅ Then = ⋅
p
n v n [2 − (1 / n)]p
lim un 1
∴ = which is finite and non-zero.
n→ ∞ vn 2p
∴ by comparison test Σun and Σvn are either both convergent or both divergent.
1
But the series Σvn = Σ is convergent if p > 1 and is divergent if p ≤ 1.
np
Hence the given series Σun is convergent if p > 1 and is divergent if p ≤ 1.
R-110

1 1 1 1 1  1  3 1  1
5 
(iii) Here un = sin =  −   +   − ....
√n n √n 
 n 3 ! n  5! n   
1  1  1
2
1  1
4 
= 1 −   +   − ..... .
n3 /2  3 !  n 5 !  n 
1
Take vn = ⋅ Apply comparison test in the usual manner. The series comes out
3 /2
n
to be
convergent.
1
(iv) Here un = cos ⋅
n
lim lim 1
We have un = cos = cos 0 = 1 which is > 0.
n→ ∞ n→ ∞ n
Hence the given series is divergent. [See corollary of theorem 2, article 5 ]

Problem 4: (i) Σ [ √ ( n + 1) − √ n] . (ii) Σ [ √ ( n2 + 1) − n] .

(iii) Σ [ √ ( n3 + 1) − √ n3 ] . (iv) Σ [ √ ( n4 + 1) − n2 ] .
Solution: Here
[ √ ( n + 1) − √ n] [ √ ( n + 1) + √ n]
un = √ ( n + 1) − √ n =
√ ( n + 1) + √ n
( n + 1) − n 1 1 1
= = = ⋅ ⋅
√ ( n + 1) + √ n √ ( n + 1) + √ n n1 /2 √ { 1 + (1 / n)} + 1
1 u 1
Take vn = ⋅ Then n = ⋅
n1 /2 v n √ { 1 + (1 / n)} + 1
lim un 1
∴ = which is finite and non-zero.
n → ∞ vn 2
∴ by comparison test Σun and Σvn are either both convergent or both divergent.
1 1 1
But for vn = = , p = which is < 1.
n1 /2 np 2
∴ Σvn is divergent and hence Σun is also divergent.
( n2 + 1) − n2 1
(ii) Here un = √ ( n2 + 1) − n = =
√ ( n2 + 1) + n √ ( n2 + 1) + n
1 1
= ⋅ ⋅
n √ {1 + (1 / n2 )} + 1
1 u 1
Take vn = ⋅ Then n = ⋅
n vn √ {1 + (1 / n2 )} + 1
lim un 1
∴ = which is finite and non-zero.
n → ∞ vn 2
∴ by comparison test Σun and Σvn are either both convergent or both divergent.
1 1
But for vn = = , p = 1.
n np
∴ Σvn is divergent and hence Σun is also divergent.
R-111

(iii) Here
[ √ ( n3 + 1) − √ n3 ] [ √ ( n3 + 1) + √ n3 ]
un = √ ( n3 + 1) − √ n3 =
√ ( n3 + 1) + √ n3
3 3
( n + 1) − n 1
= =
3 3
√ ( n + 1) + √ n √ ( n + 1) + √ n3
3

1 1
= ⋅ ⋅
n3 /2 √ [1 + (1 / n3 )] + 1
1
Take vn = ⋅ Apply comparison test in the usual manner. The given series comes out
n3 /2
to be convergent.
(iv) Here
( n4 + 1) − n4 1
un = √ ( n4 + 1) − n2 = =
√ ( n4 + 1) + n2 √ ( n4 + 1) + n2
1 1
= ⋅ ⋅
n √ [1 + (1 / n4 )] + 1
2

1
Take vn = ⋅ Apply comparison test. The given series is convergent.
n2

1 2 3


Problem 5: ( i)  3 +  3 +  3  + ....
2  3  4 
1 1
( ii) The series whose nth term is sin ⋅
n n (Kanpur 2005)
Solution: (i) Here
 n  n1 /2 1 1
un =   = = ⋅ ⋅
3 3 /2 3 /2
 ( n + 1)  n [1 + (1 / n)] n [1 + (1 / n)]3 /2
1
Take vn = ⋅
n
lim un
Then = 1 which is finite and non-zero.
n→ ∞ vn
∴ by comparison test Σun and Σvn are either both convergent or both divergent.
1 1
But for vn = = , p = 1.
n np
∴ Σvn is divergent and hence Σun is also divergent.
1 1 1 1  1  
3 5
1 1  1
(ii) Here un = sin =  −   +   − ....
n n n  n 3 !  n  5 !  n 

1  
2 4
1  1 1  1
= ⋅ 1 −   +   − ..... ⋅
2
n  3 !  n  5 !  n  
R-112

1
Take vn = ⋅
n2
2 4
un 1  1 1  1
Then =1−   +   − ...... .
vn 3 !  n 5 !  n
lim un
∴ = 1 which is finite and non-zero.
n→ ∞ vn
∴ by comparison test Σun and Σvn are either both convergent or both divergent.
1 1
But for vn = = , p = 2 > 1.
n2 n p
∴ Σvn is convergent and hence Σun is also convergent.

Comprehensive Problems 2
Test for convergence the following series :


1
Problem 1: (i)
1 + (1 / n)
n (Kanpur 2008; Avadh 12)
n2
1 + 1 
(ii) ∑ 
 n

1 1 1
Solution: Here un = = ⋅ ⋅
n1 + (1 / n) n n1 / n
1 u 1
Take vn = ⋅ Then n = ⋅
n vn n1/ n
lim un lim 1  lim 
∴ = = 1, ∵ n→ ∞ n1 / n = 1
n→ ∞ vn n → ∞ 1/n
n  
which is finite and non-zero.
∴ by comparison test Σun and Σvn are either both convergent or both divergent.
1 1
But for vn = = , p = 1.
n np
∴ Σvn is divergent and hence Σun is also divergent.
n2
un = 1 + 
1
(ii) Here ⋅
 n
1/n
 n2  n
( un )1 / n =  1 +  = 1 +  ⋅
1  1

 n   n
 
lim lim n
∴ u1n / n = 1 + 1  = e which is > 1.
 
n→ ∞ n→ ∞  n
∴ by Cauchy’s root test the given series is divergent.
R-113

n2
 n 
Problem 2: (i) ∑  
 n + 1 (Avadh 2013; Kashi 14)
− n3 /2
1 + 1 
(ii) ∑ 


√ n
n2
 n 
Solution: Here un =   ⋅
 n + 1
n
 n  1
∴ un1 / n =   = ⋅
 n + 1 [1 + (1 / n)]n
lim lim 1 1
∴ un1 / n = = which is < 1.
n→ ∞ n→ ∞ [1 + (1 / n)]n e
∴ by Cauchy’s root test the given series is convergent.
− n3 /2
un = 1 +
1  1
(ii) Here  = ⋅
 √ n 3 /2
[1 + (1 / √ n)]n
1 1 1
∴ un1 / n = = = ⋅
[{1 + (1 / √ n)} n3 /2 1 / n
] [1 + (1 / √ n)] n1 /2 [1 + (1 / √ n)]√ n
lim 1
∴ un1 / n = which is < 1.
n→ ∞ e
∴ by Cauchy’s root test the given series is convergent.
3 4 5
Problem 3: ( i) 2 + x + x2 + x3 + … , where x > 0.
2 3 4

3n + 1 n
( ii) ∑ 4n + 3
x , x > 0.
n=1
n + 1 n−1
Solution: (i) Here un = x .
n
n+2 n
∴ un + 1 = x .
n+1

un 1 n + 1 n + 1 1 n2 + 2n + 1
∴ = ⋅ ⋅ = ⋅ ⋅
un + 1 x n n+2 x n2 + 2n
lim un 1
∴ = ⋅
n→ ∞ un + 1 x

∴ by ratio test, Σun is convergent if 1 / x > 1 i. e., if x < 1


and Σun is divergent if 1 / x < 1 i. e., if x > 1.
For 1 / x = 1 i. e., x = 1, the ratio test fails.
R-114

n+1 1
When x = 1, un = =1+ ⋅
n n
lim lim 1 + 1  = 1 which is > 0.
∴ un =  
n→ ∞ n→ ∞  n
∴ Σ un is divergent for x = 1.
Hence the given series is convergent if x < 1 and is divergent if x ≥ 1.
3n + 1 n 3n + 4 n + 1
(ii) Here un = x . ∴ un + 1 = x .
4n + 3 4n + 7
un 1 3n + 1 4n + 7 1 12n2 + 25n + 7
∴ = ⋅ ⋅ = ⋅ ⋅
un + 1 x 4n + 3 3n + 4 x 12n2 + 25n + 12
lim un 1 12 1
∴ = ⋅ = ⋅
n→ ∞ un + 1 x 12 x
∴ by ratio test, Σ un is convergent if 1 / x > 1 i. e., if x < 1
and is divergent if 1 / x < 1 i. e., if x > 1.
For x = 1, the ratio test fails.
3n + 1 3 + (1 / n)
When x = 1, un = = ⋅
4n + 3 4 + (3 / n)
lim 3
∴ un = which is > 0.
n→ ∞ 4
∴ Σun is divergent for x = 1.
Hence the given series is convergent if x < 1 and is divergent if x ≥ 1.
2 32 2 43 3 ( n + 1)n n
Problem 4: x+ x + x + .... + x + .....
12
23 34
nn + 1
n
( n + 1)n n  n + 1 1 n
Solution: Here un = x =  ⋅ ⋅x .
nn + 1  n  n
n+1 1
⋅ x = 1 +  ⋅
1 1
∴ u1n / n = ⋅ ⋅ x.
n n1 / n  n  n1 / n
lim 1  lim 
∴ u1n / n = (1 + 0 ) ⋅ ⋅ x = x 1/n
∵ n → ∞ n = 1
n→ ∞ 1  
∴ by Cauchy’s root test, Σun is convergent if x < 1 and Σun is divergent if x > 1.
For x = 1, the root test fails and further investigation is required.
n
( n + 1)n
1 
. 1 +  ⋅
1
When x = 1, un = =
n n+1 n  n
lim un lim  n
1 1
Take vn = . Then = 1 +  = e which is finite and non-zero.
n n → ∞ vn n → ∞  n
∴ by comparison test Σ un and Σ vn are either both convergent or both divergent.
1 1
But for vn = = , p = 1.
n np
∴ Σ vn is divergent and so Σ un is also divergent if x = 1.
Hence the given series is convergent if x ≤ 1 and is divergent if x > 1.
R-115

x x2 x3
Problem 5: (i) 1 + + + + … ∞ , x > 0.
2
2 3 43
(ii) x + 2 x2 + 3 x3 + 4 x4 + … .
2 3
1  2
+   x +   x2 +  
3 4
(iii) x3 + .... ∞, x > 0.
2  3  4  5
Solution: (i) Omitting the first term of the series because it will not affect the
convergence or divergence of the series, we have
xn
un = ⋅
( n + 1)n
x
∴ u1n / n = ⋅
n+1
lim lim x
∴ u1n / n = = x. 0 = 0 which is < 1.
n→ ∞ n→ ∞ n + 1
Hence by Cauchy’s root test the given series is convergent.
(ii) Here un = n x n. ∴ un1 / n = n1 / n . x.
lim lim  lim 
∴ un1 / n = ( x . n1 / n ) = x .1 = x. 1/n
 ∵ n → ∞ n = 1
n→ ∞ n→ ∞  
∴ by Cauchy’s root test, Σ un is convergent if x < 1 and is divergent if x > 1.
For x = 1, the root test fails and further investigation is required.
When x = 1, the given series becomes
1 + 2 + 3 + 4 + ....
1
Sn = sum of n terms of the series = n ( n + 1).
2
Since lim Sn = ∞, hence this series is divergent.
Thus the given series is convergent if x < 1 and is divergent if x ≥ 1.
(iii) Omitting the first term of the series because it will not affect the convergence or
n
 n + 1
divergence of the series, we have un =   x n.
 n + 2
Now for complete solution of the problem see example 18.

Comprehensive Problems 3
Test for convergence the following series :
3 x2 4 x3 ( n + 1) x n
Problem 1: ( i) 2 x + + + .... + + ....
8 27 n3
(Kumaun 2003; Kanpur 11; Meerut 12, 12B)
x2 x3 x4
( ii) + + + ..... , x > 0.
2 √1 3 √ 2 4 √ 3
x x2 x3
( iii) 1+ + + + … , x > 0.
1! 2 ! 3 !
R-116

( n + 1) x n
Solution: (i) Here un = ⋅
n3
( n + 2) x n + 1
∴ un + 1 = ⋅
( n + 1)3
un ( n + 1) x n ( n + 1)3 1 ( n + 1)4 1 [1 + (1 / n)]4
∴ = ⋅ = ⋅ = ⋅ ⋅
3 n+1 x n ( n + 2) x [1 + (2 / n)]
3
un + 1 n ( n + 2) x
lim un 1
∴ = ⋅
n→ ∞ un + 1 x
∴ by ratio test, Σ un is convergent if 1 / x > 1 i. e., if x < 1
and Σ un is divergent if 1 / x < 1 i. e., if x > 1. For 1 / x = 1 i. e., x = 1, the ratio test fails.
n+1 1
When x = 1, we have un = = ⋅ [1 + (1 / n)].
n3 n2
1
Take vn = ⋅
n2
lim un lim
Then = [1 + (1 / n)] = 1 which is finite and non-zero.
n → ∞ vn n → ∞
∴ by comparison test Σ un and Σvn are either both convergent or both divergent.
1 1
But for vn = = , p = 2 > 1.
n2 n p
∴ Σvn is convergent and so Σ un is also convergent if x = 1.
Hence the given series is convergent if x ≤ 1
and is divergent if x > 1.
x n+1 x n+2
(ii) Here un = ⋅ ∴ un + 1 = ⋅
( n + 1) √ n ( n + 2) √ ( n + 1)
un 1 ( n + 2) √ ( n + 1) 1 [1 + (2 / n)] √ [1 + (1 / n)]
∴ = ⋅ = ⋅ ⋅
un + 1 x ( n + 1) √ n x 1 + (1 / n)
lim un 1
∴ = ⋅
n→ ∞ un + 1 x
∴ by ratio test, Σ un is convergent if 1 / x > 1 i. e., if x < 1
and Σ un is divergent if 1 / x < 1 i. e., if x > 1.
For 1 / x = 1 i. e., x = 1, the ratio test fails.
1 1 1
When x = 1, we have un = = ⋅ ⋅
( n + 1) √ n n3 /2 1 + (1 / n)
Take vn = 1 / n3 /2 .
lim un lim 1
Then = = 1 which is finite and non-zero.
n → ∞ vn n → ∞ 1 + (1 / n)
∴ by comparison test Σun and Σvn are either both convergent or both divergent.
1 1 3
But for vn = = , p = > 1.
n3 /2 n p 2
∴ Σvn is convergent and so Σun is also convergent if x = 1.
Hence the given series is convergent if x ≤ 1 and is divergent if x > 1.
R-117

xn − 1 xn
(iii) Here un = ⋅ ∴ un + 1 = ⋅
( n − 1) ! n!
un + 1 x n ( n − 1) ! x
∴ = ⋅ = ⋅
un n! xn − 1 n
lim un + 1 lim x
∴ = = x. 0 = 0 which is < 1.
n→ ∞ un n→ ∞ n
∴ by ratio test, the given series is convergent for all x > 0.
lim un
Note: In case we find , then
n → ∞ un + 1

lim un lim n lim un + 1


= = ∞ so that = 0 which is < 1.
n→ ∞ un + 1 n→ ∞ x n → ∞ un

∴ by ratio test, the given series is convergent for all x > 0.

12 ⋅ 22 22 ⋅ 32 32 ⋅ 42
Problem 2: + + + .... .
1! 2! 3!
n2.( n + 1)2 ( n + 1)2 ( n + 2)2
Solution: Here un = ⋅ ∴ un + 1 = ⋅
n! ( n + 1) !
un n2 .( n + 1)2 ( n + 1) ! n2 ( n + 1)
∴ = ⋅ = ⋅
2 2
un + 1 n! ( n + 1) ( n + 2) ( n + 2)2
lim un lim n [1 + (1 / n)]
∴ = =∞
n→ ∞ un + 1 n → ∞ [1 + (2 / n)]2
lim un + 1
or = 0 which is < 1.
n→ ∞ un
Hence by ratio test, the given series is convergent.
x x2 x3 xn
Problem 3: ( i) 1+ + + + …+ + .....
2 5 10 n2 + 1

2 6 14 3 2n − 2 n − 1
( ii) 1+ x + x2 + x + …+ x + .....
5 9 17 2n + 1
Solution: (i) Omitting the first term of the series as it will not affect the convergence
or divergence of the series, we have
xn
un = ⋅
2
n +1
xn + 1 un 1 ( n + 1)2 + 1
∴ un + 1 = ⋅ ∴ = ⋅ ⋅
2
( n + 1) + 1 un + 1 x n2 + 1
lim un lim 1 ( n + 1)2 + 1 1
∴ = ⋅ = ⋅
n→ ∞ un + 1 n→ ∞ x n2 + 1 x
R-118

∴ by ratio test, Σ un is convergent if1 / x > 1i. e., if x < 1and Σ un is divergent if1 / x < 1i. e., if
x > 1.
For 1 / x = 1 i. e., x = 1 the ratio test fails and further investigation is required.
1 1 1
When x = 1, we have un = = . ⋅
n + 1 n 1 + (1 / n2 )
2 2

1
Take vn = and apply comparison test. In this case the series comes out to be
n2
convergent.
Hence the given series is convergent if x ≤ 1 and is divergent if x > 1.
2n − 2 n − 1 2n + 1 − 2 n
(ii) Here un = x . ∴ un + 1 = x .
2n + 1 2n + 1 + 1
[Note that if we put n = 1in this value of un, we get u1 = 0 while the first term of the given
series is 1. We do not bother about it because it will not affect the convergence or
divergence of the series.]
un 1 2n − 2 2 n + 1 + 1
Now = ⋅
un + 1 x 2n + 1 2n + 1 − 2

1 2n [1 − (1 / 2n − 1)] 2n + 1 [1 + (1 / 2 n + 1)]
= ⋅
x 2n [1 + (1 / 2 n )] 2 n + 1 [1 − (1 / 2n )]

1 [1 − (1 / 2n − 1)] [1 + (1 / 2 n + 1)]
= ⋅ ⋅
x [1 + (1 / 2n )] [1 − (1 / 2n )]
lim un 1  lim 1 
∴ = ⋅ ∵ n → ∞ = 0, etc.
n→ ∞ un + 1 x  2n 
∴ by ratio test, Σ un is convergent if 1 / x > 1 i. e., if x < 1
and Σ un is divergent if 1 / x < 1 i. e., if x > 1.
For 1 / x = 1 i. e., x = 1, the ratio test fails and further investigation is required.
2n − 2 1 − (1 / 2n − 1)
When x = 1, un = = ⋅
2n + 1 1 + (1 / 2n )
lim
∴ un = 1 which is > 0.
n→ ∞
∴ Σ un is divergent for x = 1.
Hence the given series is convergent if x < 1 and is divergent if x ≥ 1.
x x2 x3
Problem 4: + + + ..... .
1⋅2 2 ⋅3 3 ⋅4 (Gorakhpur 2012)
xn xn + 1
Solution: Here un = ⋅ ∴ un + 1 = ⋅
n ( n + 1) ( n + 1) ( n + 2)
un 1 ( n + 1) ( n + 2) 1 n + 2
∴ = ⋅ = ⋅ ⋅
un + 1 x n ( n + 1) x n
lim un 1
∴ = ⋅
n→ ∞ un + 1 x
R-119

∴ by ratio test, Σ un is convergent if 1 / x > 1 i. e., if x < 1


and Σ un is divergent if 1 / x < 1 i. e., if x > 1.
For 1 / x = 1 i. e., x = 1, the ratio test fails and further investigation is required.
1 1 1
When x = 1, we have un = = ⋅ ⋅
n ( n + 1) n 1 + (1 / n)
2

Take vn = 1 / n2 and apply comparison test. In this case the series comes out to be
convergent.
Hence the given series is convergent if x ≤ 1 and is divergent if x > 1.

1 1. 2 1. 2 . 3 1. 2 . 3 . 4
Problem 5: + + + + ….
3 3.5 3.5.7 3.5.7.9
1 . 2 . 3 . 4 . ..... . n
Solution: Here un = ⋅
3 . 5 . 7 . 9 ..... (2n + 1) [Note that the nth term of the
A.P. 3, 5, 7, 9, ... is 3 + ( n − 1) 2]

1 . 2 . 3 . 4 .... n ( n + 1)
∴ un + 1 = ⋅
3 . 5 . 7 . 9 . .... (2n + 1) (2n + 3)
un 2n + 3
∴ = ⋅
un + 1 n+1
lim un lim 2n + 3
∴ = = 2 which is > 1.
n → ∞ un + 1 n → ∞ n + 1
Hence by ratio test the given series is convergent.
n ! 3n n3
Problem 6: (i) ∑ n
n
(ii) ∑ ( n − 1) !
n ! 3n ( n + 1) !
Solution: (i) Here un = ⋅ ∴ un + 1 = ⋅ 3n + 1.
nn ( n + 1)n + 1
( n + 1)n + 1
n
n ! 3n 1 ( n + 1)n 1 
= 1 +  ⋅
un 1
∴ = ⋅ =
un + 1 nn ( n + 1) ! . 3n + 1 3 nn 3  n
lim un e
∴ = which is < 1 because 2 < e < 3.
n→ ∞ un + 1 3
Hence by ratio test the given series is divergent.
n3 ( n + 1)3
(ii) Here un = ⋅ ∴ un + 1 = ⋅
( n − 1) ! n!
un n3 n! n . n3 1
∴ = ⋅ = = n. ⋅
un + 1 ( n − 1) ! ( n + 1)3
( n + 1)3 [1 + (1 / n)]3
lim un lim un + 1
∴ =∞ or = 0 which is < 1.
n→ ∞ un + 1 n→ ∞ un
Hence by ratio test the given series is convergent.
R-120

 3 n − 1  xn 
Problem 7: (i) ∑  n 
 2 
(ii) ∑  .
 x + n
3n − 1 3 ( n + 1) − 1 3n + 2
Solution: (i) Here un = ⋅ ∴ un + 1 = = ⋅
2n 2n + 1 2n + 1
un 3n − 1 2 n + 1 3n − 1 [3 − (1 / n)]
∴ = ⋅ = 2. = 2. ⋅
un + 1 2 n 3n + 2 3n + 2 [3 + (2 / n)]
lim un
∴ = 2 which is > 1.
n→ ∞ un + 1
Hence by ratio test the given series is convergent.
xn xn + 1
(ii) Here un = ⋅ ∴ un + 1 = ⋅
x+ n x+ n+1
un xn x + n + 1 1 n [1 + {(1 + x ) / n}]
∴ = ⋅ = ⋅
un + 1 x+ n xn + 1 x n [1 + ( x / n)]
1 [1 + {(1 + x ) / n}]
= ⋅ ⋅
x [1 + ( x / n)]
lim un 1
∴ = ⋅
n→ ∞ un + 1 x
∴ by ratio test, Σ un is convergent if 1 / x > 1 i. e., if x < 1
and Σ un is divergent if 1 / x < 1 i. e., if x > 1.
For 1 / x = 1 i. e., x = 1, the ratio test fails and further investigation is required.
1 1 1
When x = 1, we have un = = ⋅ ⋅
n + 1 n [1 + (1 / n)]
1 lim un lim 1
Take vn = ⋅ Then = = 1 which is finite and non-zero.
n n → ∞ vn n → ∞ [1 + (1 / n)]
∴ by comparison test Σ un and Σvn are either both convergent or both divergent.
1 1
But for vn = = , p = 1.
n np
∴ Σvn is divergent and so Σ un is also divergent when x = 1.
Hence the given series is convergent if x < 1 and is divergent if x ≥ 1.
xn nn
Problem 8: (i) ∑ a+ n
(ii) ∑ n!

xn xn + 1
Solution: Here un = ⋅ ∴ un + 1 = ⋅
a + √n a + √ ( n + 1)
un 1 a + √ ( n + 1) 1 √ {1 + (1 / n)} + ( a / √ n)
∴ = ⋅ = ⋅ ⋅
un + 1 x a + √n x [1 + ( a / √ n)]
un 1
∴ lim = ⋅
un + 1 x
R-121

∴ by ratio test, Σ un is convergent if 1 / x > 1 i. e., if x < 1


1
and Σ un is divergent if < 1 i. e., if x > 1.
x
For 1 / x = 1 i. e., x = 1, the ratio test fails.
1 1 1
When x = 1, we have un = = ⋅ ⋅
a + √ n n1 /2 1 + ( a / √ n)
1
Take vn = ⋅
n1 /2
lim un lim 1
Then = =1
n → ∞ vn n → ∞ 1 + ( a / √ n)
which is finite and non-zero.
∴ by comparison test Σ un and Σvn are either both convergent or both divergent.
1 1 1
But for vn = = , p = which is < 1.
1 /2 p 2
n n
∴ Σvn is divergent and so Σ un is also divergent for x = 1.
Hence the given series is convergent if x < 1 and is divergent if x ≥ 1.
nn
(ii) Here un = ⋅
n!
( n + 1)n + 1
∴ un + 1 = ⋅
( n + 1) !
un nn ( n + 1) ! nn 1
∴ = ⋅ = = ⋅
+
un + 1 n ! ( n + 1) n 1 ( n + 1)n [1 + (1 / n)]n
lim un 1
∴ = which is < 1.
n → ∞ un + 1 e
∴ by ratio test the given series is divergent.
Problem 9: Test for convergence the series whose n th term is
n ( n + 1)2
2
2n − 1
( i) ( ii)
n! 3n + 1
√n n3 − 1 n
( iii) x n, ( x > 0 ) ( iv ) x , (x > 0)
√ ( n2 + 1) n3 + 1
 n −1  n 3n − 2 n − 1
(v )   x , ( x > 0 ) ( vi) x , (x > 0)
 n3 + 1 3n + 1
xn
(vii) , x > 0, a > 0.
xn + a n
Solution: (i) Proceed as in problem 2.
2n − 1
(ii) Here un = ⋅
3n + 1
2n + 1 − 1
∴ un + 1 = ⋅
3n + 1 + 1
R-122

un 2n − 1 3n + 1 + 1 2n [1 − (1 / 2n )] 3n + 1 [1 + (1 / 3n + 1 )]
∴ = ⋅ = ⋅
un + 1 3n + 1 2n + 1 − 1 3n [1 + (1 / 3n )] 2n + 1 [1 − (1 / 2n + 1 )]

3 [1 − (1 / 2n )] [1 + (1 / 3n + 1 )]
= ⋅ ⋅ ⋅
2 [1 + (1 / 3n )] [1 − (1 / 2n + 1 )]
lim un 3
∴ = which is > 1.
n→ ∞ un + 1 2
Hence by ratio test the given series is convergent.
√n
(iii) Here un = x n.
√ ( n2 + 1)
√ ( n + 1)
∴ un + 1 = xn + 1 ⋅
√ {( n + 1)2 + 1}
un 1 √n √ {( n + 1)2 + 1}
∴ = ⋅ ⋅
un + 1 2
x √ ( n + 1) √ ( n + 1)

1 √ [ {1 + (1 / n)}2 + (1 / n2 )]
= ⋅ ⋅
x √ [1 + (1 / n2 )] . √ [1 + (1 / n)]
lim un 1
∴ = ⋅
n→ ∞ un + 1 x
∴ by ratio test, Σ un is convergent if 1 / x > 1 i. e., if x < 1
and Σ un is divergent if 1 / x < 1 i. e., if x > 1.
For 1 / x = 1 i. e., x = 1, the ratio test fails.
√n 1 1
When x = 1, we have un = = ⋅ ⋅
√ ( n2 + 1) n1 /2 √ {1 + (1 / n2 )}
1
Take vn = and apply comparison test.
1 /2
n
The series comes out to be divergent when x = 1.
Hence the given series is convergent if x < 1 and is divergent if x ≥ 1.
n3 − 1 n ( n + 1)3 − 1 n + 1
(iv) Here un = x . ∴ un + 1 = x .
n3 + 1 ( n + 1)3 + 1
un 1 n3 − 1 ( n + 1)3 + 1
∴ = ⋅ ⋅ ⋅
un + 1 x n3 + 1 ( n + 1)3 − 1
lim un 1
∴ = ⋅
n→ ∞ un + 1 x
∴ by ratio test, Σ un is convergent if 1 / x > 1 i. e., if x < 1
and Σ un is divergent if 1 / x < 1 i. e., if x > 1.
For 1 / x = 1 i. e., x = 1, the ratio test fails.
n3 − 1 1 − (1 / n3 )
When x = 1, un = = ⋅
3
n +1 1 + (1 / n3 )
R-123

lim
∴ un = 1 which is > 0.
n→ ∞
∴ Σ un is divergent when x = 1.
Hence the given series is convergent if x < 1 and is divergent if x ≥ 1.
 n −1  n
(v) Here un =   x .
 n3 + 1

 n  n+1
∴ un + 1 =   x .
3
 ( n + 1) + 1
un 1 √ ( n − 1) √ [( n + 1)3 + 1] 1 √ ( n − 1) √ [( n + 1)3 + 1]
∴ = ⋅ ⋅ = ⋅ ⋅ ⋅
un + 1 x √ ( n3 + 1) √n x √n √ ( n3 + 1)
lim un 1
∴ = ⋅
n→ ∞ un + 1 x
∴ by ratio test, Σ un is convergent if 1 / x > 1 i. e., if x < 1
and is divergent if 1 / x < 1 i. e., if x > 1.
For 1 / x = 1 i. e., x = 1, the ratio test fails.
 n − 1  1  1 − (1 / n) 
When x = 1, un =   = ⋅  ⋅
 n3 + 1 n 1 + (1 / n3 )
Take vn = 1 / n and apply comparison test.
The series comes out to be divergent when x = 1.
Hence the given series is convergent if x < 1 and is divergent if x ≥ 1.
3n − 2 n − 1 1 − (2 / 3n ) n − 1
(vi) Here un = x = x .
3n + 1 1 + (1 / 3n )
1 − (2 / 3n + 1 ) n
∴ un + 1 = x .
1 + (1 / 3n + 1 )
un 1 1 − (2 / 3n ) 1 + (1 / 3n + 1 )
∴ = ⋅ ⋅ ⋅
un + 1 x 1 + (1 / 3n ) 1 − (2 / 3n + 1 )
lim un 1
∴ = ⋅
n→ ∞ un + 1 x
∴ by ratio test Σ un is convergent if 1 / x > 1 i. e., if x < 1
and Σ un is divergent if 1 / x < 1 i. e., if x > 1.
For 1 / x = 1 i. e., x = 1, the ratio test fails.
1 − (2 / 3n )
When x = 1, un = ⋅
1 + (1 / 3n )
lim
∴ un = 1 which is > 0.
n→ ∞
∴ Σun is divergent for x = 1.
Hence the given series is convergent if x < 1 and is divergent if x ≥ 1.
R-124

an 1
(vii) Case 1: Let x = a. Then un = = ⋅
a + an
n 2
n
∴ Sn = u1 + u2 + … + un = ⋅
2
lim lim n
∴ Sn = = ∞ and so Σun is divergent when x = a.
n→ ∞ n→ ∞ 2
xn 1
Case 2: Let x > a. Then un = = ⋅
x + an
n
1 + ( a / x )n
lim 1  a lim  a = 0
n
∴ un = ∵ x > a ⇒ < 1 ⇒   
n→ ∞ 1+ 0  x 
n→ ∞ x  
= 1 which is > 0.
∴ the given series is divergent for x > a.
xn
Case 3: Let x < a. Then un = ⋅
x + an
n

xn + 1
∴ un + 1 = ⋅
x n + 1 + an + 1

un xn x n + 1 + an + 1
∴ = ⋅
un + 1 n
x + a n
xn + 1

1 an + 1 [1 + ( x / a)n + 1 ] a [1 + ( x / a)n + 1]
= ⋅ = ⋅ ⋅
x an [1 + ( x / a)n ] x [1 + ( x / a)n ]

lim un a  x lim  x  n 
∴ = , ∵ x < a ⇒ < 1 ⇒   = 0
n→ ∞ un + 1 x  a n → ∞  a  
which is > 1 because x < a.
∴ by ratio test the given series is convergent for x < a.
Hence the given series is convergent if x < a and is divergent if x ≥ a.
Problem 10: Examine the convergence of the series
1 x x2 x3
+ + + +…
1p 3p 5p 7p
xn − 1 xn
Solution: Here un = ⋅ ∴ un + 1 = ⋅
(2n − 1)p (2n + 1)p
p p
un 1 (2n + 1) 1 [2 + (1 / n)]
∴ = ⋅ = ⋅ ⋅
un + 1 x (2n − 1)p x [2 − (1 / n)]p

lim un 1 2p 1
∴ = ⋅ = ⋅
n→ ∞ un + 1 x 2p x
R-125

∴ by ratio test, Σun is convergent if 1 / x > 1 i. e., if x < 1


and Σun is divergent if 1 / x < 1 i. e., if x > 1.
For 1 / x = 1 i. e., x = 1, the ratio test fails.
1
When x = 1, we have un = ⋅
(2n − 1)p
In this case apply comparison test and proceed as in problem 3 part (ii) of
Comprehensive Problems 1.
Ans. The given series is convergent if x < 1 and is divergent if x > 1.
In case x = 1, the given series is convergent if p > 1 and is divergent if p ≤ 1.

Comprehensive Problems 4
Test for convergence the following series :

1 1⋅3 2 1. 3 . 5 3
Problem 1: 1 + x+ x + x +…
2 2 ⋅4 2.4.6
Solution: Omitting the first term of the series because it will not affect the
convergence or divergence of the series, we have
1 . 3 . 5 . ..... (2n − 1) n
un = x .
2 . 4 . 6 . ..... (2n)
1 . 3 . 5 . .... (2 n − 1) (2n + 1) n + 1
∴ un + 1 = x .
2 . 4 . 6 . .... (2n) (2n + 2)
un 1 2n + 2 lim un 1
∴ = ⋅ ⋅ ∴ = ⋅
un + 1 x 2n + 1 n→ ∞ un + 1 x

∴ by ratio test, Σun is convergent if 1 / x > 1 i. e., if x < 1 and Σun is divergent if
1 / x < 1 i. e., if x > 1.
For 1 / x = 1 i. e., x = 1, the ratio test fails and further investigation is required. In this
case we apply Raabe’s test.
u 2n+2
When x = 1, we have n = ⋅
un + 1 2n + 1

 u   2n + 2  n
∴ n  n − 1 = n  − 1 = ⋅
u   2n + 1  2n + 1
 n+1 

lim  u  lim n 1
∴ n  n − 1 = = which is < 1.
n→ ∞ u  n→ ∞ 2n + 1 2
 n+1 
So by Raabe’s test Σ un is divergent for x = 1.
Hence the given series is convergent if x < 1 and is divergent if x ≥ 1.
R-126

1 x2 1 . 3 . 5 x4 1 . 3 . 5 . 7 . 9 x6
Problem 2: 1 + ⋅ + ⋅ + ⋅ + .....
2 4 2.4.6 8 2 . 4 . 6 . 8 . 10 12
Solution: Omitting the first term of the series because it will not affect the
convergence or divergence of the series, we have
1 . 3 . 5 . 7 . 9 . .... (4n − 3) x2n
un = ⋅ ⋅
2 . 4 . 6 . 8 . 10 . .... (4n − 2) 4n
[ Note that the nth term of the
A.P. 1, 5, 9, 13, ... is 1 + ( n − 1) 4 i. e. 4n − 3 ]

1 . 3 . 5 . .... (4n − 3) (4n − 1) (4n + 1) x2 n + 2


∴ un + 1 = ⋅ ⋅
2 . 4 . 6 . .... (4n − 2) (4n) (4n + 2) 4n + 4

un 1 4n (4n + 2) 4n + 4 1 16n2 + 24n + 8


∴ = ⋅ ⋅ = ⋅ ⋅
un + 1 x2 (4n − 1) (4n + 1) 4n x2 16n2 − 1

lim un 1 16 1
∴ = ⋅ = ⋅
n→ ∞ un + 1 x2 16 x2

∴ by ratio test, Σ un is convergent if 1 / x2 > 1 i. e., if x2 < 1


and Σ un is divergent if 1 / x2 < 1 i. e., if x2 > 1.
For 1 / x2 = 1 i. e., x2 = 1, the ratio test fails and in this case we apply Raabe’s test.
u 16n2 + 24n + 8
When x2 = 1, we have n = ⋅
un + 1 16n2 − 1
 u   16n2 + 24n + 8  24n2 + 9n
∴ n  n − 1 = n  − 1 = ⋅
u   2
16n − 1  16n2 − 1
 n+1 

lim  u  lim 24n2 + 9n 24 3


∴ n  n − 1 = = = which is > 1.
n→ ∞ u  n→ ∞ 16n2 − 1 16 2
 n+1 
∴ by Raabe’s test Σ un is convergent for x2 = 1.
Hence the given series is convergent if x2 ≤ 1 and is divergent if x2 > 1.
22 4 22 . 42 22 . 42 . 62
Problem 3: x2 + x + x6 + x8 + ....
3.4 3.4.5.6 3.4.5.6.7.8 (Kanpur 2014)
Solution: Omitting the first term of the series because it will not affect the
convergence or divergence of the series, we have
22 . 42 . 62 . ...... (2 n)2
un = x2 n + 2 .
3 . 4 . 5 . 6 . 7 . 8 .... (2n + 1) (2n + 2)
22 . 42 . 62 … (2n)2 (2n + 2)2
∴ un + 1 = x2 n + 4
3 . 4 . 5 . 6 . 7 . 8. …
(2n + 1) (2n + 2) (2n + 3) (2n + 4)
R-127

un 1 (2n + 3) (2n + 4) 1 4n2 + 14n + 12


∴ = ⋅ = ⋅ ⋅
un + 1 x2 (2n + 2)2 x2 4n2 + 8n + 4
lim un 1
∴ = ⋅
n→ ∞ un + 1 x2

∴ by ratio test, Σun is convergent if 1 / x2 > 1 i. e., if x2 < 1


and Σun is divergent if 1 / x2 < 1 i. e., if x2 > 1.
For 1 / x2 = 1 i. e., x2 = 1, the ratio test fails and we apply Raabe’s test in this case.
u 4n2 + 14n + 12
When x2 = 1, we have n = ⋅
un + 1 4n2 + 8n + 4
 u   4n2 + 14n + 12  6n2 + 8n
∴ n  n − 1 = n  − 1 = ⋅
u   4n2 + 8n + 4  4n2 + 8n + 4
 n+1 
lim  u  lim 6n2 + 8n 6 3
∴ n  n − 1 = = = which is > 1.
n→ ∞ u  n→ ∞ 2
4n + 8n + 4 4 2
 n+1 
2
∴ by Raabe’s test, Σ un is convergent for x = 1. Hence the given series is convergent if
x2 ≤ 1 and is divergent if x2 > 1.
x 1 x3 1 . 3 x5 1 . 3 . 5 x7
Problem 4: (i) 1+ + ⋅ + + +…
1 2 3 2.4 5 2.4.6 7 (Kashi 2014)
3 5 7
x 1 x 1. 3 x 1. 3 . 5 x
(ii) + ⋅ + + + ….
1 2 3 2.4 5 2.4.6 7 (Gorakhpur 2012, 14)
2 3 4
x 1 x 1.3 x 1.3.5 x
(iii) + . + ⋅ + ⋅ + … , x > 0.
1 2 3 2.4 5 2.4.6 7
Solution: (i) Omitting the first two terms of the series because it will not affect the
convergence or divergence of the series, we have
1 . 3 . 5 . .... (2n − 1) x2n + 1
un = ⋅ ⋅
2 . 4 . 6 . .... (2n) 2n + 1
1 . 3 . 5 ..... (2 n − 1) (2n + 1) x2 n + 3
∴ un + 1 = ⋅ ⋅
2 . 4 . 6 ..... (2n) (2n + 2) 2n + 3
un 1 2n + 2 2n + 3 1 4n2 + 10 n + 6
∴ = ⋅ ⋅ = ⋅ ⋅
un + 1 x2 2n + 1 2n + 1 x2 4n2 + 4n + 1
lim un 1
∴ = ⋅
n→ ∞ un + 1 x2

∴ by ratio test, Σ un is convergent if 1 / x2 > 1 i. e., if x2 < 1


and is divergent if 1 / x2 < 1 i. e., if x2 > 1.
For 1 / x2 = 1 i. e., x2 = 1, the ratio test fails and we apply Raabe’s test in this case.
R-128

u 4n2 + 10 n + 6
When x2 = 1, we have n = ⋅
un + 1 4n2 + 4n + 1
 u   4n2 + 10 n + 6  6n2 + 5n
∴ n  n − 1 = n  − 1 = ⋅
u  2
 4n + 4n + 1  4n2 + 4n + 1
 n+1 
lim  u  6 3
∴ n  n − 1 = = which is > 1 and so
n→ ∞ u  4 2
 n+1 
by Raabe’s test Σ un is convergent for x2 = 1.
Hence the given series is convergent if x2 ≤ 1 and is divergent if x2 > 1.
(ii) Omitting the first term of the series because it will not affect the convergence
or divergence of the series, we have
1 . 3 . 5 . ..... (2n − 1) x2n + 1
un = ⋅
2 . 4 . 6 . ..... (2n) 2n + 1
Now proceed as in part (i).
Ans. Convergent if x2 ≤ 1 and divergent if x2 > 1.
(iii) Omitting the first term of the series because it will not affect the convergence or
divergence of the series, we have
. . . .... (2n − 1) x n + 1
135
un = ⋅ ⋅
. . .... (2n) 2n + 1
246
1 . 3 . 5 . ..... (2 n − 1) (2n + 1) x n + 2
∴ un + 1 = ⋅ ⋅
2 . 4 . 6 . ..... (2n) (2n + 2) 2n + 3
un 1 2n + 2 2n + 3
∴ = ⋅ ⋅ ⋅
un + 1 x 2n + 1 2n + 1
Now proceed as in part (i).
Ans. Convergent if x ≤ 1 and divergent if x > 1.
a a ( a + 2) a ( a + 2) ( a + 4)
Problem 5: + x+ x2 + .....
a + 3 ( a + 3) ( a + 5) ( a + 3) ( a + 5) ( a + 7)
a ( a + 2) ( a + 4) … ( a + 2n − 2)
Solution: Here un = x n − 1.
( a + 3) ( a + 5) ( a + 7) … ( a + 2n + 1)
[Note that the nth term of the A.P. 0, 2, 4, 6, ... is 0 + ( n − 1) 2 i. e. 2n − 2 and the nth
term of the A.P. 3, 5, 7, ... is 3 + ( n − 1) 2 i. e., 2n + 1]
a ( a + 2) … ( a + 2 n − 2) ( a + 2n)
∴ un + 1 = x n.
( a + 3) ( a + 5) … ( a + 2 n + 1) ( a + 2n + 3)
un 1 a + 2n + 3 1 2n + a + 3
∴ = ⋅ = ⋅
un + 1 x a + 2n x 2n + a
lim un 1
∴ = ⋅
n→ ∞ un + 1 x
∴ by ratio test, Σ un is convergent if 1 / x > 1 i. e., if x < 1 and is divergent if
1 / x < 1 i. e., if x > 1.
For 1 / x = 1 i. e., x = 1, the ratio test fails and we apply Raabe’s test in this case.
R-129

un 2n + a + 3
When x = 1, we have = ⋅
un + 1 2n + a
 u   2n + a + 3  3n
∴ n  n − 1 = n  − 1 = ⋅
u   2 n + a  2 n + a
 n+1 

lim  u  lim 3n 3
∴ n  n − 1 = = which is > 1
n→ ∞ u  n →∞ 2n+ a 2
 n+1 
and so by Raabe’s test Σ un is convergent for x = 1.
Hence the given series is convergent if x ≤ 1 and is divergent if x > 1.
4 . 7 . … (3n + 1) n
Problem 6: Σ x .
1. 2 . … . n
4 . 7 . … (3n + 1) n
Solution: Here un = x .
1. 2 . … . n
4 . 7 . … (3n + 1) (3n + 4) n + 1
∴ un + 1 = x .
1. 2 . … . n ( n + 1)
un 1 n+1 lim un 1
∴ = ⋅ ⋅ ∴ = ⋅
un + 1 x 3n + 4 n → ∞ un + 1 3 x
1 1 1
∴ by ratio test, Σ un is convergent if > 1 i. e., if x < and Σ un is divergent if <1
3x 3 3x
1
i. e., if x > ⋅
3
1 1
For = 1 i. e., x = , the ratio test fails and further investigation is required. In this
3x 3
case we apply Raabe’s test.
1 u 3 ( n + 1) 3n + 3
When x = , we have n = = ⋅
3 un + 1 3n + 4 3n + 4
 u   3n + 3  −n −1
∴ n  n − 1 = n  − 1 = = ⋅
u   3 n + 4  3 n + 4 3 + (4 / n)
 n+1 
lim  u  lim −1 1
∴ n  n − 1 = = − which is < 1.

n → ∞  un + 1  n → ∞ 3 + (4 / n) 3

So by Raabe’s test Σ un is divergent for x = 1 / 3. Hence the given series is convergent if
x < 1 / 3 and is divergent if x ≥ 1 / 3.
Problem 7: Apply Cauchy’s condensation test to discuss the convergence of the series


1

n=2 ( n log n) ( log log n)p
1
Solution: Here f ( n) = ⋅
( n log n) (log log n)p
an 1
∴ an f ( an ) = = ⋅
n p
n n
( a log a ) (log log a ) ( n log a) [log ( n log a)]p
R-130

Now a is a positive integer greater than 1 and may be so chosen that log e a > 1 so
that n log a > n.
1
Then an f ( an ) < ⋅
( n log a) (log n)p


1 1
But the series is convergent when p > 1.
log a n (log n)p
Therefore Σ an f ( an ) is also convergent and consequently the given series is
convergent when p > 1.
Let p ≤ 1.
Now if we take a = 2, then log e a < 1 so that n log e a < n.
1
∴ an f ( an ) > ⋅
( n log a) (log n)p

∑ ∑ an
1 1
But the series is divergent when p ≤ 1and therefore f ( an ) is
log a n (log n)p
also divergent. It follows by Cauchy’s condensation test that the given series is divergent
when p ≤ 1.

Comprehensive Problems 5
Test for convergence the following series :

Problem 1: x2 ( log 2)q + x3 ( log 3)q + x4 ( log 4)q + …


Solution: Adding one more term, say 1, to the beginning of the series because it will
not affect the convergence or divergence of the series, we have
un = x n (log n)q . ∴ un + 1 = x n + 1 [log ( n + 1)]q .
un 1 (log n)q 1 (log n)q
∴ = ⋅ =
un + 1 x [log ( n + 1)]q x [log {n (1 + 1 / n)}]q

1 (log n)q 1 (log n)q


= ⋅ = ⋅
x [log n + log (1 + 1 / n)]q x  1 
q
(log n)q 1 + log (1 + 1 / n)
 log n 

1 1
= ⋅ ⋅
x  q
1 
1 + log n log (1 + 1 / n)
 
lim un 1  lim 1 lim 1 
∴ = ⋅ ∵ n → ∞ log n = 0 and n → ∞ n = 0 
n → ∞ un + 1 x  
∴ by ratio test, Σ un is convergent if 1 / x > 1 i. e., if x < 1
and is divergent if 1 / x < 1 i. e., if x > 1.
For 1 / x = 1 i. e., x = 1, the ratio test fails and we apply log test in this case.
R-131

un 1
When x = 1, we have = ⋅
un + 1  q
1 
1 + log n log (1 + 1 / n)
 
un  1 1 1 1 
∴ n log = − qn log 1 +  − 2 + 3 − …  ,
un + 1  log n  n 2 n 3 n 

expanding log {1 + (1 / n)} in ascending powers of 1 / n


  1 1 
= − qn log 1 +  − + … 
  n log n 2n2 log n  

 
2 
1 1 1 1 1 
= − qn   − + … −  − + … + … 
  n log n 2n2 log n  2  n log n 2 n2 log n  
 
applying the expansion for log (1 + x )
q
=− + all such terms that tend to zero as n → ∞.
log n
lim un
∴ n log = 0 which is < 1 and so by log test Σun is divergent for x = 1.
n→ ∞ un + 1
Hence the given series is convergent if x < 1 and is divergent if x ≥ 1.
1 1 1
Problem 2: + +… +…
( log 2)p ( log 3)p ( log n)p
Solution: Adding one more term, say 1, to the beginning of the series because it will
not affect the convergence or divergence of the series, we have
1 1
un = ⋅ ∴ un + 1 = ⋅
(log n)p [log ( n + 1)]p
un [log ( n + 1)]p [log n + log {1 + (1 / n)}]p
∴ = =
p
un + 1 (log n) (log n) p
p
 1 
(log n)p 1 + log (1 + 1 / n)
 log n 
=
(log n)p
p
 1 
= 1 + log {1 + (1 / n)} ⋅
 log n 
lim un
∴ = 1 and so the ratio test fails to decide the convergence or
n→ ∞ un + 1
divergence of the series and we proceed to apply log test.
un  1 1 1 1  
We have n log = np log 1 +  − + − … 
un + 1  log n  n 2n2 3n3  
  1 1  
= np log 1 +  − + … 
2
  n log n 2n log n  
R-132

 
2 
1 1 1 1 1 

= np  − + … −  − + …  + …
  n log n 2n2 log n  2  n log n 2n2 log n  
 
p
= + all such terms that tend to zero.
log n
lim un
∴ for all p, n log = 0 which is < 1.
n→ ∞ un + 1
Hence by log test, the given series is divergent for all p.
22 x2 33 x3 44 x4 55 x5
Problem 3: (i) x+ + + + +…
2! 3! 4! 5! (Gorakhpur 2013)
2 2 3 3 4 4 5 5
2 x 3 x 4 x 5 x
(ii) 1+ + + + + ....
2! 3! 4! 5!
nn x n ( n + 1)n + 1 x n + 1
Solution: (i) Here un = ⋅ ∴ un + 1 = ⋅
n! ( n + 1) !
un 1 nn ( n + 1) ! 1 nn ( n + 1)
∴ = ⋅ ⋅ = ⋅
un + 1 x n ! ( n + 1)n + 1 x ( n + 1)n + 1
1 nn 1 nn 1 1
⋅ = = ⋅ = ⋅ ⋅
x ( n + 1) n n
x n {1 + (1 / n)}n x {1 + (1 / n)}n
lim un 1  lim 1 + 1  = e 
n
∴ = ⋅ ∵   
n → ∞ un + 1 ex  n→ ∞  n 
1 1
∴ by ratio test, Σ un is convergent if > 1 i. e., if x <
ex e
1 1
and Σ un is divergent if < 1 i. e., if x > ⋅
ex e
1 1
For = 1 i. e., x = , the ratio test fails and we proceed to apply log test in this case.
ex e
1 un 1
When x = , we have = e. ⋅
e un + 1 {1 + (1 / n)}n
un
∴ n log = n [1 − n log {1 + (1 / n)}]
un + 1
 1 1 1  
= n 1 − n  − + − .... 
  n 2n2 3n3  
1 1  1 1
=n − + … = − + ….
 2n 3n2  2 3n
lim un 1
∴ n log = which is < 1.
n→ ∞ un + 1 2
∴ by log test, Σ un is divergent for x = 1 / e.
Hence the given series is convergent if x < 1 / e and is divergent if x ≥ 1 / e.
nn x n
(ii) Here un = ⋅
n!
R-133

[Note that if we put n = 1 in this value of un, we get u1 = x while the first term of the given
series is 1. But it will not affect the convergence or divergence of the series and so we do
not bother about it.]
Now proceed as in part (i).
1! 2! 2 3! 3 4! 4
Problem 4: 1 + x+ x + x + x +…
2 32 43 54 (Kanpur 2014)
( n − 1) ! n − 1 n! n
Solution: Here un = x . ∴ un + 1 = x .
nn − 1 ( n + 1)n
n
1 ( n − 1) ! ( n + 1)n 1 ( n + 1)n 1 
= 1 +  ⋅
un 1
∴ = ⋅ ⋅ = ⋅
un + 1 x nn − 1 n! x nn x n
lim un e  lim 1 + 1 
n 
∴ = ⋅ ∵   = e
n→ ∞ un + 1 x  n →∞  n 
∴ by ratio test, Σun is convergent if e / x > 1 i. e., if x < e
and Σun is divergent if e / x < 1 i. e., if x > e.
For e / x = 1 i. e., x = e, the ratio test fails and we proceed to apply logarithmic test in
this case.
n
When x = e, we have n = 1 +  .
u 1 1
un + 1 e  n
 1 
= n − 1 + n log 1 + 
un
∴ n log [∵ log e = 1]
un + 1   n  
 1 1 1 1  
= n − 1 + n  − + − + ..... 
  n 2n 2
3n3
4n4  
1 1 1
=− + − + .... .
2 3n 4n2
lim un 1
∴ n log = − which is < 1.
n→ ∞ un + 1 2
∴ by log test, Σun is divergent for x = e.
Hence the given series is convergent if x < e and is divergent if x ≥ e.

Comprehensive Problems 6
Test for convergence the following series :
22 22 . 42 22 . 42 . 62
Problem 1: 1 + + + + ……
32 32 . 52 32 . 52 . 72 (Kashi 2013; Meerut 13)
2 2 2 2
2 .4 .6 ……(2n − 2)
Solution: Here, un =
32 .52 .72 ……(2n − 1)2
22 .42 .62 ……(2n − 2)2 (2n)2
and un + 1 = ⋅
3 .5 .7 ……(2n − 1)2 (2 n + 1)2
2 2 2
R-134

un (2n + 1)2
∴ lim = lim = 1 so that ratio test fails.
un + 1 (2n)2
Now we apply Raabe’s test. We have
 u  (2n + 1)2  n (4n + 1)
lim n  n − 1 = lim n  − 1 = lim = 1.
u   4n 2  4n2
 n+1 
Hence Raabe’s test also fails. Now we apply Gauss test. We can write
un 4n2 + 4n + 1 1 1 a b
= =1+ + =1+ + n ⋅
un + 1 4n 2 n 4n 2 n np
1
Here a = 1, bn = , p = 2 > 1. Consequently the series is divergent by Gauss’s test.
4
12 12 . 32 12 . 32 . 52
Problem 2: + + +…
2 2 2
2 2 .4 22 . 42 . 62 (Kumaun 2003)
12 . 32 . 52 . .... (2n − 1)2
Solution: Here un = ⋅
22 . 42 . 62 . ....(2n)2
12 . 32 . 52 . … (2n − 1)2 (2n + 1)2
∴ un + 1 = ⋅
22 . 42 . 62 . .... (2n)2 (2n + 2)2
un (2 n + 2)2 4n2 + 8n + 4
∴ = = ⋅
un + 1 (2n + 1)2 4n2 + 4n + 1
lim un 4
∴ = = 1 and so the ratio test fails and further investigation is
n→ ∞ un + 1 4
required. Now we shall apply Raabe’s test.
 u   4n2 + 8n + 4  4n2 + 3n
We have n  n − 1 = n  − 1 = ⋅
u  2 2
 n+1   4n + 4n + 1  4n + 4n + 1
lim  u  4
∴ n  n − 1 = = 1 and so the Raabe’s test fails and further
n→ ∞   4
 un + 1 
investigation is required. Now we shall apply De Morgan’s and Bertrand’s test.
We have
  u    4n2 + 3n 
 n  n − 1 − 1 log n =  − 1 log n
u  2
  n + 1    4n + 4n + 1 
− n −1 log n − 1 − (1 / n)
= log n = ⋅ ⋅
4n2 + 4n + 1 n 4 + (4 / n) + (1 / n2 )
   u   lim  log n − 1 − (1 / n) 
(log n) n  n − 1 − 1 =
lim
∴  ⋅
n→ ∞  u  → ∞ 2 
   n + 1   n  n 4 + (4 / n) + (1 / n )
−1
= 0. = 0 which is < 1.
4
∴ by De Morgan’s and Bertrand’s test the given series is divergent.
R-135

a a ( a + 1) a ( a + 1) ( a + 2)
Problem 3: + + + .....
b b ( b + 1) b ( b + 1) ( b + 2)
a ( a + 1) ( a + 2) … ( a + n − 1)
Solution: Here un = ⋅
b ( b + 1) ( b + 2) … ( b + n − 1)
a ( a + 1) ( a + 2) … ( a + n − 1) ( a + n)
∴ un + 1 = ⋅
b ( b + 1) ( b + 2) … ( b + n − 1) ( b + n)
un b+ n n+ b
∴ = = ⋅
un + 1 a + n n + a
lim un
∴ = 1and so the ratio test fails to decide the convergence or divergence of
n→ ∞ un + 1
the series and we proceed to apply Raabe’s test.
 u  n+ b  n ( b − a)
We have n  n − 1 = n  − 1 = ⋅
u  n+ a  n+ a
 n+1 
lim  u 
∴ n  n − 1 = b − a.
n→ ∞ u 
 n+1 
∴ by Raabe’s test, Σun is convergent if b − a > 1
and Σun is divergent if b − a < 1.
For b − a = 1, Raabe’s test fails and we proceed to apply De Morgan’s and Bertrand’s
test.
 u  n
When b − a = 1, we have n  n − 1 = ⋅
u  n+ a
 n+1 
  u    n  a
∴  n  n − 1 − 1 log n =  − 1 log n = − log n
u   +  +
  n + 1   n a n a

log n −a
= ⋅ ⋅
n 1 + ( a / n)
   u    log n −a 
( log n) n  n − 1 − 1 =
lim lim
∴  n ⋅ 1 + ( a / n)
n→ ∞    →∞

u
  n + 1   n  
−a  lim log n 
= 0. = 0 which is < 1. ∵ n → ∞ = 0
1+ 0  n 
∴ the given series is divergent for b − a = 1.
Hence the given series is convergent if b − a > 1 and is divergent if b − a ≤ 1.

a (1 − a) (1 + a) a (1 − a) (2 − a)
Problem 4: 1 + +
12 12 . 22
(2 + a) (1 + a) a (1 − a) (2 − a) (3 − a)
+ + .....
12 . 22 . 32
R-136

Solution: Omitting the first term of the series because it will not affect the
convergence or divergence of the series, we have
( n − 1 + a) … (2 + a) (1 + a) a (1 − a) (2 − a) (3 − a)
…( n − a)
un = ⋅
12 . 22 . 32 . .... n2
( n + a) ( n − 1 + a) … (2 + a) (1 + a) a (1 − a) (2 − a)
… ( n − a) ( n + 1 − a)
∴ un + 1 = ⋅
12 . 22 . 32 . … n2 . ( n + 1)2

un ( n + 1)2 n2 + 2n + 1
∴ = = ⋅
un + 1 ( n + a) ( n + 1 − a) n2 + n + a − a2
lim un
∴ =1
n→ ∞ un + 1
and so the ratio test fails and we proceed to apply Raabe’s test.
 u   n2 + 2n + 1  n2 + n (1 − a + a2 )
We have n  n − 1 = n  − 1 = ⋅
u   n2 + n + a − a2  n2 + n + a − a2
 n+1 
lim  u 
∴ n  n − 1 = 1 and so the Raabe’s test also fails and we
n→ ∞ u 
 n+1 
proceed to apply De Morgan’s and Bertrand’s test.
  u    n2 + n (1 − a + a2 ) 
We have  n  n − 1 − 1 log n =  − 1 log n
u  2 2
  n + 1    n + n + a − a 
n ( − a + a2 ) − a + a2 log n a2 − a + {( a2 − a) / n}
= log n = ⋅ ⋅
n2 + n + a − a2 n 1 + (1 / n) + {( a − a2 ) / n2 }
   u   a2 − a + 0
(log n) n  n − 1 − 1  = 0.
lim
∴ = 0 which is < 1.
n→ ∞    1+ 0 + 0
 
u
   n + 1 
Hence by De Morgan’s and Bertrand’s test, the given series is divergent.
α α (α + 1)2 α (α + 1)2 (α + 2)2
Problem 5: 1 + x+ x2 + x3 + …
1. β 1 . 2 β ( β + 1) 1 . 2 . 3 β ( β + 1) ( β + 2)
Solution: Omitting the first term of the series because it will not affect the
convergence or divergence of the series, we have
α (α + 1)2 (α + 2)2 … (α + n − 1)2
un = x n.
1 . 2 . 3 . … n β ( β + 1) ( β + 2) … ( β + n − 1)
α . α2
[Note that if we put n = 1 in this value of un, we get un = x which is different
1. β
α
from the new first term x but it does not affect the convergence or divergence
1. β
of the series.]
R-137

α (α + 1)2 … (α + n − 1)2 (α + n)2


∴ un + 1 = x n + 1.
1 . 2 . 3 . … n ( n + 1) β (β + 1) … (β + n − 1) (β + n)
un 1 ( n + 1) (β + n) 1 n2 + n (β + 1) + β
∴ = ⋅ = ⋅ ⋅
un + 1 x (α + n)2 x n2 + 2 α n + α2
lim un 1
∴ = ⋅
n→ ∞ un + 1 x
∴ by ratio test, Σ un is convergent if 1 / x > 1 i. e., if x < 1
and Σ un is divergent if 1 / x < 1 i. e., if x > 1.
For 1 / x = 1 i. e., x = 1, the ratio test fails and we proceed to apply Raabe’s test.
u n2 + n (β + 1) + β
When x = 1, n = ⋅
un + 1 n2 + 2αn + α2
 u   n2 + n (β + 1) + β 
∴ n  n − 1 = n  − 1
u  2 2
 n+1   n + 2αn + α 
n2 ( β + 1 − 2α ) + ( β − α2 ) n
= ⋅
n2 + 2αn + α2
lim  u 
∴ n  n − 1 = β + 1 − 2α.
n→ ∞  
 un + 1 
∴ for x = 1, by Raabe’s test Σ un is convergent if β + 1 − 2α > 1 i. e., if β > 2α
Σ un is divergent if β + 1 − 2α < 1 i. e., if β < 2α
and the Raabe’s test fails if β + 1 − 2α = 1 i. e., if β = 2α and we apply De Morgan’s and
Bertrand’s test in this case.
 u  n2 + ( β − α2 ) n
For x = 1, when β = 2α, we have n  n − 1 = ⋅
u  n2 + 2αn + α2
 n+1 
  u    n2 + (β − α2 ) n 
∴  n  n − 1 − 1 log n =  − 1 log n
u  2 2
  n + 1    n + 2αn + α 
( β − α2 − 2α ) n − α2 log n ( β − α2 − 2α ) − (α2 / n)
= log n = ⋅ ⋅
2 2
n + 2α n + α n 1 + (2α / n) + (α2 / n2 )
   u 
lim
(log n) n  n − 1
   lim log n 
∴ − 1 = 0, ∵ n → ∞ = 0
n→ ∞  u 
   n + 1    n 
which is < 1.
∴ for x = 1, when β = 2α, Σun is divergent by De Morgan’s and Bertrand’s test.
Hence the given series is convergent if x < 1, divergent if x > 1 and when x = 1 then
convergent if β > 2α and divergent if β ≤ 2α.
2 /3 2 /3 2 /3
 1   1. 3   1. 3 . 5 
Problem 6:   +   +   +…
2 . 4  2 . 4 . 6  2 . 4 . 6 . 8 
2 /3
 1 . 3 . 5 . … (2n − 1) 
Solution: Here un =  
2 . 4 . 6 . 8 . … (2n + 2)
R-138

2 /3
 1 . 3 . 5 . ..... (2n − 1) (2n + 1) 
and un + 1 =   .
2 . 4 . 6 . ..... (2 n + 2) (2n + 4)
2 /3 2 /3
un  2n + 4   1 + (2 / n) 
∴ =  =  .
un + 1  2n + 1  1 + (1 / 2n)
lim un
∴ = 1 and so the ratio test fails and we proceed to apply log test.
n→ ∞ un + 1
2 /3
un  1 + (2 / n) 
We have n log = n log  
un + 1 1 + (1 / 2n)
2n   2  1 
=  log 1 +  − log 1 + 
3  n  2 n 
2n  2 1  2    1 1 1 2 1 1 3  
2 3
  −   +   − …  −  −   +   − … 
1 2
=
3   n 2 n   
3 n  
 2n 2 2n  
3 2n   

2n  3 1 15 1 63 1 
= ⋅ − ⋅ + ⋅ + …
3  2 n 8 n2 24 n3 
5 1 21 1
=1− ⋅ + ⋅ +…
4 n 12 n2
lim un
∴ n log = 1 and so the log test also fails to decide the convergence or
n→ ∞ un + 1
divergence of the given series and further investigation is required.
  un  
(log n)   n log
lim
Now − 1
n→ ∞   
   un + 1  
lim   5 21  
= (log n)  1 − + + … − 1
n → ∞    4 n 12n 2  
lim   5 21 
= (log n)  − + + … 
n → ∞   4n 12n2 
lim   log n   5 21  
=   − + + …
n → ∞   n   4 12n  

= 0 .  −  = 0 which is < 1.
5
 4
∴ the given series Σ un is divergent.

Problem 7: x + x1+ 1 /2 + x1 + 1 /2 + 1 /3 + x1 + 1 /2 + 1 /3 + 1 /4 + …
Solution: Here un = x1 + 1 /2 + 1 /3 + … + 1 / n,
and un + 1 = x1 + 1 /2 + 1 /3 + … + 1 / n + 1 /(n + 1) .
un 1
∴ = ⋅
un + 1 x1 /(n + 1)
lim un 1
∴ = = 1 and so the ratio test fails and we proceed to apply log test.
n→ ∞ un + 1 x0
R-139

1 /(n + 1)
= n log  
un 1 1
We have n log = n log
un + 1 x1 /(n + 1)  x
n 1 1 1
= log = log ⋅
n+1 x 1 + (1 / n) x
lim un 1
∴ n log = log ⋅
n→ ∞ un + 1 x
∴ by log test, Σun is convergent if log (1 / x ) > 1 i. e., if 1 / x > e i. e., if x < 1 / e,
Σun is divergent if log (1 / x ) < 1 i. e., if 1 / x < e i. e., if x > 1 / e
and the log test fails if log (1 / x ) = 1 i. e., if 1 / x = e i. e., if x = 1 / e.
When x = 1 / e, we have
un n
n log = ⋅
un + 1 n + 1
  un  
(log n)   n log
lim
∴ − 1
n→ ∞   

un + 1 
  
lim   n  lim   −1  
= (log n)  − 1  = (log n)  
n→ ∞  n+1   n→ ∞   n + 1 
lim   log n   − 1 
=   ⋅ 
n→ ∞  n  1 + (1 / n)
 lim log n 
= 0 . ( − 1) = 0, ∵ n → ∞ = 0
 n 
which is < 1.
∴ the given series Σ un is divergent if x = 1 / e.
Hence the given series is convergent if x < 1 / e and is divergent if x ≥ 1 / e.

Comprehensive Problems 7
Problem 1: Examine the convergence of the series
1 1 1 1
− + − + ….
1. 2 3 . 4 5 . 6 7 . 8

Solution: The given series is an alternating series


u1 − u2 + u3 − … + ( − 1)n − 1 un + … , ( un > 0 for all n).
1
Here un = > 0 for all n.
(2n − 1) 2n
1 1
We have un + 1 − un = −
(2 n + 1) (2n + 2) (2n − 1) 2n

4n2 − 2n − 4n2 − 6n − 2
=
(2n + 1) (2n + 2) (2n − 1) (2n)
R-140

− 8n − 2
=
(2n + 1) (2n + 2) (2n − 1) (2n)
< 0 for all n.
Thus un + 1 < un for all n.
1
Also lim un = lim =0⋅
(2n − 1) 2n
Hence by alternating series test the given series is convergent.
Problem 2: Show that the series
log 2 log 3 log 4
− + − …converges.
22 32 42
Solution: The given series is an alternating series
u1 − u2 + u3 − … + ( − 1)n − 1 un + … , ( un > 0 for all n) ⋅
log ( n + 1)
Here un = ⋅
( n + 1)2
log n log ( n + 1)
Since → 0 as n → ∞, therefore lim un = lim
n ( n + 1)2
 log ( n + 1) 1 
= lim  ⋅  =0⋅
 n+1 n + 1
Now we shall show that for the given series un + 1 ≤ un for all n.
log x
Let f ( x) = ⋅
x2
x2 .(1 / x ) − 2 x log x 1 − 2 log x
Then f ′ ( x) = = < 0,
4
x x3
whenever x > e1 /2 .
∴ The function f ( x ) is monotonically decreasing for all x > e1 /2 ⋅
But 2 < e < 3 and so 1 < e1 /2 < 2.

Therefore f ( n + 2) ≤ f ( n + 1) for all n i.e., un + 1 ≤ un for all n.


Hence the given series satisfies all the conditions of the alternating series test and is
therefore convergent.
1 + 1 + 1 + … + 1 

n  2 3 n ⋅
Problem 3: Examine the convergence of the series Σ ( − 1)  
n=1 n
 
 
Solution: The given series is an alternating series Σ ( − 1)n un , un > 0 for all n.
1 1 1
1+ + + …+
Here un = 2 3 n > 0 for all n.
n
R-141

1 1 1 1 1 1
1+ + + …+ 1+ + + …+
2 3 ( n + 1) 2 3 n
We have un + 1 − un = −
n+1 n
 1 
− ( n + 1) 1 + + + … + 
1 1 1 1 1
n 1 + + + … +
 2 3 n + 1  2 3 n 
=
n ( n + 1)
− 1+ + + …+ 

n 1 1 1
n + 1  2 3 n 
=
n ( n + 1)
 1 1   1 1 1
 n + 1 + n + 1 + … + upto n terms − 1 + 2 + 3 + … + n 
=    
n (n + 1)
 1   1 1  1 1
 n + 1 − 1 +  n + 1 − 2  + … +  n + 1 − n 
     
=
n ( n + 1)
< 0 for all n.
∴ un + 1 < un for all n.
Also lim (1 / n) = 0. So by Cauchy’s first theorem on limits
1 1 1
+ + …+
lim 1 2 n = 0 i.e., lim u = 0.
n
n
Hence by alternating series test, the given series is convergent.
∞  1 ( − 1)n + 1 
Problem 4: Test the convergence of the series ∑  +
 n √ n 
.
n=1
Solution: Let Sn denote the sum of the first n terms of the given series.
n 1 ( − 1)r + 1  n n
( − 1)r + 1
∑ ∑ ∑
1
Then Sn =  + = + ⋅
 r √ r  r √r
r =1 r =1 r =1

( − 1)n + 1
Now the series ∑ √n
is an alternating series and is convergent by Leibnitz’s
n=1
test.
 n 
lim ( − 1)r + 1 

n→ ∞

 r =1
∑ √r 
is a definite real number, say S.
 

lim  n 
∑ ∑
1  1
Also the series is a divergent series. ∴ = ∞.
n n→ ∞  r
n=1  r =1 
 
( − 1)r + 1 
n n
lim lim
∑ ∑
 1
Now Sn = + = ∞ + S = ∞.
n→ ∞ n→ ∞  r √r 
 r = 1 r = 1 
Hence the given series is a divergent series and it diverges to ∞.
R-142

Comprehensive Problems 8
Problem 1: Test for convergence the following series :
1 1 1 1 1 1 1
( i) 1− + − +… ( ii) − + − +…
1! 2 ! 3 ! log 2 log 3 log 4 log 5

( − 1)n − 1

1 1 1
( iii) 1− + − +… ( iv )
22 32 42 √ [ n ( n + 1) ( n + 2)]
n=1
( − 1)n − 1 ( n + 1)
(v ) ∑ 2n
1 1 1 1
( vi) − + − + … , x > 0, a > 0
x x + a x + 2a x + 3a

log   − log   + log   − log   + ....


2 3 4 5
( vii)
 1  2  3  4

( viii) log   − log   + log   − log   + .... .


1 2 3 4
 2  3  4  5
Solution: (i) The given series is an alternating series
u1 − u2 + u3 − u4 + … + ( − 1)n − 1 un + … , ( un > 0 for all n).
1
Here un = > 0 for all n.
( n − 1) !
1 1
We have un + 1 − un = −
n ! ( n − 1) !
1− n
= ≤ 0, V n ∈ N.
n!
Thus un + 1 ≤ un for all n.
lim lim 1
Also un = = 0.
n→ ∞ n→ ∞ ( n − 1) !
Hence by Leibnitz’s test for alternating series, the given series is convergent.
(ii) The given series is an alternating series
u1 − u2 + u3 − u4 + … + ( − 1)n − 1 un + … , ( un > 0 for all n).
1
Here un = which is > 0 for all n.
log ( n + 1)
1 1
We have un + 1 − un = −
log ( n + 2) log ( n + 1)
log ( n + 1) − log ( n + 2)
= < 0 for all n.
log ( n + 1) log ( n + 2)
Thus un + 1 < un for all n i. e., each term is numerically less than the preceding term.
lim lim 1
Also un = = 0.
n→ ∞ n → ∞ log ( n + 1)
Hence by Leibnitz’s test for alternating series, the given series is convergent.
R-143

(iii) The given series is an alternating series ∑ (− 1)n − 1 un ,
n=1
1
where un = > 0, for all n ∈ N.
n2
1 1
We have un + 1 − un = −
( n + 1)2 n2
n2 − ( n + 1)2 − 2n − 1
= = < 0 for all n ∈ N.
n2 ( n + 1)2 n2 ( n + 1)2
Thus un + 1 < un for all n i. e., each term is numerically less than the preceding term.
lim 1lim
Also un = = 0.
n→ ∞ n→ ∞
n2
Hence by Leibnitz’s test for alternating series, the given series is convergent.

(iv) The given series is an alternating series ∑ ( − 1)n − 1 un ,
n=1
1
where un = > 0, for all n ∈ N.
∈ [ n ( n + 1) ( n + 2)]
We have
1 1
un + 1 − un = −
√ [( n + 1) ( n + 2) ( n + 3)] √ [ n ( n + 1) ( n + 2)]
√ n − √ ( n + 3)
= < 0 for all n.
√ [ n ( n + 1) ( n + 2) ( n + 3)]
Thus un + 1 < un for all n.
lim lim 1
Also un = = 0.
n→ ∞ n→ ∞ √ [ n ( n + 1) ( n + 2)]
Hence by Leibnitz’s test for alternating series, the given series is convergent.

(v) The given series is an alternating series ∑ ( − 1)n − 1 un ,
n=1
n+1 1 1
where un = = + > 0, for all n.
2n 2 2n
n+2 n+1
We have un + 1 − un = −
2 ( n + 1) 2n
n2 + 2n − n2 − 2n − 1
=
2n ( n + 1)
−1
= < 0 for all n.
2n ( n + 1)
Thus un + 1 < un for all n i. e., each term is numerically less than the preceding term.

lim lim 1 1  1
But un =  +  = i. e, ≠ 0.
n→ ∞ n→ ∞  2 2 n 2
Hence the given series is an oscillatory series.
R-144


(vi) The given series is an alternating series ∑ ( − 1)n − 1 un ,
n=1
1
where un = > 0, for all n. [Note that x > 0, a > 0]
x + ( n − 1) a
1 1
We have un + 1 − un = −
x + na x + ( n − 1) a
x + ( n − 1) a − x − na
=
( x + na) [ x + ( n − 1) a]
−a
= < 0, for all n.
( x + na) [ x + ( n − 1) a]
Thus un + 1 < un for all n i. e., each term is numerically less than the preceding term.
lim lim 1
Also un = = 0.
n→ ∞ n→ ∞ x + ( n − 1) a
Hence by Leibnitz’s test for alternating series, the given series is convergent.

(vii) The given series is an alternating series ∑ ( − 1)n − 1 un ,
n=1
n + 1
un = log   1
where  = log 1 +  > 0, for all n.
 n   n
 n + 2  n + 1
We have un + 1 − un = log   − log  
 n + 1  n 
n+ 2 n   n2 + 2n 
= log  ⋅  = log  2  < 0 for all n
 n + 1 n + 1  n + 2 n + 1
n2 + 2n n2 + 2n
because 0 < < 1 ⇒ log < 0.
n2 + 2n + 1 n2 + 2n + 1
[Note that log e x > 0 if x > 1 and log e x < 0 if 0 < x < 1]
Thus un + 1 < un for all n.
lim lim
log 1 +  = log (1 + 0 ) = log 1 = 0.
1
Also un =
n→ ∞ n→ ∞  n
Hence by Leibnitz’s test for alternating series, the given series is convergent.
(viii) The given series is
log   − log   + log   − log   + …
1 2 3 4
 2  3  4  5

= − log   + log   − log   + log   − …


2 3 4 5
 1  2  3  4
[∵ − log x = log x − 1 = log (1 / x )]

n + 1
( − 1)n un , where un = log   1
= ∑  n 
 = log 1 +  > 0, for all n.
 n
n=1
R-145

 n + 2  n + 1
We have un + 1 − un = log   − log  
 n + 1  n 
n+ 2 n 
= log  ⋅ 
 n + 1 n + 1
 n2 + 2n 
= log   < 0 for all n.
 n2 + 2n + 1
lim lim
log 1 + 
1
Also un =
n→ ∞ n→ ∞  n
= log (1 + 0 ) = log 1 = 0.
Hence by Leibnitz’s test for alternating series, the given series is convergent.

Problem 2: Test the absolute convergence or conditional convergence of the following series :
1 1 1 1
( i) 1 − x + x2 − x3 + … (0 < x < 1) ( ii) − + − + …( p> 0)
1p 2p 3p 4p
1 1 1 x2 x4 x6
( iii) 1− + − +… ( iv ) 1− + − +…
2 √2 3 √3 4 √4 2! 4! 6!
1 1 1 1 1 1 1 1
(v ) − + − +… ( vi) 1− + − + −…
1. 2 3 . 4 5 . 6 7 . 8 2 4 8 16
(Meerut 2012B)
sin n α cos n α
( vii) ∑ (− 1)n 3
n
,α ∈R ( viii) ∑ ( − 1)n
n√ n
, α ∈ R.

( − 1)n − 1 n2
( ix ) ∑ 2n + 3
( x) ∑ ( − 1)n − 1
( n + 1) !
∞ ∞ 1 1 
∑ ∑
1
( xi) ( − 1)n sin ( xii) ( − 1)n − 1  + ⋅
2
n=1
n
n=1  n ( n + 1)2 

Solution: (i) Let the given series be ∑ un ,where un = ( − 1)n − 1 x n − 1.
n=1

Then the series Σ|un| = 1 + x + x2 + x3 + … which is a convergent series because it is an


infinite geometric series whose common ratio x is such that 0 < x < 1.
Since the series Σ|un| is convergent, therefore the given series is absolutely
convergent.
(ii) The given series is an alternating series


1
( − 1)n − 1 un , where un = > 0, for all n. Also it is given that p > 0.
n=1 np
1 1
Since for all n, therefore un + 1 < un for all n.
<
( n + 1)p
np
lim lim 1
Also un = = 0. [∵ p > 0 ]
n→ ∞ n → ∞ np
Hence by Leibnitz’s test for alternating series, the given series is convergent for all p > 0.
R-146


Now the series ∑ |( − 1)n − 1 un| is the series
n=1
1 1 1 1
+ + +
+ .... ...(1)
p p p
1 2 3 4p
The series (1) is convergent if p > 1 and is divergent if p ≤ 1.
Hence the given series is absolutely convergent if p > 1and semi-convergent if 0 < p ≤ 1.

(iii) The given series is an alternating series ∑ ( − 1)n − 1 un,
n=1
1
where un = > 0, for all n.
n√ n
1 1
We have un + 1 < un for all n because < ⋅
( n + 1) √ ( n + 1) n √ n
Thus each term of the given series is numerically less than the preceding term.
lim lim 1
Also un = = 0.
n→ ∞ n→ ∞ n √n
Hence by Leibnitz’s test for alternating series, the given series is convergent.

∑ |(− 1)n − 1 un| = ∑ ∑ ∑
1 1 1
Now the series = = ,
n√ n n3 /2 np
n=1
where p = 3 / 2 > 1.
∴ the series ∑ |( − 1)n − 1 un| is convergent and hence the given series is absolutely
convergent.
(iv) We shall test the given series for absolute convergence.
Let the given series be Σun.
For x = 0, the series Σ | un| obviously converges.
Let x ≠ 0.
x2n − 2
We have un = ( − 1)n − 1
(2n − 2) !
x2 n
and un + 1 = ( − 1)n ⋅
(2n) !
|un|  un   x2 n − 2 (2n) ! 1
Now =  = ⋅ = (2n − 1) 2n.
|un + 1| un + 1 (2n − 2) ! x2n  x2
lim |un| lim (2n − 1) 2n
∴ = =∞
n → ∞ |un + 1| n → ∞ x2
lim |un + 1|
so that = 0 which is < 1.
n → ∞ |un|
∴ by D’Alembert’s ratio test, the series ∑
|un| is convergent for all x ∈ R.
Hence the given series is absolutely convergent and so also convergent for all real
numbers x.
R-147

(v) The given series is an alternating series




1
( − 1)n − 1 un, where un = > 0, for all n.
(2n − 1) 2n
n=1
Applying Leibnitz’s test for alternating series, show that the given series is
convergent.
Now let Σun ′ denote the series obtained from the given series by making all its terms
positive i. e.,
1 1 1
un ′ = |( − 1)n − 1 un| = = ⋅ ⋅
(2n − 1) (2n) n2 2 [2 − (1 / n)]
We shall apply comparison test to check the convergence of Σun ′.
1
Take vn = ⋅
n2
un ′ 1
Then = ⋅
vn 2 [2 − (1 / n)]
lim un ′ 1
∴ = which is finite and non-zero.
n → ∞ vn 4
Hence by comparison test Σ un ′ and Σ vn are either both convergent or both
1 1
divergent. But for vn = = , p = 2 which is > 1. Therefore Σ vn is convergent and
n2 n p
so Σ un ′ is also convergent.
Hence the given series is absolutely convergent.

(vi) The given series is an alternating series ∑ (− 1)n − 1 un ,
n=1
1
where un = > 0, for all n.
2n − 1
Applying Leibnitz’s test for alternating series, show that the given series is convergent.
Now the series
∞ ∞
∑ ∑
1 1 1 1
|( − 1)n − 1 un| = =1+ + + + ....
n−1 2
n=1 2
n=1
2 2 23
1
which is convergent because it is an infinite geometric series whose common ratio is
2
1
such that 0 < < 1.
2
Since the series ∑ |( − 1)n − 1 un| is convergent, therefore the given series is absolutely
convergent.

sin n α
(vii) Let the given series be ∑ un , where un = (− 1)n n3

n=1

We have ( −1)n sin n α


|un| =  
 n3 
1 1
= ⋅ |sin n α|≤ ⋅ [∵ |sin n α|≤ 1]
3
n n3
R-148


1
Since the series is convergent, therefore by comparison test the series Σ|un| is
n3
also convergent and hence the given series is absolutely convergent.

cos n α
(viii) Let the given series be ∑ un ,where un = (− 1)n n√ n

n=1
 cos n α
We have |un| = ( − 1)n  = 1 ⋅ |cos n α|
 n √ n  n
3 /2
1
≤ ⋅ [∵ | cos n α|≤ 1]
n3 /2


1
Now the series is convergent if p > 1.
np

1
∴ the series is convergent and so by comparison test the series Σ |un| is also
n3 /2
convergent.
Hence the given series is absolutely convergent.


1
(ix) The given series is an alternating series ( − 1)n − 1 un, where un = >0
2n + 3
n=1
for all n.
1 1 −2
We have un + 1 − un = − = < 0 for all n.
2n + 5 2n + 3 (2n + 3) (2 n + 5)
Thus un + 1 < un for all n.
lim lim 1
Also un = = 0.
n→ ∞ n → ∞ 2n + 3
Hence by Leibnitz’s test for alternating series, the given series is convergent.
Now let Σun ′ denote the series obtained from the given series by making all its terms
positive
1 1 1
i. e., un ′ = |( − 1)n − 1 un| = = ⋅ ⋅
2n + 3 n 2 + (3 / n)
We shall apply comparison test to check the convergence of Σun ′.
1 u ′ 1
Take vn = ⋅ Then n = ⋅
n vn 2 + (3 / n)
lim un ′ 1
∴ = which is finite and non-zero.
n → ∞ vn 2
∴ by comparison test Σ un ′ and Σvn are either both convergent or both divergent.
1 1
But for vn = = , p = 1. Therefore Σvn is divergent and so Σun ′ is also divergent.
n np

∑ (− 1)n − 1 2n + 3 is convergent but the series
1
Thus the given series
n=1


( − 1)n − 1 1 
∑ 
2n + 3
n=1 
is divergent. Hence the given series is a semi-convergent series.
R-149

n2
(x) Let the given series be ∑ un, where un = (− 1)n − 1 (n + 1) ! ⋅
n=1
We shall test the given series for absolute convergence i. e., we shall check the
convergence of the series Σ|un|.
For this we shall apply D’ Alembert’s ratio test.
|un|  un 
We have = 
|un + 1| un + 1

n2 ( n + 2) ! n2 ( n + 2)
= ⋅ =
( n + 1) ! ( n + 1)2 ( n + 1)2
n [1 + (2 / n)]
= ⋅
[1 + (1 / n)]2
lim |un| lim |un + 1|
∴ = ∞ so that = 0 which is < 1.
n → ∞ |un + 1| n → ∞ |un|
∴ by ratio test, Σ|un| is convergent.
Since the series Σ∞un∞ is convergent, therefore the given series is absolutely
convergent and so convergent also.

(xi) The given series is an alternating series ∑ (− 1)n un,
n=1
1
where un = sin > 0 for all n ∈ N.
n
 1  1
Now sin   < sin
 n + 1 n
⇒ un + 1 < un, for all n ∈ N.
lim lim 1
Also un = sin = 0.
n→ ∞ n→ ∞ n
∴ by Leibnitz’s test for alternating series, the given series is convergent.
Now let Σ un ′ denote the series obtained from the given series by making all its
terms positive i. e.,
1
un ′ = |( − 1)n un| = sin
n
1 1 1 1 1 1 
= − ⋅ + … = 1 − ⋅ + .... .
n 3 ! n3 n 3 ! n2 
We shall apply comparison test to check the convergence of Σ un ′.
1
Take vn = ⋅
n
lim un ′ lim  1 1 
Then = 1− ⋅ + …
n → ∞ vn n → ∞  3 ! n2 
= 1 which is finite and non-zero.
∴ by comparison test Σ un ′ and Σ vn are either both convergent or both divergent. But
1
Σ vn = Σ is divergent and so Σ un ′ is also divergent.
n
R-150

∑  (− 1)n sin n is


1 1
Since the given series Σ ( − 1)n sin is convergent but the series
n
divergent, therefore the given series is semi-convergent.
(xii) The given series is an alternating series

∑ (− 1)n − 1 un, where un = n2 + (n + 1)2 > 0, for all n.
1 1
n=1

First by applying Leibnitz’s test show that the given series is convergent.
Now let Σ un ′ denote the series obtained from the given series by making all its
terms positive i. e.,
1 1 1  1 
un ′ = |( − 1)n − 1 un | = + = 1 + ⋅
n2 ( n + 1)2 n2  (1 + 1 / n)2 
Applying comparison test we see that Σ un ′ is convergent. Hence the given series is
absolutely convergent.

Problem 3: Show that the series ∑ (− 1)n + 1 [√ (n + 1) − √ n] is semi-convergent.
n=1

Solution: The given series is an alternating series



∑ ( − 1)n + 1 un , where un = √ ( n + 1) − √ n > 0 for all n.
n=1

[ √ ( n + 1) − √ n] [ √ ( n + 1) + √ n]
Now un = √ ( n + 1) − √ n =
√ ( n + 1) + √ n
( n + 1) − n 1
= =
√ ( n + 1) + √ n √ ( n + 1) + √ n
1
and un + 1 = ⋅
√ ( n + 2) + √ ( n + 1)
1 1
Since < , therefore un + 1 < un for all n.
√ ( n + 2) + √ ( n + 1) √ ( n + 1) + √ n
lim lim 1
Again un = = 0.
n→ ∞ n→ ∞ √ ( n + 1) + √ n
Hence by Leibnitz’s test for alternating series, the given series is convergent.
Now let Σ un ′ denote the series obtained from the given series by making all its
terms positive i. e.,
un ′ = |( − 1)n + 1 { √ ( n + 1) − √ n}|
1 1 1
= √ ( n + 1) − √ n = = ⋅ ⋅
√ ( n + 1) + √ n n1 /2 √ {1 + (1 / n)} + 1
We shall apply comparison test to check the convergence of Σ un ′.
R-151

1 u ′ 1
Take vn = ⋅ Then n = ⋅
n1 /2 vn √ {1 + (1 / n)} + 1
lim un ′ 1
∴ = which is finite and non-zero.
n → ∞ vn 2
Hence by comparison test Σ un ′ and Σ vn are either both convergent
1 1 1
or both divergent. But for vn = = , p= which is < 1 so that Σ vn is
1 /2 p 2
n n
divergent. Therefore Σ un ′ is also divergent.

Since the given series ∑ (− 1)n + 1 [√ (n + 1) − √ n] is convergent and the series
n=1

∑ |(− 1)n + 1 [√ (n + 1) − √ n]| is divergent, therefore the given series is
n=1
semi-convergent.

( − 1)n − 1
Problem 4: Show that the series ∑ ( n+ a)2
is semi-convergent.
n=1

Solution: The given series is an alternating series




1
( − 1)n − 1 un, where un = > 0 for all n.
n=1 ( √ n + √ a)2

We have un + 1 < un for all n because


1 1
< for all n.
[ √ ( n + 1) + √ a]2 ( √ n + √ a)2
lim lim 1
Also un = = 0.
n→ ∞ n → ∞ ( √ n + √ a)2

Hence by Leibnitz’s test for alternating series, the given series is convergent.
1 1 1
Let un ′ = |( − 1)n − 1 un| = un = = ⋅ ⋅
2
( √ n + √ a) n [1 + √ ( a / n)]2
1 lim un ′
Take vn = ⋅ Then = 1 which is finite and non-zero. So by comparison test
n n → ∞ vn
Σ un ′ and Σ vn are either both convergent or both divergent.
1
But Σ vn = Σ is divergent and so Σ un ′ is also divergent.
n
∞ ∞
( − 1)n − 1  ( − 1)n − 1 
Since the given series ∑ ( n+ 2
is convergent but the series ∑  2
is
n=1 a) n = 1 ( √ n + √ a) 
divergent, therefore the given series is semi- convergent.
R-152

2 2 2
1. 3  1. 3 . 5 
Show that the series   − 
1
Problem 5:  +   − .... is conditionally
 2  2 . 4  2 . 4 . 6
convergent.
Solution: The given series is an alternating series
∞ 2
1 . 3 . 5 … (2n − 1)
∑ ( − 1)n − 1 un, where un = 
 2 . 4 . 6 … (2n) 
 > 0 for all n.
n=1
2
1 . 3 . 5 . … (2 n − 1) (2n + 1)
We have un + 1 =   ⋅
 2 . 4 . 6 … (2n) (2n + 2) 
2
un + 1  2n + 1 
∴ =  < 1 for all n.
un  2n + 2 
∴ un + 1 < un for all n.
Also it can be easily shown that
√ ( n + 1) 1 . 3 . 5 … (2n − 1) 1
< < ⋅
2n + 1 2 . 4 . 6 . … (2n) √ (2n + 1)
n+1 1
∴ < un < for all n.
(2n + 1)2 2n + 1
lim n+1 lim 1
Since =0 and = 0,
n → ∞ (2 n + 1)2 n → ∞ 2n + 1
lim
therefore un = 0.
n→ ∞
Hence by Leibnitz’s test for alternating series, the given series is convergent.
Now let un ′ = |( − 1)n − 1 un| = un.
To check the convergence of the series Σun ′ we proceed to apply D’ Alembert’s ratio
test.
2
un ′  2n + 2 
We have = 
un + 1′  2n + 1 
4n2 + 8n + 4
= ⋅
4n2 + 4n + 1
lim un ′
∴ =1
n → ∞ un + 1′
and so the ratio test fails to decide the convergence or divergence of the series Σun ′ and
we proceed to apply Raabe’s test.
 u ′   4n2 + 8n + 4 
We have n  n − 1 = n  − 1
u 
 n + 1′
2
  4n + 4n + 1 
4n2 + 3n
= ⋅
4n2 + 4n + 1
R-153

lim  u ′ 
∴ n  n − 1 = 1 and so the Raabe’s test also fails
n→ ∞ u ′ 
 n+1 
and we proceed to apply De Morgan’s and Bertrand’s test.
  u ′    4n2 + 3n 
We have  n  n − 1 − 1 log n =  − 1 log n
u ′   2
+ + 
  n + 1   4 n 4 n 1
− n −1
= (log n) .
2
4n + 4n + 1
log n − 1 − (1 / n)
= ⋅ ⋅
n 4 + (4 / n) + (1 / n2 )

lim    u ′  
∴ (log n) . n  n − 1 − 1 = 0 which is < 1.
n→ ∞   ′ 

u
   n + 1 
∴ by De Morgan’s and Bertrand’s test, Σun ′ is divergent.
∞ ∞
Since the given series ∑ ( − 1)n − 1 un is convergent but the series ∑ |( − 1)n − 1 un|is
n=1 n=1
divergent, therefore the given series is conditionally convergent.
1 1 1
Problem 6: Test the series − + − …, p> 0
2 ( log 2)p 3 ( log 3)p 4 ( log 4)p
for convergence and absolute convergence.
Solution: The given series is an alternating series


1
( − 1)n − 1 un, where un = > 0 for all n.
n=1 ( n + 1) [log ( n + 1)]p

It is given that p > 0.


1 1
Since < for all n,
p
( n + 2) [log ( n + 2)] ( n + 1) [log ( n + 1)]p
therefore un + 1 < un for all n.

lim lim 1
Also un = = 0. [∵ p > 0 ]
n→ ∞ n→ ∞ ( n + 1) [log ( n + 1)]p
Hence by Leibnitz’s test for alternating series, the given series is convergent for all p > 0.

Now the series ∑ |( − 1)n − 1 un| is the series
n=1
1 1 1
+ + + .... ...(1)
p p
2 (log 2) 3 (log 3) 4 (log 4)p
The series (1) is convergent if p > 1 and is divergent if p ≤ 1.
Hence the given series is absolutely convergent if p > 1and semi-convergent if 0 < p ≤ 1.
R-154

Problem 7: Test for convergence the following series :


∞ ∞  1 
n+2
∑ ∑
1
( − 1)n ( − 1)n − 1  + ⋅
5
( i) ( ii)
n 5
n=1
2 +5 n=1  √ n √ ( n + 1) 

Solution: (i) Let the given series be ∑ un,
n=1
n+2
where un = ( − 1)n ⋅
2n + 5
We shall apply D’ Alembert’s ratio test to check the absolute convergence of the given
series i. e., to check the convergence of the series Σ | un| .
|un|  un  n + 2 2n + 1 + 5
We have = = ⋅
|un + 1| un + 1 2n + 5 n+3

1 + (2 / n) 2n + 1 [1 + (5 / 2n + 1 )]
= ⋅
1 + (3 / n) 2n [1 + (5 / 2n )]
1 + (2 / n) 1 + (5 / 2n + 1)
= ⋅2. ⋅
1 + (3 / n) 1 + (5 / 2n )
lim |un|
∴ = 2 which is > 1.
n → ∞ |un + 1|

∴ by ratio test, the series Σ|un| is convergent.


Hence the given series is absolutely convergent and so also convergent.
(ii) The given series is an alternating series


1 1
( − 1)n − 1 un, where un = + > 0 for all n.
n=1 √ n5 √ ( n + 1)5

By Leibnitz’s test for alternating series it can be easily shown that the given series is
convergent.
Now we shall apply comparison test to check the absolute convergence of the given
series i. e., to check the convergence of the series
∞ ∞
∑ ∑
1
|( − 1)n − 1 un| = un , where un = > 0, for all n.
n=1 n=1 |n + |( n + 1)5
5

1 1
We have un = ⋅ ⋅
n5 /2 1 + √ {1 + (1 / n)}5
1
Take vn = ⋅
n5 /2
lim un 1
Then = which is finite and non-zero .
n→ ∞ vn 2
∴ by comparison test Σ un and Σ vn are either both convergent or both divergent.
R-155

1 1 5
But for vn = = , p= > 1. Therefore Σ vn is convergent and so Σ un is also
n5 /2 np 2
convergent.
Hence the given series is absolutely convergent.
Problem 8: Show that the series 1 − 2 + 3 − 4 + 5 − 6 + … oscillates infinitely.
Solution: Let Sn denote the sum of the given series upto n terms.
Then S2 n = (1 − 2) + (3 − 4) + (5 − 6) + ..... + [(2n − 1) − 2n]
= − 1 − 1 − 1 − .... upto n terms = − n
and S2 n + 1 = S2 n + (2n + 1) = − n + (2n + 1) = n + 1.
lim lim
We have S2n = − ∞ and S2n + 1 = ∞.
n→ ∞ n→ ∞
∴ the given series oscillates infinitely.
Problem 9: Discuss the convergence including absolute convergence of the series
1 − 2 x + 3 x2 − 4 x3 + …
Solution: Let the given series be Σ un.
First we shall test the convergence of Σ | un| .
|un|  un   nx n − 1 
We have =  = 
|un + 1| un + 1 ( n + 1) x n
n 1 1 1
= = ⋅ ⋅
n + 1 |x| 1 + (1 / n) |x|
lim |un| 1
∴ = ⋅
n → ∞ |un + 1| |x|
1
So by ratio test the series Σ | un|is convergent if > 1i. e., if|x|< 1. Thus the given series
|x|
is absolutely convergent if |x|< 1.
Since an absolutely convergent series is always convergent, therefore Σ un is convergent
if |x|< 1.
2 3 4 5
Problem 10: Show that the series − + − + .... converges conditionally.
2 2 2
1 2 3 42

Solution: The given series is an alternating series ∑ (− 1)n − 1 un , where
n=1
n+1
un = > 0 for all n.
n2
n+2 n+1 n2 ( n + 2) − ( n + 1)3
We have un + 1 − un = − =
2 2
( n + 1) n n2 ( n + 1)2
n3 + 2n2 − n3 − 3n2 − 3n − 1
=
n2 ( n + 1)2
R-156

− n2 − 3n − 1
= < 0 for all n.
n2 ( n + 1)2
∴ un + 1 < un for all n.
lim lim n+1 lim 1 
⋅ 1 +  = 0.
1
Also un = =
n→ ∞ n→ ∞ n 2 n→ ∞ n  n
∴ by Leibnitz’s test for alternating series, the given series is convergent.
n+1 1 1 + 1  ⋅
Now |( − 1)n − 1 un| = un = =  
n 2 n  n
1 lim un
Take vn = ⋅ Then = 1 which is finite and non-zero. Therefore by
n n→ ∞ vn
comparison test Σ un and Σ vn are either both convergent or both divergent. But
1
Σ vn = Σ is divergent and so Σ un is also divergent.
n
Since the given series Σ ( − 1)n − 1 un is convergent but the series Σ|( − 1)n − 1 un| is
divergent, therefore the given series converges conditionally.

sin ( nθ)
Problem 11: Show that the series ∑ n2
is absolutely convergent.
n=1

sin ( nθ)
Solution: Let the given series be ∑ un , where un = n2

n=1

We have sin ( nθ)


|un| =   = 1 ⋅ |sin ( nθ)|≤ 1 ⋅ [∵ |sin ( nθ)|≤ 1]
 n2  n2 n2


1
Since the series is convergent, therefore by comparison test the series Σ|un|is also
n2
convergent and hence the given series Σ un is absolutely convergent.
Problem 12: Define absolute convergence. Show that the series

1 1 1 1 1 1 1 1
1− − + − − + ..... + − − +…
23 43 33 63 83 (2n − 1)3 (4n − 2)3 (4n)3
is absolutely convergent.
Solution: Absolutely convergent series: Definition:
∞ ∞
A series ∑ unis said to be absolutely convergent if the series ∑ |un| is convergent.
n=1 n=1
 ∞
1 

1 1
The given series is  − − ⋅
3 3
n = 1  (2n − 1) (4n − 2) (4n)3 
It will be an absolutely convergent series if the series
∞ 
1 

1 1
 + +  is convergent.
3 3
n = 1  (2 n − 1) (4 n − 2 ) (4 n)3 
R-157

1 1 1 1  1 1 1
Let un = + + = ⋅ + + ⋅
3 3 3 3 3 3
(2n − 1) (4n − 2) (4n) n  {2 − (1 / n)} {4 − (2 / n)} 64 
1
Take vn = ⋅
n3
lim un 1 1 1
Then = + + which is finite and non-zero.
n→ ∞ vn 8 64 64
∴ by comparison test Σun and Σvn are either both convergent or both divergent.
1 1
But for vn = = , p = 3 which is > 1.
n3
np
∴ Σvn is convergent and so Σun is also convergent.
Hence the given series is absolutely convergent.
Problem 13: Prove that the series
a − b 2 ( a − b ) ( a − 2b ) 3 ( a − b ) ( a − 2b ) ( a − 3b ) 4
z+ z + z + z +…
2! 3! 4!
1
is absolutely convergent if |z|< ⋅
|b|
Solution: Omitting the first term of the series, the given series is

( a − b ) ( a − 2b ) … ( a − nb ) n + 1
∑ un where un =
( n + 1) !
z .
n=1
To check the absolute convergence of the given series we shall apply D’ Alembert’s
ratio test.
|un|  un  ( a − b ) ( a − 2b ) … ( a − nb ) n + 1
We have =  = z
|un + 1| un + 1  ( n + 1) !
( n + 2) ! 1  

( a − b ) ( a − 2b ) … ( a − nb ) {a − ( n + 1) b} z n + 2
n + 2 1
 =
 n+2 1 
= ⋅  ⋅ 
 z a − nb − b  z nb + ( b − a 
)
n+2 1 1  1 
= ⋅ ⋅ ⋅ 
n |z| |b| 1 + {( b − a) / nb}

= 1 +  ⋅
2 1 1 1
⋅ ⋅ ⋅
 n  |z | |b| |1 + {( b − a) / nb }|
lim |un| 1 1
∴ = ⋅ ⋅
n → ∞ |un + 1| |z | |b|
1 1 1
∴ by ratio test, Σ|un| is convergent if ⋅ > 1 i. e., if |z |< ⋅
|z | |b| |b|
1
Hence the given series is absolutely convergent if |z |< ⋅
|b|
R-158

Hints to Objective Type Questions

Multiple Choice Questions


1
1. (c). We know that Σ is convergent if p > 1.
np
2. (a). By article 8, Cauchy’s root test, we know that a series Σ un of positive terms is
lim
convergent if un1 / n < 1.
n→ ∞
3. (d). If un denotes the n th term of the given series, then
12 . 32 . 52 . … . (2 n − 1)2
un = x n −1.
22 . 42 . 62 . … . (2 n)2

12 . 32 . 52 . … . (2 n − 1)2 (2 n + 1)2
∴ un +1 = x n.
2 2 2 2 2
2 . 4 . 6 . … . (2 n) (2 n + 2)

un (2 n + 2)2 1 [2 + (2 / n)]2 1
∴ = ⋅ = ⋅ ⋅
un +1 (2 n + 1)2 x [2 + (1 / n)]2 x
lim un 1
∴ = ⋅
n→ ∞ un +1 x
1
4. (b). We know that the series Σ is convergent if p > 1 and divergent if p ≤ 1.
np
2 1
Since < 1, therefore Σ is divergent.
3 n 2 /3
5. (c). By Alternating series test the series
1 1 1 1 1
1− + − + − + …
2 3 4 5 6
is convergent.
1 1 1
6. (a). The series1 − + − + … is absolutely convergent because the series
22 32 42
1 1 1 1
1+ + + + … i. e., Σ is convergent.
22 32 42 n2
7. (c). By Leibnitz’s test (i. e., Alternating series test) the series Σ un i. e.,
∞ 1
Σ ( −1)n +1 is
n =1 n
1 1
convergent but the series Σ |un|i. e., Σ is divergent because here p = < 1.
n1 /2 2
∴ the given series is semi-convergent.
R-159

Fill in the Blank(s)


1. 0. See article 3. 2. 1. See example 1.
nn
3. 1. See article 6. 4. ⋅
( n + 1)n + 1
5. l < 1. See article 9, Theorem 1.
6. l < 1. See article 9, Theorem 1, Note 1.

x2 n−2
7. ⋅
( n + 1) n

8. For the given series, un = (2 n − 1) x n−1 and un+1 = (2 n + 1) x n.

un (2 n − 1) 1 n [2 − (1 / n)] 1 2 − (1 / n) 1
∴ = ⋅ = ⋅ = ⋅
un+1 (2 n + 1) x n [2 + (1 / n)] x 2 + (1 / n) x

lim un 2−0 1 1
∴ = ⋅ = ⋅
n→ ∞ un+1 2 + 0 x x

9. For the given series,


1. 2 . 3 . 4 . … . n 1. 2 . 3 . 4 . … . n
un = = ⋅
3 . 5 . 7 . 9 . … . {3 + 2 ( n − 1)} 3 . 5 . 7 . 9 . … . (2 n + 1)
10. For the given series,

12 . 32 . 52 . … . (2 n − 1)2
un =
22 . 42 . 62 . … . (2 n)2

12 . 32 . 52 . … . (2 n − 1)2 (2 n + 1)2
and un+1 = ⋅
22 . 42 . 62 . … . (2 n)2 (2 n + 2)2

un (2 n + 2)2 n2 [2 + (2 / n)]2 [2 + (2 / n)]2


∴ = = = ⋅
2 2 2
un+1 (2 n + 1) n [2 + (1 / n)] [2 + (1 / n)]2

lim un (2 + 0 )2
∴ = = 1.
n→ ∞ un +1 (2 + 0 )2
11. Σ un is convergent but Σ |un| is divergent.
12. Σ |un| is convergent.
13. alternating series.
14. 0. See article 18, Alternating series test (Leibnitz’s Theorem).
15. 1. See article 13, Raabe’s test.
R-160

True or False:
1. T . See article 5, Theorem 2, Corollary.
lim
2. F. We know that un = 0 is a necessary but not a sufficient condition
n→ ∞
lim
for a series Σ un to be convergent. If un = 0, the series Σ un is not
n→ ∞
necessarily convergent, it may also be divergent.
lim
3. F. If un1 / n = 1, the series Σ un of positive terms may be convergent
n→ ∞
or divergent.
4. T . See article 9, Theorem 1, Note 1.
1
5. F. The given series is Σ which is convergent.
n2
1 1
6. T . The given series is Σ which is divergent because Σ is divergent if p ≤ 1.
n np
1 1
7. T . Obviously cos > 0 for every natural number n. So, Σ cos is a series having
n n
1 lim lim 1
all terms positive. We have un = cos so that un = cos = 1
n n→ ∞ n→ ∞ n
which is > 0.
∴ Σ un is divergent.
∞ 1
8. F. The series Σ sin is divergent. See example 8 part (ii).
n=1 n
n1 /2 n1 /2 1
9. T . Let un = = = ⋅
n + 1 n [1 + (1 / n )] n [1 + (1 / n2 )]
2 2 2 3 /2
1
Take vn = ⋅
3 /2
n
lim un lim 1
Then = = 1 which is finite and non-zero.
n → ∞ vn n → ∞ [1 + (1 / n2 )]
Therefore, by comparison test, Σ un and Σ vn are either both convergent or
3
both divergent. But for vn , p = > 1. Therefore, Σ vn is convergent.
2
So, Σ un is also convergent.
1
10. F. Let un = ⋅
1 + (1 / n)
lim lim 1 1
We have un = = = 1 which is > 0.
n→ ∞ n → ∞ 1 + (1 / n) 1 + 0
∴ Σ un is divergent.
11. T . The given series is the exponential series
x x2 x3
1+ + + +…
1! 2 ! 3 !
which is absolutely convergent for all values of x. See example 40.
R-161

12. F. If we make all the terms of the given series positive, then the resulting series is
1 1 1 1 1
+ + + + … i. e., Σ
1 2 3 4 1 /2
n
which is divergent. So, the given series is not absolutely convergent.
13. T . See example 39.
14. F. By Alternating series test the given series is convergent.
15. F. If we make all the terms of the given series positive, then the resulting series is
1 1 1
1+ + + +…
1! 2 ! 3 !
which is convergent because it is an exponential series. So, the given series
is absolutely convergent.
16. F. By alternating series test, the given series is convergent. By comparison test,
the series
1 1
Σ |un| = Σ =Σ is divergent.
( n + a )2 n [1 + ( a / n )]2
1
Take vn = ⋅
n
So, the given series is semi-convergent and not absolutely convergent.
17. T. See article 20, Theorem 1.
18. F. For example, the series
1 1 1
1− + − + …
2 3 4
is convergent but it is not absolutely convergent because the series
1 1 1
1+ + + + …
2 3 4
is divergent.
∞ 1
19. T. By Alternating series test, the series Σ ( −1)n sin is convergent but the
n=1 n
∞ 1
series Σ |un| = Σ sin is divergent. See problem 2(xi) of Comprehensive
n=1 n
Problems 8. Thus, the given series is convergent but is not absolutely
convergent.
20. T. For the given series, we have
1 1
un = = ⋅
n + n+1 n [1 + 1 + (1 / n) ]
1
Take vn = ⋅
n
By applying comparison test we see that the given series is divergent.
21. F. For the given series of positive terms, we have
( n + 1 − n) ( n + 1 + n) ( n + 1) − n
un = n + 1 − n = =
n+1+ n n+1+ n
1 1
= = ⋅
n+1+ n n [1 + 1 + (1 / n)]
R-162

1
Take vn = ⋅
n
By applying comparison test we see that the given series is divergent.
22. T . For the given series of positive terms, we have

un = n4 + 1 − n2

( n4 + 1 − n2 ) ( n4 + 1 + n2 ) ( n4 + 1) − n4
= =
n4 + 1 + n2 n4 + 1 + n2
1 1
= = ⋅
4
n +1+ n 2 n [1 + 1 + (1 / n4 )]
2

1
Take vn = ⋅
n2
By applying comparison test we see that the given series is convergent.
23. F. For the given series of positive terms, we have
( n3 + 1 − n3 ) ( n3 + 1 + n3 )
un = n3 + 1 − n3 =
n3 + 1 + n3
1 1
= = ⋅
3 /2
3
n +1+ n 3 n [1 + 1 + (1 / n3 )]
1
Take vn = ⋅
3 /2
n
By applying comparison test we see that the given series is convergent.
24. T . If we make all the terms of the given series Σ un positive, then the
resulting series is
1 1
Σ |un| i. e., Σ i. e., Σ which is convergent.
n n n3 /2
So, the given series is absolutely convergent.
1 n
25. F. For the given series of positive terms, we have un = (1 + ) ⋅
n
lim lim  n
1
We have un = 1 +  = e > 1.
n→ ∞ n→ ∞  n
So, the given series is divergent.
26. T . The given statement is true.
1 1 1
27. T . For the given series, we have un = = for which p = < 1.
n n1 /2 2
So, the given series is divergent.
28. F. The given statement is false because if l > 1, then by Ratio test Σ un is
convergent.
❍❍❍
R-163

Chapter-6
The Riemann Integral

Comprehensive Problems 1
1 2
Problem 1: (i) Let f ( x ) = x (0 ≤ x ≤ 1). Let P be the partition {0, , , 1} of [0, 1] .
3 3
Compute U ( P, f ) and L ( P, f ).
(ii) Let f ( x ) = x (0 ≤ x ≤ 3). Let P be the partition {0, 1, 2, 3} of [0, 3] . Compute U ( P, f ) and
L ( P, f ).
Solution: (i) The partition P divides the interval [0, 1] into the sub-intervals

I1 = 0,  , I2 =  ,  , I3 =  , 1 ⋅
1 1 2 2
 3   3 3   3 
The lengths of these sub-intervals are given by
1 1 2 1 1 2 1
∆1 = − 0 = , ∆2 = − = , ∆3 = 1 − = ⋅
3 3 3 3 3 3 3
Also if M r and mr be respectively the l.u.b. and g.l.b. of the function f ( x ) = x in
I r , r = 1, 2 , 3, then here we get
1 2 1 2
M1 = , m1 = 0 ; M2 = , m2 = ; and M3 = 1, m3 = ⋅
3 3 3 3
3
∴ U ( P, f ) = Σ M r ∆ r = M1 ∆1 + M2 ∆2 + M3 ∆3
r =1
1 1 2 1 1 1 2 1 6 2
= ⋅ + ⋅ + 1⋅ = + + = =
3 3 3 3 3 9 9 3 9 3
3
and L ( P, f ) = Σ mr ∆ r = m1 ∆1 + m2 ∆2 + m3 ∆3
r =1

1 1 1 2 1 1 2 1
=0⋅ + ⋅ + ⋅ = + = ⋅
3 3 3 3 3 9 9 3
(ii) Proceed as in problem 1 (i). Ans. U ( P, f ) = 6 and L ( P, f ) = 3.
1 1
Problem 2: Show by definition that
∫0 x4 dx =
5

Solution: Let f ( x ) = x4 defined on [0, 1].


n −1 n
Let P = 0, , , … , , = 1 be the partition of [0, 1] obtained by dissecting [0, 1]
1 2
 n n n n 
into n equal parts. Then
R-164

r −1 r
I r = r th sub-interval =  ,
 n n 
1
and ∆ xr = length of I r = , r = 1, 2, … , n.
n
Let mr and M r be respectively the infimum and supremum of f in I r . Since f ( x ) = x4 is
an increasing function in [0, 1], therefore
( r − 1)4 r4
mr = and M r =
, r = 1, 2 , … , n.
n4 n4
n n  ( r − 1)4 1  1 n
Now L ( P, f ) = Σ mr ∆ xr = Σ  ⋅ = Σ ( r − 1)4
r =1 r =1 4 5 r =1
 n n  n
1 4
= [1 + 24 + … + ( n − 1)4 ]
n5
1 ( n − 1) n (2n − 1)
= [3 ( n − 1)2 + 3 ( n − 1) − 1]
n5 30
 n n ( n + 1) (2n + 1) 
∵rΣ r4 = (3n2 + 3n − 1)
 = 1 30 
1
= ( n − 1) n (2 n − 1) (3 n2 − 3 n − 1)
30 n5
1  1  1  3 1
= 1 −   2 −   3 − − 2  ⋅
30  n  n  n n 
1
lim L ( P, f ) = 1 ⋅1⋅ 2 ⋅ 3 = 1 ⋅

∫0 f ( x ) dx =
n→ ∞ 30 5
n n  r4 1  1 n
Again U ( P, f ) = Σ M r ∆ xr = Σ  4 ⋅ = 5 Σ r4
r =1 r =1  n n  n r =1

1 n ( n + 1) (2n + 1)
= ⋅ (3n2 + 3n − 1)
n5 30
=
1 1 + 1   2 + 1   3 + 3 − 1  ⋅
     
30  n  n  n n2 
1
lim U ( P, f ) = 1 ⋅1 ⋅ 2 ⋅ 3 = 1 ⋅

∫0 f ( x ) dx =
n→ ∞ 30 5
1 1
Since
∫0 f =
∫0 f, therefore f is Riemann-integrable in [0, 1] and
1 1 1
∫0 f ( x ) dx =
∫0 x4 dx =
5

Problem 3: Let f ( x ) = x − 1 /2 on [1, 4]. Consider the partition obtained by dividing [1, 4] into n
4
equal parts and hence show that
∫1 x − 1 /2 dx = 2.

Solution: Obviously f ( x ) = x − 1 /2 = 1 / √ x is bounded on [1, 4] because


R-165

1
≤ f ( x ) ≤ 1 V x ∈[1, 4].
2
Let P = 1, 1 + , 1 + , … , 1 + = 4 be the partition of [1, 4] obtained by
3 6 3r 3n
, … ,1 +
 n n n n 
dividing [1, 4] into n equal parts. Then

3 ( r − 1)
I r = r th sub-interval = 1 +
3r 
,1 +
 n n 
3
and ∆xr = length of I r = , r = 1, 2, … , n.
n
Let mr and M r be the infimum and supremum respectively of f in I r . Since
f ( x ) = 1 / √ x is a decreasing function in [1, 4], therefore
1 1
Mr = and mr = ⋅
1 + 3 ( r − 1) 1 + 3r 
 n   n 
 
n n  
 1 3
Now U ( P, f ) = Σ M r ∆ xr = Σ ⋅ 
r =1 r =1   3 ( r − 1) n 
 1 + 
 
 n 
 
 
3 1 1 1 1
=  + + + …+  = Q, say.
n  √1 1 + 3  1 + 6  1 + 3 ( n − 1) 
      
  n  n  n 
  3 
 1 +  + √1
  3    n 
We have Q =  1 +  − √1 ⋅
 n √1
 
  6 1 + 3  
 1 +  + 


  n n 
+  1 +  − 1 + 3  ⋅ 
6
 
 n  
n  1 + 3
  
 n

  9 1 + 6  
 1 +  +  

+  1 +  −
9 1 + 6  ⋅  
 n  n 
+…
 
 n  n  1 + 6 
  
 n

  3n   3 ( n − 1) 
 1 +  + 1 +
 
 
1 + 3 ( n − 1) ⋅  n  n 
+  1 +
3n 
 −  
 n  n  1 + 3 ( n − 1)

 
 n 
R-166

2 1 + 
3
  2 √1   1 + 3   ⋅  n
>  1 +  − √1 ⋅ +  1 +  −
3 6
 
 n  √1  n  n  1 + 
3
    
 n
3 ( n − 1)
2 1 + 
 1 + 3 ( n − 1)  ⋅  n 
+ … +  1 +
3n 
 −  
 n  n  1 + 3 ( n − 1)

 
 n 
   1 + 3   + …
= 2   1 +  − √1 +  1 +  −
3 6
 
   n   n  n 
 

 1 + 3 ( n − 1) 
+  1 +
3n 
 −  
 n  n  
 

 
= 2  1 +
3n 
 − √1 = 2 (2 − 1) = 2.
 n 
 
Thus Q > 2.
  3    3 
 √1 + 1 −    1 +  + √1
 
1 − 3  ⋅  n +   3   n 
Again Q =  √1 −    1 +  − √1 ⋅
 n  √1 n  3 
   1 + 
 n
  6 1 + 3  
 1 +  + 


  n n 
+  1 +  − 1 +  ⋅ 
6 3
  +…
 n  n  1 + 6
  
 n
  3 ( n − 1)  3 ( n − 2)  
 1 +  + 1 + 
 3 ( n − 1) 
1 + 3 ( n − 2) ⋅   n   n 
+  1 + −  
  n   n  1 + 3 ( n − 1)
 
 n 

2 1 + 
3
 1 − 3   ⋅ 2 √1 +  1 + 3  − √1 ⋅  n
<  √1 −       +…
 n   √1 n 1 + 3 
    
 n
3 ( n − 1)
2 1 + 
 3 ( n − 1) 1 + 3 ( n − 2)  ⋅  
+  1 +
n
−  
  n   n  1 + 3 ( n − 1)
 
 n 
R-167

 1 − 3   +  1 + 3  − √1 +  1 + 6  − 1 + 3   + …
= 2  √1 −           
   n    n n  n 
 

 3 ( n − 1) 1 + 3 ( n − 2) 
+  1 +  −  
 n   n  
 

 1 + 3 ( n − 1) − 1 − 3   .
=2    
 n   n 

 1 + 3 ( n − 1) − 1 − 3   ⋅
Thus 2 < Q < 2      ...(1)
 n   n 

4

∫1 f ( x) dx = n → ∞
Now lim U ( P, f ) = lim Q.
n→ ∞

Since lim 2=2


n→ ∞
lim 2  1 + 3 ( n − 1) − 1 − 3   = 2 [ √ 4 − √1] = 2,
and    
n → ∞   n   n 

therefore by Sandwich theorem, lim Q = 2.


n→ ∞
n n  1 3
Again L ( P, f ) = Σ mr ∆ xr = Σ  ⋅ 
r =1 r =1
 (1 + 3 r / n) n
3 1 1 1 1 
=  + + + …+ ⋅
n  (1 + 3 / n) (1 + 6 / n) (9 / n) (1 + 3n / n) 

Proceeding as above we can show that lim L ( P, f ) = 2.


n→ ∞
4
∫1 f ( x) dx = n → ∞
∴ lim L ( P, f ) = 2.

4 4
Hence
∫1 f ( x ) dx =
∫1 f ( x ) dx = 2.

4
f ( x ) = x − 1 /2 is Riemann-integrable on [1, 4] and
∫1 x
− 1 /2
∴ dx = 2.

Problem 4: If f ( x ) = cos x V x ∈ [0, π / 2] , show that f is integrable on [0, π / 2] and


π /2
∫0 cos x dx = 1.

Solution: Obviously f ( x ) = cos x is bounded on [0, π / 2] because


0 ≤ f ( x ) ≤ 1 V x ∈[0, π / 2] .
π 2π r π nπ π 
Let P = 0 , , ,…, ,…, =  be the partition of [0, π / 2] obtained by
 2 n 2 n 2 n 2 n 2
dissecting [0, π / 2] into n equal parts. Then
R-168

( r − 1) π r π 
I r = r th sub-interval =  ,
 2n 2n 
π
and ∆xr = length of I r = , r = 1, 2, … , n.
2n
Let mr and M r be the infimum and supremum respectively of f in I r . Since
f ( x ) = cos x is a decreasing function in [0, π / 2], therefore
( r − 1) π rπ
M r = cos and mr = cos ⋅
2n 2n
n
Now U ( P, f ) = Σ M r ∆ xr
r =1
n
 ( r − 1) π  π 
= Σ ⋅
 
cos
r =1 2n  2n 
π  π 2π ( n − 1) π 
= cos 0 + cos + cos + … + cos
2n  2n 2n 2n 
( n − 1) π  nπ
cos 0 + ⋅ sin
π  2 2 n  4 n
= ⋅
2n sin ( π / 4n)
n −1  nβ
 cos  α + β  sin 
 ∵ cos α + cos (α + β ) + … + cos {α + ( n − 1) β } =  2  2 
 sin (β / 2) 
 
( π / 4n) π  1  π
= ⋅ 2 cos  1 −   sin ⋅
sin ( π / 4n) 4  n  4
π /2
lim U ( P, f ) = 1. 2 cos π sin π = 1.

∫0 f ( x ) dx =
n→ ∞ 4 4
n n
 r π π 
L ( P, f ) = Σ mr ∆ xr = Σ ⋅
 
Again cos
r =1 r =1 2n  2n 
π  cos π + cos 2 π + … + cos nπ 
=
2n  2n 2n 2n 
π ( n − 1) π  nπ
cos  + ⋅ sin
=
π
⋅  2n 2 2n  4n
2n sin ( π / 4n)

( π / 4n) π 1  π
= ⋅ 2 cos  1 +   sin ⋅
sin ( π / 4n)  4  n   4
π /2
lim L ( P, f ) = 1. 2 cos π sin π = 1.

∫0 f ( x ) dx =
n→ ∞ 4 4
π /2 π /2
Hence
∫0 f ( x ) dx =
∫0 f ( x ) dx = 1.
π /2
∴ f ( x ) = cos x is Riemann-integrable on [0, π / 2] and
∫0 cos x dx = 1.
R-169

Problem 5: Show that f ( x ) = 3 x + 1 is integrable on [1, 2] and


2 11
∫1 (3 x + 1) dx = 2 ⋅
Solution: First, we observe that f ( x ) is bounded on [1, 2], for evidently
4 ≤ f ( x ) ≤ 7 V x ∈[1, 2].

Let P = 1, 1 + , 1 + , … , 1 + = 2 be the partition of [1, 2] obtained by dissecting


1 2 n
 n n n 
[1, 2] into n equal parts. Then
r −1
I r = r th sub-interval = 1 + ,1 + 
r
 n n 
1
and ∆ xr = length of I r = , r = 1, 2, … , n.
n
Let mr and M r be respectively the infimum and supremum of f in I r . Since
f ( x ) = 3 x + 1 is an increasing function in [1, 2], therefore
r − 1 3 ( r − 1)
mr = 3 1 +  +1=4 +
 n  n
M r = 3 1 +  + 1 = 4 +
r 3r
and ⋅
 n n
n n
 3 ( r − 1) 1 
Now L ( P, f ) = Σ mr ∆ xr = Σ 4 + ⋅ 
r =1 r =1  n  n
1 n  3 ( r − 1) 1  3 n 
= Σ 4 + = 4n + n Σ ( r − 1)
n r =1 n  n  r =1 
3
=4+ [1 + 2 + … + ( n − 1)]
n2
3 ( n − 1) n
= 4 + 1 −  ⋅
3 1
=4+ ⋅
n2 2 2 n
2 3 11
∫1 f ( x) dx = n → ∞
∴ lim L ( P, f ) = 4 + = ⋅
2 2
n n
 3r  1 
U ( P, f ) = Σ M r ∆ xr = Σ 4+ ⋅
 
Again
r =1 r =1 n  n 
n 3r  1  n 
Σ  4 +
1 3
=  = 4n + Σ r
n r =1  n  n  n r =1
3 n ( n + 1)
= 4 + 1 +  ⋅
3 3 1
=4+ (1 + 2 + … + n) = 4 + ⋅
n n 2 2 n
2
3 11
∫1 f ( x) dx = n → ∞ U (P, f ) = 4 + 2 = 2 ⋅
∴ lim

2 2
Since
∫1 f ( x ) dx =
∫1 f ( x ) dx, f is Riemann-integrable on [1, 2]
2 11
and
∫1 f ( x) dx = 2 ⋅
R-170

Problem 6: Give an example to prove that a bounded function need not be R-integrable.
Solution: Give any one of the examples given in Ex. 5 and Ex. 6 of Text Book.
Problem 7: Give an example of a discontinuous function which is R-integrable on [0, 1].
Solution: Give Ex. 12 of Text Book.
n 1 1
Problem 8: Let f be defined on [0, 1] by f ( x ) = , when < x ≤ , n = 1, 2, 3, … and
n+1 n+1 n
1 π2
f ( x ) = 1 , x = 0. Then, show that f is Riemann integrable on [0, 1]and
∫0 f ( x ) dx =
6
−1 .

Solution: Here for n = 1, 2, 3, …


1 1
f ( x) = when < x ≤1
2 2
2 1 1
f ( x) = when x≤
3 3 2
3 1 1
f ( x) = when < x≤
4 4 3
… … … … …
n −1 1 1
f ( x) = when < x≤
n n n −1

n 1 1
f ( x) = when < x≤
n+1 n+1 n
… … … … …
We have
n −1
f  + 0  = lim  h = 1 −
1 1
+
n  h→0  n  n

f  − 0  = lim  n − h = n
1
and  
n  h→0  n+1  n+1

f  + 0  ≠ f  − 0  , f is not continuous at x =
1 1 1
Since
n  n  n
1 1 1
The set of point of discontinuity , , , … of f has only one limit point at x = 0.
2 3 4
Consequently by the theorem 4 of article 7 of Text Book, the given function is
R-integrable.
1 ∞ 1 /n n
Now
0 ∫
f ( x ) dx = Σ

n = 1 1 /( n +1) n + 1
dx

∞ n 1 1  ∞ 1
= Σ  −  = Σ
n=1 n + 1  n n + 1 n = 1 ( n + 1)2

1 1 1
= + + + ...
2 2
2 3 42
R-171

1 1 1 1  π2
= + + + + … − 1 = − 1.
 12 22 32 42  6

Problem 9: A function f ( x ) is defined on [0, 1] as follows :


n 1 1
f ( x) = , when ≤ x ≤ ( n = 1, 2, 3, ...)
n+2 n+1 n
1 1
and f ( x ) = 1 , when x = 0. Show that f ( x ) is R-integrable on [0, 1] and
∫0 f ( x ) dx =
2
.

(Garhwal 2008)
Solution: Proceed as in problem 8.
Problem 10: Let a function f ( x ) be defined on [0, 1] as follows :
1 1 1
f ( x) = , when < x< for r = 1, 2, 3, … where a is an integer greater
−1 ar −1
ar ar
1 a
than 1. Show that
∫0 f ( x ) dx exists and is equal to
a+1
.

Solution: Here, for r = 1, 2, 3, … we have

f  r + 0  = lim f  r + h =
1 1 1
a  h→0 a  ar − 1

f  r − 0  = lim f  r − h = r
1 1 1
and
a  h→0 a  a
1 1 1 
since + h lies in the interval  r , 
ar  a ar − 1 
1  1 1
and − h lies in the interval  , 
ar  ar + 1 ar 

Thus f  r + 0  ≠ f  r − 0  , f is not continuous at x = r , r = 1, 2, 3, … and the set


1 1 1
a  a  a
1 1 1
of points of discontinuity , , , … of f has only one limit point at x = 0. Con-
a a2 a3
sequently by theorem 4 of article 7 of Text Book that f ∈ R [0, 1]. Hence we have
1 ∞ 1 /ar − 1
1
∫0 f ( x ) dx = Σ
r = 1 ∫1 / a ar −1
r
dx

∞ 1  1 1
= Σ − r
r = 1 ar − 1 
a r −1 a 

1 1 1 1 1
= 11 −  +
1 1
   − 2  + 2  2 − 3  + ....
a a a a  a a a 

1 1 1 1 1 a
=1− + − + − …= = .
a a2 a3 a4
a+
1 a +1
a
R-172

Problem 11: Calculate the values of upper and lower integrals for the function f defined on [0, 2]
as follows :
f ( x ) = x2 when x is rational
3
and f ( x) = x when x is irrational.
Solution: First, we observe that f is bounded, for evidently
0 ≤ f ( x ) < 8 V x ∈ [0, 2] .
Now x2 − x3 = x2 (1 − x ) .
∴ x2 > x3 if 0 < x < 1 and x2 < x3 if 1 < x ≤ 2.
If P is any partition of [0, 2], then any sub-interval of P, however small it may be, will
contain rational as well as irrational points.
Let mr and M r be respectively the infimum and supremum of f in the r th sub-interval.
Then for all values of r
 x3 , if 0 < x < 1
mr = 
 x , if 1 < x < 2
2

 x2 , if 0 < x < 1
and Mr = 
 x3 , if 1 < x < 2.
2 1 2
1 4 1 1 3 2
Hence
∫0 f ( x ) dx =
∫0 x3 dx +
4 ∫1 x2 dx =
[x ]0 + [x ]1
3
1 1 1 7 31
= + (8 − 1) = + =
4 3 4 3 12
2 1 2 3 1 1 4 2
∫0 f ( x) dx = ∫0 x2 dx +
∫1 x dx = 3 [ x
3 1
and ]0 + [x ]1
4
1 1 1 15 49
= + (16 − 1) = + = ⋅
3 4 3 4 12
Alternative Solution.
2n 2 ( n + 1) 2 (2n − 1) 2 . 2n
P = 0, = 2
2 4
Let , ,…, , ,…, ,
 2n 2n 2n 2n 2n 2n 
be the partition of [0, 2] obtained by dissecting [0, 2] into 2n equal parts. Then

2 ( r − 1) 2r   r − 1 r 
I r = r th subinterval =  , = , ,
 2n 2n   n n 
2 1
and ∆ xr = length of I r = = , r = 1, 2, … , 2 n.
2n n
( r − 1)3
 , for r = 1, … , n
 n3
Also mr = 
( r − 1)2
, for r = n + 1 . … , 2n
 n2
R-173

 r2
 2 , for r = 1, … , n

and Mr =  n
3
 r , for r = n + 1, … , 2n.
 n3
2n n 2n
∴ L ( P, f ) = Σ mr ∆ xr = Σ mr ∆ xr + Σ mr ∆ xr
r =1 r =1 r = n+1

n  ( r − 1)3 1  2n  ( r − 1)2 1 
= Σ  ⋅ + Σ  ⋅ 
r =1 3 n  r = n+1 2
 n  n n 

n 2n n
1 1 1
= Σ ( r − 1)3 + Σ ( r − 1)2 − Σ ( r − 1)2
n4 r = 1 n3 r = 1 n3 r =1

2
1  ( n − 1) n  1 (2 n − 1) 2 n (4 n − 1) 1 ( n − 1) n (2 n − 1)
= + −
n4  2  n3 6 n3 6
2
1 1 1 1  1 1  1  1
= 1 −  +  2 −   4 −  − 1 −   2 −  ⋅
4 n 3 n  n 6  n  n

2 lim L ( P, f ) = 1 + 8 − 1 = 31 ⋅

∫0 f ( x ) dx =
n→ ∞ 4 3 3 12

2n n 2n
Also U ( P, f ) = Σ M r ∆ xr = Σ M r ∆ xr + Σ M r ∆ xr
r =1 r =1 r = n+1

n  r2 1  2n  r3 1 
= Σ  2 ⋅  + Σ  3 ⋅ 
r =1  n n  r = n+1  n n 

n 2n n
1 1 1
= Σ r2 + Σ r3 − Σ r3
n3 r = 1 n4 r =1 n4 r =1

2 2
1 n ( n + 1) (2n + 1) 1  2n (2n + 1) 1  n ( n + 1)
= ⋅ + ⋅ −
n3 6 n4  2  n4  2 
2 2
1 1  1  1 1 1
= 1 +   2 +  +  2 +  − 1 +  ⋅
6 n  n  n 4 n
2
lim U ( P, f ) = 1 + 4 − 1 = 49 ⋅

∫0 f ( x ) dx =
n→ ∞ 3 4 12
Problem 12: Let f ( x ) be a function bounded on [ a, b ] and let P1 and P2 be two partitions of
[ a, b ] such that P1 ⊂ P2 . Then prove that
U ( P1, f ) − L ( P1, f ) ≥ U ( P2 , f ) − L ( P2 , f ).
R-174

Solution: P1 ⊂ P2 ⇒ P2 is refinement of P1

⇒ U ( P2 , f ) ≤ U ( P1, f )
[Refer theorem 3 of article 2 of Text Book]
⇒ U ( P1, f ) ≥ U ( P2 , f ). ...(1)
Again P1 ⊂ P2 ⇒ P2 is a refinement of P1
⇒ L ( P1, f ) ≤ L ( P2 , f ) [Refer theorem 3 of article 2 of Text Book]
⇒ − L ( P1, f ) ≥ − L ( P2 , f ). ...(2)
From (1) and (2), by adding we get
U ( P1, f ) − L ( P1, f ) ≥ U ( P2 , f ) − L ( P2 , f ).

Hints to Objective Type Questions

Multiple Choice Questions


1. (b). See article 2, Theorem 3 of Text Book.
2. (a). See article 2, Theorem 5 of Text Book.
3. (c). The function f is said to be Riemann integrable on [ a, b ] iff
b b

∫a f =
∫a f. See article 4 of Text Book.

4. (d). See article 8, Theorem 6 of Text Book.


Fill in the Blank(s)
1. a = x0 < x1 < … < xn − 1 < xn = b. See article 2, Definition 1 of Text Book.

2. P* ⊃ P. See article 2, Definition 2 of Text Book.


3. common refinement. See article 2, Definition 2 of Text Book.
4. ≤. See article 2, Definition 3 of Text Book.
5. ≥. See article 2, Theorem 3 of Text Book.
6. ≥. See article 2, Theorem 4 of Text Book.
7. supremum. See article 3, definition of lower Riemann integral of Text Book.
8. infimum. See article 3, definition of upper Riemann integral of Text Book.
9. exceed. See article 3, Theorem 1 of Text Book.
b
10.
∫a f. See article 4 of Text Book.
b b
The function f is called Riemann integrable on [ a, b ] iff
∫a f =
∫a f.
11. ε. See article 6, Theorem of Text Book.
12. φ ( b ) − φ ( a). See article 9, Theorem 5 of Text Book i. e., fundamental theorem
of integral calculus.
R-175

True or False
1. T . See article 7, Theorem 1 of Text Book.
2. F. Every bounded function f defined on [ a, b ] is not necessarily Riemann
integrable on [ a, b ]. See Example 6 of Text Book.
3. F. If a function f is discontinuous on [ a, b ] but the set of points of discontinuity
of f on [ a, b ] is finite or has only a finite number of limit points even then f is
Riemann integrable on [ a, b ]. See article 7, Theorem 3 and Theorem 4 of
Text Book.
4. T . See article 7, Theorem 2 of Text Book.
5. F. If f is Riemann integrable on [ a, b ], then | f | is also Riemann integrable on
[ a, b ]. See article 8, Theorem 6 of Text Book.
6. T . See article 9, Theorem 1 of Text Book.
7. T . See article 9, Theorem 2 of Text Book.
b
8. F. We have
∫a f ( x ) dx = φ ( b ) − φ ( a). See article 9, Theorem 5 of Text

Book.
9. F. We have m ( b − a) ≤ L ( P, f ). See article 2, Theorem 1 of Text Book.
10. T . See article 2, Theorem 2 of Text Book.
11. F. We have U ( P * , f ) ≤ U ( P, f ). See article 2, Theorem 3 of Text Book.
12. F. If f is a bounded function on [ a, b ], then it is not necessarily Riemann
b b
integrable on [ a, b ] i. e., it is not necessary that
∫ a f = ∫a f. See Example 6 of

Text Book.
13. T . If f is continuous on [ a, b ], then f is Riemann integrable on [ a, b ] and so we
b b
must have
∫ a f = ∫a f.
14. T . See article7, Theorem 3. If the set of points of discontinuity of f on [ a, b ] is
finite, then f is Riemann integrable on [ a, b ].

❍❍❍
A-176

Chapter-7
Convergence of Improper Integrals

Comprehensive Problems 1
∞ dx
Problem 1: Evaluate
∫1 x

Solution: We have
∞ dx x dx
∫1 = lim
∫1 = lim [log x] 1
x
x x→ ∞ x x→ ∞

= lim [log x − 0 ] = ∞.
x→ ∞
Thus the limit does not exist finitely and therefore the given integral is divergent ( i. e., the
integral does not exist).
∞ dx
Problem 2: Evaluate

3 ( x − 2)2

Solution: We have
∞ dx x dx
∫3 ( x − 2) 2
= lim
x→ ∞ ∫3 ( x − 2)2
, (By def.)

x
lim  ( x − 2) 
x −1

∫3 ( x − 2)− 2 dx =
= lim
 
x→ ∞ x → ∞  − 1 
3
x
= lim  − 1  = lim  − 1 
+ 1 = 1,
x → ∞  x − 2  3 x → ∞  x − 2 
which is a definite real number. Therefore the given integral is convergent and its value
is 1.

Problem 3: Test the convergence of
∫0 e2 x dx.
∞ x 2x
Solution: We have
∫0 e2 x dx = lim
x→ ∞ ∫0 e dx, (By def.)
x
lim  e2 x  1 lim
=   = [ e2 x
− 1] = ∞.
x → ∞  2  2 x→ ∞
0

Thus the limit does not exist finitely and therefore the given integral is divergent ( i. e. the
integral does not exist).
R-177
∞ dx
Problem 4: Show that
∫0 (1 + x )2 /3
is not convergent.

∞ dx ∞
∫0 ∫0
Solution: We have = lim (1 + x )− 2 /3 dx, (By def.)
(1 + x ) 2 /3 x→ ∞
x
lim  (1 + x )  = lim 3 [(1 + x )1 /3 − 1] = ∞.
1 /3
=  
x → ∞  1 / 3  x→ ∞
0
Thus the limit does not exist finitely and therefore the given integral is divergent( i. e., the
integral does not exist).
0
Problem 5: Test the convergence of
∫− ∞ sinh x dx;

Solution: We have
0 0
∫− ∞ ∫
sinh x dx = lim sinh x dx, (By def.)
x→ ∞ − x
lim 0 e x − e− x
=
x→ ∞ − x 2 ∫ dx [Note]

1 0 0 
e x dx − lim
∫ ∫− x e
− x
=  lim dx 
2  x→ ∞ − x x→ ∞ 
 e− x 
0 
1  lim
{e x }0− x − lim 
=  
2  x→ ∞ x → ∞  −1  
 x 
lim
1 
=  lim { e0 − e − x } + { e0 − e x }
2 x → ∞ x→ ∞ 
1
= [1 − ∞ ] = − ∞.
2
Thus the given integral diverges to − ∞.
0
Problem 6: Test the convergence of
∫− ∞ cosh x dx.

Solution: We have
0 0 e x + e− x
1  0 0 
∫ − ∞ cosh x dx = ∫ − ∞ ∫− ∞ e ∫− ∞ e
x − x
dx = dx + dx 
2 2  
1
=
[1 + ∞ ] = ∞.
2
(As proved in Example 4 of Text Book prove it here.)
Thus the given integral diverges to ∞.

Problem 7: Test the convergence
∫0 cos x dx
∞ x
∫0 ∫0 cos x dx = x → ∞
Solution: We have cos x dx = lim lim [sin x ]0x
x→ ∞
= lim sin x − sin 0 = lim sin x.
x→ ∞ x→ ∞
Thus the limit dose not exist finitely. Since the limit of sin x oscillates between + 1 and
− 1 when x → ∞. Hence the given integral oscillates and so not convergent.
R-178

∫− ∞ e− x
Problem 8: Test the convergence of dx.

Solution: We have
∞ 0 ∞
∫− ∞ e−
∫− ∞ e−
∫0 e − x dx
x x
dx = dx +

= + ∞ + 1 = ∞. (Refer Ex. 4 and Ex. 2 of Text Book)


Thus the limit does not exist finitely. Therefore the given integral is divergent (i. e., the
integral does not exist).
∞ dx
Problem 9: Evaluate

− ∞ x2 + 2 x + 2

∞ dx ∞ dx
Solution: We have
∫− ∞ 2
x +2x+2
=
∫− ∞ ( x + 1)2 + 1
c dx x2 dx
∫− x ∫c
= lim + lim ,
x1 → ∞ 1 ( x + 1) + 1 2 x2 → ∞ ( x + 1)2 + 1
where c is any real number
lim
= [tan− 1 ( x + 1)] −c + lim [tan− 1 ( x + 1)] x2
c
x1 → ∞ x1 x2 → ∞

= lim [tan− 1 ( c + 1) + tan− 1 (1 − x )]


1
x1 → ∞

+ lim [tan− 1 ( x + 1) − tan− 1 ( c + 1)]


2
x2 → ∞
= tan− 1 ( c + 1) − tan− 1 ( − ∞ ) + tan− 1 ( ∞ ) − tan− 1 ( c + 1)
= − ( − π / 2) + π / 2 = π.
Hence the given integral is convergent and its value is π.
1 dx
Problem 10: Evaluate
0 x3


Solution: Here the integrand ( i. e., 1 / x3 ) becomes infinite at the lower limit x = 0.
1
1 dx 1 dx  1   1 1 

∫0 = lim
x3 ε → 0 ∫0 + ε = lim  − 
x3 ε → 0  2 x2  ε
= lim  − +
ε → 0  2 2ε2 
1
= − + ∞ = ∞.
2
Hence the limit does not exist finitely. Therefore the given integral diverges to ∞ and its
value does not exist.
1 dx
Problem 11: Evaluate
0 1− x


Solution: Here the integrand [ i. e., 1 / (1 − x )] becomes infinite at the upper limit
( i. e., x = 1).
R-179

1 dx 1− ε dx

∫0 = lim
1− x ε→ 0 ∫0 = lim [ − log (1 − x )] 10 − ε
1− x ε→ 0
lim
= [ − log ε + 0 ] = − [ − ∞ ] = ∞.
ε→ 0
Hence the limit does not exist finitely and therefore the given integral does not exist i. e.,
is divergent.
1 dx
Problem 12: Evaluate

− 1 x2 /3

(Gorakhpur 2011)
Solution: Here the integrand becomes infinite at x = 0 and 0 lies between − 1 and 1.
1 dx 0 dx 1 dx

∫− 1 x2 /3
=
∫ −1 x2 /3
+
∫0 x2 /3
0 −ε dx 1 dx
= lim
∫ −1 ∫ ε′ + 0
+ lim
ε→ 0 2 /3
x ε′ → 0 x2 /3

= lim [3 x1 /3 ] −− 1ε + lim [3 x1 /3 ] 1ε′


ε→ 0 ε′ → 0
lim lim
= [ − 3ε1 /3 + 3] + [3 − 3 ( ε′ )1 /3 ]
ε→ 0 ε′ → 0
= 3 + 3 = 6.
Hence the given integral is convergent if n > 0 and is divergent if n ≤ 0.

Comprehensive Problems 2
∞ sin2 x
Problem 1: Show that the integral
∫π x2
dx is convergent.

Solution: Proceed as in Ex. 10 of Text Book, by taking a = π.

∞ cos x
Problem 2 (i): Test the convergence of the integral
∫0 1 + x2
dx.

(Purvanchal 2010; Bundelkhand 11; Rohilkhand 12; Gorakhpur 12)


Solution: Proceed as in Ex. 9 of Text Book by taking m = 1 and a = 1.

∞ sin x
Problem 2(ii): Test the convergence of the integral
∫π x2
dx.

sin x
Solution: Let f ( x ) = ⋅ Then f ( x ) is bounded in the interval ( π, ∞ ). Take
x2
1
g ( x) = ⋅ Then g ( x ) is positive in the interval ( π, ∞ ).
x2

We have sin x
| f ( x )| =   = |sin x | ≤ 1 , since |sin x|≤ 1.
 x 
2
x2 x2
R-180
∞ sin x ∞ dx
∴ by comparison test,
∫π x2
dx is convergent if
∫π x2
is convergent.

∞ dx
But the comparison integral
∫π x2
is convergent because here n = 2 which is > 1.

∞ sin x
Hence
∫π x2
dx is also convergent.

∞ sin mx
Problem 2(iii): Test the convergence of the integral
∫0 x2 + a2
dx
(Garhwal 2008)
sin mx 1
Solution: Here f ( x ) = . Let g ( x ) = ⋅
x2 + a2 x2 + a2
Obviously g ( x ) is positive in the interval (0, ∞ ).
sin mx |sin mx | 1
We have | f ( x )| = = ≤ , since |sin mx |≤ 1.
2 2 2 2
x + a x + a x + a2
2

Thus | f ( x )|≤ g ( x ) when x ≥ 0.


∞ sin mx ∞ dx
∴ by comparison test,
0 ∫
x2 + a2
dx is convergent if
∫0 x + a2
2
is convergent.

∞ ∞ x
lim  tan−1 
dx dx 1 x
But
∫0 2
x + a 2
= lim
x→ ∞ ∫0 2
x + a 2
=
x → ∞  a a 0

1 π
lim  tan−1 − 0  = ⋅ ⋅
1 x
=
x → ∞  a a  a 2

= a definite real number.


∞ dx

∫0 x2 + a2
is convergent

∞ sin mx
Hence
∫0 x2 + a2
dx is also convergent.

∞ x3
Problem 2(iv): Test the convergence of
∫0 ( x2 + a2 )2
dx.

x3
Solution: Let f ( x ) = ⋅ Then f ( x ) is bounded in the interval (0, ∞ ).
( x + a2 )2
2

Let b > 0.
We can write
∞ x3 b x3 ∞ x3
∫0 ( x2 + a2 )2
dx =
∫0 ( x2 + a2 )2
dx +
∫b ( x2 + a2 )2
dx.

The first integral on the right hand side is a proper integral because the interval of
integration (0, b ) is finite and the integrand f ( x ) is bounded in this interval. So we need
∞ x3
to check the convergence of the integral
b ( x + a2 )2
2
dx only.

R-181

We can write
x3 1
f ( x) = = ⋅
x {1 + ( a2 / x2 )}2
4
x {1 + ( a2 / x2 )}2
Take g ( x ) = 1 / x.

lim f ( x) 1
We have = lim = 1, which is finite and non-zero.
x → ∞ g ( x) x → ∞ {1 + ( a / x2 )}2
2

∞ ∞
Therefore
∫b f ( x ) dx and
∫b g ( x ) dx either both converge or both diverge. But the
∞ ∞ 1 ∞
comparison integral
∫b g ( x ) dx i. e.,
∫b x
dx is divergent. Therefore
∫b f ( x ) dx is also

divergent.
∞ x3
Hence
∫0 ( x + a2 )2
2
dx is divergent.

∞ dx
Problem 2(v): Test the convergence of the integral
∫1 3

√ ( x + 1) (Gorakhpur 2012; 15)

1 1
Solution: Let f ( x ) = = ⋅
3 3 /2
√ ( x + 1) x √ {1 + (1 / x2 )}

1 lim f ( x) 1
Take g ( x ) = ⋅ We have = lim = 1, which is finite
x3 /2 x → ∞ g ( x) x → ∞ √ {1 + (1 / x3 )}
∞ ∞
and non-zero. Therefore
∫1 f ( x ) dx and
∫1 g ( x ) dx are either both convergent or

∞ ∞ dx
both divergent. But the comparison integral
∫1 g ( x) dx i. e., ∫1 x3 /2
is convergent

∞ ∞ dx
because here n = 3 / 2 which is > 1. Hence
∫1 f ( x ) dx i. e.,
∫1 3
√ ( x + 1)
is also convergent.

∞ 1 − cos x
Problem 2(vi): Test the convergence of the integral
∫0 x2
dx.

Solution: We have
∞ 1 − cos x ∞ 2 sin2 ( x / 2)
∫0 x2
dx =
∫0 x2
dx

∞ 2 sin2 t
=
∫0 4t2
⋅ 2dt .
putting
x
= t so that dx = 2 dt
2
∞ sin2 t
=
∫0 t2
dt.

Now proceed as in Ex. 10 of Text Book.


R-182

dx ∞
Problem 2(vii): Test the convergence of
∫2
√ ( x2 − x − 1)
;

1 1
Solution: Let f ( x) = = ⋅
√ ( x − x − 1) √ {( x − 1 )2 − 5 }
2
2 4
Obviously f ( x ) is bounded in the interval (2, ∞ ).
1 1
We can write f ( x ) = ⋅ Take g ( x ) = ⋅
x √ {1 − (1 / x ) − (1 / x2 )} x
We have
lim f ( x) 1
= lim = 1,
x → ∞ g ( x ) x → ∞ √ {1 − (1 / x ) − (1 / x2 )}
which is finite and non-zero. Therefore
∞ ∞
∫2 f ( x ) dx and
∫2 g ( x ) dx

either both converge or both diverge.


But the comparison integral
∞ ∞ 1

2 ∫g ( x ) dx i. e.,
2 x ∫
dx is divergent because here n = 1. Hence

∞ ∞ dx
∫2 f ( x ) dx i. e.,
∫2 √ ( x2 − x − 1)
is also divergent.

∞ dx
Problem 2(viii): Test the convergence of the integral
∫2 √ ( x2 − 1)

Solution: Proceed exactly as in problem 2 (vii). Here also the given integral is
divergent.
∞ x2 dx
Problem 2(ix): Test the convergence of the integral
∫0 (1 + x )3
.

x2
Solution: Let f ( x ) = . Then f ( x ) is bounded in the interval (0, ∞ ). Let b > 0.
(1 + x )3
We can write
∞ x2 b x2 ∞ x2
∫0 (1 + x ) 3
dx =
∫0 (1 + x ) 3
dx +
∫b (1 + x )3
dx.

The first integral on the right hand side is a proper integral because the interval of
integration (0, b ) is finite and the integrand f ( x ) is bounded in this interval. So we need
∞ x2
to check the convergence of the integral
b (1 + x )3 ∫
dx only.

2
x 1
We can write f ( x ) = =
3 3
x (1 / x + 1) x (1 / x + 1)3
1
Take g ( x) = ⋅
x
R-183

f ( x) 1
We have lim = lim = 1,
x → ∞ g ( x ) x → ∞ (1 / x + 1)3
which is finite and non-zero.
∞ ∞
Therefore
∫b f ( x ) dx and
∫b g ( x ) dx either both converge or both diverge. But the
∞ ∞1 ∞
comparison integral
∫b g ( x ) dx i. e.,
∫b x
dx is divergent. Therefore
∫b f ( x ) dx is also

divergent.
∞ x2
Hence
∫0 (1 + x )3
dx is divergent.

∞ x3 /2
Problem 2(x): Text the convergence of the integral
∫0 2 2
(b x + c )
dx

x3 /2
Solution: Let f ( x ) = ⋅ Then f ( x ) is bounded in the interval (0, ∞ ).
2 2
b x + c
Let b > 0.
We can write
∞ x3 /2 b x3 /2 ∞ x3 /2
∫0 2 2
(b x + c )
dx =
∫0 2 2
(b x + c )
dx +
∫b 2 2
(b x + c )
dx

The first integral on the right hand side is a proper integral because the interval of
integration (0, b ) is finite and the integrand f ( x ) is bounded in this interval. So we need
∞ x3 /2
to check the convergence of the integral
b ( b2 x2 + c )
dx only.

We can write
x3 /2 x3 /2 1
f ( x) = = = .
b2 x2 + c x2 ( b2 + c / x2 ) x1 /2 ( b2 + c / x2 )
1
Take g ( x) = .
x1 /2
f ( x) 1 1
We have lim = lim = ⋅
x → ∞ g ( x ) x → ∞ b2 + c / x2 b2
which is finite and non-zero.
∞ ∞
Therefore
∫b f ( x ) dx and
∫b g ( x ) dx either both converge or both diverge. But the
∞ ∞ 1 ∞
comparison integral
∫b g ( x ) dx i. e.,
∫b x 1 /2
dx is divergent. Therefore
∫b f ( x ) dx is

also divegent.
∞ x3 /2
Hence
∫0 2 2
(b x + c )
dx is divergent.
R-184

Problem 3: Show that the following integrals are convergent.


∞ x2 ∞ dx
(i )
∫0 2
(a + x ) 2 2
dx ; ( ii )
∫1 (1 + x ) √ x

Solution: (i) Let b > 0. Then we have


∞ x2 b x2 ∞ x2
∫0 2
(a + x ) 2 2
dx =
∫0 2
(a + x ) 2 2
dx +
∫b ( a + x2 )2
2
dx.

The first integral on the right hand side is convergent because it is a proper integral. So
we need to check the convergence of
∞ x2
∫b ( a2 + x2 )2
dx.

x2
Let f ( x ) = ⋅ Then f ( x ) is bounded in the interval ( b, ∞ ). Take µ = 4 − 2 = 2.
( a + x2 )2
2

Then
lim x µ f ( x ) = lim x2 ⋅ x2 1
= lim = 1,
x→ ∞ x→ ∞ ( a + x2 )2 x → ∞ {1 + ( a2 / x2 )}2
2

which is a definite real number.


∞ x2 ∞ x2 dx
Since µ > 1, therefore by µ-test
∫b (a + x )2 2 2
dx is convergent. Hence
∫0 ( a + x2 )2
2
is

also convergent because it is the sum of two convergent integrals.


3
( ii ) Apply µ-test by taking µ = ⋅
2
∞ x3 /2 dx
Problem 4: Test the convergence of
∫b √ ( x4 − a4 )
, where b > a.

x3 /2
Solution: Let f ( x ) = ⋅ Then f ( x ) is bounded in the interval ( b, ∞ ). Take
4
√( x − a4)
3 1
µ =2− = ⋅ Then
2 2
lim x µ f ( x ) = lim x1 /2 ⋅ x3 /2
x→ ∞ x→ ∞ x √ {1 − ( a 4 / x 4 )}
2

lim 1
= = 1,
x → ∞ √ { 1 − ( a 4 / x 4 )}

which is finite and non-zero.


Since µ < 1, therefore by µ-test the given integral is divergent.

∫a xn − 1 e − x
Problem 5: Show that the integral dx is convergent, where a > 0.

Solution: Proceed exactly in the same way as in Ex. 17 of Text Book.


R-185

1 dx
Problem 6(i): Test the convergence of the integral
∫0 x3 (1 + x2 )

1
Solution: In the given integral the integrand f ( x ) = is unbounded at the
x3 (1 + x2 )
lower limit of integration x = 0. Take g ( x ) = 1 / x3 .
lim f ( x) 1
Then = lim = 1,
x → 0 g ( x ) x → 0 1 + x2

which is finite and non-zero. Therefore, by comparison test,


1 1
∫0 f ( x ) dx and
∫0 g ( x ) dx
1 dx
either both converge or both diverge. But the comparison integral
∫0 x3
is divergent

because here n = 3 which is > 1.


1 dx
Hence the given integral
∫0 x (1 + x2 )
3
is also divergent.

1 dx
Problem 6(ii): Test the convergence of the integral
∫0 ( x + 1) √ (1 − x2 )

1
Solution: In the given integral the integrand f ( x ) = is unbounded at
( x + 1) √ (1 − x2 )
the upper limit of integration x = 1.
1
Take g ( x) = ⋅
√ (1 − x2 )

Then lim f ( x ) = lim 1 1


= , which is finite and non-zero.
x → 1 g ( x) x → 1 x + 1 2
Therefore, by comparison test
1 1
∫0 f ( x ) dx and
∫0 g ( x ) dx either both converge or both diverge.

1 1 dx lim 1− ε dx
But
∫0 g ( x ) dx =
∫0 √ (1 − x ) 2
=
ε→ 0 ∫0 √ (1 − x2 )

= lim [sin−1 x ] 10 − ε = lim [sin− 1 (1 − ε)]


ε→ 0 ε→ 0
= sin− 1 1 = π / 2,
which is a definite real number.
1

∫0 g ( x ) dx is convergent.

1 dx
Hence
∫0 ( x + 1) √ (1 − x2 )
is also convergent.
R-186

π /2 cos x
Problem 6(iii): Test the convergence of the integral
∫0 x2
dx.

cos x
Solution: In the given integral the integrand f ( x ) = is unbounded at the lower
x2
limit of integration x = 0. Take g ( x ) = 1 / x2 .
lim f ( x) lim  cos x 2  lim
Then =  2 ⋅x  = cos x = 1,
x → 0 g ( x) x → 0  x  x→0
which is finite and non-zero.
π /2 π /2
Therefore, by comparison test,
∫0 f ( x ) dx and
∫0 g ( x ) dx either both converge

or both diverge.
π /2 π /2 1 lim 1 π /2
But
∫0 g ( x ) dx =
∫0 x2
dx =
ε→ 0 ∫ε
x2
dx
π /2
lim  1 
= lim  − +  = ∞.
2 1
= −
ε → 0  x  ε ε → 0  π ε 
π /2

∫0 g ( x ) dx is divergent.
π /2 cos x
Hence the given integral
∫0 x2
dx is also divergent.

π /4 1
Problem 6(iv): Test the convergence of
∫0 √ ( tan x )
dx.
(Agra 2012)
Solution: Here the integrand f ( x ) = 1 / √ (tan x ) is unbounded at x = 0.
1
Take µ = ⋅
2
lim x µ . f ( x ) = lim x1 /2 ⋅ 1
We have
x→0 x→0 √ (tan x )
 x 
= lim   ⋅ √ (cos x ) = 1.1 = 1 .
x→0  sin x 
Since 0 < µ < 1, therefore by µ-test the given integral is convergent.
π /2 sin x
Problem 6(v): Test the convergence of
0 x1 + n ∫
dx.

π /2 sin x π /2 sin x
  ⋅ 1 dx.
Solution: We have
0 ∫x1+ n
dx =
0
 ∫

 x  xn
sin x
Now lim = 1. Therefore for n ≤ 0, the integrand is bounded throughout the
x→0 x
interval (0, π / 2) and so the given integral is a proper integral and hence it is convergent
if n ≤ 0.
If n > 0, the integrand is unbounded only at x = 0. In this case, we have
µ
lim x µ sin x = lim  sin x  ⋅ x 
  
x→0 x n + 1 x→0  x  x n
R-187

 sin x  
= lim  x µ − n ⋅  
x→0   x  
= 1 if µ − n = 0 i. e., µ = n.
∴ by µ-test if 0 < µ < 1 i. e., 0 < n < 1, the given integral is convergent and if µ ≥ 1 i. e., n ≥ 1,
the given integral is divergent.
Hence the given integral is convergent if n < 1 and divergent if n ≥ 1.
Problem 7: Test the convergence of the following integrals :
∞ dx ∞ dx
(i)

0 x 1 /3 1
(1 + x )/2
(ii)
0
x (1 + x2 )

∞ x1 /2 ∞ x
(iii)
∫0 2
x +4
dx (iv)
∫0 1 + x2
sin x dx

Solution: (i) Let a > 0. Then we have


∞ dx a dx ∞ dx
∫0 x1 /3
(1 + x 1 /2
)
=
∫0 x 1 /3
(1 + x 1 /2
)
+
∫a x1 /3 (1 + x1 /2 )

The first integral on the right hand side is convergent because it is a proper integral. We
observe that in this integral the range of integration (0, a) is finite and the integrand
1
is bounded throughout the interval (0, a). So we need to check the
x1 /3 (1 + x1 /2 )
∞ dx
convergence of

a x1 /3 (1 + x1 /2 )
only.

1 1
Let f ( x) = = . Then f ( x ) is bounded in the
x1 /3 (1 + x1 /2 ) x5 /6 (1 / x1 /2 + 1)
interval ( a, ∞ ).
1 f ( x) 1
Take g ( x ) = ⋅ We have lim = lim = 1,
x5 /6 x → ∞ g ( x ) x → ∞ (1 / x1 /2 + 1)
which is finite and non-zero.
∞ ∞
Therefore
∫a f ( x ) dx and
∫a g ( x ) dx either both converge or both diverge. But the
∞ b dx ∞
comparison integral
∫0 g ( x ) dx i. e.,
∫a 5 /6
x
is divergent. Therefore
∫a f ( x ) dx is also

divergent.
∞ dx
Hence
∫0 x1 /3 (1 + x1 /2 )
is divergent.

(ii) Proceed as in part (i). Ans. Divergent.


(iii) Proceed as in part (i). Ans. Divergent.
(iv) Proceed as in problem 2(i). Ans. Convergent.
∞ sin x
Problem 8: Examine the convergence of the integral
0 x3 /2
dx

Solution: Proceed as in problem 2(i). Ans. Convergent.
R-188
∞ 2 x2
Problem 9: Show that the integral
∫0 e− a cos bx dx is absolutely convergent.

Solution: We have
∞ 2 2 ∞ 2 2 ∞ − a2 x2
∫0 e − a x cos bx dx ≤
  ∫0 e − a x  dx =
  ∫0 e dx

c 2 x2 ∞ 2 x2
=
∫0 e− a dx +
∫c e− a dx, where c > 0.
c 2 x2
But the integral
∫0 e− a dx is a proper integral and hence convergent.
∞ − a2 x2
Also
∫c e dx is convergent by µ-test, for we have
µ
lim x µ e − a2 x2
= lim x
x→ ∞ x→ ∞ a4 x4
1 + a2 x2 + +…
2!
=0, for all values of µ.
∞ − a2 x2
Taking µ > 1, we see that
∫c e dx is convergent.
∞ − a2 x2

∫0 e dx is convergent.

 − a2 x2
cos bx dx is convergent, by comparison test.
Hence
∫0 e 
From the above discussion it follows that the given integral is absolutely convergent.
∞ cos xy
Problem 10: Evaluate
∫0 1 + x2 dx with the help of differentiation w.r.t., parameter.
∞ ∞ cos xy
Solution: Let φ (α ) =
∫0 f ( x, α ) dx =
∫0 1 + x2 dx …(1)

cos xy
We have f ( x, α ) =
1 + x2

− x sin xy
⇒ fα ( x, α ) = …(2)
1 + x2

cos xy 1 1
Now ≤ ≤
2 2
1+ x 1+ x x2

x sin xy | x|
and ≤ ⋅
2
1+ x 1 + x2
R-189
∞ dx ∞ | x|
Here
∫0 2
x
and
∫0 1 + x2 dx are convergent.
∞ ∞

∫0 f ( x, α ) dx and
∫0 fα ( x, α ) dx

are uniformly convergent by Weierstrass’s M test.


⇒ φ′ (α ) exists.
We have

φ′ (α ) =
∫0 fα ( x, α ) dx

∞ − x sin xy
=
∫0 1 + x2
dx

∞ sin xα ∞ sin xα
=−
∫0 x
dx +
∫0 x (1 + x2 )
dx …(3)

α α ∞ cos xα
Also
∫0 φ (α ) dα = ∫0 dα
∫0 1 + x2
dx

∞ α cos xα
=
∫0 dx ∫0 1 + x2

∞ sin x α
=
∫0 x (1 + x2 )
dx. …(4)

Proceeding as in example 2, we have


∞ sin xα π
0 x ∫ dx = , for α > 0
2
…(5)

Putting the values of (4) and (5) in (3), we get


π α
φ′ (α ) = − +
2 0
φ (α ) dα, [α > 0 ]
∫ …(6)

Since φ (α ) is continuous, differentiating (6), we have


φ′ ′ (α ) = φ (α )
⇒ φ (α ) = A e α + B e − α , …(7)
where A and B are constants
⇒ φ (0 ) = A + B …(8)
Putting α = 0 in (1) and (6), we get
π
φ (0 ) = …(9)
2
π
and φ′ (0 ) = −
2 …(10)
Differentiating (7) w.r.t. α and putting α = 0, we get
φ′(0 ) = A − B …(11)
R-190

From (8), (9), (10) and (11), we get


A = 0 and B = π /2.
Substituting these values in (7), we get
π
φ (α ) = e − α , where α > 0
2
∞ cos xy π
or

0 1 + x2
dx = e − α .
2
∞ − x2
Problem 11: Evaluate
∫0 e cos αx dx using differentiation with respect to parameter.
∞ − x2 ∞
Solution: Let F(α ) =
∫0 e cos αx dx =
∫0 f ( x, α )dx …(1)
2
where f ( x, α ) = e − x cos αx
2
Now fα ( x, α ) = − xe − x sin αx
2 2
We have |xe − x sin αx|≤ xe − x
2 2
and |e − x cos αx|≤ e − x [∵|cos α x|≤ | ]
∞ − x2 ∞ − x2
∫0 e dx and
∫0 xe dx are both convergent.
∞ ∞
Therefore, by Weierstrass’s M–test both
∫0 f ( x, α )dx and
∫0 fα ( x, α )dx are uniformly
convergent.
Thus, F ′(α ) exists.
∞ ∞ − x2
We have F ′(α ) =
∫0 fα ( x, α )dx = ∫0 − xe sin αx dx
∞ ∞ − x2
α
=  e − x sin αx  −
1 2
 2 
0 2 ∫0 e cos αx dx

α
=−
F(α )
2
F ′(α ) α
or =−
F(α ) 2
d α
or {log F(α )} = − .
dα 2

Integrating w.r.t. α, we get


α2
log F(α ) = − + c1, where c1 is constant of integration.
4
2 /4
or F(α ) = c e − α , where c = e c1 …(2)
Putting α = 0 in equation (1) and (2), we get
∞ − x2 π
F(0 ) =
0
e

dx =
2
…(3)
R-191

and F(0 ) = c …(4)


From equations (3) and (4), we get
π
c= .
2
Substituting the above value of c in equation (2), we get
π − α 2 /4
F(α ) = e
2
∞ − x2 π − α 2 /4
or
∫0
e cos αx dx =
2
e .

Hints to Objective Type Questions

Multiple Choice Questions


∞ dx
1. (d). We know that the integral
∫a xn
, where a > 0, is convergent if and only if

n > 1. See article 4, Theorem of Text Book.


b dx
2. (a). We know that the integral
∫ a ( x − a)n is convergent if and only if

n < 1. See article 10, Theorem of Text Book.


3. (c). See Ex. 39 of Text Book.

∫0
n−1
4. (c). See Ex. 42 of Text Book. The integral x e− x
dx is convergent if n > 0

and is divergent if n ≤ 0.
5. (a). See Ex. 10 of Text Book.

Fill in the Blank(s)


1. proper integral. See article 1 of Text Book. Definition 4 of a proper integral.
2. bounded. See article 1 of Text Book, Definition 5 of an improper integral.
3. improper integral of the first kind. See article 1 of Text Book, Definition 6.
4. second. See article 1, Definition 7 of an improper integral of the second kind.
∞ dx
5.

first. In the definite integral
0 1 + x2
, the range of integration is [0, ∞[ which

1
is an infinite interval and the integrand is bounded in [0, ∞[ because
1 + x2
1
≤ 1, V x ∈ [0, ∞[.
1 + x2
So this integral is an improper integral of the first kind.
R-192
4 dx
6. second. In the definite integral
∫0 ( x − 2) ( x − 3)
, the range of integration [0, 4]

1
is a finite inter val but the integrand is unbounded at the points
( x − 2) ( x − 3)
x = 2 and x = 3 of this inter val. So this integral is an improper integral of the
second kind.
7. n > 1. See article 4, Theorem of Text Book.
8. n ≥ 1. See article 10, Theorem of Text Book.
9. n > 0. See Ex. 42 of Text Book.
10. 0. See Ex. 41 of Text Book.
True or False
1. F. See problem 2 of Comprehensive Problems 1
∞ cos x
2. T . The integral

a 1 + x2
dx is convergent. See Ex. 9 and problem 2 (i) of

Comprehensive Problems 2 of Text Book.


∞ dx
3. F. The integral
a ∫xn
, where a > 0, is divergent when n ≤ 1. See article 4,

Theorem of Text Book.


∞ dx
4. F. The integral

a ( x − a)n
is convergent when n < 1. See article 10, Theorem of

Text Book.
1 dx
5. F. See problem 6 (i) of Comprehensive Problems 2. The integral
∫0 x3 (1 + x2 )
is divergent.
6. T . See Ex. 29 of Text Book.
7. T . See problem 2 (v) of Comprehensive Problems 2.
8. F. See problem 2 (viii) of Comprehensive Problems 2.

❍❍❍
Krishna’s

MATHEMATICAL METHODS
C hapters

1. The Laplace Transform

2. The Inverse Laplace Transform

3. Applications of Laplace Transform


4. Fourier Transforms

5. Finite Fourier Transforms


M-3
Chapter-1
The Laplace Transform

Comprehensive Problems 1
Problem 1(i): Find L {2t3 − 6t + 8}.
Solution: We have L {2t3 − 6t + 8} = 2L {t3 } − 6L {t} + 8L {1}
3 1 1
=2 −6 + 8⋅ , p> 0
p4 p2 p
12 6 8
= − + , p > 0.
p4 p2 p
Problem 1(ii): Find L {sin t cos t}.
1 1
Solution: We have L {sin t cos t} = L { sin 2t} = L {sin 2t}
2 2
1 2 1
= ⋅ , p> 0 = , p > 0.
2 p2 + 4 2
p +4
Problem 1(iii): Find L {cosh2 2t}.
1
Solution: We have L { cosh2 2t} = L { (1 + cosh 4t)}
2
1 1 1 p 
= [ L {1} + L { cosh 4t}] =  +  , p> 4
2 2  p p2 − 42 

p2 − 8
= , p > 4.
p ( p2 − 16)
Problem 1(iv): Find L {6 sin 2t − 5 cos 2t}.
Solution: L {6 sin 2t − 5 cos 2t} = 6L {sin 2t} − 5L { cos 2t}
2 p 12 − 5 p
= 6⋅ − 5⋅ , p> 0 = , p > 0.
2
p +2 2 2
p +2 2 p2 + 4
Problem 2(i): Find L {3 cosh 5t − 4 sinh5t}.
Solution: L {3 cosh 5t − 4 sinh 5t} = 3L{ cosh 5t} − 4L {sinh 5t}
p 5
= 3⋅ − 4⋅ , p> 5
p − 52
2 p2 − 52
3 p − 20
= , p > 5.
p2 − 25
M-4

Problem 2(ii): Find L {7e2 t + 9e −2 t + 5 cos t + 7t 3 + 5 sin 3t + 2 }.


Solution: We have
L {7e2 t + 9e −2 t + 5 cos t + 7t 3 + 5 sin 3t + 2}
= 7L{ e2 t } + 9L{ e −2 t } + 5L{ cos t} + 7L{ t 3 } + 5L{sin 3t} + 2 L{1}
1 1 p 3! 3 1
=7⋅ + 9⋅ + 5⋅ + 7⋅ + 5⋅ + 2⋅ ,
p−2 p+ 2 p + 12
2 p4 p2 + 32 p

p > 2, p > − 2 and p > 0 i.e., p > 2

16 p − 4 5p 42 + 2 p3 15
= + + + , p > 2.
p2 − 4 p2 + 1 p4 p2 + 9

Problem 2(iii): Find L {2e3 t − e −3 t }. (Rohilkhand 2006)


Solution: L {2e3 t − e −3 t } = 2 L { e3 t } − L { e −3 t } = 2 ⋅
1 1
− ,
p−3 p+ 3
p > 3 and p > − 3 i. e. p > 3
p+ 9
= , p > 3.
p2 − 9
Problem 2(iv): Find the Laplace Transform of the function F ( t) = ( e at − 1) / a.
Solution: L { F ( t)} = L {( e at − 1)/a} = (1/a) [ L { e at } − L { 1}]
1 1 1
=  −  , p > 0 and p > a
a  p − a p
1
= , p > 0 if a < 0 and p > a if a > 0.
p ( p − a)
e t , 0 < t ≤ 1
Problem 3(i): Find L { F( t)}, if F ( t) = 
 0, t > 1.
∞ −pt 1 −pt t ∞ −.pt
Solution: L {F ( t)} =
∫0 e F( t) dt =
∫0e e dt +
∫1 e ⋅ 0 dt

1
1 −(p −1) t  e −(p −1) t 
[1 − e −(p −1) ], p ≠ 1.
1
=
∫ 0
e dt =  −
 p − 1 
 =
( p − 1)
0
0, 0 < t < 2
Problem 3(ii):Find L { F ( t)}, where F ( t) = 
4, t > 2.
∞ −pt
Solution: We have L {F( t)} =
∫0 e F ( t) dt

2 −pt ∞ −pt
=
∫0 e F ( t) dt +
∫2 e F ( t) dt

2 −pt ∞ −pt ∞
e −pt dt
=
∫0 e ⋅ 0 dt +
∫2 e ⋅ 4 dt = 4
∫2
M-5


 e −pt  4 −pt 4 −2 p
= 4  = − lim e + e
 − p  2 p t →∞ p

⋅ 0 + e −2 p , provided p > 0.
4 4
=−
p p
4 −2 p
Hence L {F ( t)} = e , p > 0.
p

Problem 4(i): Find the Laplace transform of the following function :


 x / a, 0 < x < a
f ( x) = 
 1, x > a.

∞ −px
Solution: We have L { f ( x )} =
∫a e f ( x ) dx

a −px ∞ −px
=
∫0 e f ( x ) dx +
∫a e f ( x ) dx

ax ∞ −px
e −px dx +
=
∫0 a ∫a e ⋅1 dx

a
1  e −px  1 a e −px ∞ −px
= − ⋅ x −
a  − p  a 0 − p ∫
⋅ 1dx +
0
e dx

0
a ∞
1  a e −ap  1  e −px   e −px 
= − +   +  
a  p  ap  − p   − p  a
0
e −ap
( e −ap − 1) − lim e − px + e −ap
1 1 1
=− −
p ap2 p x→∞ p

(1 − e −ap ) − ⋅ 0, provided p > 0.


1 1
=
ap2 p

(1 − e −ap ), p > 0.
1
Hence L { f ( x )} =
ap2
0, 0 < t < 1

Problem 4(ii): Find L {F ( t)}, where F( t) =  t, 1 < t < 2 (Lucknow 2010)
0, t > 2.

Solution: Here F ( t) is not defined at t = 0, t = 1 and t = 2.


∞ −pt
∴ L {F( t)} =
∫0 e F ( t) dt

1 −pt 2 −pt ∞ −pt


=
∫0 e ⋅ 0 dt +
∫1 e ⋅ t dt +
∫2 e ⋅ 0 dt
M-6

2
2 −pt  e −pt e −pt 
=

1
e ⋅ t dt =  − t
 p
− 
p2 1
, p≠0

 2 1  −2 p  1 1  −p
=− +  e +  +  e , p ≠ 0.
 p 2   p 
 p   p2 
Problem 5(i): Find Laplace Transform of the function F ( t), where
sin t, 0 < t < π
F( t) = 
 0, t > π. (Lucknow 2007)

∞ −pt
Solution: L {F( t)} =
∫0 e ⋅ F( t) dt

π −pt ∞ −pt
=
∫0 e ⋅ sin t dt +
∫π e ⋅ 0 dt

π
 e −pt  e− p π + 1
= ( − p sin t − cos t) = ⋅
2 p2 + 1
 p + 1  0

e t , 0 < t<5
Problem 5(ii):Find L {F ( t)}, if F ( t) = 
3, t > 5.
∞ −pt 5 −pt ∞ −pt
Solution: L {F( t)} =
∫0 e ⋅ F( t) dt =
∫0 e ⋅ e t dt +
∫5 e ⋅ 3 dt

5 ∞
5 −( p −1)t ∞ −pt  e −( p −1)t   e −pt 
=
0 ∫
e dt + 3
5
e dt =  −
∫  + 3 −
 ( p − 1)  0 

p 
5
−5( p −1)
1− e 3 −5 p
= + e , p > 0.
p −1 p

1, 0 < t<2


Problem 6(i): Find L {F ( t)}, if F ( t) = 
t, t > 2.
∞ −pt 2 − pt ∞ −pt
Solution: L {F( t)} =
∫0 e ⋅ F( t) dt =
∫ 0 1⋅ e dt +
∫ 2 te dt

2 ∞
 e −pt    e −pt   e −pt  
=  +   ⋅t −


 2  
 ⋅1
 − p 0   − p   p   2

e −2 p 1 1 lim 1 lim e −2 p
+ e −2 p − e −2 pt +
t 2
=− + −
p p p t → ∞ e pt p p2 t → ∞ p2

[1 + e −2 p ] +
1 −2 p 1
lim e −2 pt .
1 t 1
= e − lim −
p p2 p t→∞ e pt p2 t→∞
M-7

Now if p > 0, we have lim e −pt = 0


t →∞

and
lim t
=0 Form ∞ 
t → ∞ e pt  ∞ 

lim 1
= = 0.
t → ∞ pe pt

[1 + e −2 p ] +
1 1 −2 p
∴ L {F ( t)} = e , p > 0.
p p2

t, 0 < t<4


Problem 6(ii):Find L { F ( t)} if F ( t) = 
5, t > 4.
Solution: We have
∞ −pt 4 −pt ∞ −pt
L {F( t)} =
∫ 0 F( t) e dt =
∫0 t e dt +
∫4 5 e dt

4 ∞
 e −pt 1 −pt   5 
= − t − e  +  − e −pt 
 p p2  0  p 4

= − e −4 p −
1 −4 p
− 1) + e −4 p , p > 0
4 5
(e
p p2 p

1 + ( p − 1) e −4 p
= , p > 0.
p2
2 (1 − e − π p ) sin 2t, 0 < t< π
Problem 7(i): Prove that L { H ( t)} = , where H ( t) = 
p2 + 4  0, t > π.
∞ −pt
Solution: We have L {H( t)} =
∫0 e H ( t) dt

π −pt ∞ −pt
=
∫0 e H ( t) dt +
∫π e H ( t) dt

π −pt ∞ −pt π −pt


=
∫0 e ⋅ sin2t dt +
∫π e ⋅ 0 dt =
∫0 e sin 2t dt

π
 e −pt 
= ( − p sin 2t − 2 cos 2t)
2
 p + 4  0

e −pπ 1
= ( − 2 cos 2 π ) − ⋅ ( − 2 cos 0 )
p2 + 4 p2 + 4
2 e −pπ
(1 − e −πp ).
2 2
= − =
p2 + 4 p2 + 4 p2 + 4
M-8

 1  1
Problem 7(ii): Prove that L  = ⋅
 √ ( π t )  √ p (Avadh 2014)
 1  ∞ −pt 1
Solution: We have L 
 √ ( π t )
=
 0
e
∫ ⋅
√ ( πt)
dt

1 ∞ −pt 1 1 ∞ −pt −1 /2
=
√π 0 ∫
e ⋅
√t
dt =
√π 0
e
∫t dt

1 Γ( 2 )
1
1 ∞ −pt (1 /2)−1
=
√π 0∫e t dt = ⋅
√ π p1 /2
, p> 0

1 √π 1
= ⋅ = , p > 0.
√ π π1 /2 √ p
 cos √ t  π  −1 /4 p
Problem 8: Show that L   =
  e .
 √t   p (Kashi 2014; Purvanchal 07)
cos √ t 1  1 2 1 1 
Solution: We have = 1 − ( √ t) − ( √ t)4 − ( √ t)6 + ...
√t √t  2! 4! 6! 
=t −1 / 2 −
1 1 /2
t +
1 3 /2
t −
1 5 /2
t + ...
2! 4! 6!
 cos √ t  −1 /2 } − 1 L { t1 /2 } + 1 L { t 3 /2 } − 1 L { t 5 /2 } + ...
∴ L  = L {t
 √ t  2! 4! 6!
1 3 5 7
Γ( ) Γ( ) Γ( ) Γ( )
= 2 − 1 2 + 1 2 − 1 2 + ..., p > 0
p1 /2 2 ! p3 /2 4 ! p5 /2 6 ! p7 /2
1 3 1
⋅√π ⋅ ⋅√π
√π 1 2 1
= − + 2 2 ⋅
p1 /2 1.2 p3 /2 1.2.3.4 p 5 /2
5 3 1
⋅ ⋅ ⋅√π
1
−2 2 2 ⋅ + ...
1.2.3.4.5.6 p7 /2

 π   1 1  1  1  
2
13  π  −1 /4 p
=   1 −   +   − + ... =   e .
 p   1!  4 p 2!  4 p 3! 

 p

Problem 9: Show that t 2 is of exponential order 3.


Solution: We have,
lim lim  t 2  lim  2t 
{e −at F( t)} =   =  , [by L’ Hospital Rule]
t→ ∞ t → ∞  e at  t → ∞  ae at 
lim 2
= = 0, if a > 0.
t → ∞ a2 e at
M-9

Therefore F( t) = t 2 is of exponential order.


Now | t 2 | = t 2 < e 3 t for all t > 0.
∴ the given function is of exponential order 3.

Comprehensive Problems 2
Problem 1(i): Find L {t3 e −3 t }. (Kanpur 2008)
Solution: We have,
L { t3 } = 3 !/ p4 = 6 / p4 = f ( p), say.
∴ From first shifting theorem, we have
L {t3 e −3 t } = f ( p + 3) = 6 / ( p + 3)4 .
Problem 1(ii): Find L {e −at t n −1 / ( n − 1) !}

Solution: We have,
 t n −1  1 ( n − 1) ! 1
L = ⋅ = = f ( p), say.
( n − 1)! ( n − 1) ! pn pn
∴ From first shifting theorem, we have
 t n −1 
L e −at
1
 = f ( p + a) = .
 ( n − 1) ! ( p + a)n

Problem 1(iii): Find L {tn e at }. (Avadh 2010)


n!
Solution: We have L {t n} = = f ( p), say
pn +1
∴ From first shifting theorem, we have
n!
L {e at tn} = f ( p − a) = ⋅
( p − a)n + 1
Problem 2(i): Find L {e3 t sin 4t}. (Lucknow 2010)
Solution: We have L {sin 4t} = 4 / ( p2 + 42 ) = 4 / ( p2 + 16) = f ( p), say.
∴ From first shifting theorem, we have
L { e3 t sin 4t} = f ( p − 3) = 4 / {( p − 3)2 + 16} = 4 / ( p2 − 6 p + 25).

Problem 2(ii): Find L {e3 t cos 5t}.


Solution: We have
L { cos 5t} = p / ( p2 + 52 ) = p / ( p2 + 25) = f ( p), say.
∴ From first shifting theorem, we have
( p − 3) p−3
L { e3 t cos 5t} = f ( p − 3) = = ⋅
( p − 3)2 + 25 p2 − 6 p + 34
M-10

Problem 3(i): Find L {e t cos2 t}.


Solution: We have
1 1
L { cos2 t} = L { (1 + cos 2t)} = [ L {1} + L { cos 2t}]
2 2
1 1 p  p2 + 2
=  + 2  = = f ( p), say.
2  p p + 22  p ( p2 + 4)
∴ From first shifting theorem, we have
( p − 1)2 + 2 p2 − 2 p + 3
L { e t cos2 t} = f ( p − 1) = = ⋅
( p − 1) {( p − 1)2 + 4} ( p − 1) ( p2 − 2 p + 5)
Problem 3(ii): Find L {e − 4 t cosh 2t}.
Solution: We have
L { cosh 2t} = p / ( p2 − 22 ) = p / ( p2 − 4) = f ( p), say.
∴ From first shifting theorem, we have
( p + 4) p+ 4
L { e −4 t cosh 2t} = f ( p + 4) = = .
( p + 4)2 − 4 p2 + 8 p + 12
Problem 4(i): Find L {e − t (3 sin 2 t − 5 cosh 2 t)}.
Solution: We have
2 5p
L {3 sin 2t − 5 cosh 2t} = 3 ⋅ − = f ( p), say.
p2 + 22 p2 − 22
∴ From first shifting theorem, we have
L { e −t (3 sin 2t − 5 cosh 2t)} = f ( p + 1)
6 5 ( p + 1) 6 5 ( p + 1)
= − = − ⋅
( p + 1)2 + 4 ( p + 1)2 − 4 p2 + 2 p + 5 p2 + 2 p − 3
Problem 4(ii): Find L {e − 2 t (3 cos 6t − 5 sin 6t)}.
Solution: We have L {3 cos 6t − 5 sin 6t}
p 6 3 p − 30
= 3⋅ − 5⋅ = = f ( p), say.
2
p +6 2 2
p +6 2 p2 + 36
∴ From first shifting theorem, we have
L { e −2 t (3 cos 6t − 5 sin 6t)} = f ( p + 2)
3 ( p + 2) − 30 3 p − 24
= = .
2
( p + 2) + 36 2
p + 4 p + 40
Aliter: By definition, L { e −2 t (3 cos 6t − 5 sin 6t)}
∞ −pt
⋅ e −2 t (3 cos 6t − 5 sin 6t) dt
=
∫0 e

∞ −( p +2 ) t ∞ −( p +2 )t
=3
∫0 e cos 6t dt − 5
∫0 e sin6t dt
M-11


 3e − ( p + 2) t 
= { − ( p + 2) cos 6t + 6 sin 6t}
2 2
 ( p + 2) + 6  0

 5 ⋅ e −( p +2 )t 
− {− ( p + 2) sin 6t − 6 cos 6t}
2 2
 ( p + 2) + 6  0
3 ( p + 2) 30 3 p − 24
= − = .
2
p + 4 p + 40 2
p + 4 p + 40 2
p + 4 p + 40
Problem 5: Find the Laplace transform of the function ( t + 3)2 . e t .
Solution: We have,
L {( t + 3)2 } = L { t 2 + 6t + 9}
2! 1! 9 2 + 6 p + 9 p2
= + 6⋅ + = = f ( p), say.
p3 p2 p p3
∴ From first shifting theorem, we have
L {( t + 3)2 e t } = f ( p − 1)
2 + 6 ( p − 1) + 9 ( p − 1)2 9 p2 − 12 p + 5
= = .
( p − 1)3 ( p − 1)3
Problem 6: If L { F ( t)} = f ( p), find L { F ( t) cos ωt}.
Solution: We have, L { F ( t)} = f ( p), given.
∴ From first shifting theorem,
1 iωt
L { F ( t) cos ωt} = L {F ( t) ⋅ (e − e −iωt )}
2
= [ L {F ( t) ⋅ e i ω t } − L {F ( t) e −i ω t }] = [ f ( p − iω) − f ( p + iω)].
1 1
2 2
Problem 7: Applying change of scale property, find
(i) L {sinh 3t} and (ii) L { cos 5t}.
Solution: (i) We have,
L {sinh t} = 1 / ( p2 − 1) = f ( p), say.
∴ by the change of scale property, we have
1 p  1 1 3
L {sinh 3t} = f   = ⋅ = .
3  3  3 ( p / 3)2 − 1 p2 − 9
p
(ii) We have, L { cos t} = , p> 0
2
p +1
= f ( p), say.
∴ by the change of scale property, we have
1 p  1 p/5 p
L { cos 5t} = f   = ⋅ = , p > 0.
5  5  5 ( p / 5)2 + 1 p2 + 25
M-12

e t − a , t > a
Problem 8: Find L {G ( t)}, where G ( t) = 
0 , t < a.
Solution: Let F ( t) = e t .
∞ −pt 1
Then L {F ( t)} = L ( e t ) =
∫0 e ⋅ e t dt =
p −1
, p > 1 = f ( p), say.

 F ( t − a) = e t − a , t > a
Now let G ( t) = 
 0 , t < a.
∴ From second shifting theorem, we have
e −ap
L { G ( t)} = e −ap ⋅ f ( p) = , p> 1.
p −1
sin ( t − π / 3), t > π /3
Problem 9: Find L {F ( t)}, where F ( t) = 
 0 , t < π /3.
Solution: Let φ ( t) = sin t.
φ ( t − 1 π ), t > π / 3

Then F ( t) =  3
 0 , t < π / 3.
1
We have L { φ ( t)} = L {sin t} = = f ( p), say.
p2 + 1
∴ From second shifting theorem, we have
L { F ( t)} = e −( π /3)p . f ( p) = e − π p /3
1
, p > 0.
p2 + 1
e −3 /( p + 1)
Problem 10: If L {F ( t)} = (1 / p) e −1 / p , prove that L { e − t F (3t)} = .
p+1
Solution: Given that L {F ( t)} = (1 / p) e −1 / p = f ( p), say.
∴ By the change of scale property
e −1 /( p /3) = e −3 / p = f1( p), say.
1 1 1 1
L {F (3t)} = f ( p / 3) = ⋅
3 3 ( p / 3) p
∴ From first shifting theorem,
L { e −t F (3t)} = f1 ( p + 1) = e −3 /( p + 1) .
1
( p + 1)

Comprehensive Problems 3
Problem 1(i): Find L {t cos at}. (Lucknow 2008)
p
Solution: Since L {cos at} = , p > 0,
p2 + a2
d d  p  p2 − a2
∴ L { t cos at} = − L {cos at} = −  = , p > 0.
dp dp  p + a  (
 2 2 p2 + a2 )2
M-13

Problem 1(ii): Find L {t cosh 3 t}.


Solution: We have, L { cosh 3t} = p /( p2 − 9), p > 3.
d  p  p2 + 9
∴ L { t cosh 3t} = −  2 = 2 , p> 0 ⋅
dp  p − 9  ( p − 9)2

Problem 2(i): Find L {t2 sin at}. (Lucknow 2006)


a
Solution: Since L {sin at} = ,
p2 + a2
d2
∴ L { t2 sin at} = ( − 1)2 L {sin at}
dp2

d2  a  d  − 2ap  2a (3 p2 − a2 )
=   =  = , p > 0.
dp2  p2 + a2  dp ( p2 + a2 )2  ( p2 + a2 )3
Problem 2(ii): Find L {t 2 e2 t }.
1
Solution: Since L { e2 t } = , p > 2.
p−2
d2 d2  1  2
∴ L { t 2 e2 t } = ( − 1)2 L { e2 t } =   = .
dp2 dp 
2 p − 2  ( p − 2)3
6 p4 − 36 p2 + 6
Problem 3(i): Show that L { t3 cos t} = .
( p2 + 1)4
p
Solution: Since L { cos t} = , p > 0,
p2 + 1

d3  p  6 p4 − 36 p2 + 6
∴ L { t3 cos t} = ( − 1)3  = .
 
dp3  p2 + 1 ( p2 + 1)4
n!
Problem 3(ii): Show that L { t n . e at } = , p > a.
( p − a)n +1 (Avadh 2011)
1
Solution: Since L { e at } = , p > a,
p− a
dn  1  ( − 1)n n ! n!
∴ L { tn e at } = ( − 1)n ⋅   = ( − 1)n ⋅ = , p > a.
dpn  p − a  ( p − a)n +1 ( p − a)n +1
Problem 4(i): Find L { t (3 sin 2t − 2 cos 2t)}.
Solution: We have L { 3 sin 2t − 2 cos 2t}
6 2p 6 − 2p
= 3L { sin 2t} − 2L { cos 2t} = − = ⋅
p2 + 4 p2 + 4 ( p2 + 4)
d  6 − 2 p  8 + 12 p − 2 p2
∴ L { t (3 sin 2t − 2 cos 2t)} = ( − 1) ⋅  = ⋅
dp  p2 + 4  ( p2 + 4)2
M-14

Problem 4(ii): Find L.T. of f ( t) = sin α t + t cos α t.


Solution: L { f ( t)} = L { sin αt + t cos αt}
 d 
= L { sin αt} + L { t cos αt} = L { sin αt} +  − L { cos αt}
 dp 
α d  p  α 2
α − p 2
= −   = −
 
p2 + α2 dp  p2 + α2  p2 + α2 ( p2 + α2 )2
(α + 1) p2 + (α − 1) α2
= ⋅
( p2 + α2 )2
2 a ( p − a)
Problem 5: Prove that L {t e at sin at} = ⋅
( p2 − 2ap + 2a2 )2 (Kanpur 2010)
Solution: Let F( t) = sin at
a
L {F( t)} = L {sin at} = = f ( p) say
p2 + a2
d d a 
∴ L {tsin at} = ( −1) f ( p) = ( −1)  
dp dp  p + a 
 2 2

 −2ap  2ap
= − = 2 = f1( p) say
2 2 2 2 2
 ( p + a )  ( p + a )
2 a ( p − a)
∴ L {t e at sin at} = f1( p − a) =
{( p − a)2 + a2 }2
2 a ( p − a)
= ⋅
( p + 2a2 − 2ap)2
2

√π
Problem 6: Given L {sin √ t} = e −1 /4 p , show that
2 p3 /2
 cos √ t   π  −1 /4 p
L =   e .
 √t   p
Solution: Let F ( t) = sin √ t.
cos √ t
Then F ′ ( t) = and F (0 ) = 0.
2 √t
∴ From L { F ′ ( t)} = pL { F ( t)} − F (0 ),
 cos √ t  π −1 /4 p
we have L  = p L {sin √ t} = p⋅ 3 /2 e ,
 2 √t  2p
substituting the given value of L {sin √ t}
1  π  −1 /4 p
=   e .
2  p

 cos √ t   π  −1 /4 p
∴ L =   e .
 √t   p
M-15

Find L 
sinh t 
Problem 7: ⋅
 t 
Solution: Let F ( t) = sinh t.
lim F( t) lim sinh t
Now = = 1.
t→0 t t→0 t
Since L {sinh t} = 1/( p2 − 1) = f ( p),
∞ ∞
L 
sinh t  dx
therefore
 t 
= ∫p f ( x ) dx =
∫p x2 − 1
∞  ∞
x − 1 1 − 1/ x  p −1 1 p+ 1
=  log
1 1 1
=  log  = 0 − log = log ⋅
 2 
x + 1 p  2 1 + 1/ x  p 2 p+ 1 2 p −1

 t sin x 
Problem 8: Find L 
 0 x
∫ dx  .

Solution: First show that
L 
sin t  −1
 = cos p.
 t 
 t  1
Now using the theorem that if L {F ( t)} = f ( p), then  F ( x ) dx  = f ( p),
 0  p

 sin x  1
t
dx  = cot −1 p.
we have L
 0 x
∫  p

Problem 9: Prove that if L { F ( t)} = f ( p), then


∞ F ( t) ∞
∫0 t
dt =
∫0 f ( x ) dx provided that the integral converges.

Solution: From article 20, we have


∞ −pt F ( t) ∞
L 
F ( t) 
 t 
=
0
e
t ∫dt =
p
f ( x ) dx

0 ∞
=
∫ p f ( x) dx + ∫ 0 f ( x ) dx

p ∞
=−
∫ 0 f ( x) dx + ∫ 0 f ( x ) dx.

Taking limit as p → 0 + , (assuming the integral converges), we have


∞ F( t) ∞
0 t ∫ dt =
0 ∫
f ( x ) dx.

Problem 10: Find the Laplace transform of F ( t) defined as


t + 1 , 0 ≤ t ≤ 2
F ( t) = 
 3, t>2.
Also determine L { F ′ ( t)}.
M-16

∞ −pt
Solution: We have L {F ( t)} =
∫0 e F ( t) dt

2 −pt ∞ −pt
=
∫0 e F ( t) dt +
∫2 e F ( t) dt
2 −pt ∞ −pt
=
∫0 e ( t + 1) dt +
∫2 e ⋅ 3 dt

2 ∞
 e −pt e −pt   e −pt 
= ( t + 1) −  +3 
 −p 2
p  0  − p  2
 3 e −2 p e −2 p 1 1  3 e −2 p
= − + + +
 − p p2 p p2  p

[1 + p − e −2 p ].
1
=
p2
Now L { H ′ ( t)} = pL { H ( t)} − H (0 )
1 + p − e −2 p  1 − e −2 p
= p  − 1, [ H (0 ) = 1] = ⋅
 p2  p
 

Comprehensive Problems 4
∞ sin t π
Problem 1: Show that
∫0 t
dt =
2

(Rohilkhand 2010, 14; Meerut 13, 13B; Purvanchal 07)


Solution: Let F ( t) = sin t
so that f ( p) = L { F ( t)} = L {sin t} = 1 / ( p2 + 1).
∞ −pt sin t ∞
From article 20, L  ⋅ sin t =
1

t  0
e ⋅
t∫ dt =
p
f ( x ) dx

[ ]p = 2π − tan−1 p.
∞ ∞
dx + tan−1 x
1
=
∫p x2 + 1
Taking limit as p → 0, we have
∞ sin t π

0 t
dt = ⋅
2
∞ e− t − e− 3t
Problem 2: Evaluate
∫0 t
dt.
(Rohilkhand 2010)
Solution: Taking a = 1, b = 3 in Ex. 21, we have
∞ e − t − e −3 t 3
0 ∫ t
dt = log = log 3.
1

t e − 3 t sin t dt =
3
Problem 3: Show that

0 50

M-17

d
Solution: We have L { t sin t} = − L {sin t}
dp
∞ −pt d  1  2p
or
∫0 e t sin t dt = −   =
dp  p2 + 1 ( p2 + 1)2

∞ −3 t sin t dt = 3 ⋅
Putting p = 3, we have
∫0 t e 50

t e − 2 t cos t dt =
3
Problem 4: Show that
∫0 25

d
Solution: We have L { t cos t} = − L { cos t}
dp
∞ −pt d  p  p2 − 1
or
∫ 0
e t cos t dt = −   =
dp  p2 + 1 ( p2 + 1)2
Taking p = 2, we have
∞ −2 t cos t dt = 3 ⋅
∫0 t e 25
∞ e − x sin x π
Problem 5: Prove that
∫ 0 x
dx = ⋅
4 (Rohilkhand 2009, 11)
1
Solution: We have L {sin x } = , p> 0
p2 + 1
= f ( p), say.
sin x
Now lim exists and is equal to 1.
x→0 + x
∞ ∞ 1
L 
sin x 
So
 x
=
 p ∫
f ( x ) dx =
p x2 + 1
dx

[ ] p = 2π − tan−1 p = cot−1 p.

= tan−1 x

∞ −px sin x
L 
sin x 
But
 x 
=
0
e
∫ x
dx.

∞ −px sin x
dx = cot −1 p.

∫0
e
x
Putting p = 1 on both sides, we have
∞ e − x sin x π
dx = cot −11 =
∫0 x 4

∞ −3 t cos 4t dt.
Problem 6: Evaluate
∫0 te (Gorakhpur 2007, 09)
Solution: We have
d
L {t cos 4t} = − L {cos 4t}
dp
M-18

∞ −pt d  p  16 − p2
or
∫ 0
e t cos 4t dt = −   =
dp  p2 + 16  ( p2 + 16)2

Taking p = 3, we have
∞ −3 t 7
∫0 e t cos 4t dt = ⋅
625

Comprehensive Problems 5
 t   1 
Problem 1: Given L 2    =
1 1
, show that =L ⋅
 π 3 / 2 1 / 2
 √ ( πt) 
  p p
Solution: Let F ( t) = 2 √ ( t/ π), so that F ′( t) = 1/√ ( πt).
∴ We have, L { F ′ ( t)} = p. L { F ( t)} − F (0 )
 1    t 1
or L  = p L 2    − 0 = p ⋅ 3 /2 ,
 √ ( π t )   π  p
substituting the given value of L {2 √ ( t/ π)}
1
= .
√p
Problem 2: Find (i) L { F ( t)} and (ii) L { F ′ ( t)} , for the function given by
2t , 0 ≤ t ≤ 1
F ( t) = 
 t, t > 1.
∞ −pt 1 −pt ∞ −pt
Solution: (i) We have, L {F( t)} =
∫0 e F ( t) dt =
∫0 e ⋅ 2t dt +
∫1 e ⋅1 dt

1 ∞
  e −pt   e −pt     e −pt   e −pt  
=   ⋅ 2t −   ⋅ 2 +   ⋅t −   ⋅1
   2     2  
  − p   p   0   − p   p   1

2e −p 2e −p 2 1 lim t 1 lim 1 e −p e −p
=− − + − − + +
p p2 p2 p t → ∞ e pt p2 t → ∞ e pt p p2
2e −p e −p 2 e −p e −p
=− −2 + + + , p> 0
p p2 p2 p p2
 lim t lim 1 
∵ if p > 0, then t → ∞ pt = 0 and also t → ∞ pt = 0 
 e e 
2  1 1  −p
= − +  e , p > 0.
 
p2  p p2 
(ii) Now to find L { F ′ (t ) }:
First we observe that F ( t) is discontinuous at t = 1.
We have F (1 − 0 ) = lim F ( t)
t→1− 0

= lim F (1 − h), h is positive and sufficiently small


h→0
M-19

= lim 2 (1 − h) = 2
h→0

and F (1 + 0 ) = lim F ( t) = lim F (1 + h) = lim (1 + h) = 1.


t→1+ 0 h→0 h→0
Since F (1 − 0 ) ≠ F (1 + 0 ), therefore F ( t) is discontinuous at t = 1.
∴ Using the formula for the Laplace transform of the derivative of F ( t) given in article 13,
Note 2,
we have L { F ′ ( t)} = pL { F ( t)} − F (0 ) − e −p [ F (1 + 0 ) − F (1 − 0 )]
 2  1 1  −p 
= p − +  e  − 0 − e −p [1 − 2]
2  p 
 p  p2  
2 e −p 1
− e −p − e −p + e −p = − = (2 − e −p ).
2 1
=
p p p p p
Alternative method for finding L {F ′ (t )}:
2, 0 < t < 1
We have F ′ ( t) = 
1, t > 1.
∴ by definition of Laplace transform of a function, we have
∞ −pt 1 −pt ∞ −pt
L {F ′ ( t)} =
∫0 e F ′ ( t) dt =
∫0 e F ′ ( t) dt +
∫1 e F ′ ( t) dt
1 ∞
1 −pt ∞ −pt  e −pt   e −pt 
=
0
e
∫ ⋅ 2 dt +
1
e

⋅1 dt = 2   + 
 p  0  − p  1
 −pe  
1 −p 
e
=2 +  + 0 +  , p> 0
 − p p   p 
2 e −p 1
= − = (2 − e −p ), p > 0.
p p p
25 30 9
Problem 3: Show that L {(5 e2 t − 3)2 } = − + , p > 4.
p−4 p−2 p
Solution: L {(5 e2 t − 3)2 } = L {25 e4 t − 30 e2 t + 9}
25 30 9
= − + , p > 4, p > 2 and p > 0 i. e. p > 4.
p−4 p−2 p
 sin2 t  1  p2 + 4 
Problem4: Show that L   = log  ⋅

(Rohilkhand 2010; Kashi 14)
 t  4  p2 
1
Solution: Let F ( t) = sin2 t = (1 − cos 2t).
2
11 p 
∴ L { F ( t)} =  −  = f ( p), say.
2
2  p p + 4 
lim 1  lim  sin t 
Now t → 0  f ( t) = t → 0   ⋅ sin t = 1⋅ 0 = 0 i. e., the limit exists.
 t  t 
M-20

∞ 1 ∞1 x 
L  F( t) =  dx=  log x − log ( x2 + 4)
1 1 1

t  p ∫
f ( x ) dx =  −
2 p  x x2 + 4  ∫2  2  p

1  x2  1  lim x2 p2 
=  log 2  =  log − log 
4  x + 4  p 4  x → ∞ 2
x +4 2
p + 4 

1  lim 1 p2  1  p2 
=  log − log  =  log 1 − log 2 
4  x → ∞ 2
1 + (4/ x ) 2
p + 4  4  p + 4 
 p2  2 p2 + 4
=
1 0 − log  = − 1 log p 1
= log .
4  p2 + 4  4 p2 + 4 4 p2
 
∞ sin2 t π
Problem 5: Show that
∫0 t2
dt = .
2
(Rohilkhand 2003)

Solution: First proceeding as in problem 4 show that


sin2 t  1 p2 + 4
L  = log = f ( p), say.
 t  4 p2
Now from problem 9 of Comprehensive Problems 3, we know that if L { F ( t)} = f ( p),
∞ F ( t) ∞
then
0 ∫ t
dt =
0 ∫
f ( x ) dx, provided that the integral converges.

∞ sin2 t ∞ 1  sin2 t  ∞1 x2 + 4

∫0 t2
dt =
∫0 
t  t
 dt =



0 4
log
x2
dx. …(1)

1 ∞ x2 + 4
Now let I =
4 0 ∫
log
x2
dx.

Put x = 2 tan θ. Then dx = 2sec2 θ dθ.


1 π /2   1  π /2
∫ ∫
∴ I = 2 2
 log  2   2 sec θ dθ = − 0 sec θ log sin θ dθ
4 0  sin θ 
π /2 π /2
= −[tan θ log sin θ]
θ
+
0 ∫
tan θ ⋅ cot θ dθ,

integrating by parts taking sec2 θ as second function


π /2

=− lim tan θ log sin θ + lim
tan θ log sin θ + dθ
θ → π /2 θ→ 0 0
π /2
= 0 + [θ] = π / 2.
0
∞ sin t
2 π
∴ from (1), we have
∫0 t2
dt = ⋅
2
∞ cos 6t − cos 4t
dt = log   ⋅
2
Problem 6: Show that
∫ 0 t  3
Solution: Let F ( t) = cos 6t − cos 4t.
M-21
p p
Then L { F ( t)} = L { cos 6t} − L { cos 4t} = − = f ( p), say.
p2 + 62 p2 + 42

Now lim F( t) = lim cos 6t − cos 4t  Form 0 


t→0 t t→0 t  0 
− 6 sin 6t + 4 sin 4t
= lim , by L’ Hospital’s rule
t→0 1
= 0, i.e., the limit exists.
∞ ∞ 
L 
F( t)  x x

 t 
= ∫
p
f ( x ) dx =  2
p x + 6


2

2 2  dx
x + 4 

1 ∞ 1 x2 + 62 
= log ( x2 + 62 ) − log ( x2 + 42 ) =  log 
2 p 2
 x2 + 42  p

1 x2 + 62 1 p2 + 62
= lim log − log
2 x→∞ x2 + 42 2 p2 + 42
1 1 + (62 / x2 ) 1 p2 + 62
= lim log − log
2x→∞ 1 + (42 / x2 ) 2 p2 + 42
1 p2 + 62 1 p2 + 62 1 p2 + 42
=0 − log =− log = log .
2 p2 + 42 2 p2 + 42 2 p2 + 62
∞ −pt cos 6t − cos 4t 1 p2 + 42

∫0 e t
dt =
2
log
p2 + 62

Taking limit as p → 0, we have


∞ cos 6t − cos 4t 2
42 1
= log   = log ⋅
1 2 2
0∫ t
dt = log
2 62 2  3 3
2
Problem 7: Prove that L { J0 ( a √ t)} = (1 / p) e − (a /4 p).
t2 t4 t6
Solution: We know that J0 ( t) = 1 − + − + ...
22 22 .42 22 .42 .62
a2 t a4 t2 a6 t3
∴ J0 ( a √ t) = 1 − + − + ...
22 22 . 42 22 .42 .62
a2 a4 a6
∴ L { J0 ( a √ t)} = L {1} − L { t} + L { t2 } − L { t3 } + ...
22 22 .42 22 .42 .62
1 a2 1 ! a4 2! a6 3!
= − ⋅ + ⋅ − ⋅ + ...
p 22 p2 2
2 .42 p3 2 .42 .62 p4
2

 2 3 
1  1  a2  1  a2  1  a2  2
1 −   +   −   + ... = ⋅ e − (a /4 p) .
1
=
   
p  1!  4 p  2!  4 p   
3!  4 p   p
 
Problem 8: If F ( t) = t2 , 0 < t < 2 and F ( t + 2) = F ( t), find L { F ( t)}.
M-22

Solution: Here F ( t) is a periodic function with period T = 2.


∴ from article 22 theorem, we have
2
 t2 −pt  2 2 −pt

2 2 −pt  −  +

T −pt e te dt
L {F ( t)} = 0 ∫
e F ( t) dt
=
0
t e dt  p
=


0 p 0
−pt −2 p −2 p
1− e 1− e 1− e
 2 
2  t  1 2 −pt 
− e −2 p +  − e −pt  +
4
p p  p 0 p 0
e dt


= 
−2 p
1− e
2
2  e −pt 
− e −2 p −
4 4 −2 p
e + − 
2  p 
p p p2 
= 0
1 − e −2 p
− e −2 p −
4 4 −2 p 2 −2 p 2
e − e +
− (4 p2 + 4 p + 2) e −2 p + 2
p p2 p3 p3
= = .
1 − e −2 p p3 (1 − e −2 p )
sin t, 0 < t < π
Problem 9: Compute L { F ( t)}, if F ( t) =  where F ( t)has period 2π.
0, π < t < 2 π,
Solution: Here F ( t) is a periodic function with period T = 2 π.
T −pt π −pt 2π
0 ⋅ e −pt dt

∫ e
L {F ( t)} = 0
F ( t) dt

= 0
e sin t dt +
∫π
1 − e −pT 1 − e −2 πp
π
1 π −pt 1  e −pt 
=
1 − e −2 πp
∫ 0
e sin t dt = 
1 − e −2 πp  p2 + 1
( − p sin t − cos t)
 0

1  e −pπ 1  1 + e −pπ
= + = 2
−2 πp  p2 + 1 p + 1 ( p + 1) [1 − ( e −pπ )2 ]
2
1− e 
1 + e −p π 1
= = ⋅
( p2 + 1) (1 − e −pπ ) (1 + e −pπ ) ( p2 + 1) (1 − e −pπ )
Problem 10: Find the Laplace transform of the Heaviside’s unit step function H ( t − a).
0, t < a
Solution: We have H ( t − a) = 
1, t > a.
∞ −pt ∞ −pt
∴ L {H( t − a)} =
∫0 e H ( t − a) dt =
∫a e ⋅1 dt

 e −pt   e −pt  e −ap e −ap
=  =  lim + =0+ , p> 0
 − p  a  t→∞ − p  p p

e −ap
= , p > 0.
p
M-23

t  1 − e −2 x 
Problem 11: Find Laplace transform of
∫ 0  x
 dx.


Solution: We have
L {1 − e −2 t } = L {1} − L { e −2 t } = −
1 1
p p+ 2
1 − e −2 t  ∞1 1 
∴ L
 t
=

 −
p  p p+ 2∫
 dp

  F( t)  ∞ 
∵ L {F( t)} = f ( p) ⇒ L  t  = p f ( x ) dx 
 

∞ ∞
∞   p+ 2    2 
= [log p − log ( p + 2)] = −  log   = −  log 1 +  
p  p   p  p
 p 
  2   2
= −  log 1 − log 1 +   = log 1 +  ⋅
  p    p
Now if L {F ( t) = f ( p)} then we have
 t  f ( p)
0

L  F ( u) du =
 p

t −2 x   1
 1 − e  2
 dx  = log 1 +  ⋅
∴ L
∫0  x 
  p  p
∞ e− u log ( p + 1)
Problem 12: If E ( t) =
∫ t u
du, show that L { E ( t)} =
p

Solution: We have
 ∞ e −u   ∞ e −tv 
L {E ( t)} = L 
 t u ∫
du = L 
  1 v
dv ,
 ∫
putting u = tv so that du = t dv
∞ −pt  ∞ e −tv 
=
0
e
∫ 
 1 v ∫
dv  dt

∞ 1  ∞ −pt −tv 
=
∫1  e
v 0 ∫ e dt dv,

changing the order of integration
∞11 ∞1 1 1 
=
∫ ⋅
1 v p+ v
dv =  −
1 p v p+ v ∫
 dv

1 ∞
= [log v − log( p + v )]
p 1

1 
− log  + 1
p 1
= = log ( p + 1).
p  v   1 p
M-24

e − t erf √ t dt.
Problem 13: Evaluate
∫0
1
Solution: We have L { erf √ t} = ⋅
p √ (1 + p2 )
∞ −pt 1

∫0 e erf √ t dt =
p √ (1 + p2 )

Taking p = 1, we get
∞ −1 1
∫0 e erf √ t dt = ⋅
√2
δ( t − 4)
Problem 14(i): Evaluate L  ⋅
 t 
δ( t − 4) ∞ −pt 1
Solution: We have L 
 t 
=
0
e
t ∫
δ( t − 4)dt

e −4 p
= .
4
Problem 14(ii): Evaluate L {e −4 t δ ( t − 3)} ⋅
∞ −pt −4 t
L {e −4 t δ ( t − 3)} =
solution: We have
∫0 e e δ ( t − 3)dt

∞ −(p +4)t
=
∫0 e δ( t − 3)dt

= e −3(p +4).
Problem 15(i): Evaluate L {cos t log t δ ( t − π )} ⋅

e −pt cos t log t δ ( t − π )dt
solution: We have L {cos t log t δ ( t − π )} =
∫0
= cos π log π e −pπ = − log π e −pπ ⋅
Problem 15(ii): Evaluate L {t3 δ ( t − 5)} ⋅

e −pt t3 δ ( t − 5)dt
solution: We have L {t3 δ ( t − 5)} =
∫0
= (5)3 e −5 t = 125 e −5 t ⋅
∞ −3 t δ( t − 4)dt .
Problem 16(i): Evaluate
∫ −∞ e
[ ]t =4 = e −12 ⋅

e −3 t δ( t − 4)dt = e −3 t
solution: We have
∫0
∞ −3 t δ ′( t − 2)dt .
Problem 16(ii): Evaluate
∫ −∞ e
[ ]t =2 = 3e −6 ⋅
∞ −3 t δ ′( t − 2)dt = 3e −3 t
solution: We have
∫ −∞ e
M-25

H ints to O bjective T ype Q uestions

Multiple Choice Questions


1. (c). See Note 1 of article 7. 2. (c). See part (i) of article 8.
3. (b). See article 11. 4. (a). See article 13.
5. (a). See part (vii) of article 8. 6. (b). See article 18.
7. (a). See Ex. 19. 8. (c). See article 12.
2
9. (d). Since e t is not of exponential order as t → ∞.
10. (b). Use change of scale property.
d d  a  2 ap
11. (c). L { t sin at} = − L {sin at} = −   = ⋅
dp  2 2 
dp  p + a  ( p + a2 )2
2

12. (a). See deduction (iv) of Ex. 25.


13. (b). See Problem 1 of Comprehensive Problems 4.
14. (a). Refer Ex. 27.
Fill in the Blank(s)
1. See article 1. 2. See article 2.
3. See part (vi) of article 8. 4. See article 10.
5. See article 19. 6. See Ex. 25.
7. See Problem 5 of Comprehensive Problems 5.
8. See article 22.
True or False
1. T. See article 3. 2. T. See Ex. 6.
3. F. See Ex. 8. 4. F. See part (ii) of article 8.
5. F. See article 18. 6. F. See part (iv) of article 8.
 sin2λ  1 1 2λ λ
7. T . L {sin λt ⋅ cos λt} = L   = L {sin 2λt} = ⋅ 2 = , p > 0.
 2  2 2 p + 4λ 2 p + 4λ2
2

8. T. See Problem 1(i), of Comprehensive Problems 2.

❍❍❍
M-26

Chapter-2
The Inverse Laplace Transform

Comprehensive Problems 1
 4 
Problem 1: Find L−1  .
( p − 2) 
 4  −1  1  = 4e2 t .
Solution: L−1  =4L  
 p − 2  p − 2
 p 6p 3 
Problem 2: Find L−1  + + ⋅
2
 p + 2 2
p − 16 p − 3 

 p 6p 3 
Solution: L−1  + + 
2
 p + 2
2
p − 16 p − 3 
 p   p  −1  1 
= L−1  +6L
−1
 2 + 3L  
 p − 3
2 2
 p + ( √ 2)  2
 p − 4 
= cos √ 2t + 6 cosh 4t + 3e 3 t .
 2p + 1 
Problem 3: Find L−1  ⋅
 p ( p + 1) (Kashi 2014; Purvanchal 14)
 2 p + 1   p + ( p + 1)  −1  1  + L−1 1  = e −t + 1.
Solution: L−1  −1
=L   =L    
 p ( p + 1)   p ( p + 1)   p + 1  p
2 p − 5 
Problem 4: Find L−1  ⋅
2
 p − 9  (Meerut 2013)
 2 p − 5  −1 
 p  
−1  
Solution: L−1 
1
2  = 2L  2 2  −5 L  2 2 
 p − 9   p − 3   p + 3 
= 2 cosh 3t − (5/3)sinh3t.
3 p − 2 7 
Problem 5: Find L−1  − ⋅
 p5 / 2 3 p + 2 
3 p − 2 7 
Solution: We have L−1  − 
 p5 / 2 3 p + 2 

 1  −1  1  7 −1  
= 3L−1 
1
 − 2 L  5 /2  − L  
 p
3 / 2
  p  3  p + (2 / 3 ) 
M-27

t 1 /2 t 3 /2
− . e − (2 /3) t
7
=3. −2.
Γ (3 / 2) Γ (5 / 2) 3

= 6   − t   − e −2 t /3 .
t 8 t 7
 π 3  π 3

 6 3 + 4p 8 − 6 p 
Problem 6: Find L−1  − + ⋅
 2 p − 3 9 p − 16 16 p2 + 9 
2

 6 3 + 4p 8 − 6 p 
Solution: We have L−1  − + 
2 p − 3 9 p 2 − 16 16 p 2 + 9 

= 3L−1 
1  −1 
 1  −1   p 
 − 3L  2  − 4L  2 
 p − (3 / 2)  9 p − 16  9 p − 16 

  
−1  p 
+ 8L−1 
1
2  − 6L  2 
16 p + 9  16 p + 9 

  1 −1   4 −1  p 
= 3L−1 
1 1
− L  2 − L  2 
 p − (3 / 2 )  3 2
 p − (4 / 3)  9 2
 p − (4/3) 
  3 −1  
+ L−1 
1 1 p
2 2 − L  2 2 
2  p + (3/4)  8  p + (3/4) 

1 1 4 4 4t
= 3 . e(3 /2)t − . . sinh t − . cosh
3 (4 / 3) 3 9 3
1 1 3t 3 3
+ . sin − cos t
2 3/4 4 8 4

1 4t 4 4t 2 3t 3 3
= 3 e 3 t /2 − sinh − cosh + sin − cos t.
4 3 9 3 3 4 8 4

 3 3p + 2 3 p − 27 6 − 30 √ p 
Problem 7: Find L−1  + − + ⋅
2
 p − 3 p3 2
p +9 p4 

 3 p + 2 3 p − 27 6 − 30 √ p 
Solution: We have L−1 
3
+ − + 
2
 p − 3 p3 2
p +9 p 4 
 30 
= L−1 
3 3 2 3p 27 6
+ + − + + − 
 p − 3 2 p2 p3 2
p +9 2
p +9 p4 p7 /2 

  −1  1   1 
−1  −1   p 
= 3L−1 
1
2 2  + 3L  2  + 2 L  3  − 3 L  2 2
 p − ( √ 3)   p   p   p − 3 
   1  −1 
 1 
+ 27 L−1  −1
1
2 2 +6L  4  − 30 L  7 /2 
 p + 3   p   p 
M-28

1 t2 − 1 t3 − 1 27
=3. sinh √ 3t + 3 ⋅ + 2. − 3.cos 3t + sin 3t
√3 (2 − 1) ! (3 − 1) ! 3

t 4 −1 t7 /2 − 1
+ 6. − 30 .
(4 − 1) ! Γ ( 7 / 2)
= √ 3 sinh √ 3t + 3t + t 2 − 3 cos 3t + 9 sin 3t + t 3 − 16t 2 √ ( t/ π ).
3 ( p − 1) 4 p − 18 ( p + 1) (2 − √ p)
2 2
Problem 8: Find L−1  + + ⋅
 2p 5 9 − p2 p5 /2 
Solution: We have
3 ( p2 − 1)2 4 p − 18 ( p + 1)(2 − √ p)
L−1  + + 
 2 p5 9 − p2 p5 /2 

3 1 p 1 
= L−1  . − 3 ⋅
1 3 18 2 1 2
+ −4⋅ + + − + − 
2 p p3 2 p5 p2 − 9 p2 − 9 p3 /2 p p5 /2 p2 
1 −1  1   1  3 −1  1   p 
= L   − 3L−1  3  + L  5  − 4L
−1
 2 
2  p  2  p  2
 p − 3 
 p

   1   1   1 
+ 18 L−1  −1 −1 −1
1
+ 2 L  3 /2  + 2 L  5 /2  − 2 L  
 p2 − 32   p   p   p2 

1 3
= − 3 ( t2 /2 !) + ( t4 / 4 !) − 4 cosh 3t + (18 / 3) sinh 3t
2 2
+ 2. {t1 /2 /Γ (3 / 2)} + 2 {t3 /2 /Γ (5 / 2)} − {t /1 !}
1 3 1 4 8
= − t2 + t − 4 cosh 3t + 6 sinh 3t + 4 √ ( t/ π ) + t √ ( t/ π ) − t.
2 2 16 3

1 1 t2 t4 t6
Problem 9: Show that L−1  cos  = 1 − + − + ...
2 2 (6 !)2
 p p (2 !) (4 !)
1 1
Solution: We have L−1  cos 
 p p

 1   
2 (1 / p)4 (1 / p)6
= L−1  1 −
(1 / p)
+ − + ... 
 
  p 2 ! 4 ! 6 !  
1  1  1  1  1  1  1 
= L−1   − L−1   + L−1   − L−1   + …
2 !  3  4 !  5  6 !  p7 
  p p  p 
t2 t4 t6
=1− + − + ... .
(2 !)2 (4 !)2 (6 !)2
M-29

Comprehensive Problems 2
 
Problem 1(i): Find L−1 
1
2 ⋅
 p − 6 p + 10 
Solution: We have
  −1 
   1 
L−1  3 t −1
1 1 3t
2 =L  2  = e .L  2  = e sin t.
 p − 6 p + 10  ( p − 3) + 1  p + 1

 p −1 
Problem 1(ii): Find L−1  ⋅
2
 ( p + 3) ( p + 2 p + 2)

 p −1  
−1  4p + 1 
Solution: L−1 
4
 = L − + 
2
( p + 3) ( p + 2 p + 2)  5 ( p + 3 ) 2
5 ( p + 2 p + 2)

4 −1  1  1 −1 4 ( p + 1) − 3 
=− L  + L  
5  p + 3 5  ( p + 1)2 + 1 

4 −3 t e − t  −1  4 p 3 
=− e + L  2 − 
5 5  2
 p + 1 p + 1

4 − 3t 1 − t
=− e + e (4 cos t − 3 sin t).
5 5

   p 
Problem 2: Evaluate (i) L−1  (ii) L−1 
1
 ⋅
( p + a)n  ( p + 1)5 /2 
   1  n−1
Solution: (i) L−1  −at L−1 −at t
1
=e  =e
n
( p + a)   n
p  Γ ( n)

t n−1
= e −a t , if n is a positive integer.
( n − 1) !
   ( p + 1) − 1 
−1   p − 1 
− t −1 
L−1 
p
(ii)
5 / 2 =L  5 / 2  = e L  5 /2 
( p + 1)  ( p + 1)   p 

= e − t L−1 {(1/ p3 /2 ) − (1/ p5 /2 )} = e − t L−1 {1 / p3 /2 } − e − t L−1 {1 / p5 /2 }

t3 /2 − 1 t 5 /2 − 1  t  − 4 e −t . t  t
= e −t − e −t = 2e − t    
Γ (3/2) Γ (5/2)  π 3  π

=
2 −t  t  (3 − 2t).
e  
3  π
M-30

 p 
Problem 3: Evaluate (i) L−1   (Lucknow 2011)
5
( p + 1) 

 3 p + 2 
(ii) L−1  ⋅
2
4 p + 12 p + 9 
 
(iii) L−1 
1
.
 √ (2 p + 3)
 p  −1 
( p + 1) − 1   p − 1 
Solution: (i) L−1  =L  =e
− t L−1
 5 
5
( p + 1)   ( p + 1)5   p 
= e − t L−1 {1/ p4 } − e − t L−1 {1/ p5 } = e − t ( t3 / 3 !) − e − t ( t4 / 4 !)
= e − t (4t3 − t4 ) / 24.
 3 p + 2  3 ( p + 3 / 2) − 5 / 2 
(ii) L−1   = L−1  
4 p2 + 12 p + 9   4 ( p + 3 / 2)2 
3 p − 5 / 2  −3 t /2  3 L−1 {1/ p} − 5 L−1 {1/p2 }
= e −3 t /2 L−1  =e
 4 p2   4 8 

3 5 t  1
= e −3 t /2  − .  = e −3 t /2 (6 − 5t).
 4 8 1 ! 8
   
  −1   1 −1  
L−1 
1 1 1
(iii) =L   = L 
3 
 √ (2 p + 3) 3
 √ 2 . √ ( p + ) √ 2  √ ( p + )
 2   2 
1  t (1 /2) − 1
. e(−3 /2) t L−1   = . e −(3 /2) t .
1 1
=
√2  √ p √ 2 Γ  
1
2
1 e −(3 /2) t t −1 /2 1
= . = ⋅
√2 √π e(3 /2) t
√ (2 πt)
 p2 − 2 p + 3 
   
Problem 4: Find (i) L−1  (ii) L−1 
1
2  2 ⋅
( p − 1) ( p + 1) ( p + 2) ( p − 1) 
(Gorakhpur 2005)
 p2 − 2 p + 3   2 
Solution: (i) L−1 
 
= −1  ( p − 1) + 2 
2  L  2 
 ( p − 1) ( p + 1)   ( p − 1) ( p − 1 + 2) 
 p2 + 2 
 
= e t L−1  
2
 p ( p + 2)
 1  3 p2  
= e t L−1 
1
1 − p +  , dividing 2 + p2 by 2 + p till p2
2  2 2 ( + 2 ) 
 p 
p  
is a common factor in the remainder
M-31

 1  t  3 −2 t 
= e t L−1 
1 3 1
− +  = e . t − . 1 + e
 p2 2 p 2 ( p + 2)  2 2 

= ( t − ) e t + (3/2) e − t .
1
2

  −1 
  t −1  
L−1 
1 1 1
(ii)
2 =L  2 = e L  2 
( p + 2) ( p − 1)  ( p − 1 + 3) ( p − 1)  ( p + 3) p 
 1  1 1 1 p2  
= e t L−1   − p+  ,
2  9 p + 3  
 p  3 9 
dividing 1 by 3 + p till p2 is a common factor in the remainder
1 1 1  t 1 1 1 −3 t 
= e t L−1  ⋅
1 1 1
− ⋅ + ⋅  = e  t − + e 
2 + 
 p
3 9 p 9 ( p 3 )  3 9 9

= [(3t − 1) e t + e −2 t ].
1
9
 p  1 −1 
 32 p 
Problem 5: If L−1   = t sin t, find L  ⋅
2 2
( p + 1)  2 2 2
(16 p + 1) 

 p  1
Solution: We have L−1   = . t sin t.
2 2
( p + 1)  2
 ap  1 1 t  2a2 p  t
 
L−1  L−1 
t t
∴  = . . sin or  = sin .
2 2 2
( a p + 1)  2 2 2
2 a a a ( a p + 1)  a a

 32 p  t
Taking a = 4, we have L−1 
t
2 2  = sin .
(16 p + 1)  4 4

 e −5 p   e − 4 p 
Find (i) L−1  (ii) L−1  ⋅
4 4
Problem 6:
( p − 2)  ( p − 3) 
  2 t −1  1 
Since L−1 
1 2t 3 1 3 2t
Solution: (i)  = e L  4  = e ( t /3 !) = t e ,
( p − 2)4   p  6

 e −5 p  1 ( t − 5)3 e2 (t − 5), t > 5


∴ L−1  =
4  6
(See article 8)
( p − 2)   0 ,t<5

= ( t − 5)3 e2 (t − 5) H ( t − 5),
1
6
in terms of Heaviside’s unit step function.
  3 t −1   1  1 3 3 t
Since L−1 
1
(ii)
4  = e L  4 = t e
 ( p − 3)   p  6
M-32

−4 p  1 ( t − 4)3 e3 (t − 4), t > 4


 e  
∴ L−1   = 6
( p − 3)4   0 ,t<4

1 3 3(t − 4)
= ( t − 4) e H ( t − 4),
6
in terms of Heaviside’s unit step function.
4 − 3p 
 e 
Problem 7: Find L−1  ⋅
5 / 2
( p + 4) 
Solution: We have
  5 /2 − 1 4 t 3 /2 e − 4 t
L−1 
1  1 
−4 t L−1  −4 t t
=e  5 /2  = e = ⋅
( p + 4)5 / 2   p  Γ (5 / 2) 3 π

 e4 −3 p   e −3 p 
∴ L−1  4 −1
=e L  
( p + 4)5 /2  ( p + 4)5 / 2 

 e 4 4 ( t − 3)3 /2 e −4(t −3), t > 3



= 3√π
 0 ,t<3

 4 ( t − 3)3 /2 e − 4(t − 4), t > 3

= 3 √ π
 0 ,t<3

4 3 / 2 −4 (t − 4)
= ( t − 3) e H ( t − 3),
3√π
in terms of the Heaviside unit step function.

Problem 8: Find the inverse L.T. of e − 3 p / p3 . (Lucknow 2009)


Solution: We have L−1 {1/ p3 } = t2 /2 ! = t 2 .
1
2
L−1 {e −3 p / p3 } = ( t − 3)2 H ( t − 3).
1

2
Problem 9: Prove the following :
 e −pπ 
(i) L−1   = − sin t. H ( t − π )
 p2 + 1 (Kanpur 2008)
 p e −2 pπ /3 
(ii) L−1   = cos 3 ( t − 2 π / 3) . H ( t − 2 π / 3).
 p2 + 9 
Solution: (i) We have, L−1 {1/( p2 + 1)} = sin t.
∴ L−1 {e −pπ /( p2 + 1)} = sin ( t − π ). H ( t − π ) = − sin t H ( t − π ).
(ii) We have, L−1 { p / ( p2 + 9)} = cos 3t.
 p 
∴ L−1  ⋅ e −2 pπ / 3  = cos 3 ( t − 2 π /3) H ( t − 2 π /3).
2
 p + 9 
M-33

Problem 10: Prove the following :


 pe −ap 
(i) L−1   = cosh ω ( t − a). H ( t − a), a > 0
 p2 − ω2 
3 (1 + e −pπ )
(ii) L−1   = − sin 3 t. H ( t − π ) + sin 3t.
 p2 + 9 
Solution: (i) We have, L−1 { p / ( p2 − ω2 )} = cosh ωt
−ap 
 p e 
∴ L−1   = cosh ω ( t − a). H ( t − a).
2 2
 p − ω 
3 (1 + e −pπ )  1   e −pπ 
(ii) L−1   = 3 L−1   + 3 L−1   …(1)
 p2 + 9   p2 + 9   p2 + 9 

L−1 {1/( p2 + 9)} = sin 3t.


1
Now,
3
L−1 {e −pπ /( p2 + 9)} = sin 3 ( t − π ). H ( t − π ) = − sin 3t. H ( t − π ).
1 1

3 3
Putting in (1), we have
3 (1 + e −pπ )
L−1   = sin 3t − sin 3t. H ( t − π ).
 p2 + 9 
Problem 11: Evaluate
 p+ 2 
(i) L−1   (Kanpur 2008; Lucknow 10; Avadh 13)
2
 p − 2 p + 5 
 p+ 8 
(ii) L−1  ⋅
2
 p + 8 p + 5 
 p+ 2  −1  ( p − 1) + 3   p + 3 
Solution: (i) L−1  =L 
t −1
 =e L  2 
2
 p − 2 p + 5  ( p − 1)2 + 4   p + 4 
  p  −1 
 
= e t  L−1 
1
2 2 +3L  2 2 
  p + 2   p + 2 
= e t [cos 2t + (3 / 2) sin 2t].
 p+ 8  −1  ( p + 4) + 4  − 4 t L−1 
 p + 4 
(ii) L−1  =L  = e  2 
2
 p + 8 p + 5  2
( p + 4) − 11  p − 11
   −1 
 
= e −4 t  L−1 
p 1
2 2 +4L  2 2 
  p − ( √11)   p − ( √11) 
= e − 4 t [cosh ( √11 t) + (4/√11) sinh ( √11 t)].
M-34

 4p + 5 
Problem 12: Prove that L−1  t 1 t 1 −2 t .
 = 3t e + e − e
2
( p − 1) ( p + 2) 3 3
(Purvanchal 2007)

 4 p + 5 
 −1  
Solution: We have L−1 
1 3 1
= L  + − 
2
( p − 1) ( p + 2) 3 ( p − 1) ( p − 1)2 3 ( p + 2)

=
1 −1  1   1  1 −1  1 
−1 
L  +3L  − L  
3 ( p − 1) ( p − 1)2  3  p + 2

e t + 3e t L−1 {1/ p2 } − e −2 t
1 1
=
3 3

e t + 3e t . ( t / 1!) − e −2 t = e t + 3te t − e −2 t .
1 1 1 1
=
3 3 3 3

 4 p + 5 

Problem 13: Evaluate (i) L−1  
2
( p − 4) ( p + 3)
 5 p2 − 15 p − 11 
 
(ii) L−1  .
3
( p + 1) ( p − 2)  (Gorakhpur 2009)

Solution: (i) We have


 4p + 5  −1 
 −1 
L−1 
1 3
=L  + + 
2
( p + 3) ( p − 4)  7 ( p + 3) 7 ( p − 4) ( p − 4) 
2

1 −3 t 1 4 t
+ e + 3e4 t L−1 {1/ p2 } = − e −3 t + e4 t + 3te4 t .
1 1
=− e
7 7 7 7
 5 p2 − 15 p − 11 
 
(ii) We have L−1 
+ − 3
 ( p 1) ( p 2 ) 
 
= L−1 −
1 1 4 7
+ + − 
 3 ( p + 1) 3 ( p − 2 ) ( p − 2)2 3
( p − 2) 

= − L−1 
1  1  1 −1  1  −1  1  −1 
 1 
+ L   + 4L  −7L  
 p + 1 3  p − 2
3 2
( p − 2)  3
( p − 2) 

= − e − t + e2 t + 4e2 t L−1 {1/ p2 } − 7e2 t L−1 {1/ p3 }


1 1
3 3
1 − t 1 2t
=− e + e + 4e2 t . ( t /1 !) − 7e 2 t ( t 2 / 2 !)
3 3
1 − t 1 2t 7
=− e + e + 4te2 t − t2 e 2 t .
3 3 2

 2p + 1  1
Problem 14: Prove that L−1  t
 = t (e − e
−2 t ).
( p + 2)2 ( p − 1)2  3 (Gorakhpur 2007)
M-35

 2p + 1   
Solution: We have L−1  −1 1 1
2 2 = L  2

2 
( p + 2) ( p − 1)  3 ( p − 1) 3 ( p + 2) 

  1 −1  
= L−1 
1 1 1
2 − L  2 
3 ( p − 1)  3 ( p + 2) 
= e t L−1 {1/ p2 } − e −2 t L−1 {1/ p2 }
1 1
3 3
1 −2 t
. t = t ( e t − e −2 t ).
1 t 1
= e .t− e
3 3 3

 p  1
Problem 15: Prove that L−1   = sin t sinh t.
2 2
( p − 2 p + 2) ( p + 2 p + 2) 2
 p 
Solution: We have L−1  
2 2
( p − 2 p + 2) ( p + 2 p + 2)
1 ( p2 + 2 p + 2) − ( p2 − 2 p + 2) 
 
= L−1 
4 2 − 2 p + 2) ( p2 + 2 p + 2) 
 ( p 
 
= L−1 
1 1
− 
2 2
4 ( p − 2 p + 2) 4 ( p + 2 p + 2)
  1 −1  
= L−1 
1 1 1
2 − L  2 
4 ( p − 1) + 1 4 ( p + 1) + 1

 1  1 − t −1  1 
= e t L−1 
1
2 − e L 
2 
4  p + 1 4  p + 1
e . sin t − e − t sin t = ( e t − e − t ) sin t = sinh t sin t.
1 t 1 1 1
=
4 4 4 2

Comprehensive Problems 3
   
Problem 1: Find (i) L−1  (ii) L−1 
1 1
3  3 ⋅
( p − a)  ( p + a)  (Rohilkhand 2010)
d2  1  2
Solution: (i) Since   = ,
dp2  p − a  ( p − a)3
1 1 d2  1 
∴ =  .
( p − a)3 2 dp2  p − a 
  1 d2  1   1 −1  d2  1  
−1 
L−1 
1
Hence =L    = L  2  
3
( p − a)  2 dp2  p − a   2  dp  p − a  
 1  1 2 at
= ( − 1)2 t2 L−1 
1
= t e .
2  p − a 2
M-36

   1  2
L−1  at L−1
1 at t = 1 t2 e at .
Aliter:
3 =e  3= e
( p − a)   p  2! 2

d2  1  2
(ii) Since
2  ( p + a)  = 3
dp   ( p + a)
1 1 d2  1 
∴ =  .
( p + a)3 2 dp2  p + a 
  1 d  1   1 −1  d  1  
−1 
2 2
L−1 
1
Hence =L    = L  2  
3
( p + a)  2 dp  p + a   2
2  dp  p + a  
 1  1 2 −at
= ( − 1)2 t 2 L−1 
1
= t e .
2  p + a 2
   1  t 2 1 2 −at
L−1  = e −at L−1   = e −at
1
Aliter.  = t e .
3 3
( p + a)   p  2! 2

 p 
Problem 2: Find L−1  ⋅
2 2 2
( p − a ) 
d  1  2p
Solution: Since   =− ,
dp  p2 − a2  ( p − a2 )2
2

p 1 d  1 
we have =−  .
( p2 − a2 )2 2 dp  p − a 
 2 2

 p  1 −1  d   
L−1 
1
Hence
2 2 2  = − L   2 2


( p − a )  2  dp  p − a  

  1 1
= − ( − 1)1 t L−1 
1 1 t
2 2  = t sinh at = sinh at.
2  p − a  2 a 2 a

  1  
Problem 3: Find L−1 log 1 −  .
 
  p2  

 1  p2 − 1 
Solution: Let f ( p) = log 1 −  = log   = − 2 log p + log ( p2 − 1).
   p2 
 p2   
1 p 
∴ f ′ ( p) = − 2  − .
 p 2 
 p − 1
∴ L−1 { f ′ ( p)} = − 2 (1 − cosh t) or − t L−1 { f ( p)} = − 2 (1 − cosh t)

  1   2
or L−1 log 1 −   = (1 − cosh t).
 
  p2   t
M-37

 
Problem 4: Find L−1 
1
3 ⋅
 p ( p + 1)
 1 
Solution: We have, L−1  −t
 = e = F ( t), say.
 p + 1 
∴ By theorem I of article 14, we have
  t t −x
L−1  e dx = 1 − e −t .
1
=
 p ( p + 1) 0 ∫
F ( x ) dx =
0 ∫
  t
L−1  (1 − e − x ) dx = t + e −t − 1
1

 p2 ( p + 1)
= ∫
0

  t
L−1  ( x + e − x − 1) dx = 1 − t + t 2 − e −t .
1 1
and
 p3 ( p + 1)
= ∫
0 2

 
Problem 5: Find L−1 
1
3 2 ⋅
 p ( p + 1)

 1 
Solution: We have L−1   = sin t.
 p2 + 1
  t
L−1 
1

2
 p ( p + 1)
= ∫ 0 sin x dx = 1 − cos t,
  t
L−1 
1
2 2
 p ( p + 1)
= ∫ 0 (1 − cos x) dx = t − sin t
  t t2
L−1 
1
and
4 2
 p ( p + 1)
=
0 ∫
( x − sin x ) dx =
2
+ cos t − 1

 
Problem 6: Evaluate L−1 
1
3 ⋅
 p ( p + 1) 
Solution: We have
  −1 
   
L−1  − t −1
1 1 1
3 =L  3 = e L  3 .
 p ( p + 1)  ( p + 1 − 1) ( p + 1)  ( p − 1) p 
…(1)
 1 
Since L−1  =e ,
t
 p − 1
  t x
L−1 
1

 p ( p − 1)
=
0∫e dx = ( e t − 1),

  t x
L−1 
1
2
 p ( p − 1)
=
0 ∫
( e − 1) dx = e t − t − 1,
M-38

  t x
L−1 
1 1
and
3
 p ( p − 1)
=∫0
( e − x − 1) dx = e t − t2 − t − 1.
2
∴ From (1), we have
  −t  t 1 2 
L−1  −t
1 1 2
3  = e  e − t − t − 1 = 1 − e (1 + t + t ).
 p ( p + 1)   2  2

 p+ 2  1 p+ 2 
Problem 7: Find (i) L−1 log  (ii) L−1  log ⋅
 p+1   p p+1 
p+ 2
Solution: (i) Let f ( p) = log = log ( p + 2) − log ( p + 1).
p+1
1 1
∴ f ′ ( p) = − ⋅
p+ 2 p+1
∴ L−1 { f ′ ( p)} = e −2 t − e −t or − t L−1 { f ( p)} = e −2 t − e − t .
 p+ 2  1 −t
∴ L−1 { f ( p)} = L−1 log  = (e − e
−2 t ).
 p + 1  t
− 1  p + 2  1 −t −2 t ) = F ( t), say.
(ii) From part (i), L log  = (e − e
 p + 1  t
∴ By theorem 1 of article 14, we have
1 p+ 2 t t
L−1  log F ( x ) dx = ( e − x − e −2 x ) dx.
 p p + 1
=
 0∫ ∫ 0

 p+ 3 
Problem 8: Find (i) L−1 log  (Kanpur 2007)
 p+ 2 
1 p+ 3 
(ii) L−1  log ⋅
 p p+ 2 
p+ 3
Solution: (i) Let f ( p) = log = log ( p + 3) − log ( p + 2).
p+ 2
1 1
∴ f ′ ( p) = − ⋅
p+ 3 p+ 2
∴ L−1 { f ′ ( p)} = e −3 t − e −2 t or − t L−1 { f ( p)} = e −3 t − e −2 t
 p + 3  1 −2 t
or L−1 { f ( p)} = L−1 log  = [e − e −3 t ]
 p+ 2  t
 p + 3  1 −2 t
(ii) From part (i), L−1 log  = [e − e3 t ] = F ( t ).
 p+ 2  t
1 p+ 3 t t 1 −2 x
L−1  log − e −3 x ) dx.

 p p + 2
=

∫ 0 ∫
F ( x ) dx =
0 x
(e
M-39

 p  1  
If L−1  −1 1
Problem 9:
2 2  = t sin t , find L  2 2 ⋅
( p + 1)  2 ( p + 1) (Kanpur 2011)
Solution: Given,
 p  1
L−1   = t sin t = F ( t), say.
2 2
( p + 1)  2

  1
−1  p 
L−1 
1
∴ =L  ⋅ 
2 2 2 2
( p + 1)   p ( p + 1) 

t 1 t 1
=
∫ 0 F ( x) dx = 2 ∫ 0 x sin x dx =
2
(sin t − t cos t).

 1
Problem 10: Find L−1 tan−1  ⋅
 p
Solution: Let f ( p) = tan−1 (1 / p) = cot −1 p.
1
Then f ′ ( p) = − .
2
p +1
 1   1 
−1 
∴ L−1 { f ′ ( p)} = L−1 − =−L  2  = − sin t.
2
 p + 1  p + 1
But L−1 { f ′ ( p)} = ( − t) L−1 { f ( p)}
∴ ( − t) L−1 { f ( p)} = − sin t
L−1 { f ( p)} = or L−1 {tan−1 (1 / p)} =
sin t sin t
or ⋅
t t

Comprehensive Problems 4
 
L−1 
1
Problem 1: Use the convolution theorem to find (i) 
 ( p − 1) ( p + 2 ) 

   
L−1  L−1 
1 1
(ii) , (iii) .
 ( p − 1) ( p + 3 )   ( p + 1) ( p − 2 ) 
(Kanpur 2012)
 1  −1  1  = e −2 t = F ( t).
Solution: (i) L−1  t
 = e = F1( t), L   2
 p − 1   p + 2
∴ By the convolution theorem we have
 1 1 
L−1  ⋅  = F1* F2
 p − 1 p + 2
t t x −2(t − x)
=
∫ 0 F1 ( x) F2 (t − x) dx = ∫ 0 e ⋅e dx

t 3x
= e −2 t dx = ( e t − e −2 t ).
1
∫0 e 3
M-40

 1 
(ii)We have L−1  t
 = e = F ( t), say
 p − 1 
− 1  1  −3 t = G ( t), say
and L  =e
 p + 3
∴ by the convolution theorem, we have
 1 1  t
L−1  ⋅
 p − 1 p + 3
 = F ( t) * G ( t) =
0 ∫
F ( x ) G ( t − x ) dx

t x −3(t − x) t 4x
dx = e −3 t
=
0∫e ⋅e
0 ∫
e dx

= e −3 t [ e4 x ] 0t = e −3 t ( e 4 t − 1) = ( e t − e −3 t )
1 1 1
4 4 4
 1  −1  1  = e2 t = F ( t).
(iii) L−1  −t
 = e = F1 ( t), L   2
 p + 1   p − 2
∴ By the convolution theorem, we have
 1 1  t
L−1 
 p + 1

p − 2
=
 0 ∫
F1 ( x ) F2 ( t − x ) dx

t − x 2(t − x) t 2 t −3 x t
dx = e2 t  − e −3 x  = [ e2 t − e −t ].
1 1
=
∫0e ⋅e dx =
∫0 e ⋅e
 3  0 3

 p 
Problem 2: Use the convolution theorem to find (i) L−1  
2 2 2
( p + a ) 
 
L−1 
1
(ii)
2 2 ⋅
 p ( p + 4)  (Lucknow 2009, 11)
Solution: (i) Since L−1 { p / ( p2 + a2 )} = cos at = F1 ( t)
and L−1 {1 / ( p2 + a2 )} = (1 / a) sin at = F2 ( t),
∴ by the convolution theorem, we have
  t t
L−1 
1 1
2 2 2
( p + a ) 
= ∫0
F1 ( x ) F2 ( t − x ) dx =
∫ 0
cos ax sin a (t − x ) dx
a
1 t
=
a 0 ∫
cos ax ⋅ (sin at cos ax − cos at sin ax ) dx

1 t 1 t
= sin at
a ∫0
cos2 ax dx − cos at
a ∫0
sin ax cos ax dx

1 t 1 t
=
2a ∫
sin at (1 + cos 2ax ) dx −
0 2a ∫
cos at sin 2ax dx
0

sin at  t +
sin2at  1
cos at 1 −
1 cos 2at 
= −
2a  2a  2a  2a 

t sin at sin at . 2 sin at cos at cos at . 2 sin2 at


= + −
2a 4a2 4a2
= (1 / 2a) t sin at.
M-41

  −1 
 1 p 
L−1 
1
(ii)
2 2 =L  2 ⋅ 2 2 ⋅
 p ( p + 4)   p ( p + 4) 
1 p
Let f1 ( p) = and f 2 ( p) =
p2 ( p + 4)2
2

 1 
so that L−1 { f1 ( p)} = L−1   = t = F1 ( t), say
 p2 

 p   1 d  1  
and L−1 { f 2 ( p)} = L−1   = L−1 −   
( p2 + 4)2   2 

2 dp  p + 4 
1 −1  d  1    1 
  = − ( − 1) t L−1 
1
=− L   
 2  2
 p + 4 
2  dp  p + 4   2
1 sin 2t 1
= t. = t sin 2t = F2 ( t), say.
2 2 4
∴ By the convolution theorem, we have
 1  t
L−1  ⋅
p
2 2
 p ( p + 4) 
2 =
0∫F2 ( x ) ⋅ F1 ( t − x ) dx

t x  t t 1 t 2
=
∫ 0 4 sin2 x (t − x) dx =

4 0
x sin 2 x dx −

4 0
x sin 2 x dx

t
1  x2 
t
t  x 1  x 1
=  − cos 2 x + sin2 x  −  − cos 2 x + sin2 x + cos2 x 

4 2 4 0 4  2 2 4 0
1
= (1 − t sin 2t − cos 2t).
16
Problem 3: Use the convolution theorem to find
   
(i) L−1  (ii) L−1 
1 1
 ⋅
( p − 2) ( p2 + 1) ( p2 + 4) ( p + 2)
 1 
(i) L−1  2t −1 
 1 
Solution:  = e = F1( t), L  2  = sin t = F2 ( t).
 p− 2  p + 1
∴ By the convolution theorem, we have
  t 2 (t − x)
L−1 
1
2
( p − 2) ( p + 1)
0 ∫
 = F2 ( t) * F1 ( t) = sin x ⋅ e dx

t
= e2 t e −2 x sin x dx = e2 t ⋅ [ e −2 x ( − 2 sin x − cos x )] 0t
1
0∫ 5
1 2 t −2 t 1
= e [e ( − 2 sin t − cos t) + 1] = [ e2 t − 2 sin t − cos t ].
5 5

 1  1   1 
(ii) L−1  −1
 = sin 2t = F1 ( t), L  =e
−2 t = F ( t).
2
 p + 2
2
 p + 4  2
M-42

∴ By the convolution theorem, we have


  t 1
L−1  sin 2 x ⋅ e −2 (t − x)dx
1
( p2 + 4) ( p + 2)
 = F1 ( t) * F2 ( t) =
0 2 ∫
= [ e −2 t + sin 2t − cos 2t ].
1
8
 
Problem 4: Use the convolution theorem to find (i) L−1 
1
2 2 
 p ( p + 1) 
(Gorakhpur 2006)

 
(ii) L−1 
1
2 ⋅ (Kanpur 2011)
( p + 2) ( p − 2)
 1 
Solution: (i) L−1   = t = F1( t),
 p2 
   1 
L−1  − t −1 −t
1
2 =e L  2  = t e = F2 ( t).
( p + 1)   p 
∴ By the convolution theorem, we have
  t −x
L−1 
1
2 2
 p ( p + 1) 
 = F2 ( t) * F1( t) =
0 ∫
x e ⋅ ( t − x ) dx

= [ − x ( t − x ) e − x − ( t − 2 x ) e − x + 2 e − x ] 0t = ( t + 2) e − t + t − 2.

   1 
−2 t L−1 
(ii) L−1  −2 t . t = F ( t), say
1
2 =e  2 = e 1
( p + 2)   p 
 1 
and L−1  2t
 = e = F2 ( t), say.
 p − 2
∴ By the convolution theorem, we have

  t −2 x 2(t − x)
L−1 
1
2
( p + 2) ( p − 2)
 = F1 * F2 =
0
xe
∫⋅e dx, say

t
= e2 t . ( − x −
1 −4 x 
[ e − (4t + 1)] e −2 t ].
1 1 2t
)e =
 4 16  0 16

 p2 
 
Find L−1  ⋅
2 + 4)2 
Problem 5:
 ( p  (Kanpur 2010)
 
Solution: Proceed as in Ex. 16, here a = 2.
 p2  1
∴ L−1   = [2t cos 2t + sin 2t ].
2 2
( p + 4)  4
M-43

  1
Problem 6: Show that L−1 
1
 = erf (2 √ t) .
 p √ ( p + 4)  2
  −4 t L−1  1 
Solution: Here L−1 {1/ p} = 1 = F1 ( t) and L−1 
1
=e  
 √ ( p + 4 )   √ p

  t −1 /2 e −4 t
= e −4 t L−1  = e −4 t
1
 = = F2 ( t)
−1 /2 +1  Γ( 1 ) √ ( π t )
 p  2
Now applying the convolution theorem, we have
  t e −4 x
L−1 
1
 = F2 ( t)* F1( t) =
 p √ ( p + 4) 

0 √ ( πx )
⋅1 dx.

1
Putting 4 x = y2 so that dx = y dy or dx /√ x = dy, we have
2
  1 2 √t − y2 2 √t − y2
L−1 
1 1 2
=
 p √ ( p + 4)  √ π 0 ∫ e dy = ⋅
2 √π 0
e

dy

1 2 t − x2
= erf (2 √ t), since erf ( t) =
2 √π 0
e
∫ dx.

 2 p2 − 6 p + 5 
 
Problem 7: Using Heaviside’s expansion formula, find L−1  ⋅
3 − 2 + −
 p 6 p 11 p 6 
Solution: Here F ( p) = 2 p2 − 6 p + 5
G ( p) = p3 − 6 p2 + 11p − 6 = ( p − 1) ( p − 2) ( p − 3)
G ′ ( p) = 3 p2 − 12 p + 11.
G ( p) has 3 distinct zeroes α1 = 1, α2 = 2 and α3 = 3.
∴ By the Heaviside’s expansion formula, we have

 2 p2 − 6 p + 5  F (1) t
L−1 
F (2) 2 t F (3) 3 t
= e + e + e
 p − 6 p2 + 11p − 6  G′ (1)
3 G′ (2) G′ (3)
1 5
= e t − e2 t + . e3 t .
2 2
 p2 − 6 
 
Problem 8: Using Heaviside’s expansion formula, find L−1  ⋅
3 2
 p + 4 p + 3 p

Solution: Here F ( p) = p2 − 6,
G ( p) = p3 + 4 p2 + 3 p = p ( p + 1) ( p + 3)
and G ′ ( p) = 3 p2 + 8 p + 3.
G ( p) has 3 distinct zeroes α1 = 0, α2 = − 1 and α3 = − 2.
M-44

∴ By the Heaviside’s expansion formula, we have


 p2 − 6  F ( −1) − t F ( − 3 ) −3 t
 
L−1 
F (0 ) 0⋅ t
= e + e + e
3 + 2 + G ′ (0 ) G ′ ( − 1) G′ ( − 3)
 p 4 p 3 p 
− 6 − 5 − t 3 −3 t
= − 2 + e − t + e −3 t .
5 1
= + e + e
3 −2 6 2 2
Problem 9: Using Heaviside’s expansion formula, find

 19 p + 37   p+ 5 
(i) L−1   (ii) L−1  ⋅
( p + 1) ( p − 2) ( p + 3)
2
( p + 1) ( p + 1)

Solution: (i) Here F ( p) = 19 p + 37, G( p) = ( p + 1) ( p − 2) ( p + 3).


∴ G ( p) has 3 distinct zeroes, α1 = − 1, α2 = 2, α3 = − 3.
Also G ′ ( p) = 3 p2 + 4 p − 5.
∴ By Heaviside’s expansion formula, we have
 19 p + 37  F ( − 1) − t F ( − 3 ) −3 t
L−1 
F (2) 2 t
= e + e + e
 ( p + 1) ( p − 2 ) ( p + 3 )  G ′ ( − 1) G ′ (2 ) G ′ ( − 3)

= − 3 e − t + 5 e2 t − 2 e −3 t .
(ii) Here F ( p) = p + 5, G ( p) = ( p + 1) ( p2 + 1) = ( p + 1) ( p + i ) ( p − i ).
∴ G ( p) has 3 distinct zeroes, − 1, i, − i.
Also G ′ ( p) = 3 p2 + 2 p + 1.
∴ By Heaviside’s expansion formula, we have
 p+ 5  F ( − 1) − t F ( − i ) − it
L−1 
F ( i ) it
= e + e + e
2
( p + 1)( p + 1) G ′ ( − 1) G ′ ( i ) G ′( − i )

= 2e − t − (2 + 3i) e it + (3i − 2) e − it
1 1
2 2
= 2e − t − 2 . ( e it + e − it ) − 3i . ( e it − e − it )
1 1
2 2
= 2e − t − 2 cos t + 3 sin t.

Comprehensive Problems 5
 
Problem 1: Find L−1 
1
5 .
( p − 1) ( p + 2) (Kanpur 2009, 11)
  −1   
Solution: L−1 
1 1
5 =L  5 
( p − 1) ( p + 2) ( p − 1) ( p − 1 + 3)
 
= e t L−1 
1
5 
 p ( p + 3)
M-45

 1  1 p p2 p3 p4 p5  
= e t L−1   − + − + −  ,
5  3 9 27 81 243 243 ( p + 3)
 p  
dividing 1 by 3 + p till p5 is a common factor in the remainder
 1 
= e t L−1 
1 1 1 1 1
− + − + − 
3 p5 9p4 27 p3 81p2 243 p 243 ( p + 3)

1 t 4 1 t 3 1 t2 1 t 1 e −3 t 
= et  ⋅ − ⋅ + ⋅ − ⋅ + ⋅1 − 
3 4 ! 9 3 ! 27 2 ! 81 1 ! 243 243 

et  4 4 3 4 2 8 8  e −2 t
=
t − t + t − t +  − ⋅
72  3 3 9 27  243
1 1 1
Another method: = ⋅ ⋅
( p − 1) ( p + 2) ( p − 1) ( p + 2)
5 5

  t −1   1  t 4 e t
L−1 
1
Since
5  = e L  5 = = F1 (t)
( p − 1)   p  24
 1 
and L−1  =e
−2 t = F ( t),
2
 p + 2 
∴ by the convolution theorem, we have
 
L−1 
1
5  = F1 ( t) * F2 ( t)
( p − 1) ( p + 2)
t 1 4 e x e −2(t − x)dx = 1 e −2 t t x4 e3 x dx
=
∫0 24 x 24 0∫
− 2 t  4 3 t 8 
t e3 t + 
e t e 4 4 8 8 3t
=  − t 3 e3 t + t 2 e3 t − e − 
24  3 9 9 27  81 81 

et  4 4 3 4 2 8 8  e −2 t
=t − t + t − t +  − ⋅
72  3 3 9 27  243
  t
Problem 2: Prove that L−1 
1
2 2
 p√ ( p + a )
=
0 ∫
J0 ( ax ) dx .

Solution: We have
  −1 /2 
  1 a2  
L−1  = L−1  1 +
1
  
 √ ( p2 + a2 )  
 p p2  
 

  2 4 6  
= L−1  1 −
1 1 a 1. 3 a 1 .3 . 5 a
+ − + ... 
 2 4 6 
   
p 2 p 2 .4 p 2 . 4 .6 p

1 a 2 4 6 
= L−1  −
1 1.3 a 1.3.5 a
⋅ + − + ...
 p 2 p3 2.4 p5 2.4.6 p7 
M-46

. a4 t 4 1.3.5 a6 t6
a2 t 2 13
=1− ⋅ + ⋅ − ⋅ + ...
2 2 ! 24
. 4! 2.4.6 6 !
( at)2 ( at)4 ( at)6
=1− + − + ... = J0 ( at)
22 22 .42 22 .42 .62
and L−1 {1 / p} = 1.
∴ by the convolution theorem , we have
  t t
L−1 
1
2 2
 p √ ( p + a )
=
0 ∫
J0 ( ax ) ⋅1 dx =
0
J0 ( ax ) dx.

Problem 3: Apply Heaviside’s expansion formula, to prove that
 1  1 t
L−1   = [e − e
−t /2 {cos (1 √ 3t) + √ 3 sin (1 √3 t)}]
3
 p − 1 3 2 2

Solution: Here F ( p) = 1, G ( p) = p3 − 1 = ( p − 1) ( p2 + p + 1),


1 1
G ( p) has 3 distinct zeroes, 1, ( − 1 + √ 3i ), ( − 1 − √ 3 i ).
2 2
∴ By the Heaviside’s expansion formula, we have
 1  F (1) t F {1 ( − 1 + √ 3 i )} {1 /2 (− 1 + √3 i)}t
L−1   = e + 2 e
 p3 − 1 G′ (1) G′ {1 (− 1 + √ 3 i )}
2
F {1 ( − 1 − √ 3 i )} {1 (− 1 − √3 i)}t
+ 2 e 2
G′ {1 ( − 1 − √ 3 i )}
2
1 1 (−1+√3 i) t
1
1 (−1−√3 i) t 1
= et + e2 + e2
3 3
( − 1 + √ 3 i)2 3
( − 1 − √ 3 i)2
4 4

1 (− 1 + √3 i) t
1 t 2 (1 − √ 3 i )
= e e2
3 3 (1 + √ 3 i ) (1 − √ 3 i )
1 (− 1 − √3 i) t
2 (1 + √ 3 i )
− e2
3 (1 − √ 3 i)(1 + √ 3 i )

1 t 1 (− 1 + √3 i) t 1
1 (− 1 − √3 i) t
1
= e − (1 − √ 3 i ) e 2 − (1 + √ 3i ) e 2
3 6 6
= e t − e −t /2 [( e i√3 t /2 + e −i√3 t /2 ) − i √ 3 ( e i√3 t /2 − e −i√3 t /2 )]
1 1
3 6
= [ e − e −t /2 {cos( √ 3t) + √ 3 sin ( √ 3t)}].
1 t 1 1
3 2 2
∞ 1
Problem 4: Show that
∫0 sin x2 dx = √ ( π /2).
2

∫ 0 sin t x
Solution: Let F ( t) = 2 dx.
M-47

∞ −pt ∞ −pt  ∞ 2 
∴ L {F( t)} =
∫0 e F ( t) dt =
∫0 e ∫
 0 sin tx x  dt
 
∞
∞ −pt  ∞ ∞ x2
=
∫ ∫ 
0  0
e ⋅ sin tx2 dt dx =
 0 ∫
L {sin tx2 } dx =
0 p2 + x4
dx

1 π /2
=
2√ p 0 ∫
√ (tan θ) dθ,

p sec2 θ dθ
putting x = √( ptanθ) so that dx =
2 √ ( p tan θ)

π /2 1 /2
sin θ cos −1 /2 θ dθ
1
=
2√ p 0 ∫
3 1 1 1
Γ( ) Γ( ) Γ ( ) Γ (1− )
1 4 = 1
= ⋅ 4 4 4
2 √ p 2 Γ (1) 2√ p 2
1 π π  π , 
= ⋅ = ∵ Γ ( p) Γ (1 − p) = sin pπ 0 < p < 1
4 √ p sin (1 π ) 2 √ (2 p)  
4

π −1  1  π t1 /2 −1 1  π 
∴ F ( t) = L  = ⋅ =  ⋅
2 √2  p  2 √ 2 Γ(1 / 2) 2  2t 
1 / 2

∞ 2 dx = 1 √ ( π / 2t).

∫ 0 sin tx 2
Now taking t = 1, we have
∞ 2 dx = 1 √ ( π / 2).

∫ 0 sin x 2

Problem 5: Apply the convolution theorem to show that


t t
∫ 0 sin u cos (t − u) du = 2 sin t . (Gorakhpur 2007, 11)
t
Solution: Let F ( t) =
∫ 0 sin u cos (t − u) du.
∴ By the convolution theorem, we have
L {F ( t)} = L {sin t} ⋅ L {cos t}

1 p p
= ⋅ = ⋅
p2 + 1 p2 + 1 ( p2 + 1)2
  t
F ( t) = L−1 
p
∴  = sin t. (See Ex. 12.)
2 2
( p + 1)  2
M-48

∞ sin2 x π
Problem 6: Use Laplace transform to prove that
∫0 x2
dx = ⋅
2
∞ sin2 ( xt)
Solution: Let F ( t) =
∫ 0 x2
dx.

∞ −pt  ∞ sin2 xt 
Then L {F( t)} =
∫0 e 
 0
∫ x
dx  dt


∞ 1  ∞ −pt 
=
∫0 
x2  0
e
∫sin2 xt dt dx, changing the order of integration

∞ 1 ∞ 1  1 
L (1 − cos 2 xt) dx
=
∫0 x2
[ L {sin2 xt}] dx =
0 x2  2 ∫  

∞ 1 1 1 p 
=
∫0 2  − ⋅ 2 2  dx, p > 0
x  2 p 2 p + 4 x 
∞ 1  p2 + 4 x2 − p2 
=
∫0   dx
x2  2 p ( p2 + 4 x2 ) 

∞ 2 1 1 2x 1 π
tan−1
2
=
∫0 p ( p2 + 4 x2)
dx = ⋅ ⋅
p p 2  p 0
= ⋅ ⋅
p2 2
 π  π −1  1  π
∴ F ( t) = L−1  = L   = t.
2  p2  2
2 p  2
∞ sin2 xt π
Thus
∫0 x2
dx = t.
2
∞ sin2 x π
Putting t = 1 on both sides, we get
∫ 0 x2
dx = ⋅
2
 2  2
Problem 7: Prove that L−1 tan−1  = sin t sinh t. (Agra 2002)
 p2  t
Solution: Let f ( p) = tan−1 (2 / p2 ) = cot −1 ( p2 / 2).
1 1 − 4p
Then f ′ ( p) = − ⋅ ⋅ 2p = ⋅
1 + ( p2 / 2)2 2 p4 + 4

 − 4 p 
∴ L−1 { f ′ ( p)} = L−1  ⋅ …(1)
 p4 + 4 
− 4p − 4p − 4p
Now = =
p4 + 4 ( p2 + 2)2 − (2 p)2 ( p2 − 2 p + 2)( p2 + 2 p + 2)
M-49

( p2 − 2 p + 2) − ( p2 + 2 p + 2) 1 1
= = −
( p2 − 2 p + 2) ( p2 + 2 p + 2) p2 + 2 p + 2 p2 − 2 p + 2
1 1
= − ⋅
( p + 1)2 + 1 ( p − 1)2 + 1
 
∴ from (1), L−1 { f ′ ( p)} = L−1 
1 1
− 
2 2
( p + 1) + 1 ( p − 1) + 1
  −1 
 
= L−1 
1 1
2 −L  2 
( p + 1) + 1 ( p − 1) + 1
 1   1 
= e − t L−1  t −1
−e L  2
−t t
 = e sin t − e sin t
 p2 + 1  p + 1

= − ( e t − e − t ) sin t = − 2 sin t sinh t.

But L−1 { f ′ ( p)} = ( − 1)1 t L−1 { f ( p)} = − t L−1 { f ( p)}.

∴ − t L−1 { f ( p)} = − 2 sin t sinh t


 2  2
or L−1 { f ( p)} = L−1 tan−1  = sin t sinh t.
 p2  t

Problem 8: Show that 1 *1 *1 * … *1 ( n times) = tn −1 / ( n − 1)! , where n = 1, 2, 3, …


(Rohilkhand 2002)
t
Solution: Since F * G =
∫0 F ( x) G (t − x) dx
t
∴ 1 *1 =
∫01⋅1⋅ dx = t
t t2
∴ 1 *1 *1 = t *1 =
∫0 x ⋅1 dx = 2
t
 t2  t x2  x3  t3
and 1 *1 *1 *1 =   *1 =
2
  0 2 ∫
⋅1 dx = 

 =

 2 ⋅ 3  0 3!

Proceeding similarly, we have


1 *1 *1 * ...... *1 ( n times) = tn −1 / ( n − 1)!., where n = 1 2, 3, …

 
Problem 9: Find L−1 
1
2 2 3 / 2 ⋅
 ( p + a ) 
1
Solution: We have L { J0 ( at)} = ⋅
√ ( p2 + a2 )
M-50

Differentiating w.r.t. ‘a’, we have


d d 1
L { J0 ( at)} = ⋅
da da √ ( p2 + a2 )
−a −a
L J ( at) =
d
or or L { t J0 ′ ( at)} =
 da 0  ( p2 + a2 )3 /2 ( p2 + a2 )3 /2
 
L−1 
1 t t
∴  = − J0 ′ ( at) = J1 ( at), since J0 ′ ( x ) = − J1 ( x ).
2 2 3 / 2
 ( p + a )  a a

t t t t ( t − u)n −1
Problem 10: Prove that
∫0 ∫0 ......
∫0 F ( t) dtn =
∫ 0 ( n − 1) !
F ( u) du.

Solution: By the Convolution theorem, if f ( p) = L {F( t)}, then we have

 t  ( n − 1)!
L  ( t − u)n −1 F( u) du = L {tn −1} ⋅ L {F( t)} =
 0∫  pn
⋅ f ( p)

 t ( t − u)n −1   f ( p)
∴ L

 0 ( n − 1)!
F ( u) du = 
  pn 

t ( t − u)n − 1  f ( p)
F ( u) du = L−1 
Hence
∫0 ( n − 1) !  pn 

t t t
∫0 ∫0 ...... ∫0 F (t) dt .
= n [From article 14 theorem II]

H ints to O bjective T ype Q uestions

Multiple Choice Questions


1. (a). See part (i) of Ex 1.
−1
1  t (1 /2) t −1 /2
(c). L −1 
1
2. = = = ⋅
 P  1 π πt
 
 2
 1  t 3 /2 t 3 /2 4 t 3 /2
3. (c). L−1  = = = ⋅
 p5 / 2  Γ(5 / 2) 3
⋅ √π 3 √π
1
2 2
4. (a). See article 7.
5. (d). See article 9.
6. (c). See Theorem 1 of article 14.
M-51

7. (c). See part (iv) of article 5.


 p  1 −1  p  1
8. (d). L−1  = L  2  = cosh 2t.
2
 2 p − 8  2  p − 4  2
9. (b). See article 13.
10. (a). See Ex. 24.
11. (b). See part (i) of Problem 1 of Comprehensive Problems 3. Here a = 1.

Fill in the Blank(s)


1. See article 1.
2. See part (iii) of article 5.
3. See article 8.
4. See article 16.
5. See Ex. 23.

True or False
 3   1 
1. T . L−1  =3L
−1
 =3e
− 4 t.
 p + 4  p + 4
   1 
2. F. L−1  −t L−1  −t
1
2 =e  = e . t.
( p + 1)   p2 
3. T . See Problem 1of Comprehensive Problems 2.
4. T . See properties of convolution article 15.
5. F. See part (i) of Problem 1of Comprehensive Problems 4 .

❍❍❍
M-52
Chapter-3
Applications of Laplace Transform

Comprehensive Problems 1
dy
Problem 1(i): Solve + y = 1, given that y = 2 when t = 0.
dt
Solution: Taking the Laplace transform of both sides of the given differential
equation, we have
L { y ′} + L { y } = L {1} or p L { y } − y(0 ) + L { y } = 1/ p
or ( p + 1) L { y } − 2 = 1 / p
2p + 1 p + ( p + 1) 1 1
or L { y} = = = + ⋅
p ( p + 1) p ( p + 1) p+1 p
Taking the inverse Laplace transform, we have
 1  −1  1 
y = L−1  + L  
 p + 1  p
or y = e − t + 1, which is the required solution.
d2 y dy
Problem 1(ii): Solve + y = 0, under the conditions that y = 1, = 0 when t = 0.
dt2 dt
(Rohilkhand 2002)
Solution: Taking the Laplace transform of both sides of the given differential
equation, we have
L{ y ′ ′ } + L{ y } = 0 or p2 L{ y } − py (0 ) − y ′(0 ) + L{ y } = 0

or ( p2 + 1) L{ y } − p . 1 − 0 = 0 or L{ y } = p/( p2 + 1).
∴ y = L−1 { p/( p2 + 1)} = cos t, which is the required solution.
Problem 2(i): Solve ( D2 + 1) y = 6 cos 2t, if y = 3, D y = 1, when t = 0.
(Gorakhpur 2006, 09; Rohilkhand 07; Purvanchal 07)
Solution: Taking the Laplace transform of both sides of the given differential equation,
we have
L{ y ′ ′ } + L{ y } = 6 L{ cos 2t}
or p2 L{ y } − py(0 ) − y ′(0 ) + L{ y } = 6 p/( p2 + 22 )

or ( p2 + 1) L{ y } − 3 p − 1 = 6 p / ( p2 + 4)
3p 1 6p
or L{ y } = + +
p2 + 1 p2 + 1 ( p2 + 1) ( p2 + 4)
M-53

3p 1 2 p [( p2 + 4) − ( p2 + 1)]
= + +
p2 + 1 p2 + 1 ( p2 + 1) ( p2 + 4)

3p 1  1 1  5p 1 2p
= + + 2p  − = 2 + − ⋅
p2 + 1 p2 + 1 2 2 2 2
 p + 1 p + 4  p + 1 p + 1 p + 4

 p   1 
−1   p 
−1 
∴ y = 5L−1  + L  2  − 2L  2 
2
 p + 1  p + 1  p + 4 
or y = 5 cos t + sin t − 2 cos 2t, which is the required solution.
Problem 2(ii): Solve ( D2 + D) x = 2, when x (0 ) = 3, x ′ (0 ) = 1.
(Rohilkhand 2011; Kanpur 07)
Solution: Taking Laplace transform of both sides of the given equation, we have
L { x ′ ′} + L{ x ′} = L{2}
or p2 L{ x } − p x(0 ) − x ′(0 ) + p L{ x} − x (0 ) = 2 / p
or ( p2 + p) L{ x } = 3 p + 4 + 2 / p
3 p2 + 4 p + 2 2 2 1
or L{ x } = = + +
p2 ( p + 1) p p2 p+1

 2 1 
x = L−1  + −t
2
∴ +  = 2 + 2t + e .
 p p2 p + 1

Problem 3(i): Solve ( D2 + m2 ) x = a sin nt, t > 0, where x, Dx equal to x 0 and x1 , when
t = 0, n ≠ m.
Solution: Taking the Laplace transform of both sides of the given equation, we have
L { x ′ ′} + m2 L { x } = a L {sin nt}
or p2 L { x } − p x(0 ) − x ′(0 ) + m2 L { x } = an / ( p2 + n2 )
or ( p2 + m2 ) L { x} = p x 0 + x1 + an / ( p2 + n2 )
p x1 an
or L { x} = x 0 + +
p2 + m2 p + m2 ( p2 + m2 ) ( p2 + n2 )
2

p x1 an ( p2 + m2 ) − ( p2 + n2 ) 
= x0 + +  
p2 + m2 p2 + m2 m2 − n2  ( p2 + m2 ) ( p2 + n2 ) 

p x1 a  n n 
= x0 + +  2 − ⋅
p2 + m2 p2 + m2 2 2
( m − n )  p + n 2 2 2
p + m 
Taking inverse Laplace transform, we get
x1 a  sin nt − n sin mt  ,
x = x 0 cos mt + sin mt +  
m m2 − n2  m 

which is the required solution.


M-54

Problem 3(ii): Solve ( D + 2)2 y = 4e −2 t , y (0 ) = − 1 and y ′ (0 ) = 4.


Solution: The given equation can be written as
( D2 + 4D + 4) y = 4e −2 t .
∴ L { y ′ ′ } + 4 L { y ′ } + 4 L { y } = 4 L { e −2 t }
4
or p2 L { y } − p . y (0 ) − y ′ (0 ) + 4 [ p L { y } − y (0 )] + 4 L { y } =
p+ 2
4
or p2 L { y } + p − 4 + 4 p L { y } + 4 + 4 L { y } =
p+ 2
4 4 − 2 p − p2
or ( p2 + 4 p + 4) L { y } = − p=
p+ 2 p+ 2

4 − 2p − p
2 4 − ( p + 2)2 + 2 ( p + 2)
or L{ y}= = ⋅
( p + 2)3 ( p + 2)3
4 − ( p + 2)2 + 2 ( p + 2) 

∴ y = L−1  
 ( p + 2)3 
4 − p2 + 2 p 

= e −2 t . L−1 
 −2 t L−1  4 2 1 
=e  3 + 2 − 
 p3   p p p

or y = e −2 t (2 t2 + 2t − 1), which is the required solution.


Problem 4(i): Solve ( D2 − D − 6) y = 2, t > 0, if y = 1, Dy = 0, when t = 0.
Solution: Taking the Laplace transform of both sides of the given equation, we have
L { y ′ ′} − L { y ′} − 6 L { y } = L {2}
or p2 L { y } − p y (0 ) − y ′ (0 ) − [ p L { y } − y (0 )] − 6 L { y } = 2 / p
or p2 L { y } − p.1 − 0 − p L { y } + 1 − 6 L { y } = 2 / p
or ( p2 − p − 6) L { y } = (2 / p) + p − 1
p2 − p + 2 p2 − p + 2
or L{ y}= =
p ( p2 − p − 6) p ( p + 2) ( p − 3)
 1 4 8 
= − + + ⋅
 3 p 5 ( p + 2 ) 15 ( p − 3 ) 
1 −1  1  4 −1  1  8 −1  1 
∴ y = − L  + L  + L  
3  p 5  p + 2  15  p− 3
y = − + e −2 t +
1 4 8 3t
or e , which is the required solution.
3 5 15
Problem 4(ii): Solve ( D2 + 6D + 9) y = sin x, where y (0 ) = 1, y ′ (0 ) = 0.
Solution: Taking the Laplace transform of both sides of the given equation, we have
L { y ′ ′} + 6L { y ′} + 9L { y } = L {sin x }
or p 2 L { y } − py (0 ) − y ′(0 ) + 6 [ p L { y } − y (0 )] + 9L { y } = 1/( p2 + 1)
M-55

or ( p2 + 6 p + 9) L { y } − p. 1 − 0 − 6 ⋅1 = 1/( p2 + 1)
p+ 6 1
or L { y} = +
( p + 3)2 ( p + 3)2 ( p2 + 1)
1 3  3 1 3p − 4 
= + +  + − 
( p + 3) ( p + 3)2
 50 ( p + 3) 10 ( p + 3)
2 50 ( p2 + 1)

1  53 155 p 4 
=  + −3 + 
50  p + 3 ( p + 3)2 2 2
p + 1 p + 1

1  −1  1  + 155 L−1 
 1 
∴ y= 53 L    
50   p + 3 
2
( p + 3) 
 p   1 
− 3 L−1  +4L
−1
 2 
 p2 + 1  p + 1

[53 e −3 x + 155 e −3 x L−1 {1 / p2 } − 3 cos x + 4 sin x ]


1
=
50
[(53 + 155 x ) e −3 x − (3 cos x − 4 sin x )],
1
=
50
which is the required solution.
Problem 5: Solve ( D2 + 9) y = 18t, if y (0 ) = 0, y ( π / 2) = 0.
Solution: Taking Laplace transform of both sides of the given equation, we have
L { y ′ ′} + 9 L { y } = 18 L{ t}
or p2 L{ y } − py(0 ) − y ′ (0 ) + 9 L{ y } = 18 / p2
or ( p2 + 9) L{ y } = A + 18 / p2 , where y ′ (0 ) = A
A 18 A  ( p2 + 9) − p2 
or L { y} = + = + 2 
p2 + 9 p2 ( p2 + 9) p2 + 9 2 2
 p ( p + 9) 
A  1 1  ( A − 2) 2
= +2 −  = + ⋅
 2 
p2 + 9  p 2
p + 9 2
p +9 p2
1
∴ y = ( A − 2) ⋅ sin 3t + 2t.
3
1
But y ( π / 2) = 0. ∴ 0 = ( A − 2) sin (3 π / 2) + π or A = 3 π + 2.
3
∴ y = π sin 3t + 2t, which is the required solution.
Problem 6: Solve ( D2 + 6D + 25) y = 208 e3 t , t > 0, if y = 1, D y = 0 when t = 0.

Solution: Taking Laplace transform of both sides of the given equation, we have
L{ y ′ ′} + 6 L{ y ′} + 25 L{ y } = 208 L { e3 t }
or p2 L { y } − py (0 ) − y ′(0 ) + 6 [ p L { y } − y (0 )] + 25 L { y }= 208/( p − 3)
or ( p2 + 6 p + 25) L { y } − p − 6 = 208 / ( p − 3)
M-56

p+ 6 208
or L{y}= + ⋅
p2 + 6 p + 25 ( p − 3) ( p2 + 6 p + 25)
 ( p + 3) + 3 
y = L−1 
208
∴ + 
2 2
( p + 3) + 16 [( p + 3) − 6] [( p + 3) + 16]

 p + 3 
= e −3 t L−1 
208
+ 
2 2
 p + 16 ( p − 6) ( p + 16)
 p 4 ( p + 6) 
= e −3 t L−1 
3 4
+ + − 
 p2 + 16 p2 + 16 p−6 p2 + 16 
 3p 21 
= e −3 t L−1 
4
− − 
 p − 6 p2 + 42 p + 42 
2

= e −3 t [4e6 t − 3 cos 4t − (21/4) sin 4t ]


= 4e3 t − e −3 t (4 cos 4t + 7 sin 4t), which is the required solution.
3
4
Problem 7: Solve ( D2 − 4D + 5) y = 125t2 , if y = 0 = D y , when t = 0.
Solution: Taking Laplace transform of both sides of the given equation, we have
L { y ′ ′} − 4 L { y ′} + 5 L { y } = 125 L { t2 }
or p2 L { y} − py (0 ) − y ′(0 ) − 4 [ p L { y } − y (0 )] + 5 L { y } = 125.(2 !/ p3 )
or ( p2 − 4 p + 5) L { y } = 250 / p3
250 50 40 22 2 (24 − 11p)
or L{ y}= = + + +
p3 ( p2 − 4 p + 5) p3 p2 p p2 − 4 p + 5
50 40
22  2 − 11 ( p − 2) 
= + +2+ ⋅
p3 p2
p 2
 ( p − 2) + 1 
t2 2 − 11p 
+ 40 ⋅ + 22 . 1 + 2e2 t L−1 
t
∴ y = 50 ⋅ 
2
2! 1!  p + 1 
= 25t2 + 40 t + 22 + 2e2 t (2 sin t − 11 cos t), which is the required solution.
Problem 8: Solve ( D2 + 2D + 1) y = 3t e − t , t > 0, subject to the conditions, y = 4, D y = 2
when t = 0.
Solution: Taking the Laplace transform of both sides of the given equation, we have
L { y ′ ′} + 2L { y ′} + L { y } = 3L {te − t }
or p2 L { y } − py (0 ) − y ′ (0 ) + 2 [ pL { y } − y (0 )] + L { y } = 3L { te − t }
[ L { e − t }]
d
or ( p2 + 2 p + 1) L { y } − 4 p − 10 = − 3
dp
d 1 
or ( p + 1)2 L { y} − 4 p − 10 = − 3  
dp  p + 1
3
or ( p + 1)2 L { y } = + 4 p + 10
( p + 1)2
M-57

3 4 ( p + 1) + 6 3 4 6
or L{ y } = + = + + .
( p + 1)4 ( p + 1)2 ( p + 1)4 p + 1 ( p + 1)2
  −1  1  + 6L−1   
y = 3L−1 
1 1
∴  + 4L    
 p + 1
4
( p + 1)  2
( p + 1) 

 1   1 
− t −1 
= 3e −t L−1  −t
 + 4e + 6e L  2 
4
 p   p 
t3
= 3e − t + 4e − t + 6e − t . t
3!
1 −t 3
= e . t + 4e − t + 6te − t , which is the required solution.
2
Problem 9: Solve ( D2 − 3D + 2) y = 1 − e 2 t , y = 1, D y = 0 when t = 0.
(Avadh 2008; Gorakhpur 05)
Solution: Taking the Laplace transform of both sides of the given equation, we have
L { y ′ ′} − 3L { y ′} + 2L { y } = L {1} − L { e2 t }
1 1
or p2 L { y } − py (0 ) − y ′ (0 ) − 3 [ pL { y } − y (0 )] + 2L { y } = −
p p−2
−2
or ( p2 − 3 p + 2) L { y } − p + 3 =
p ( p − 2)
2
or ( p − 1) ( p − 2) L { y } = − + ( p − 3)
p ( p − 2)
p3 − 5 p2 + 6 p − 2 p2 − 4 p + 2
1 1 1
or L{y}= = + − = ⋅
p ( p − 1) ( p − 2)2 2 p 2 ( p − 2) ( p − 2)2
p ( p − 2)2


1  1
y = L−1   + L−1 
1  1  −1 
 1 
−L  
 p − 2
2 2
( p − 2) 
 p 2
1 1
= + e2 t − te2 t , which is the required solution.
2 2
Problem 10: Solve ( D2 + 1) y = sin t sin 2t, t > 0, if y = 1, Dy = 0 when t = 0.
1
Solution: The given equation can be written as ( D2 + 1) y = (cos t − cos 3t).
2
1 1
∴ L { y ′ ′ } + L { y } = L { cos t} − L { cos 3t}
2 2
2 p p
or p L { y } − py(0 ) − y ′(0 ) + L { y } = −
2 2
2 ( p + 1) 2 ( p + 9)
p p
or ( p2 + 1) L { y } − p = −
2 2
2 ( p + 1) 2 ( p + 9)
p p p
or L { y} = + −
p2 + 1 2 ( p2 + 1)2 2 ( p2 + 1)( p2 + 9)
p p p  ( p2 + 9) − ( p2 + 1)
= + −  
p2 + 1 2 ( p2 + 1)2 16  ( p2 + 9)( p2 + 1) 
M-58

p 1  d 1  p p
= −   − +
 dp 2 
p2 + 1 4  ( p + 1) 16 ( p + 1) 16 ( p2 + 9)
2

 p  1 −1  d 1  1 −1  p  1 −1  p 
or y = L−1  − L  − L  + L  
2
 p + 1 4 2
 dp p + 1 16 2
 p + 1 16 2
 p + 9 
 1  1
= cos t + t L−1 
1 1
− cos t + cos 3t
4  p2 + 1 16 16
15 1 1
= cos t + t sin t + cos 3t, which is the required solution.
16 4 16
Problem 11: Solve ( D3 − D) y = 2 cos t, y = 3, Dy = 2, D2 y = 1, when t = 0.
Solution: Taking the Laplace transform of both sides of the given equation, we have
L { y ′ ′ ′} − L { y ′} = 2L { cos t}
2p
or p3 L { y } − p2 y (0 ) − py ′ (0 ) − y ′ ′ (0 ) − [ p L { y } − y (0 )] =
2
p +1
2 p
or ( p3 − p) L { y } − 3 p2 − 2 p − 1 + 3 =
p2 + 1
2p
or p ( p2 − 1) L { y } = + 3 p2 + 2 p − 2
p2 + 1
2 3 p2 + 2 p − 2
or L{y}= +
( p2 − 1) ( p2 + 1) p ( p − 1) ( p + 1)
1 1 2 3 1
= − + + −
p − 1 p + 1 p 2 ( p − 1) 2 (
2 2 p + 1)
 1  −1 
 1  −1  1  3  1  1 −1  1 
∴ y = L−1  −L  2 +2L  + L−1  − L  
 p − 1 2  p + 1
2
 p − 1  p + 1  p 2
3 t 1 −t
= sinh t − sin t + 2 + e − e
2 2

(e t + e − t ) + 2 ⋅ (e t − e − t )
1 1
= sinh t − sin t + 2 +
2 2
= sinh t − sin t + 2 + cosh t + 2 sinh t
= 3 sinh t − sin t + cosh t + 2 , which is the required solution.
Problem 12: Solve ( D3 + D) y = e2 t , y (0 ) = y ′ (0 ) = y ′ ′ (0 ) = 0.
Solution: Taking the Laplace transform of both sides of the given equation, we have
L { y ′ ′ ′} + L { y ′} = L { e2 t }
or p3 L { y } − p2 y (0 ) − py ′ (0 ) − y ′ ′ (0 ) + p L { y } − y (0 ) = 1/( p − 2)
1
or ( p3 + p) L { y } =
( p − 2)
1 1 1 2p − 1
or L{ y}= =− + + ⋅
p ( p − 2) ( p2 + 1) 2 p 10 ( p − 2) 5 ( p2 + 1)
M-59

1 −1  1  1 −1  1  2 −1  p  1 −1  1 
∴ y=− L  + L  + L  2 − L  2 
2  p 10  p − 2 5  p + 1 5  p + 1
1 1 2t 2 1
or y=− + e + cos t − sin t, which is the required solution.
2 10 5 5
Problem 13: Solve ( D3 − 2D2 + 5D) y = 0, if y (0 ) = 0, y ′ (0 ) = 1, y ( π / 8) = 1.
Solution: Taking the Laplace transform of both sides of the given equation, we have
L { y ′ ′ ′ } − 2L { y ′ ′ } + 5L { y ′ } = 0
or p3 L { y } − p2 y (0 ) − py ′ (0 ) − y ′ ′ (0 ) − 2 [ p2 L { y } − py (0 ) − y ′ (0 )]
+ 5 [ pL { y } − y (0 )] = 0
or ( p3 − 2 p2 + 5 p) L { y } −
p − A − 2. ( − 1) + 5.0 = 0, where y ′ ′ (0 ) = A
A −2+ p A −2 1
or L{ y}= = +
p − 2 p2 + 5 p
3 2 2
p ( p − 2 p + 5) p − 2 p + 5
A −2 A −2 p−2 1
= − ⋅ +
5p 5 2
p − 2p + 5 p2 − 2 p + 5
A − 2 A − 2 ( p − 1) − 1 1
= − ⋅ +
5p 5 ( p − 1)2 + 4 ( p − 1)2 + 4
A −2 A −2 ( p − 1) A+3 2
= − ⋅ + ⋅ ⋅
5p 5 2
( p − 1) + 4 10 ( p − 1)2 + 4
A − 2 −1 1  A − 2 −1  p −1  A + 3 −1  2 
∴ y= ⋅ L  − L  + L  
5 5 2
( p − 1) + 4  10 2
( p − 1) + 4 
 p
A −2 A −2 t A+3 t
= − e cos 2t + e sin 2t.
5 5 10
Since y ( π / 8) = 1, therefore
A −2 A −2 A + 3 π /8 1
⋅ e π /8 ⋅
1
1= − + ⋅e ⋅
5 5 √2 10 √2
7 − A e π /8
or = ( − 2 A + 4 + A + 3)
5 10 √ 2
π /8 
 7 − A  ⋅ 1 − e
or 
 5  
   = 0 or A = 7.
 2 √ 2 
Hence the required solution is y = 1 + e t (sin 2t − cos 2t).
Problem 14: Solve ( D2 + D) y = t2 + 2t, where y (0 ) = 4, y′ (0 ) = − 2.
Solution: Taking the Laplace transform of both sides of the given equation, we have
L { y ′ ′} + L { y ′} = L { t2 } + 2L { t}
or p2 L { y } − p y (0 ) − y ′ (0 ) + pL { y } − y (0 ) = 2 ! / p3 + 2 / p2
or ( p2 + p) L { y } − 4 p + 2 − 4 = 2/ p3 + 2/ p2
or p ( p + 1) L { y } = 4 p + 2 + 2/ p3 + 2/ p2
4 p4 + 2 p3 + 2 p + 2 2 2 2
or L{y}= = + + ⋅
p4 ( p + 1) p4 p p+1
M-60

∴ y = 2L−1 {1 / p4 } + 2L−1 {1 / p} + 2L−1 {1 /( p + 1)}


y = t3 + 2 + 2e − t , which is the required solution.
1
or
3

Comprehensive Problems 2
Problem 1: Solve y ′ ′ + ty ′ − y = 0, if y (0 ) = 0, y ′ (0 ) = 1.
(Agra 2002; Purvanchal 11)
Solution: Taking the Laplace transform of both sides of the given equation, we have
L { y ′ ′} + L { ty ′} − L { y } = 0
d
or p2 y − p y (0 ) − y ′ (0 ) − [ L { y ′}] − y = 0, where y = L { y }
dp
d  d 
or ( p2 − 1) y − 1 − [ p y − y (0 )] = 0 or ( p2 − 1) y − 1 − 1 . y + p y  =0
dp  dp 
d y
or − ( p − 2/ p) y = − 1/ p …(1)
dp
which is a linear differential equation of the first order in y.
− ( p − 2 / p) dp
I. F. = e ∫
2 2
∴ = e −p /2 + 2 log p = p2 e −p /2 .
∴ solution of (1) is given by
2  1  2 e − p2 /2 dp = c − − p2 /2 2
y . p2 e − p /2 = c1 + dp = c1 + e − p /2
∫  − p . p 1 ∫ pe
2  1 1 1 4 
or y = ( c1 / p2 ) e p /2 + 1/ p2 = ( c1 / p2 ) 1 + p2 + ⋅ p + ...  + 1/ p2
 2 2 ! 4 
2 1 1 4
= ( c1 + 1)/ p + c1 + c1 p + ...
2 8
y = ( c1 + 1) L−1 {1/ p2 } + c1 L−1 { 1} + c1 L−1 { p4 } + ...
1 1

2 8
or y = ( c1 + 1). t, since L−1 { p n} = 0, n = 0, 1, 2 ,...
But given that y ′(0 ) = 1.
Therefore, y ′(0 ) = c1 + 1 = 1 or c1 = 0.
Hence y = t, which is the required solution.
Problem 2: Solve [ t D2 + ( t − 1) D − 1] y = 0, if y (0 ) = 5, y ( ∞ ) = 0.
Solution: The given equation can be written as
ty ′ ′ + ty ′ − y ′ − y = 0.
∴ L { ty ′ ′} + L { ty ′} − L { y ′} − L { y } = 0
d d
or − [ L { y ′ ′}] − [ L{ y ′}] − [ p y − y(0 )] − y = 0, where y = L { y }
dp dp
d d
or − [ p2 y − p y (0 ) − y ′(0 )] − [ p y − y(0 )] − p y + 5 − y = 0
dp dp
M-61

d d
or − ( p2 y − 5 p − A ) − ( p y − 5) − ( p + 1) y + 5 = 0, where y ′(0 ) = A
dp dp
d y 3p + 2 10
or + y= …(1)
dp 2
p + p 2
p + p
which is a linear differential equation of first order in y.
3p + 2 2 1 
∫ 2 dp ∫ +  dp
p +p  p p + 1
∴ I. F. = e =e = e [2 log p + log ( p + 1)] = p2 ( p + 1).
∴ Solution of (1) is given by
10
y . p2 ( p + 1) = c1 +
∫ p2 + p ⋅ p ∫ p dp = c1
2 ( p + 1) dp = c + 10 + 5 p2
1

c1 5
or y= +
2
p ( p + 1) p +1
 1 1 1  5
= c1  − + + , resolving into partial fractions.
 p
2 p p + 1  p +1

∴ y = L−1 { y } = c1 ( t − 1 + e − t ) + 5e − t .
But given that y ( ∞ ) = 0, therefore we must have c1 = 0.
Hence y = 5e − t , which is the required solution.

Problem 3: Solve y ′ ′ ( t) + aty ′( t) − 2a y ( t) = 1, y (0 ) = y ′(0 ) = 0, a > 0.


Solution: Taking Laplace transform of the given equation, we have
L { y ′ ′} + a L { t y ′} − 2a L { y } = L {1}
d
or p2 y − p y (0 ) − y ′(0 ) − a [ L { y ′}] − 2a y = 1/ p
dp
d 1 d y 1
or p2 y − a [ p y − y(0 )] − 2a y = or p2 y − a y − ap − 2a y =
dp p dp p
dy 1
or + ( − p/a + 3/ p) y = − , …(1)
dp ap2
which is a linear differential equation in y.
(− p / a + 3 / p) dp
I. F. = e ∫
2 2
∴ = e [− p /(2 a)] + 3 log p = p3 e − p /(2 a).
∴ solution of (1) is given by
2
y . p3 e − p /2 a = c1 −
1 1 3 e − p2 /2 a dp
a ∫ p2 ⋅ p
2 2
pe −p /2 a dp = c1 + e − p /2 a
1
= c1 −
a∫
c  
2
3 p2 /2 a 1 1 2 1  1 2 1
or y = ( c1 / p ) e + = 1 1 + p +  p  + ...  + ⋅
p3 3
p  2 a 2 !  2 a   p
3

  1  1 −1  1    1 
y = c1  L−1  L−1 { p} + ... + L−1 
1
∴ + L  + 
3
 p  2a 2   p3 
  p 8 a
M-62

t2
[ ∵ L−1 { p n} = 0, n = 0, 1, 2,...]
1
or y = ( c1 + 1) + c1 ⋅ ⋅
2! 2a
Given that y (0 ) = 0.
∴ 0 = c1 / 2a or c1 = 0.
1
Hence y = t2 , which is the required solution.
2
Problem 4: Solve ty ′ ′ + 2 y ′ + t y = 0, y (0 + ) = 1, y ( π ) = 0.
Solution: Taking the Laplace transform of both sides of the given equation, we have
d d
L { t y ′ ′} + 2L { y ′} + L { t y } = 0 or − L { y ′ ′} + 2L { y ′} − L { y} = 0
dp dp
d dy
or − [ p 2 y − py (0 + ) − y ′ (0 + )] + 2 [ p y − y (0 + )] − =0
dp dp
dy dy 1
or − ( p2 + 1) −1= 0 or =−
dp dp 2
p +1
∴ y = − tan−1 p + A.
Since y → 0 as p → ∞, therefore y → 0 as p → ∞ from which it follows that A = π / 2.
π π
∴ y = − tan−1 p + = − tan−1 p = cot −1 p i. e., L { y } = cot −1 p.
2 2
∴ y = L−1 { cot −1 p}.
Let L−1 { cot −1 p} = F ( t).
d   1 
Then L−1  cot −1 p = − tF ( t) or L−1 −  = − tF ( t)
 dp   1 + p2 
sin t
or − sin t = − t F ( t) or F ( t) = ⋅
t
y = L−1 { cot −1 p} =
sin t
Hence, ⋅
t
Problem 5: Solve ty ′ ′ ( t) + (2t + 3) y ′ ( t) + ( t + 3) y ( t) = ae − t if y ( t) and its derivatives
have transforms.
Solution: Taking the Laplace transform of both sides of the given equation, we have
L { ty ′ ′} + 2L { ty ′} + 3L { y ′} + L { ty } + 3L { y } = aL { e − t }
d d d a
or − [ L { y ′ ′}] + ( − 2) L { y ′ } + 3L { y ′ } − L { y } + 3L { y } =
dp dp dp p+1
d d
or − [ p2 z − py (0 ) − y ′ (0 )] − 2 [ pz − y (0 )]
dp dp
dz a ,
+ 3 [ pz − y (0 )] − + 3z = where L { y } = z
dp p+1
d d dz a
or − [ p2 z − pA − B ] − 2 [ pz − A ] + 3 [ pz − A ] − + 3z = ,
dp dp dp p+1
since y (0 ) = A and y ′ (0 ) = B
2 dz dz dz a
or − p − 2 pz + A − 2 p − 2z + 3 pz − 3 A − + 3z =
dp dp dp p+1
M-63

dz a
or − ( p2 + 2 p + 1) + ( p + 1) z = 2 A +
dp p+1
dz 1 2A a
or − z=− − …(1)
dp ( p + 1) ( p + 1)2 ( p + 1)3
which is a linear equation of the first order.

I. F. = e − ∫1 /( p + 1) dp = e − log ( p + 1) =
1

p+1
∴ solution of equation (1) is
1 dp dp A a
z⋅
p +1
= B − 2A
∫ ( p + 1)3 − a ∫ ( p + 1)4 = B + ( p + 1)2 + 3 ( p + 1)3 ,
A a
or z = L { y } = B( p + 1) + + ,
( p + 1) 3( p + 1)2
where B is a constant of integration. But B must vanish if z ( p) is a transform since z ( p)
must vanish as p tends to ∞.
A a
∴ L { y} = +
p + 1 3 ( p + 1)2
 1  a −1  
y = AL−1 
1
or + L  
 p + 1 3
2
 ( p + 1) 
 1  
= Ae − t + e − t L−1  =  A +  e − t ,
a at
2  3
3  p 
which is the required solution.

Comprehensive Problems 3
Problem 1: Solve ( D − 2) x + 3 y = 0; 2 x + ( D − 1) y = 0, if x (0 ) = 8 and y (0 ) = 3.
Solution: Taking Laplace transform of both sides of the given equations, we have
L { x ′} − 2 L { x } + 3L { y } = 0 and 2 L { x } + L { y ′} − L { y } = 0
or p x − x (0 ) − 2 x + 3 y = 0 and 2 x + p y − y (0 ) − y = 0.
or ( p − 2) x + 3 y = 8 and 2 x + ( p − 1) y = 3.
Solving for x and y, we have
8 p − 17 8 p − 17 5 3
x= = = +
p − 3 p − 4 ( p − 4) ( p + 1) p + 1 p − 4
2

3 p − 22 3 p − 22 5 2
and y= = = − ⋅
p − 3 p − 4 ( p − 4) ( p + 1) p + 1 p − 4
2

 5 3 
∴ x = L−1  +  = 5e
− t + 3e4 t
 p + 1 p − 4
 5 2 
and y = L−1  −  = 5e
− t − 2e4 t .
 p + 1 p − 4
M-64

Problem 2: Solve ( D2 + 2) x − Dy = 1, Dx + ( D2 + 2) y = 0, if x = 0 = Dx = y = Dy,


when t = 0.
Solution: Taking Laplace transform of both sides of the given equations, we have
L { x ′ ′} + 2L { x } − L { y ′} = L {1}
and L { x ′} + L { y ′ ′} + 2 L { y } = 0
or p2 x − px (0 ) − x ′ (0 ) + 2 x − [ p y − y (0 )] = 1/ p
and p x − x (0 ) + p2 y − py (0 ) − y ′ (0 ) + 2 y = 0
or ( p2 + 2) x − p y = 1/ p and p x + ( p2 + 2) y = 0.
Solving for x and y, we have
p2 + 2 1 1  2p p 
x= = −  + 
p ( p4 + 5 p2 + 4) 2 2
2 p 6  p + 1 p + 4 

−1 1  1 1 
and y= =  − ⋅
p4 + 5 p2 + 4 3 
2 2
p + 4 p + 1

1 −1  1  1  −1  p   p
−1  
∴ x= L   − 2L  + L  2 
2
2   p 6   p + 1  p + 4 
1 1
= − (2 cos t + cos 2t)
2 6
1  1  −1  1 
and y =  L−1  −L  2 
3 2
 p + 4   p + 1

1 1 1
= ( sin 2t − sin t) = (sin 2t − 2 sin t).
3 2 6
Problem 3: Solve ( D2 − 1) x + 5 Dy = t ; − 2Dx + ( D2 − 4) y = − 2 ,
if x = 0 = Dx = y = D y when t = 0.
Solution: Taking the Laplace transform of both sides of the two equations, we have
L { x ′ ′} − L { x } + 5 L { y ′} = L { t }
and − 2 L { x ′} + L { y ′ ′} − 4 L { y} = − 2 L {1}
or p2 x − p x (0 ) − x ′ (0 ) − x + 5 [ p y − y (0 )] = 1/p2
and − 2 { p x − x (0 )} + p2 y − p y (0 ) − y ′ (0 ) − 4 y = − 2/ p
or ( p2 − 1) x + 5 p y = 1 / p2 and − 2 p x + ( p2 − 4) y = − 2 / p.
Solving for x and y, we have
11p2 − 4 1 5 4
x= =− + −
p2 ( p2 + 1) ( p2 + 4) p2 p2 + 1 p2 + 4

− 2 p2 + 4 1 2p p
and y= = − + ⋅
p ( p2 + 1) ( p2 + 4) 2 2
p p +1 p + 4
M-65

 1  −1 
 1  −1 
 1 
∴ x = − L−1   + 5L  2  − 4L  2  = − t + 5 sin t − 2 sin 2t
2
 p   p + 1  p + 4 
 1  p   p 
and y = L−1   − 2 L−1  2 + L
−1
 2  = 1 − 2 cos t + cos 2t.
 p  p + 1  p + 4 
Problem 4: Applying Laplace transform solve the equations :
dx dy d2 x
+ = t and − y = e − t,
dt dt dt2
dx
given that x (0 ) = 0 = y (0 ) and = 0 when t = 0. (Rohilkhand 2007)
dt
Solution: Taking the Laplace transform of both sides of the two equations, we have
L { x ′} + L { y ′} = L { t} and L { x ′ ′} − L { y } = L { e −t }

or p x − x (0 ) + p y − y (0 ) = 1 / p2 and

p2 x − px (0 ) − x ′ (0 ) − y = 1 / ( p + 1)

or p x + p y = 1 / p2 and p2 x − y = 1 / ( p + 1)

or x + y = 1 / p3 and p2 x − y = 1 / ( p + 1).

Solving for x and y, we have


1 1
x= +
p3 ( p2 + 1) ( p + 1) ( p2 + 1)

1 1 p 1 p 1
= − + + − + ,
p3 p p + 1 2 ( p + 1) 2 ( p2 + 1) 2 ( p2 + 1)
2

resolving into partial fractions


11 1 p 1
= − + + +
p3 p 2 ( p + 1) 2 2
2 ( p + 1) 2 ( p + 1)

1 1 1 p 1
and y= − x= − − − ⋅
p3 p 2 ( p + 1) 2 ( p2 + 1) 2 ( p2 + 1)

x = L−1 { x } = t2 − 1 + e −t + cos t + sin t


1 1 1 1

2 2 2 2

y = L−1 { y } = 1 − e −t − cos t − sin t.


1 1 1
and
2 2 2
1
Note. To resolve into partial fractions, we write
p3 ( p2 + 1)
1 1 1
= ⋅ ⋅
p3 ( p2 + 1) p3 1 + p2
M-66

Now we divide 1 by 1 + p2 till p3 is a common factor in the remainder


1 + p2 ) 1 ( 1 − p2
1 + p2

− p2
− p2 − p4

p4
1 1  p4  1 1 p
∴ = 1 − p2 + = 3 − + 2 ⋅
p3 ( p2 + 1) 3
p  2
1 + p  p p p +1
Problem 5: Solve ( D − 2) x − ( D + 1) y = 6 e3 t , (2D − 3) x + ( D − 3) y = 6 e3 t ,
if x = 3, y = 0 when t = 0.
Solution: Taking the Laplace transform of both sides of the two equations, we
have
L { x ′} − 2 L { x } − L { y ′} − L { y } = 6 L {e3 t }
and 2 L { x ′} − 3 L { x } + L { y ′} − 3 L { y} = 6 L {e3 t }
6
or p x − x (0 ) − 2 x − { p y − y (0 )} − y =
p−3
6
and 2 { p x − x (0 )} − 3 x + p y − y (0 ) − 3 y =
p−3
6 3p − 3
or ( p − 2) x − ( p + 1) y = 3 + =
p−3 p−3
6 6 p − 12
and (2 p − 3) x + ( p − 3) y = 6 + = ⋅
p−3 p−3
Solving for x ad y, we have
3 p2 − 6 p − 1 1 2 2
x= = + +
( p − 3) ( p − 1)2 p − 1 ( p − 1)2 p−3
− 3p + 5 1 1 1
and y= = − − ⋅
( p − 3) ( p − 1)2 p − 1 ( p − 1)2 p−3
 1    −1  1  = e t + 2te t + 2e3 t .
x = L−1  −1 1
∴ +2L  + 2L  
 p − 1  ( p − 1)2   p − 3
 1 
y = L−1 
 1 
−1  −1  1  = e t − te t − e3 t .
and −L  − L  
 p − 1 
2
( p − 1)   p − 3
Problem 6: Solve ( D − 2) x − ( D − 2) y = sin t, ( D2 + 1) x + 2Dy = 0,
if x = 0 = x ′ (0 ) = y (0 ).
Solution: Taking Laplace transform of both sides of the given equations, we have
L { x ′} − 2 L { x } − L { y ′} + 2 L { y } = L {sin t}
M-67

and L { x ′ ′} + L { x } + 2 L { y ′} = 0
or p x − x (0 ) − 2 x − [ p y − y (0 )] + 2 y = 1/( p2 + 1)
and p2 x − px (0 ) − x ′ (0 ) + x + 2 [ p y − y (0 )] = 0
or ( p − 2) x − ( p − 2) y = 1/( p2 + 1) and ( p2 + 1) x + 2 p y = 0.
Solving for x and y, we have
2p 2p
x= =
( p + 1) ( p − 3 p − 2) ( p + 1) ( p + 1)2 ( p − 2)
2 3 2

1 1 4 p+ 2
= + + −
9 ( p + 1) 3 ( p + 1) 2 45( p − 2) 5 ( p2 + 1)
1 1
and y=− =−
3
p − 3p − 2 2
( p + 1) ( p − 2)
1 1 1
= + − ⋅
9 ( p + 1) 3 ( p + 1)2 9 ( p − 2)
 p  1 −1  
x = L−1 
1 4 2 1
∴ + − − + L  
9 ( p + 1) 45 ( p − 2) 5 ( p + 1) 5 ( p + 1) 3
2 2 2
( p + 1) 

= e− t + e − cos t − sin t + e − t L−1 {1/ p2 }


1 4 2t 1 2 1
9 45 5 5 3
= e− t + − cos t − sin t + te − t
1 4 2t 1 2 1
e
9 45 5 5 3
  
1 −1  
y = L−1 
1 1 1
and − + L  
9 ( p + 1) 9 ( p − 2) 3
2
( p + 1) 
= e − t − e2 t + e − t L−1 {1/ p2 } = e − t − e2 t + te − t .
1 1 1 1 1 1
9 9 3 9 9 3

Comprehensive Problems 4
∂y ∂y
Problem 1: Solve =2 + y, y ( x, 0 ) = 6 e −3 x which is bounded for x > 0, t > 0.
∂x ∂t
Solution: Taking the Laplace transform of both sides of the given equation, we have
 ∂y   ∂y  d y
L =2L   + L { y } or = 2 [ p y − y ( x, 0 )] + y
 ∂ x   ∂t  dx

− (2 p + 1) y = − 12 e −3 x
d y
or …(1)
dx
which is a linear equation.
− (2 p + 1) dx
I. F. = e ∫ = e − (2 p + 1) x.
∴ the solution of (1) is
e −(2 p + 1) x y = c − 12 e −3 x . e − (2 p + 1) x dx

= c − 12 e − (2 p + 4) x dx = c + e −(2 p + 4) x
6
∫ p+ 2
M-68

y = L { y } = ce(2 p + 1) x + e −3 x .
6
or
p+ 2
Since y ( x, t) is bounded as x → ∞, therefore we must have y ( x, p) also bounded as
x → ∞. Therefore, c = 0.
e −3 x .
6
∴ y = L{ y}=
p+ 2
 6 
∴ y = L−1  e −3 x 
 p + 2 
or y ( x, t) = 6 e −2 t −3 x , which is the required solution.
∂y ∂2 y π  ∂y 
Problem 2: Solve =3 , where y  , t  = 0,   =0
∂t ∂x2 2   ∂x  x = 0

and y ( x, 0 ) = 30 cos 5 x. (Meerut 2006)


Solution: Taking Laplace transform of both the sides of the given equation, we have
∂y  ∂2 y  d2 y
L   = 3L   or p y ( x, p) − y ( x, 0 ) = 3
 ∂t  2
 ∂x  dx2
d2 y p
or − y = − 10 cos 5 x.
dx2 3
The general solution of this equation is
y = C1e x √( p /3) + C2 e − x √( p /3) −
10
cos 5 x
− 52 − ( p / 3)
y = C1 e x √( p /3) + C2 e − x √( p /3) +
30
or cos 5 x. …(1)
75 + p
Taking the Laplace transform of the given conditions, we have
 ∂y 
y ( π /2, p) = 0, L   = 0 when x = 0
 ∂x 
y ( π /2, p) = 0, 
d y
or  = 0 when x = 0.
 dx 
∴ From (1), we have
y ( π /2, p) = 0 ⇒ C1e(π /2)√( p /3) + C2 e −(π /2)√( p /3) = 0
dy
and when x = 0, =0
dx
p  p 
⇒ C1   − C2   = 0.
 
3 3
Solving, we get C1 = 0 = C2 .
30
∴ From (1), we have y = cos 5 x.
75 + p
 30 
∴ y = L−1  cos 5 x 
 75 + p 
or y = 30 e −75 t cos 5 x, which is the required solution.
M-69

∂y ∂2 y
Problem 3: Solve = , y ( x, 0 ) = 3 sin 2 πx , y (0, t) = 0 = y (1, t), 0 < x < 1, t > 0.
∂t ∂x2 (Meerut 2006)
Solution: Taking the Laplace transform of both sides of the given equation, we have
∂y  ∂2 y  d2 y
L   = L   or p y ( x, p) − y ( x, 0 ) =
 ∂t  2
 ∂x  dx2
d2 y
or − p y = − 3 sin 2 πx.
dx2
The general solution of this equation is
y = y ( x, p) = C1 e x √p + C2 e − x √p −
3
⋅ sin 2 πx
− (2 π )2 − p
y ( x, p) = C1 e x √p + C2 e − x √p +
3
or sin 2 πx. …(1)
2
4π + p
Also taking the Laplace transform of the given conditions, we have y (0, p) = 0 and
y (1, p) = 0.
∴ from (1), y (0, p) = C1 + C2 = 0
and y (1, p) = C1 e √p + C2 e −√p + 0 = 0.
Solving, we get C1 = 0 = C2 .
3
∴ from (1), we have y ( x, p) = sin 2 πx.
4 π2 + p
  − 4 π2 t sin 2 πx,
y ( x, t) = 3 L−1 
1
∴  sin 2 πx = 3e
2
4 π + p
which is the required solution.
∂y ∂2 y
Problem4: Solve =2 , y(0, t) = 0, y (5, t) = 0, y ( x, 0 ) = 10 sin 4 πx − 5 sin 6 πx.
∂t ∂x2 (Meerut 2008)

Solution: Taking Laplace transform of both sides of the given equation, we have
∂y  ∂2 y  d2 y
L   = 2 L  2 or p y ( x, p) − y ( x, 0 ) = 2
 ∂t   ∂x  dx 2
d2 y p 1
or − y = (5 sin 6 πx − 10 sin 4 πx ), whose solution is
dx2 2 2
5 sin 6 πx 10 sin 4 π x
y = y ( x, p) = C1 e x √(p /2) + C2 e − x √(p /2) − +
72 π2 + p 32 π2 + p
…(1)
Also taking Laplace transform of the given conditions, we have
y (0, p) = 0 and y (5, p) = 0.
Now y (0, p) = 0 ⇒ C1 + C2 = 0
and y (5, p) = 0 ⇒ C1 e 5 √( p /2) + C2 e −5 √( p /2) = 0.
Solving, we get C1 = 0 = C2 .
M-70

5 sin 6 π x 10 sin 4 π x
∴ y ( x, p) = − + ⋅
72 π2 + p 32 π2 + p
Taking inverse Laplace transform, we get
2 2
y ( x, t) = − 5 e −72 π t sin 6 π x + 10 e −32 π t sin 4 πx,
which is the required solution.
∂u ∂u
Problem 5: Solve − = 1 − e − t , 0 < x < 1, t > 0, u ( x, 0 ) = x.
∂x ∂t (Meerut 2007)
Solution: The given differential equation is
∂u ∂u
− = 1 − e − t , 0 < x < 1, t > 0, u ( x, 0 ) = x.
∂x ∂t
Taking the Laplace transform of both sides of the given equation, we have
∂u ∂u
L   − L   = L {1 − e − t }
 ∂x   ∂t 
du 1 1
or − [ p u − u ( x, 0 )] = − , where u ( x, p) = L { u ( x, t)}
dx p p+1
du 1 du 1
or − pu + x = or − pu = − x, …(1)
dx p ( p + 1) dx p ( p + 1)
which is a linear differential equation.
− pdx
I. F. = e ∫ = e −px
∴ the solution of (1) is
−px  
u e −px = c + e −px − −px
1 1
∫e  p ( p + 1) −

x  dx = c −
 p2 ( p + 1)
∫e x dx

 e −px e −px 
e −px − 
1
=c−
p2 ( p + 1)  − p
x−
∫ − p
⋅ 1 dx 


e −px e −px x e −px


=c− + +
p2 ( p + 1) p p2
x 1 1 x 1
or u = L { u ( x, t)} = c e px + + − = c e px + +
p p2 p2 ( p + 1) p p ( p + 1)
x ( p + 1) − p ( x + 1) 1
= c e px + + = c e px + − ⋅
p p ( p + 1) p p+1
x +1 
u ( x, t) = L−1 { u ( x, p)} = L−1 
1
∴ − + ce px 
 p p + 1 
1   1  
−1  p2 x2 
= ( x + 1) L−1   − L−1 
px
 + c L 1 + + + ...
 
p  p + 1  
 1 ! 2 ! 

= ( x + 1) − e .t − 1 n
[ ∵ L { p } = 0, n = 0, 1, 2,...]
Hence u ( x, t) = ( x + 1) − e − t is the required solution.
M-71

Comprehensive Problems 5
Problem 1: Solve the integral equation
t
F ( t) = a sin t − 2
∫ 0 F (u)⋅ cos (t − u) du.
Solution: The given integral equation may be expressed as
F ( t) = a sin t − 2 F ( t) * cos t.
Taking the Laplace transform of both the sides, we have
L { F ( t)} = aL {sin t} − 2 L { F ( t) * cos t} = aL {sin t} − 2 L { F ( t)} . L { cos t}
a p
= − 2 L { F ( t)} ⋅
p2 +1 p2 +1
 2 p  a a
or 1 + 2  L { F ( t)} = 2 or L { F ( t)} =
 p + 1 p +1 ( p + 1)2
   1  −1  t  = ate −t
F ( t) = aL−1  − t L− 1
1
∴  = a.e  2  = ae  
 Γ (2)
2
( p + 1)   p 
t
Problem 2: Solve y ( t) = t − 1 +
∫ 0 y (Γ )⋅ sin (t − Γ ) dΓ.
Solution: Proceed as in Ex. 20.
1 1
Ans. y ( t) = − 1 + t − t2 + t3 .
2 6
Problem 3: Show that the solution of the integral equation
t 3
F ( t) = 4t − 3
∫ 0 F (u) sin (t − u) du is F ( t) = t +
2
sin 2t.

Solution: The given integral equation may be expressed as


F ( t) = 4 t − 3 F ( t) * sin t.
Taking the Laplace transform of both the sides, we have
L {F ( t )} = 4 L { t} − 3 L { F ( t ) *sin t } = 4 L { t} − 3 L { F ( t)} . L {sin t}
1 1
=4⋅ − 3 L {F ( t )} ⋅
p 2 2
p +1
 3  4 4 ( p 2 + 1)
or 1 + 2  L { F ( t )} = 2 or L { F ( t )} =
 p + 1 p p 2 ( p 2 + 4)

 4 ( p + 1) 
2  − 1   
−1  
F ( t) = L − 1 
1 1
∴  = 4 L  2 + L  2 
2 2
 p ( p + 4) ( p + 4) 2
  p ( p + 4)
…(1)

 
 1
We have L − 1 
1
 = sin 2t
( p 2 + 4) 2
  1 t
L −1 
1 1 1

2 =
 p ( p + 4) 2 0 ∫
sin 2 x dx = [ − cos 2 x ] 0t = [ − cos 2t + 1]
4 4
M-72

  1 t t
 x − sin 2 x  = 1  t − 1 sin 2t
L −1 
1 1
and
2 2 =
 p ( p + 4) 4 0 ∫
(1 − cos 2 x ) dx =
4  2  0 4  2 

Putting these values in (1), we get

F ( t) = 4  sin 2t + t − sin 2t = 2 sin 2t + t − sin 2t


1 1 1 1
 2 4 8  2
3
or F ( t) = sin 2t + t.
2
t −2 u du.
Problem 4: Solve the integral equation F ( t) = 1 + 2
∫ 0 F (t − u)⋅ e
Solution: The given integral equation may be expressed as
F ( t) = 1 + 2 F ( t) * e − 2 t
Taking the Laplace transform of both the sides, we have
L { F ( t)} = L {1} + 2 L { F ( t) * e − 2 t } = + 2 [ L { F ( t)} . L { e − 2 t }]
1
p
1 2
= + L { F ( t)}
p p+ 2
 2  1 p+ 2
or 1 −  L { F ( t)} = or L { F ( t)} = ⋅
 p + 2 p p2
 p + 2  −1  1  −1  1 
∴ F ( t) = L−1   = L   + 2 L  2  = 1 + 2t.
2
 p   p 
 p
1 t
Problem 5: Solve y ( t) = t +
6 0 ∫
( t − u)3 ⋅ y ( u) du.

Solution: The given integral equation may be expressed as


1
y ( t) = t + y ( t) * t 3
6
Taking the Laplace transform of both the sides, we have
1 1 1
L { y ( t )} = L { t} +L { y (t ) * t 3 } = + [ L { y ( t )} L { t 3 }]
6 p2 6
1 3! 1 1
= + L { y ( t)} = + L { y ( t)}
p 2 6p 4 p 2 p4
 1  1
or 1 − 4  L { y ( t)} = 2
 p  p
p2 p2
or L { y ( t)} = =
p 4 − 1 ( p 2 + 1) ( p 2 − 1)
 p2 
∴ y ( t) = L − 1  
2 2
( p + 1) ( p − 1)
1 ( p 2 + 1) + ( p 2 − 1) −1 
 
= L −1 
1 1
2 2 =L  2
+
2 
2 ( p + 1) ( p − 1)  2 ( p − 1) 2 ( p + 1)
M-73

1 − 1  1  1 − 1  1 
L =  + L  
2 2
( p − 1) 2 2
2 ( p + 1)
1 1 1
= sinh t + sin t = (sinh t + sin t).
2 2 2
1 t
Problem 6: Solve the integral equation F ( t) = t2 −
2 0 ∫
( t − u) F ( u) du. (Agra 2003)

Solution: Proceed as in problem 5. Ans. F ( t) = 1 − cos t.


t
Problem 7: Solve 2 F ( t) = 2 − t +
∫ 0 F (t − u) F (u) du.
Solution: Proceed as in Ex. 22. Ans. F( t) = 1, − 1.
t F ( u) du
∫ 0 √ (t − u) = 1 + t + t
Problem 8: Solve 2.

Solution: The given equation can be written as


t − 1 /2 du = 1 + t + t2 F ( t) * t − 1 /2 = 1 + t + t2 .
∫0 F (u) . (t − u) or

Taking the Laplace transform of both the sides, we get


L {F ( t) * t − 1 /2 } = L {1 + t + t 2 }
or L { F ( t)} . L { t − 1 /2 } = L {1} + L { t} + L { t2 }
1
Γ( )  1
2 = 1 + 1 + 2 ! or L { F ( t)} = 1 1 2 
or L { F ( t)}  1 /2 + 3 /2 + 5 /2 
p1 /2 p p2 p3 √π  p p p 

 
 1 2  1  t − 1 /2 t 1 /2 2 t 3 /2 
⋅ L−1
1 1
∴ F ( t) =  1 /2 + +  =  + +
√π  p p3 /2 p5 /2  √ π  Γ (1 ) 3 5 
Γ( ) Γ( ) 
 2 2 2 

=
1  t − 1 /2 + 2 t1 /2 + 2 ⋅ 2 ⋅ 2 t 3 /2  = 1  t − 1 /2 + 2 t 1 /2 + 8 t 3 /2  ⋅
π  3 1  π  3 
Problem 9: Convert the given differential equation
F ′ ′( t) + 2 F ′( t) − 8F ( t) = 5t2 − 3 t, F (0 ) = − 2, F ′ (0 ) = 3
into an integral equation.
Solution: We have
F ′ ′( t) + 2F ′( t) − 8F ( t) = 5t2 − 3t …(1)
F (0 ) = − 2, F ′ (0 ) = 3. …(2)
Method 1: Let F ′ ′( t) = G ( t). …(3)
Integrating (3), we get
t
F ′ ( t) =
∫0 G (u) du + C1. …(4)

For t = 0, (4) gives


F ′ (0 ) = 0 + C1 or 3 = C1.
M-74
t
∴ F ′ ( t) =
∫0 G (u) du + 3. …(5)

Again integrating between the limits 0 to t, we get


t t
F ( t) =
∫0 (t − u) G (u) du + 3 ∫0 du + C2
t
or F ( t) =
∫0 (t − u) G (u) du + 3 t + C2 .
For t = 0, this gives
− 2 = 0 + 0 + C2 or C2 = − 2.
t
∴ F ( t) =
∫0 (t − u) G (u) du + 3t − 2 …(6)

Putting the values of F ′ ′ ( t), F ′( t), F ( t) from (3), (5) and (6) in (1), we get
t t
∫0 G (u) du + 6 − 8 ∫0 (t − u) G (u) du − 24 t + 16 = 5t
G ( t) + 2 2 − 3t

t
∫0 [2 − 8 t + 8 u] G (u) du − 5t
or G ( t) + 2 − 21 t + 22 = 0,

which is the required integral equation.


Method 2: Integrating (1) between the limits 0 to t, we get
t t
F ( u) du =  u3 − u2 
5 3
[ F ′ ( u)] tu = 0 + 2 [ F ( u)] tu = 0 − 8
0  3 2 ∫  0
t 5t 3 3t 2
or F ′ ( t) − F ′ (0 ) + 2F ( t) − 2 F (0 ) − 8
∫0 F (u) du = 3

2
t 5 3 3 2
or F ′ ( t) + 2 F ( t) − 8
∫0 F (u) du = 3 t − t − 1.
2
[Using (2)]

Again integrating between the same limits, we get


t t 5t 4 t 3
[ F ( u)] tu = 0 + 2
0 ∫
F ( u) du − 8 ( t − u) F ( u) du =
0 12 ∫

2
−t

t 5t 4 t 3
or F ( t) − F (0 ) +
0 ∫ ( − 8t + 8u + 2) F ( u) du =
12

2
−t

t 5t 4 t 3
or F ( t) +
0 ∫
( − 8t + 8u + 2) F ( u) du =
12

2
− t − 2,

which is the required integral equation.


Remark: Here, we have two answers for the same problem, but both of these
lead to the same conclusion.
Problem 10: Convert the given integral equation
t
F ( t) = t2 − 3t + 4 − 3
∫ 0 (t − u)
2 F ( u) du

into differential equation and associated conditions.


Solution: The given integral equation is
t
F ( t) = t 2 − 3t + 4 − 3
∫0 (t − u)
2 F ( u) du …(1)
M-75

We know that,
d b (t) b (t) ∂K db da

dt a (t)
K ( u, t) du =
∫ a (t) ∂t
du + K ( b, t)
dt
− K ( a, t)
dt
…(2)

Differentiating (1) using the fact (2), we get


t
F ′ ( t) = 2 t − 3 − 3
∫0 2 (t − u) F (u) du. …(3)

Again differentiating, we get


t
F ′ ′ ( t) = 2 − 6
∫0 F (u) du. …(4)

Finally differentiating once more, we get


F ′ ′ ′ ( t) = − 6 F ( t) or F ′ ′ ′ ( t) + 6 F ( t) = 0. …(5)
Putting t = 0 in (1), (3) and (4) respectively, we get
F (0 ) = 4, F ′ (0 ) = − 3, F ′ ′ (0 ) = 2 . …(6)
Hence, (5) is the required differential equation and its associated conditions are
given by (6).

H ints to O bjective T ype Q uestions

Multiple Choice Questions


1. (a). See Problem 1(i) of Comprehensive Problems 1.
2. (b). See article 4.
3. (a). See article 5, definition (3).
Fill in the Blank(s)
1. See article 1.
2. See article 1.
3. See article 4.
4. See article 4.
5. See article 5, definition (1).
True or False
1. F. If y ( x, t) is a function of x and t, then
 ∂2 y  2
L  = p y ( x, p) − py ( x, 0 ) − yt ( x, 0 )
 ∂ t2 
where L { y ( x, t)} = y ( x, p). See article 4.
2. T . See Example 16.
3. T . See article 5, definition (2).
4. F. See article 5, definition (4).
❍❍❍
M-76

Chapter-4
Fourier Transforms

Comprehensive Problems 1
Problem 1: Using Fourier cosine integral formula, prove that
∞ cos λx π
dλ = e − x , x ≥ 0.

0 λ2 + 1 2 (Gorakhpur 2016)
Solution: The Fourier cosine integral formula is
2 ∞  ∞ 
f ( x) =
π 0
cos λ x 

∫0 ∫
f ( u) cos λu du dλ.

…(1)

Let f ( x ) = e − x then f ( u) = e −u.


∴ from equation (1), we get
2 ∞  ∞ −u 
e−x =
π 0 ∫
cos λ x 
 0 ∫
e cos λudu dλ

2 ∞ 1  ∞ −ax a 
=

π 0
cos λ x
1 + λ2


∵ 0 e

cos bxdx = 
a2 + b2 
∞ cos λ x π
dx = e − x .
or
∫0 1+ λ 2 2
Problem 2: Using Fourier cosine integral formula, show that
2a ∞ cos ux
e − ax =
π 0 u2 + a2∫ dx, a > 0, x ≥ 0.

Solution: Proceed as in problem 1.


1, when |x|≤ 1
Problem 3: Express the function f ( x ) =  as a Fourier integral.
0, when |x|>1
∞ sin λ cos λ x
Hence evaluate
∫0 λ
dλ.

Solution: The Fourier integral formula is


1 ∞  ∞ 
f ( x) =
π −∞ ∫
f (v ) 
 0 ∫
cos w ( x − v ) dw  dv

1 ∞  ∞ 
=

π w =0  −∞ ∫
f ( v ) cos w ( v − x ) dv  dw.

…(1)

Given that
 1, when |x|≤ 1
f ( x) = 
 0, when |x|> 1.
M-77

 1, when − 1 ≤ v ≤ 1
∴ f (v ) =  …(2)
0, when v < −1 or v > 1.
∞ −1 1
Now
∫− ∞ f ( v ) cos w ( v − x ) dv =
∫− ∞ f ( v ) cos w ( v − x ) dv +
∫−1 f ( v ) cos w ( v − x ) dv


+
∫1 f ( v ) cos w ( v − x ) dv

1
=
∫ −1 cos w ( v − x ) dv [ Using (2)]

1
sin w ( v − x )
=
 w  −1
sin w (1 − x ) − sin w ( −1 − x )
=
w
sin w (1 − x ) + sin w (1 + x ) 2sin w cos wx
= = ⋅ …(3)
w w
From (1) and (3), we get
1 ∞ 2 sin w cos wx
f ( x) =
π w = 0 ∫ w
dw

1 ∞ 2 sin λ cos λ x
or f ( x) =
π 0 ∫ λ
dλ …(4)

which is the required representation of f ( x ) as Fourier integral.


Now from equation (4), we have
∞ sin λ cos λ x π
0 ∫ λ
=
2
f ( x)

 π / 2, | x |< 1
= [ Using given f ( x ) ]
 0, | x |> 1.

1, when 0 ≤ x ≤ π
Problem 4: Express f ( x ) =  as a Fourier sine integral.
0, when x > π
∞ 1 − cos πλ
Hence evaluate
0 ∫ λ
sin xλ dλ.

Solution: The Fourier sine integral formula is


2 ∞  ∞ 
f ( x) =
π 0∫sin λ x 
 0

f ( v ) sin λv dv  dλ

…(1)

[ Using λ in place of w ]
1, 0 ≤ x ≤ π
Given that f ( x ) = 
0 , x > π.
 1, 0 ≤ v ≤ π
∴ f (v ) =  …(2)
0 , v > π.
∞ π ∞
Now
∫0 f ( v ) sin λ v dv =
∫0 f ( v ) sin λ v dv +
∫π f ( v ) sin λ v dv
M-78

π
=
∫0 sin λ v dv [ Using (2)]

π
− cos λ v  1 − cos λ π
= = ⋅ …(3)
 λ  0 λ
∴ from (1) and (3), we get
2 ∞ 1 − cos λ π
f ( x) =
π 0 ∫ λ
sin λx dλ

∞ 1 − cos λ π π
or
∫0 λ
sin λ x dλ =
2
f ( x)

π /2, 0 ≤ x ≤ π
= [ Using given f ( x ) ]
0 , x > π.
At the point of discontinuity of f ( x ), x = π
∞ 1 − cos λ π π  f ( π − 0 ) + f ( π + 0 )
∫0 λ
sin λ x dx =
2  2 
π 1 + 0  = π ⋅
=
2  2  4

Problem 5: Using Fourier sine integral formula, show that


6 ∞ u sin ux du
e x − e2 x =

π 0 ( u2 + 1)( u2 + 4)
, x ≥ 0.

Solution: Proceed as in Ex. 2. Here a = 1, and b = 2.

Problem 6: Using Fourier integral formula, show that


1 ∞ u[sin ux − sin u( π − x )]
f ( x) =
π 0 ∫ u2 − 1
du

 0, when x ≤ 0 and x ≥ π
where f ( x ) is given by f ( x ) = 
 cos x, when 0 ≤ x≤ π .

Solution: Proceed as in Ex. 3.

Comprehensive Problems 2
Problem 1(i): Find the Fourier transform of f ( x ), if
 √ (2 π )
 , |x|≤ ε
f ( x ) =  2ε
 0 , |x|> ε.

Solution: We have,
1 ∞ 1 ε ipx √ (2 π )
F { f ( x )} =
√ (2 π ) −∞∫e ipx f ( x ) dx =
√ (2 π ) − ε
e .


dx
M-79

ε
1  e ipx  e ipε − e −ip ε sin pε
=   = = .
2ε  ip  −ε 2ipε pε

Problem 1(ii): Find the Fourier transform of


 x, |x| ≤ a
f ( x) =  (Gorakhpur 2004; Kanpur 08)
 0, |x|> a.
Solution: We have
a
~ 1 ∞ 1 a 1  e ipx e ipx 
f ( p) =
√ (2 π) − ∞∫f ( x ) ⋅ e ipx dx =
√ (2 π ) −a∫x e ipx dx =
√ (2 π)
x
 ip
− 
( ip)2  − a

⋅ [ pa ( e ipa + e − i p a ) + i ( e ipa − e −ipa )]


1 1
= ⋅
√ (2 π ) ip2
i 1
=− ⋅ ⋅ [ pa.2 cos pa + i.2i sin pa]
p √ (2 π)
2

⋅   ( ap cos ap − sin ap).


i 2
=−
p2  π
2 2
Problem 2(i): Show that the Fourier transform of f ( x ) = e − x /2 is e −p /2 .
(Kanpur 2011)
Solution: We have
1 ∞ 1 ∞ − x 2 /2 ipx
F { f ( x )} =

√ (2 π ) − ∞
f ( x ) e ipx dx =

√ (2 π ) −∞
e e dx

2 1 2 1
1 ∞ − x + ipx 1 ∞ − ( x − 2 ipx)
=
√ (2 π ) −∞ ∫
e 2 dx =
√ (2 π ) −∞
e 2
∫dx

2 1 2 1 2 1
1 ∞ − ( x − ip) − p 1 ∞ − ( x − ip) − p2 /2
=
√ (2 π ) −∞ ∫
e 2 2 dx =
√ (2 π ) −∞
e 2 e
∫ dx

2
e − p /2 ∞ − y2
=
√π −∞
e

dy, putting ( x − ip)/ √ 2 = y, so that dx = √2 dy

2
e − p /2 ∞ − y2
=
√π
√ π, since
−∞
e

dy = √ π

2
= e − p /2 .

Problem 2(ii): Find the Fourier transform of the function


1 + x , for − a < x < 0
 a

 x
f ( x ) = 1 − , for 0 < x < a
 a
 0 , otherwise.

M-80

Solution: We have
~ 1 ∞ 1  0 a 
f ( p) = ⋅
√ (2 π ) − ∞∫ f ( x ) e ipx dx =

√ (2 π )  −a
f ( x ) e ipx dx +
0∫ f ( x ) e ipx dx 

 ipx 
0 a
  
ipx ipx ipx
1   x e e  x e e
= 1 +  −  + 1 −  + 
√ (2 π )   a ip 2
a( ip)  −a   a  ip a( ip) 0 
2
 
 
1 1 1 e −ipa e ipa 1 1 
= − + + − −
√ (2 π )  ip a( ip) 2 a( ip)2 a( ip)2 ip a( ip)2 
 
 2 
( e ipa + e −ipa ) −
1 1
=  
√ (2 π )  a( ip)2 a( ip)2 
1 2
= ⋅ [cos pa − 1]
√ (2 π ) a( ip)2
1 2 1 2
= ⋅ [cos pa − 1] = ⋅ [1 − cos pa].
ap2 π ap2 π
1 − x2 , | x |≤ 1
Problem 3(i): Find the Fourier transform of F ( x ) =  and hence evaluate
 0, | x | > 1
∞  x cos x − sin x  x
0 

x3
∫  cos dx.
 2
(Gorakhpur 2004, 12; Kanpur 10; Purvanchal 14)
Solution: We have
~ 1 ∞ ipx 1 1
F ( p) =

√ (2 π ) −∞
e F( x ) dx =
√ (2 π ) −1 ∫
(1 − x2 ) e ipx dx

1
1  1 − x2 ipx  2 1 e ipx
= 

√ (2 π )  ip
e 

+
 −1 √ (2 π ) −1
x⋅
ip ∫
dx

  ipx 1 
√2   xe  −
1 e ipx  √ 2  e ip + e −ip 1 
=
ip√ π   ip  −1
1⋅
ip ∫
dx =
 
ip√ π  ip

( ip)2
[ e ipx ]1−1 

 −1 
√ 2  2 cos p e ip − e −ip  √ 2  2 cos p 2isin p
=  + = − + 
ip√ π  ip p2  √ π  p2 ip3 
2  pcos p − sin p
= −2 ⋅ .
π  p3


We know that if
~ 1 ∞
F ( p) =

√ (2 π ) − ∞
F ( x ) e ipx dx

∞ ~
F ( p) e − ipx dp.
1
then F ( x) =

√ (2 π ) − ∞
M-81

∞ 2 √ (2 / π )( p cos p − sin p) 1 − x2 , | x |< 1


. e −ipx dp =
1
∴ −
√ (2 π ) − ∞ ∫ p3

 0, | x | > 1
∞  p cos p − sin p ∞  p cos p − sin p
or −
∫ −∞  p3
 cos px dp + i



−∞  ∫ p3
 sin px dp


 π (1 − x2 ) , | x |< 1

= 2
 0, | x | > 1
∞ p cos p − sin p  π (1 − x2 ) , | x |< 1
or −
−∞∫ p3
cos px dp =  2

0 , | x |> 1,

since integrand in the second integral on L.H.S. is an odd function.
1
Taking x = , we have
2
∞ p cos p − sin p . p π 3π
dp = 1 −  =
1

−∞∫ p3
cos
2 2 4 8
∞ p cos p − sin p p 3π
or 2
∫0 p3
⋅ cos
2
dp = −
8
∞  x cos x − sin x  x 3π
or
∫0  x3  ⋅ cos dx = −
 2 16

Problem 3(ii): Find the cosine transform of the function f ( x ), if


cos x, 0 < x < a
f ( x) = 
 0, x > a.
Solution: We have
 2  ∞ f ( x ) ⋅ cos px dx =
~ a
2
fc ( p) =  
 π 0 ∫  
 π ∫0 cos x cos px dx
1 a
=
√ (2 π ) 0 ∫
[cos (1 + p) x + cos (1 − p) x ] dx

a
1  sin(1 + p) x sin(1 − p) x  1  sin(1 + p) a sin(1 − p) a 
= + = + .
√ (2 π )  1 + p 1 − p  0 √ (2 π )  1 + p (1 − p) 

 sin x , 0 < x < a


Problem 3(iii): Find the sine transform of the function f ( x ) = 
 0, x > 0.
(Gorakhpur 2013)
Solution: We have
2 ∞ 2 a
fs ( p) =
π 0 ∫
f ( x ) sin px dx =
π 0 ∫
sin x sin px dx

1 a
=
2π 0 ∫
[cos (1 − p) x − cos (1 + p) x ] dx
a
1  sin(1 − p) x sin(1 + p) x  1  sin(1 − p) a sin(1 + p) a 
= − = − ⋅
2 π  1 − p 1 + p  0 2 π  1 − p 1 + p 
M-82

Problem 4(i): Find the Fourier sine and cosine transform of f ( x ), if


 x, 0 < x <1

f ( x ) = 2 − x, 1 < x < 2 (Kanpur 2007, 11, 14; Gorakhpur 16)
0, x > 2.

Solution: We have
~ ∞  1 2 
fs ( p) =   2
2
 π ∫0 f ( x ) sin px dx =  
 π
 0

⋅  x sin px dx +
1 ∫
(2 − x ) sin px dx 

 
1 2 
2 x 1   2 − x  1 
=   ⋅   − cos px + sin px  + −   cos px − sin px  
 π 
 p  1 
p2  0   p  p2
 
2 2 
sin2 p = 2   ⋅
1 2 sin p
=   ⋅  sin p − (1 − cos p).
 π  p
2 p2   π  p2
~
2 ∞ 2 1 2 
Again fc ( p) =  
 π ∫0 f ( x )cos px dx =  
 π
 0

⋅  x cos px dx +
1 ∫
(2 − x ) cos px dx 

 
1
 2 − x 
2
2 x 1 cos px  
=   ⋅   sin px + cos px  +   sin px − 
 π 
 p 
p2
 0  p  p2 1 
 

= 2 1  2  cos p
[2 cos p − 1 − cos 2 p] = 2   ⋅ (1 − cos p).
 
 π p2  π  p2

Problem 4(ii): Find the Fourier sine and cosine transform of the function f ( x ) = x m − 1.
Solution: If f ( x ) = x m − 1, then
 2  ∞ f ( x ) sin px dx = 2 ∞ m −1
~
fs =  
 π 0 ∫ π 0
x
∫ sin px dx
~ 2 ∞ 2 ∞ m −1
and fc ( p) =
π 0 ∫ f ( x ) cos px dx =
π 0
x
∫ cos px dx

m − 1 cos px dx − i ∞
x m − 1 sin px dx
Now
∫x ∫0
∞ m −1
=
∫0x (cos px − i sin px ) dx
∞ − ipx m − 1 Γm  ∞ − az n − 1 Γn 
=
∫0 e x dx =
( ip)m 

∵ 0 e z dz = 
an 
Γn −m Γm  mπ mπ 
=
( i) =  cos − isin  …(1)
pm pm  2 2 
Equating real and imaginary parts on both sides of (1), we have
∞ m −1 Γm mπ ∞ m −1 Γm mπ
0∫ x cos px dx =
pm
cos
2
and
0
x sin px dx =
pm
sin
∫ 2

~ 2 Γm mπ ~ 2 Γm mπ
∴ fs ( p) = sin and fc ( p) = cos ⋅
π pm 2 π pm 2
M-83

Problem 5(i): Find the Fourier sine transform of x /(1 + x2 ).


Solution: We have
~
2 ∞  2  ∞ x sin px dx
fs ( p) =  
 π ∫0 f ( x )sin px dx =   ∫
 π  0 1 + x2

2 ∞ ( x2 + 1 − 1)
=  
 π ∫0 x(1 + x2 )
sin px dx

2 ∞ sin px  2  ∞ sin px dx


=  
 π ∫0 x
dx −   ∫
 π  0 x(1 + x2 )

2 ⋅ π −  2  ⋅ ∞ sin px dx ⋅
=  
 π 2
  ∫
 π  0 x(1 + x2 )
…(1)

Differentiating both sides w.r.t. p, we have


d ~ ∞ cos px
fs ( p) = −  
2
dp  π  ∫
0 (1 + x2 )
dx. …(2)

Differentiating again, w.r.t. p, we have


d2 ~  2  ∞ x sin px dx = f ( p)
~ ~

dp2
fs ( p) =   ∫
 π  0 1 + x2 s or ( D2 − 1) fs ( p) = 0,

~
whose general solution is fs ( p) = Ae p + Be −p . …(3)
~
Now when p = 0, from (1), fs ( p) = √ ( π / 2) and from (2),
d ~ ∞ dx
fs ( p) = −  
2
dp  π  0 1 + x2

π
= −   (tan−1 x ) 0∞ = −   .
2
 π 2
d ~
Also from (3), fs ( p) = Ae p − Be −p .
dp
~
∴ when p = 0, fs ( p) = √ ( π /2 ) ⇒ A + B = √ ( π / 2)
d ~
and fs ( p) = − √ ( π /2 ) ⇒ A − B = − √ ( π /2).
dp
Solving, A = 0 and B = √( π /2).
~
∴ From (3), we have fs ( p) = √ ( π / 2) ⋅ e −p .
e ax + e −ax
Problem 5(ii): Find the cosine transform of ⋅
e πx + e − πx
(Purvanchal 2014; Kanpur 14)
e ax + e −ax
Solution: If f ( x ) = , then
e πx e − πx
M-84

~
2 ∞ e ax + e −ax 2 ∞ e ax + e −ax e ipx + e −ipx
fc ( p) =  
 π ∫0 eπ x + e − πx
cos px dx =  
 π ∫0 e πx + e − πx

2
dx

1 ∞ e(a + ip) x + e −(a + ip) x 1 ∞ e(a − ip) x + e −(a − ip) x


=
√ (2 π ) 0 ∫ e π x + e − π x
dx +
√ (2 π ) 0 ∫
e πx + e − πx
dx

1 1 a + ip 1 1 a − ip
= ⋅ sec + ⋅ sec
√ (2 π ) 2 2 √ (2 π ) 2 2
 ∞ e az + e − az 1 a
∵ From definite integrals,

π
0 e +ez − π z ∫
dz = sec 
2 2 
1 1
= +
a + ip a − ip
2 √ (2 π ) cos 2 √ (2 π ) cos
2 2
a − ip a + ip  cos a   cos ip
cos + cos    
= 2 2 = 2 . 2  2
 
a + ip a − ip  π cos a + cos ip
2 √ (2 π )cos cos
2 2
 2  ⋅ cos( a /2)cosh( p /2) =  2  cos( a /2) ⋅ ( e
p /2
+ e − p /2 )
=     ⋅
 π cos a + cosh p  π 2 cos a + e + e −p
p

1
Problem 6: Find the sine transform of and deduce that
e πx − e − πx
1
Fs ( cosech πx ) = tanh ( p /2).
√ (2 π )
1
Solution: If f ( x ) = , then we have
e − e − πx
π x

2 ∞ 1 1 ∞ e ipx − e −ipx


Fs { f ( x )} =  
 π ∫0 e πx − e − πx
sin px dx =
√ (2 π ) i 0 ∫e πx − e − πx
dx

1 1 ip
= ⋅ tan [ From Definite integrals]
√ (2 π ) i 2 2
 1  1
or Fs  = tanh( p / 2) …(1)
 e πx − e −πx  2 √ (2 π )
1 e p /2 − e − p /2 1 e p −1
= = ⋅
2 √ (2 π ) e p /2 + e −p /2 2 √ (2 π ) e p + 1

Deduction: From (1), we have

Fs  =
1 1
 tanh ( p /2)
2sinh πx  2 √ (2 π )
1
or Fs {cosech πx } = tanh ( p /2).
√ (2 π )
M-85

Problem 7: Find the Fourier sine transform of f ( x ) , if


0, 0 < x < a

f ( x ) =  x, a ≤ x ≤ b
0, x > b.

~
2 ∞
Solution: We have fs ( p) =  
 π ∫0 f ( x ).sin px dx

 2   a 0.sin px dx + b ∞ 
=   
 π  0 ∫ ∫ a xsin px dx + ∫ b 0 ⋅ sin px dx
 b
  
b
  − x cos px  +  sin px  
2 b 2
=  
 π ∫
a
x sin px dx =

 
 π p  a  p2  
 a 

=  2   − b cos pb + a cos pa + sin pb − sin pa  .


   
 π  p p2 

Problem 8: Find f ( x ) if its sine transform is π /2.


~
Solution: We have fs ( p) = π/2.
∴ Applying Fourier sine inversion formula, we have
∞~  2  ∞ π ⋅ sin px dp
f ( x ) =  
2
 π 0 s ∫
f ( p) sin px dp =  
 π 0 2∫
 π ∞
=  
2 ∫0 sin px dp. …(1)


∞ − ipx  e − ipx 
Now, we have
∫ 0
e dp =  
 ( − ix )  p = 0
= 1 /( ix ) = − i / x


or
∫0 (cos px − isin px) dp = − i / x.
Equating imaginary parts on both sides, we have

∫0 sin px dp = 1 / x.
Hence from (1), f ( x ) = √ ( π /2) ⋅ (1 / x ).

Problem 9: Find f ( x ) if (i) its sine transform is e −ap , (ii) its cosine transform is e −ap .
(Kanpur 2012)
~
Solution: (i) Here fs ( p) = e −ap .
∞~ ∞ −ap
f ( x ) =   2
2

 π 0 s ∫
f ( p) sin px dp =  
 π ∫0 e sin px dp


−ap
= 2 ⋅ e 
( − asin px − x cos px ) = 2 ⋅ x
   2   2 .
 π   a + x2  0  π  a + x2

M-86
~
(ii) Here fc ( p) e −ap .

2 ∞~ 2 ∞ −ap


∴ f ( x) =  
 π ∫0 fc ( p) cos px dp =  
 π ∫0 e cos px dp


−ap
= 2 ⋅ e 
( − a cos px + x sin px ) = 2 ⋅ a
   2   2 .
 π   a + x2  0  π  a + x2

~
Problem 10: Find the inverse Fourier transform of f ( p) = e −|p | y.
 − p, p ≤ 0
Solution: We have |p| = 
 p, p ≥ 0.
∞ ~ ∞ −| p| y −ipx
f ( p) ⋅ e −ipx dp =
1 1
∴ f ( x) =

√ (2 π ) − ∞ √ (2 π ) −∞
e .e

dp

0 ∞ −py −ipx
e py. e −ipx dp +
1 1
=

√ (2 π ) − ∞ √ (2 π ) 0
e .e

dp

1 0 ( y − ix) p 1 ∞ −p ( y + ix)
=

√ (2 π ) − ∞
e dp +
√ (2 π ) 0
e dp

0 ∞
1  e( y − ix) p  1  e −p ( y+ ix) 
=   +  
√ (2 π )  ( y − ix )  √ (2 π )  −( y + ix ) 
−∞ 0

1  1 1  y √2
= + = .
√ (2 π )  ( y − ix ) ( y + ix )  √ π( y2 + x2 )

e −ap
~
Problem 11: ⋅ Hence deduce Fs−1 {1 / p}.
Find f ( x ) if fs ( p) =
p (Gorakhpur 2009)
Solution: Using Fourier cosine inversion formula, we have

2 ∞ e −ap
Fs−1 {e −ap / p} = f ( x ) =  
 π ∫0 p
sin px dp.

df 2 ∞ −ap 2 ⋅ a



dx
=  
 π ∫0 e cos px dp =   2
 π  a + x2

2 a dx
f = √ (2 / π )tan−1( x / a) + A
or f =  
 π ∫ a2 + x2 + A or

But when x = 0, f = 0.
∴ A = 0.

Hence f ( x ) = Fs−1 {e −ap / p} = √ (2 / π ) tan−1 ( x / a). …(1)


M-87

Deduction: Now putting a = 0 in (1), we have

Fs−1 {1 / p} = √ (2 / π ).tan−1 ∞ = √ ( π /2).

Problem 12: Solve the integral equation


∞ (1 − λ ), for 0 ≤ λ ≤ 1
∫0 f ( x ) cos λ x dx = 
0, for λ >1
∞ sin2 t
Hence evaluate
∫0 t2
dt.
(Gorakhpur 2007, 09, 11, 13)

Solution: We have
∫0 f ( x ) cos λ x dx = fc ( λ ).

~ 1 − λ, 0 ≤ λ ≤ 1
∴ fc ( λ ) =  …(1)
0, λ > 1.

Now using Fourier cosine inversion formula, we have


2 ∞
f ( x) =
π 0 ∫
fc ( λ ) cos λ x dλ

2 1
=
π 0 ∫
(1 − λ ) cos λ x dλ

1
2 (1 − λ ) sin λ x  1 sin λ x dx
=
π

x
 −
0 0
( −1)
∫ x
1
2  cos λ x  2 (1 − cos xc )
= − 2  = ⋅
π  x 0 π x2
~ 2 ∞ 2 ∞ 2 (1 − cos x )
Thus fc ( λ ) =
π 0∫f ( x ) cos λ x dx =

π 0 π x2
cos λ x dx. …(2)

Now from (1) and (2), we get


2 ∞  1 − cos x  1 − λ ,0 ≤ λ ≤ 1


π 0  x2 
 cos λ x dx = 
0, λ> 1

Now letting λ → 0, we get


2 ∞ (1 − cos x )
π 0 ∫ x2
dx = 1

∞ 2sin2 x / 2dx π
or
∫0 x2
=
2
∞ sin2 t π x
or
∫0 t 2
= ⋅
2
[ Putting
2
= t so that dx = 2 dt ]
M-88

H ints to O bjective T ype Q uestions

Multiple Choice Questions


1. (a). See article 11 Theorem 1.
2 (b). See article 14 (ii).
3. (b). See Example 6.
e −ax ~
2 ∞ e −ax
4. (a). If f ( x ) =
x
, then we have fs ( p) =  
 π ∫0 x
sin px dx.

Differentiating both sides, w.r.t. p, we have


d ~ 2 ∞ −ax
dp
fs ( p) =  
 π ∫0 e cos px dx


−ax
= 2  e 
( − a cos px + psin px ) =
a
⋅   .
2
   2
 π   a + p2  0 a2 + p2  π

~
fs ( p) = a   −1( p / a) + c.
2 dp

 π ∫ a2 + p2 + c = √ (2 / π )tan
~
But when p = 0, fs ( p) = 0. ∴ c = 0.
~
Hence fs ( p) = √ (2 / π ) ⋅ tan−1( p / a).

Fill in the Blank(s)


1. See article 4 2. See article 6
3. See article 12 4. See article 14 (i).
5. See Ex. 5.
True or False
1. T. See article 10
2. F. See article 11 Theorem 3.
3. T. See Problem 9(i) of Comprehensive Problems 2.
4. F. See article 14 (iii).
5. T. See article 20.
6. T. See article 19.

❍❍❍
C-89

Chapter-5
Finite Fourier Transforms

Comprehensive Problems 1

Problem 1(i): Find the finite Fourier sine and cosine transforms of f ( x ) = 1.
(Gorakhpur 2014)
Solution: We have
~ π π π
f s( p) =
0 ∫
f ( x ) sin px dx =
0 ∫
1⋅ sin px dx = sin px dx
0 ∫
π
 cos px  cos pπ 1 1
= − =− + = [1 − ( −1)p ]
 p  0 p p p
~ π π
and fc ( p) =
∫0 f ( x ) cos px dx =
∫0 1⋅ cos px dx
π
= [(1 / p) sin px] = 0, if p = 1, 2, 3,...
0
Problem 1(ii): Find the finite Fourier sine and cosine transforms of f ( x ) = x.
Solution: We have
π
~ π π  − x cos px  1 π
fs ( p) =
∫0 f ( x ) sin px dx =
∫0
x sin px dx = 
 p
 +
0 ∫
p 0
cos px dx

π
π ( −1) p +1  sin px  π ( −1)p +1
= +   = ⋅
 
p  p2 0 p
π
~ π π  x sin px  1 π
Also fc ( p) =
∫0 f ( x ) cos px dx =
∫ 0
x cos px dx = 
 p
 −
0 ∫
p 0
sin px dx

π
 cos px  ( −1) p − 1
=  = , if p = 1, 2, 3,...
 
 p2 0 p2
~ π π2
and if p = 0, then fc ( p) =
∫0 x ⋅1 dx =
2

Problem 2(i): Find the finite Fourier sine transforms of


1 − x  and x ⋅
 
 π 4π
M-90

Solution: Finite Fourier sine transform of (1 − x / π )


π
π x  x   cos px   π  1   cos px 
=
∫ 1 −  sin px dx =  1 −   −
0  π 
 π  p 0
 − −  −
0  π  p 
 dx ∫
π
1 1  sin px  1
=
−   = ⋅
p π  p2  p
0
And finite Fourier sine transform of x/4 π
π
π x    x   cos px   π 1  cos px 
=
∫   sin px dx. =  
0 4π  
 4π 
 −
 p 

0

∫0 4 π ⋅  − p 
 dx

π
1 1  sin px  ( − 1)p +1
=− cos pπ +  2  = ⋅
4p 4 π  p  4p
0
Problem 2(ii): Find the finite Fourier cosine transform of
1 − x  and x ⋅
 
 π 4π
Solution: Finite Fourier cosine transform of (1 − x / π )
π
π  x  x  sin px  π  1  sin px
=
∫ 1 −  cos px dx =  1 − 
0  π    π p  0

∫0 − 
 π
dx.
p
π
1  cos pπ  1
= − 2  = [1 − ( −1)p ].
π  p  πp2
0
And finite Fourier cosine transform of x/4 π
π
π  x    x   sin px   π  1  sin px
=
∫   cos px dx =  
0 4π  
⋅

 4π  p  

0

∫0  ⋅
4π 
dx
p
π
1  cos px  1
=   = [( −1)p − 1] .
4 π  p2  4 πp2
0
Problem 3: Find the finite Fourier sine transform of f ( x ) if
 x, 0 ≤ x ≤ π /2
(i) f ( x) = 
 π − x, π /2 ≤ x ≤ π .
 − x, x< c
(ii) f ( x) = 
 π − x, x > c, where 0 ≤ c ≤ π .
Solution: (i) We have
~ π π /2 π
fs ( p) =
∫0 f ( x ) sin px dx =
∫0 x sin px dx +
∫ π /2 ( π − x) sin px dx
= [ x ( − cos px )/ p + (sin px )/ p2 ] 0π /2 + [( π − x ) ( − cos px )/ p − (sin px)/ p2 ]π
π /2
= (2/ p2 ) sin ( pπ /2).
M-91

(ii) We have
~ π c π
fs ( p) =
∫0 f ( x ) sin px dx =
∫0 − x sin px dx + ∫ c ( π − x) sin px dx
c π
 cos px  c cos px   cos px   1 π
=  x.
 p
 − 0 1.
0 p
dx +
∫ ( π − x )  −
  p
 −
 c p c
cos px dx

c 1 c π−c 1 π
= cos pc −
2 [sin px] 0 + cos pc −
2 [sin px]c
p p p p
c  1 π−c  1
= cos pc −   sin pc + cos pc +   (sin pc )
 2   2
p  p  p  p 
= ( π / p) cos pc.

Problem 4(i): Find the finite Fourier cosine transform of f ( x ) if


 1, 0 < x < π / 2
f ( x) = 
−1, π / 2 < x < π. (Kanpur 2012)
Solution: We have
~ π π /2 π
fc ( p) =
∫0 f ( x ) cos px dx =
∫0 1 . cos px dx +
∫ π /2 (−1) cos px dx
π /2 π
= [(sin px )/ p] − [(sin px )/ p] = (2 / p) sin ( pπ / 2), p > 0.
0 π /2
But if p = 0, then
~ π π /2 π
fc ( p) =
∫0 f ( x ) ⋅1 dx =
∫0 1⋅ dx +
∫ π /2 (− 1) dx = 0.
Problem 4(ii): Find the finite cosine transform of (1 − x / π )2 . (Avadh 2013)
Solution: We have
~ π  2
x
fc ( p) =
∫0 1 −  cos px dx
 π
π
 2
sin px  π
=  1 −  ⋅ 1 − x  sin px dx
x 2
  π  p
 +
 0 π p 0 ∫ 
 π

π
2  x   − cos px  1 sin px  2
=0 +  1 −   − ⋅  = 2 , p > 0.
π p  π  p  pπ p 0 πp
π
~ π  x
2  π  x 
3
π
If p = 0, fc ( p) =
∫ 1 −  ⋅1dx =  − 1 −   = ⋅
0  π  3  π 
0
3

Problem 5: Find the finite Fourier sine transforms of


(i) x ( π − x ) and (ii) x ( π2 − x2 ).
M-92

~ π
Solution: (i) fs {x ( π − x )} =
∫0 x ( π − x ) sin px dx

π
  cos px   π  cos px 
=  x ( π − x)  −
  p
 +
 0 0
( π − 2 x) 

 p 
 dx

π
  sin px   cos px   2
= ( π − 2 x )   − 2⋅   = [1 − ( −1)p ] .
 2   3 
  p   p   0 p3
~ π
(ii) fs {x ( π2 − x2 )} =
∫0 x ( π2 − x2 ) sin px dx

π
  cos px   π 2  cos px 
=  x ( π2 − x2 )  −
  p 0
 +
0
( π − 3 x2 ) 
 p  ∫
 dx

π
  sin px   π  sin px 
= ( π2 − 3 x2 )

 2 

 p   0

0
(− 6 x) 

 p2 
∫  dx

π
  cos px   sin px   6π
= 6x− +  = ( − 1)p +1.
 3   4 
  p   p   0 p3

Problem 6: Find the finite Fourier sine transform of f ( x ) if


π x2
(i) f ( x ) = sin nx, (ii) f ( x) = − x+ ⋅
3 2π
~ π 1 π
Solution: (i) fs ( p) =
∫0 sin nx sin px dx = 2 ∫0 [cos ( p − n) x − cos ( p + n) n] dx

π
1  sin ( p − n) x sin ( p + n) x 
= −  = 0, if p ≠ n.
2  p− n p+ n 0

π π
∫0 ∫0
If p = n, then ~ sin2 nx dx
f s ( p) = sin nx sin px dx =

π
1 π π
(1 − cos 2nx ) dx =  x −
1 sin 2nx 
=
2 0 ∫ 2 
= ⋅
2n  0 2
~ ~
Hence fs ( p) = 0 if p ≠ n, and f s ( p) = π /2 if p = n.
~ π π x2 
(ii) fs ( p) =
0  3∫
 − x+  sin px dx
2 π 
π
 π x2   − cos px   π  −1 + x  cos px dx
=  − x +
3
  2 π


 p
 +
  0
0 ∫ 
 π

p
M-93
π
π  x   sin px   π  1   sin px 
=
6p
{( −1)p + 2} +   −1 +   2   −
  
π  p 
0
  ∫
0  π   p2 
 dx

π 1 π
= {( −1)p + 2} +
3 [cos px] 0
6p πp
π 1
= {( −1)p + 2} + {( −1)p − 1}.
6p πp3

Problem 7: Find the finite sine transform of f ( x ), if


(i) f ( x ) = cos kx (ii) f ( x ) = x3 (iii) f ( x ) = e c x .
~ π
Solution: We have f s ( p) =
0 ∫
f ( x ) sin px dx.

~ π 1 π
(i) f s ( p) =
0 ∫
cos kx ⋅ sin px dx =
2 0 ∫
[sin ( k + p) x + sin ( p − k ) x ] dx

π
1  cos ( k + p) x ( p − k) x
= − − cos
2  ( k + p) ( p − k )  0

1  cos ( k + p) π cos ( p − k ) x 1 1 
= − − + +
2  k+ p p− k k + p p − k 

1  ( p − k ) cos ( k + p) π + ( k + p) cos ( p − k ) π p − k + k + p
= − + 
2  p2 − k2 p2 − k2 
1
= [ − p{cos( k + p) π + cos ( p − k ) π}
2 ( p − k2 )
2

+ k {cos ( k + p) π − cos ( p − k ) π} + 2 p]
1
= [ − p⋅ cos kπ cos pπ − k ⋅ sin kπ sin pπ + p]
( p2 − k2 )
p p
= [1 − cos kπ cos pπ ] = [1 − ( −1)p cos kπ ].
p2 − k2 p2 − k2
π
~ π 3  1  3 π 2
(ii) fs ( p) =
∫ 0
x sin px dx =  − x3 cos px  +
 p 0 p 0
x cos px dx

 π 
π3 3   x2  2 π
x sin px dx 
=−
p
cos pπ +  sin px  −
p   p 
0 p 0  ∫
 
π
π3 6  x sin px 
=− cos pπ −  − cos px + 
p p2  p p2  0

 6 π2   6 π2 
= π −  cos pπ = π ( − 1)p  − ⋅
 3 p   3 p 
p p
M-94

π
~ π cx  ec x 
(iii) fs ( p) =
∫ 0
e sin px dx = 
2
 c + p
2
( c sin px − p cos px )
 0

(1 − cos pπ e c π ) = [1 − ( − 1)p e c π ].
p p
=
c2 + p2 c2 + p2
cosh {c ( π − x )}
Problem 8: Find finite Fourier cosine transform of f ( x ) if f ( x ) = ⋅
sinh ( πc )
Solution: We have
~ π π cosh { c ( π − x )}
fc ( p) =
∫0 f ( x ) cos px dx =
∫0 sinh ( πc )
⋅ cos px dx

1 π c (π − x)
+ e − c (π − x)} cos px dx
1
= ⋅
sinh ( πc ) 2 0
{e

 c π π − cx π cx 
cos px dx + e − c π
1
= 
2 sinh ( πc ) 
e
0
e
∫ 0
e cos px dx 


1  e c π⋅ e − c x
=  ( − c cos px + p sin px )
2 sinh ( πc )  c2 + p2
π
e− c π ⋅ e c x 
+ ( c cos px + p sin px )
c2 + p2  0

1  {− c cos ( π p) + ce cπ} + {c cos ( π p) − ce − c π } 


=  
2 sinh ( πc )  c2 + p2 
c ( e c π − e −c π ) c ⋅ 2 sinh ( π c ) c
= = = ⋅
2 ( c2 + p2 ) sinh ( π c ) 2 ( c2 + p2 ) sinh ( π c ) c2 + p2

sin k ( π − x )
Problem 9: Find finite Fourier sine transform of f ( x ) , if f ( x ) = ⋅
sin ( kπ )
Solution: We have
~ π π sin {k ( π − x )}
fs ( p) =
∫0 f ( x ) sin px dx =
∫0 sin ( kπ )
⋅ sin px dx

1 π
= ⋅
2 sin ( kπ ) 0 ∫
[cos {px − k ( π − x )} − cos {px + k ( π − x )}] dx

π
1  sin {px − k ( π − x )} sin {px + k ( π − x )} 
= ⋅ − 
2 sin ( kπ )  p+ k p− k 0
1  sin pπ + sin kπ sin pπ − sin kπ 
= ⋅ −
2 sin ( kπ )  p+ k p− k 

1  1 1  p
= ⋅ +  sin ( kπ ) = 2 , k ≠ 0, 1, 2, …
2 sin ( kπ )  p + k p − k  p − k2
M-95

Problem 10: Find the finite Fourier sine and cosine transforms of f ( x ) = x2 , 0 < x < 4.
(Gorakhpur 2008, 14, 15)
Solution: We have
~ l 4 2
fs ( p) =
∫0 f ( x) sin ( pπx/l )dx = ∫0 x sin ( pπx /4) dx, as l = 4 (given)

4
 − x2 cos ( pπx /4)  4 4
=
 pπ /4
 +
 0 ∫
2 x⋅

cos ( pπx /4) dx
 0
4
64 8  x sin ( pπx /4) cos ( pπx /4) 
=− cos pπ +  + 
pπ pπ  pπ /4 ( pπ /4)2  0
64 128
=− cos pπ + (cos pπ − 1).
pπ p 3 π3
~ l 4 2
Also fc =
∫0 f ( x ) cos ( pπx / l ) dx =
∫0 x cos ( pπx /4) dx, as l = 4

4
 x2 sin ( pπx /4)  4 4
=
 pπ /4
 −
 0 ∫
2x ⋅

⋅ sin ( pπx /4) dx
 0
4
8  − x cos ( pπx /4) sin ( pπx /4)  128
=0 −  +  = 2 2 cos p π, if p > 0
p π  pπ /4 ( pπ /4)2
 0 p π
4
~ 4 2  x3  64
and if p = 0, then fc ( p) =
∫ 0
x dx =   =
 3  0 3

~ cos(2 pπ / 3)
Problem 11: Find f ( x ) if fc ( p) = , if 0 < x < 1.
(2 p + 1)2
1~ 2 ∞ ~ pπx
Solution: We have f ( x ) = fc (0 ) + Σ fc ( p) cos
l l p =1 l
1 2 ∞ cos (2 pπ /3) ∞ cos (2 pπ /3)
= ⋅1 + Σ cos pπx = 1 + 2 Σ cos pπx.
1 1 p = 1 (2 p + 1)2 p = 1 (2 p + 1)2

Problem 12: When f ( x ) = sin mx, where m is a positive integer, show that
~ ~
fs( p) = 0 if p ≠ m and fs( p) = π /2 if p = m.
~ π π
Solution: We have fs ( p) =
0 ∫
f ( x ) sin px dx =
0 ∫
sin mx sin px dx

1 π
=
2 0 ∫
[cos ( m − p) x − cos ( m + p) x ] dx
M-96

π
1  sin ( m − p) x sin ( m + p) x 
= −  = 0, if m ≠ p.
2  m− p m+ p 0
~ π 2 px dx = 1 π
If m = p, then fs ( p) =
∫0 sin 2 0 ∫ (1 − cos 2 px ) dx

π
1  sin 2 px  π
=  x − 2p  = 2 ⋅
2  0

H ints to O bjective T ype Q uestions

Multiple Choice Questions


1. (c). See article 8.
2. (a). See Problem 1(ii) of Comprehensive Problems 1.
2 ∞ ~
3. (c). We have f ( x ) = Σ fs ( p) sin px
π p =1
2 ∞ 2 π ( − 1)p −1 ∞ ( − 1)p −1
= Σ sin px = 4 Σ sin px.
π p =1 p3 p = 1 p3
4. (d). See Problem 2(i) of Comprehensive Problems 1.
Fill in the Blank(s)
1. See article 1
2. See article 3
3. See article 9
4. See Ex.1.
True or False
1. T. See article 2.
2. F. See article 8.
3. T. See Problem 1(i) of Comprehensive Problems 1.
4. T. See Ex. 5.

❍❍❍

You might also like